Π£Π½ΠΈΠ²Π΅Ρ€ΡΠ°Π»ΡŒΠ½Ρ‹ΠΉ внСшний Π½Π°ΠΊΠΎΠΏΠΈΡ‚Π΅Π»ΡŒ для всСх iOS-устройств, совмСстим с PC/Mac, Android
Header Banner
8 800 100 5771 | +7 495 540 4266
c 9:00 Π΄ΠΎ 24:00 ΠΏΠ½-ΠΏΡ‚ | c 10:00 Π΄ΠΎ 18:00 сб
0 Comments

Π‘ΠΎΠ΄Π΅Ρ€ΠΆΠ°Π½ΠΈΠ΅

РасчСт сопротивлСния Ρ†Π΅ΠΏΠΈ

РасчСт сопротивлСния Ρ†Π΅ΠΏΠΈ Π½Π΅ΠΎΠ±Ρ…ΠΎΠ΄ΠΈΠΌ ΠΏΡ€ΠΈ Ρ€Π΅ΡˆΠ΅Π½ΠΈΠΈ Ρ€Π°Π·Π»ΠΈΡ‡Π½Ρ‹Ρ… Π·Π°Π΄Π°Ρ‡ ΠΏΠΎ элСктротСхникС. Π‘ΡƒΡ‚ΡŒ Π·Π°ΠΊΠ»ΡŽΡ‡Π°Π΅Ρ‚ΡΡ Π² ΠΏΡ€ΠΈΠ²Π΅Π΄Π΅Π½ΠΈΠΈ слоТной Ρ€Π°Π·Π²Π΅Ρ‚Π²Π»Π΅Π½Π½ΠΎΠΉ элСктричСской Ρ†Π΅ΠΏΠΈ ΠΊ Ρ†Π΅ΠΏΠΈ с СдинствСнным эквивалСнтным сопротивлСниСм, ΠΊΠΎΡ‚ΠΎΡ€ΡƒΡŽ Π½Π°Π·Ρ‹Π²Π°ΡŽΡ‚Β ΠΏΡ€ΠΎΡΡ‚ΠΎΠΉ элСктричСской Ρ†Π΅ΠΏΡŒΡŽ.Β 

ΠŸΡ€ΠΈΠΌΠ΅Ρ€ 1

Β 

ЦСпь Π² Π΄Π°Π½Π½ΠΎΠΌ ΠΏΡ€ΠΈΠΌΠ΅Ρ€Π΅ состоит ΠΈΠ· Π΄Π²ΡƒΡ… ΠΏΠΎΡΠ»Π΅Π΄ΠΎΠ²Π°Ρ‚Π΅Π»ΡŒΠ½ΠΎ соСдинСнных сопротивлСний, ΡΠ»Π΅Π΄ΠΎΠ²Π°Ρ‚Π΅Π»ΡŒΠ½ΠΎ, ΠΈΡ… ΠΎΠ±Ρ‰Π΅Π΅ сопротивлСниС Π±ΡƒΠ΄Π΅Ρ‚ Ρ€Π°Π²Π½ΠΎ суммС ΠΈΡ… сопротивлСний. ΠŸΠΎΠ΄Ρ€ΠΎΠ±Π½Π΅Π΅ ΠΎ Π²ΠΈΠ΄Π°Ρ… соСдинСний тут.

Допустим, Ρ‡Ρ‚ΠΎ R1=10 Ом R2=20 Ом, Ρ‚ΠΎΠ³Π΄Π°Β 

ΠŸΡ€ΠΈΠΌΠ΅Ρ€ 2

Β 

Π”Π²Π° сопротивлСния соСдинСны ΠΏΠ°Ρ€Π°Π»Π»Π΅Π»ΡŒΠ½ΠΎ, Π·Π½Π°Ρ‡ΠΈΡ‚ ΠΏΡ€ΠΈ сворачивании схСмы, ΠΎΠ±Ρ‰Π΅Π΅ сопротивлСниС Π±ΡƒΠ΄Π΅Ρ‚ Ρ€Π°Π²Π½ΠΎ (значСния R1,R2Β Ρ‚Π°ΠΊΠΈΠ΅ ΠΆΠ΅ ΠΊΠ°ΠΊ ΠΈ Π² ΠΏΡ€ΠΈΠΌΠ΅Ρ€Π΅ 1)Β 

МоТно Π·Π°ΠΌΠ΅Ρ‚ΠΈΡ‚ΡŒ, Ρ‡Ρ‚ΠΎ ΠΏΡ€ΠΈ ΠΏΠ°Ρ€Π°Π»Π»Π΅Π»ΡŒΠ½ΠΎΠΌ соСдинСнии ΠΎΠ±Ρ‰Π΅Π΅Β ΡΠΎΠΏΡ€ΠΎΡ‚ΠΈΠ²Π»Π΅Π½ΠΈΠ΅Β ΠΌΠ΅Π½ΡŒΡˆΠ΅, Ρ‡Π΅ΠΌ ΠΏΡ€ΠΈ ΠΏΠΎΡΠ»Π΅Π΄ΠΎΠ²Π°Ρ‚Π΅Π»ΡŒΠ½ΠΎΠΌ Π² нСсколько Ρ€Π°Π·.Β 

ΠŸΡ€ΠΈΠΌΠ΅Ρ€ 3

Β 

Π’ Π΄Π°Π½Π½ΠΎΠΌ ΠΏΡ€ΠΈΠΌΠ΅Ρ€Π΅ ситуация Π°Π½Π°Π»ΠΎΠ³ΠΈΡ‡Π½Π° ΠΏΡ€ΠΈΠΌΠ΅Ρ€Ρƒ 2, Π·Π° Ρ‚Π΅ΠΌ лишь ΠΈΡΠΊΠ»ΡŽΡ‡Π΅Π½ΠΈΠ΅ΠΌ, Ρ‡Ρ‚ΠΎ сопротивлСний Ρ‚Ρ€ΠΈ. Π’ΠΎΠ³Π΄Π° ΠΎΠ±Ρ‰Π΅Π΅ сопротивлСниС Π±ΡƒΠ΄Π΅Ρ‚ Ρ€Π°Π²Π½ΠΎ (R1,R2Β ΠΏΡ€Π΅ΠΆΠ½ΠΈΠ΅, R3=105 Ом)Β 

Β 

ΠŸΡ€ΠΈΠΌΠ΅Ρ€ 4

Β 

Π§Ρ‚ΠΎΠ±Ρ‹ Ρ€Π°ΡΡΡ‡ΠΈΡ‚Π°Ρ‚ΡŒ ΠΎΠ±Ρ‰Π΅Π΅ сопротивлСниС смСшанного соСдинСния ΠΏΡ€ΠΎΠ²ΠΎΠ΄Π½ΠΈΠΊΠΎΠ², Π½Π΅ΠΎΠ±Ρ…ΠΎΠ΄ΠΈΠΌΠΎ для Π½Π°Ρ‡Π°Π»Π° Π½Π°ΠΉΡ‚ΠΈ ΠΎΠ±Ρ‰Π΅Π΅ сопротивлСниС рСзисторов R1Β ΠΈ R2 соСдинСнных ΠΏΠ°Ρ€Π°Π»Π»Π΅Π»ΡŒΠ½ΠΎ, Π° Π·Π°Ρ‚Π΅ΠΌ ΠΎΠ±Ρ‰Π΅Π΅ сопротивлСниС, ΠΊΠ°ΠΊ сумму R12Β ΠΈ R3 соСдинСнных ΠΏΠΎΡΠ»Π΅Π΄ΠΎΠ²Π°Ρ‚Π΅Π»ΡŒΠ½ΠΎ.Β 

Β ΠŸΡ€ΠΈΠΌΠ΅Ρ€ 5

Данная элСктричСская Ρ†Π΅ΠΏΡŒ слоТнСС, Ρ‡Π΅ΠΌ ΠΏΡ€Π΅Π΄Ρ‹Π΄ΡƒΡ‰ΠΈΠ΅, Π½ΠΎ ΠΊΠ°ΠΊ ΠΌΠΎΠΆΠ½ΠΎ ΡƒΠ²ΠΈΠ΄Π΅Ρ‚ΡŒ, ΠΎΠ½Π° Ρ‚Π°ΠΊΠΆΠ΅ состоит ΠΈΠ· ΠΏΠΎΡΠ»Π΅Π΄ΠΎΠ²Π°Ρ‚Π΅Π»ΡŒΠ½ΠΎ ΠΈΠ»ΠΈ ΠΏΠ°Ρ€Π°Π»Π»Π΅Π»ΡŒΠ½ΠΎ соСдинСнных сопротивлСний, ΠΊΠΎΡ‚ΠΎΡ€Ρ‹Π΅ ΠΌΠΎΠΆΠ½ΠΎ постСпСнно ΡΠ²ΠΎΡ€Π°Ρ‡ΠΈΠ²Π°Ρ‚ΡŒ, приводя Ρ†Π΅ΠΏΡŒ ΠΊ СдинствСнному эквивалСнтному ΡΠΎΠΏΡ€ΠΎΡ‚ΠΈΠ²Π»Π΅Π½ΠΈΡŽ R.

R4=20 Ом, R5=40 Ом, R

6=15 Ом 

ΠŸΡƒΡ‚Π΅ΠΌ сворачивания Ρ†Π΅ΠΏΠΈ с ΠΏΠΎΠΌΠΎΡ‰ΡŒΡŽ ΠΏΡ€Π΅ΠΎΠ±Ρ€Π°Π·ΠΎΠ²Π°Π½ΠΈΠΉ ΠΏΠΎΡΠ»Π΅Π΄ΠΎΠ²Π°Ρ‚Π΅Π»ΡŒΠ½ΠΎ ΠΈ ΠΏΠ°Ρ€Π°Π»Π»Π΅Π»ΡŒΠ½ΠΎ соСдинСнных ΠΏΡ€ΠΎΠ²ΠΎΠ΄Π½ΠΈΠΊΠΎΠ², ΠΌΠΎΠΆΠ½ΠΎ максимально ΡƒΠΏΡ€ΠΎΡΡ‚ΠΈΡ‚ΡŒ для дальнСйшСго расчСта сколь ΡƒΠ³ΠΎΠ΄Π½ΠΎ ΡΠ»ΠΎΠΆΠ½ΡƒΡŽ схСму. Π˜ΡΠΊΠ»ΡŽΡ‡Π΅Π½ΠΈΠ΅ΠΌ слуТат Ρ†Π΅ΠΏΠΈ содСрТащиС сопротивлСния, соСдинСнныС ΠΏΠΎ схСмС Π·Π²Π΅Π·Π΄Π° ΠΈ Ρ‚Ρ€Π΅ΡƒΠ³ΠΎΠ»ΡŒΠ½ΠΈΠΊ.Β Β 

  • ΠŸΡ€ΠΎΡΠΌΠΎΡ‚Ρ€ΠΎΠ²: 54377
  • ЀизичСская Ρ„ΠΎΡ€ΠΌΡƒΠ»Π° расчСта (опрСдСлСния) эквивалСнтного сопротивлСния Π² Ρ†Π΅ΠΏΠΈ

    Если элСктричСская Ρ†Π΅ΠΏΡŒ содСрТит нСсколько рСзисторов, Ρ‚ΠΎ для подсчёта Π΅Ρ‘ основных ΠΏΠ°Ρ€Π°ΠΌΠ΅Ρ‚Ρ€ΠΎΠ² (силы Ρ‚ΠΎΠΊΠ°, напряТСния, мощности) ΡƒΠ΄ΠΎΠ±Π½ΠΎ всС рСзистивныС устройства Π·Π°ΠΌΠ΅Π½ΠΈΡ‚ΡŒ Π½Π° ΠΎΠ΄Π½ΠΎ эквивалСнтноС сопротивлСниС Ρ†Π΅ΠΏΠΈ. Волько для Π½Π΅Π³ΠΎ Π΄ΠΎΠ»ΠΆΠ½ΠΎ Π²Ρ‹ΠΏΠΎΠ»Π½ΡΡ‚ΡŒΡΡ ΡΠ»Π΅Π΄ΡƒΡŽΡ‰Π΅Π΅ Ρ‚Ρ€Π΅Π±ΠΎΠ²Π°Π½ΠΈΠ΅: Π΅Π³ΠΎ сопротивлСниС Π΄ΠΎΠ»ΠΆΠ½ΠΎ Π±Ρ‹Ρ‚ΡŒ Ρ€Π°Π²Π½Ρ‹ΠΌ суммарному Π·Π½Π°Ρ‡Π΅Π½ΠΈΡŽ сопротивлСний всСх элСмСнтов, Ρ‚ΠΎ Π΅ΡΡ‚ΡŒ показания Π°ΠΌΠΏΠ΅Ρ€ΠΌΠ΅Ρ‚Ρ€Π° ΠΈ Π²ΠΎΠ»ΡŒΡ‚ΠΌΠ΅Ρ‚Ρ€Π° Π² ΠΎΠ±Ρ‹Ρ‡Π½ΠΎΠΉ схСмС ΠΈ Π² ΠΏΡ€Π΅ΠΎΠ±Ρ€Π°Π·ΠΎΠ²Π°Π½Π½ΠΎΠΉ Π½Π΅ Π΄ΠΎΠ»ΠΆΠ½Ρ‹ ΠΈΠ·ΠΌΠ΅Π½ΠΈΡ‚ΡŒΡΡ. Π’Π°ΠΊΠΎΠΉ ΠΏΠΎΠ΄Ρ…ΠΎΠ΄ ΠΊ Ρ€Π΅ΡˆΠ΅Π½ΠΈΡŽ Π·Π°Π΄Π°Ρ‡ называСтся ΠΌΠ΅Ρ‚ΠΎΠ΄ΠΎΠΌ свёртывания Ρ†Π΅ΠΏΠΈ.

    ΠœΠ΅Ρ‚ΠΎΠ΄ свёртывания Ρ†Π΅ΠΏΠΈ

    Π’Π½ΠΈΠΌΠ°Π½ΠΈΠ΅! Расчёт эквивалСнтного (ΠΎΠ±Ρ‰Π΅Π³ΠΎ ΠΈΠ»ΠΈ суммарного) сопротивлСния Π² случаС ΠΏΠΎΡΠ»Π΅Π΄ΠΎΠ²Π°Ρ‚Π΅Π»ΡŒΠ½ΠΎΠ³ΠΎ ΠΈΠ»ΠΈ ΠΏΠ°Ρ€Π°Π»Π»Π΅Π»ΡŒΠ½ΠΎΠ³ΠΎ ΠΏΠΎΠ΄ΠΊΠ»ΡŽΡ‡Π΅Π½ΠΈΡ выполняСтся ΠΏΠΎ Ρ€Π°Π·Π½Ρ‹ΠΌ Ρ„ΠΎΡ€ΠΌΡƒΠ»Π°ΠΌ.

    ΠŸΠΎΡΠ»Π΅Π΄ΠΎΠ²Π°Ρ‚Π΅Π»ΡŒΠ½ΠΎΠ΅ соСдинСниС элСмСнтов

    Π’ случаС ΠΏΠΎΡΠ»Π΅Π΄ΠΎΠ²Π°Ρ‚Π΅Π»ΡŒΠ½ΠΎΠ³ΠΎ ΠΏΠΎΠ΄ΠΊΠ»ΡŽΡ‡Π΅Π½ΠΈΡ всС ΠΏΡ€ΠΈΠ±ΠΎΡ€Ρ‹ ΡΠΎΠ΅Π΄ΠΈΠ½ΡΡŽΡ‚ΡΡ ΠΏΠΎΡΠ»Π΅Π΄ΠΎΠ²Π°Ρ‚Π΅Π»ΡŒΠ½ΠΎ Π΄Ρ€ΡƒΠ³ с Π΄Ρ€ΡƒΠ³ΠΎΠΌ, Π° собранная Ρ†Π΅ΠΏΡŒ Π½Π΅ ΠΈΠΌΠ΅Π΅Ρ‚ Ρ€Π°Π·Π²Π΅Ρ‚Π²Π»Π΅Π½ΠΈΠΉ.

    ΠŸΡ€ΠΈ Ρ‚Π°ΠΊΠΎΠΌ ΠΏΠΎΠ΄ΠΊΠ»ΡŽΡ‡Π΅Π½ΠΈΠΈ сила Ρ‚ΠΎΠΊΠ°, проходящая Ρ‡Π΅Ρ€Π΅Π· ΠΊΠ°ΠΆΠ΄Ρ‹ΠΉ рСзистор, Π±ΡƒΠ΄Π΅Ρ‚ одинаковая, Π° ΠΎΠ±Ρ‰Π΅Π΅ ΠΏΠ°Π΄Π΅Π½ΠΈΠ΅ напряТСния складываСтся ΠΈΠ· суммарных ΠΏΠ°Π΄Π΅Π½ΠΈΠΉ напряТСния Π½Π° ΠΊΠ°ΠΆΠ΄ΠΎΠΌ ΠΈΠ· ΠΏΡ€ΠΈΠ±ΠΎΡ€ΠΎΠ².

    ΠŸΠΎΡΠ»Π΅Π΄ΠΎΠ²Π°Ρ‚Π΅Π»ΡŒΠ½ΠΎΠ΅ ΠΏΠΎΠ΄ΠΊΠ»ΡŽΡ‡Π΅Π½ΠΈΠ΅ ΠΏΡ€ΠΈΠ±ΠΎΡ€ΠΎΠ²

    Π§Ρ‚ΠΎΠ±Ρ‹ ΠΎΠΏΡ€Π΅Π΄Π΅Π»ΠΈΡ‚ΡŒ суммарноС Π·Π½Π°Ρ‡Π΅Π½ΠΈΠ΅ Π² этом случаС, Π²ΠΎΡΠΏΠΎΠ»ΡŒΠ·ΡƒΠ΅ΠΌΡΡ Π·Π°ΠΊΠΎΠ½ΠΎΠΌ Ома, ΠΊΠΎΡ‚ΠΎΡ€Ρ‹ΠΉ записываСтся ΡΠ»Π΅Π΄ΡƒΡŽΡ‰ΠΈΠΌ ΠΎΠ±Ρ€Π°Π·ΠΎΠΌ:

    I = U/R.

    Из Π²Ρ‹ΡˆΠ΅ΡΡ‚ΠΎΡΡ‰Π΅Π³ΠΎ выраТСния ΠΏΠΎΠ»ΡƒΡ‡Π°Π΅ΠΌ Π·Π½Π°Ρ‡Π΅Π½ΠΈΠ΅ R:

    R = U/I (1).

    ΠŸΠΎΡΠΊΠΎΠ»ΡŒΠΊΡƒ ΠΏΡ€ΠΈ ΠΏΠΎΡΠ»Π΅Π΄ΠΎΠ²Π°Ρ‚Π΅Π»ΡŒΠ½ΠΎΠΌ соСдинСнии:

    • I = I1 = I2 =…= IN (2),
    • U = U1 + U2 +…+ UN (3),

    Ρ„ΠΎΡ€ΠΌΡƒΠ»Π° для расчёта эквивалСнтного сопротивлСния (RΠΎΠ±Ρ‰ ΠΈΠ»ΠΈ Rэкв) ΠΈΠ· (1) – (3) Π±ΡƒΠ΄Π΅Ρ‚ ΠΈΠΌΠ΅Ρ‚ΡŒ Π²ΠΈΠ΄:

    • Rэкв = (U1 + U2 + …+ UN)/I,
    • Rэкв = R1 + R2 + … + RN (4).

    Π’Π°ΠΊΠΈΠΌ ΠΎΠ±Ρ€Π°Π·ΠΎΠΌ, Ссли имССтся N ΠΏΠΎΡΠ»Π΅Π΄ΠΎΠ²Π°Ρ‚Π΅Π»ΡŒΠ½ΠΎ соСдинённых ΠΎΠ΄ΠΈΠ½Π°ΠΊΠΎΠ²Ρ‹Ρ… элСмСнтов, Ρ‚ΠΎ ΠΈΡ… ΠΌΠΎΠΆΠ½ΠΎ Π·Π°ΠΌΠ΅Π½ΠΈΡ‚ΡŒ Π½Π° ΠΎΠ΄Π½ΠΎ устройство, Ρƒ ΠΊΠΎΡ‚ΠΎΡ€ΠΎΠ³ΠΎ:

    RΠΎΠ±Ρ‰ = NΒ·R (5).

    ΠŸΠ°Ρ€Π°Π»Π»Π΅Π»ΡŒΠ½ΠΎΠ΅ соСдинСниС

    ΠŸΡ€ΠΈ Ρ‚Π°ΠΊΠΎΠΌ ΠΏΠΎΠ΄ΠΊΠ»ΡŽΡ‡Π΅Π½ΠΈΠΈ Π²Ρ…ΠΎΠ΄Ρ‹ ΠΎΡ‚ всСх устройств соСдинСны Π² ΠΎΠ΄Π½ΠΎΠΉ Ρ‚ΠΎΡ‡ΠΊΠ΅, Π²Ρ‹Ρ…ΠΎΠ΄Ρ‹ – Π² Π΄Ρ€ΡƒΠ³ΠΎΠΉ Ρ‚ΠΎΡ‡ΠΊΠ΅. Π­Ρ‚ΠΈ Ρ‚ΠΎΡ‡ΠΊΠΈ Π² Ρ„ΠΈΠ·ΠΈΠΊΠ΅ ΠΈ элСктротСхникС Π½Π°Π·Ρ‹Π²Π°ΡŽΡ‚ΡΡ ΡƒΠ·Π»Π°ΠΌΠΈ. На элСктричСских схСмах ΡƒΠ·Π»Ρ‹ ΠΏΡ€Π΅Π΄ΡΡ‚Π°Π²Π»ΡΡŽΡ‚ собой мСста развСтвлСния ΠΏΡ€ΠΎΠ²ΠΎΠ΄Π½ΠΈΠΊΠΎΠ² ΠΈ ΠΎΠ±ΠΎΠ·Π½Π°Ρ‡Π°ΡŽΡ‚ΡΡ Ρ‚ΠΎΡ‡ΠΊΠ°ΠΌΠΈ.

    ΠŸΠ°Ρ€Π°Π»Π»Π΅Π»ΡŒΠ½ΠΎΠ΅ соСдинСниС

    РасчСт эквивалСнтного сопротивлСния Ρ‚Π°ΠΊΠΆΠ΅ выполняСм с ΠΏΠΎΠΌΠΎΡ‰ΡŒΡŽ Π·Π°ΠΊΠΎΠ½Π° Ома.

    Π’ этом случаС ΠΎΠ±Ρ‰Π΅Π΅ Π·Π½Π°Ρ‡Π΅Π½ΠΈΠ΅ силы Ρ‚ΠΎΠΊΠ° складываСтся ΠΈΠ· суммы сил Ρ‚ΠΎΠΊΠΎΠ², ΠΏΡ€ΠΎΡ‚Π΅ΠΊΠ°ΡŽΡ‰ΠΈΡ… ΠΏΠΎ ΠΊΠ°ΠΆΠ΄ΠΎΠΉ Π²Π΅Ρ‚Π²ΠΈ, Π° Π²Π΅Π»ΠΈΡ‡ΠΈΠ½Π° падСния напряТСния для ΠΊΠ°ΠΆΠ΄ΠΎΠ³ΠΎ устройства ΠΈ ΠΎΠ±Ρ‰Π΅Π΅ напряТСниС ΠΎΠ΄ΠΈΠ½Π°ΠΊΠΎΠ²Ρ‹Π΅.

    Если ΠΈΠΌΠ΅ΡŽΡ‚ΡΡ N рСзистивных устройств, ΠΏΠΎΠ΄ΠΊΠ»ΡŽΡ‡Π΅Π½Π½Ρ‹Ρ… Ρ‚Π°ΠΊΠΈΠΌ ΠΎΠ±Ρ€Π°Π·ΠΎΠΌ, Ρ‚ΠΎ:

    I = I1 + I2 Β + … + IN (6),

    U = U1 = U2 = … = UN (7).

    Из Π²Ρ‹Ρ€Π°ΠΆΠ΅Π½ΠΈΠΉ (1), (6) ΠΈ (7) ΠΈΠΌΠ΅Π΅ΠΌ:

    • RΠΎΠ±Ρ‰ = U/(I1 + I2 + …+ IN),
    • 1/Rэкв = 1/R1 + 1/R2 +…+ 1/RN (8).

    Если имССтся N ΠΎΠ΄ΠΈΠ½Π°ΠΊΠΎΠ²Ρ‹Ρ… рСзисторов, ΠΈΠΌΠ΅ΡŽΡ‰ΠΈΡ… ΠΏΠΎΠ΄ΠΊΠ»ΡŽΡ‡Π΅Π½ΠΈΠ΅ Π΄Π°Π½Π½ΠΎΠ³ΠΎ Ρ‚ΠΈΠΏΠ°, Ρ‚ΠΎ Ρ„ΠΎΡ€ΠΌΡƒΠ»Π° (8) прСобразуСтся ΡΠ»Π΅Π΄ΡƒΡŽΡ‰ΠΈΠΌ ΠΎΠ±Ρ€Π°Π·ΠΎΠΌ:

    RΠΎΠ±Ρ‰ = R Β· R / NΒ·R = R / N (9).

    Если соСдинСны нСсколько ΠΊΠ°Ρ‚ΡƒΡˆΠ΅ΠΊ индуктивности, Ρ‚ΠΎ ΠΈΡ… суммарноС ΠΈΠ½Π΄ΡƒΠΊΡ‚ΠΈΠ²Π½ΠΎΠ΅ сопротивлСниС рассчитываСтся Ρ‚Π°ΠΊ ΠΆΠ΅, ΠΊΠ°ΠΊ ΠΈ для рСзисторов.

    Расчёт ΠΏΡ€ΠΈ смСшанном соСдинСнии устройств

    Π’ случаС смСшанного ΠΏΠΎΠ΄ΠΊΠ»ΡŽΡ‡Π΅Π½ΠΈΡ ΠΏΡ€ΠΈΡΡƒΡ‚ΡΡ‚Π²ΡƒΡŽΡ‚ участки с ΠΏΠΎΡΠ»Π΅Π΄ΠΎΠ²Π°Ρ‚Π΅Π»ΡŒΠ½Ρ‹ΠΌ ΠΈ ΠΏΠ°Ρ€Π°Π»Π»Π΅Π»ΡŒΠ½Ρ‹ΠΌ ΠΏΠΎΠ΄ΠΊΠ»ΡŽΡ‡Π΅Π½ΠΈΡΠΌΠΈ элСмСнтов.

    ΠŸΡ€ΠΈ Ρ€Π΅ΡˆΠ΅Π½ΠΈΠΈ Π·Π°Π΄Π°Ρ‡ΠΈ ΠΈΡΠΏΠΎΠ»ΡŒΠ·ΡƒΡŽΡ‚ ΠΌΠ΅Ρ‚ΠΎΠ΄ сворачивания Ρ†Π΅ΠΏΠΈ (ΠΌΠ΅Ρ‚ΠΎΠ΄ эквивалСнтных ΠΏΡ€Π΅ΠΎΠ±Ρ€Π°Π·ΠΎΠ²Π°Π½ΠΈΠΉ). Π•Π³ΠΎ ΠΈΡΠΏΠΎΠ»ΡŒΠ·ΡƒΡŽΡ‚ для вычислСния ΠΏΠ°Ρ€Π°ΠΌΠ΅Ρ‚Ρ€ΠΎΠ² Π² Ρ‚ΠΎΠΌ случаС, Ссли Π΅ΡΡ‚ΡŒ ΠΎΠ΄ΠΈΠ½ источник энСргии.

    ΠŸΡ€Π΅Π΄ΠΏΠΎΠ»ΠΎΠΆΠΈΠΌ, Π·Π°Π΄Π°Π½Π° ΡΠ»Π΅Π΄ΡƒΡŽΡ‰Π°Ρ Π·Π°Π΄Π°Ρ‡Π°. ЭлСктричСская схСма (см. рис. Π½ΠΈΠΆΠ΅) состоит ΠΈΠ· 7 рСзисторов. РассчитайтС Ρ‚ΠΎΠΊΠΈ Π½Π° всСх рСзисторах, Ссли ΠΈΠΌΠ΅ΡŽΡ‚ΡΡ ΡΠ»Π΅Π΄ΡƒΡŽΡ‰ΠΈΠ΅ исходныС Π΄Π°Π½Π½Ρ‹Π΅:

    • R1 = 1Ом,
    • R2 = 2Ом,
    • R3 = 3Ом,
    • R4 = 6Ом,
    • R5 = 9Ом,
    • R6 = 18Ом,
    • R7 = 2,8Ом,
    • U = 32Π’.

    ЭлСктричСская схСма

    Из закона Ома имССм: 

    I = U/R,

    Π³Π΄Π΅ R – суммарноС сопротивлСниС всСх ΠΏΡ€ΠΈΠ±ΠΎΡ€ΠΎΠ².

    Π•Π³ΠΎ Π±ΡƒΠ΄Π΅ΠΌ Π½Π°Ρ…ΠΎΠ΄ΠΈΡ‚ΡŒ, воспользовавшись ΠΌΠ΅Ρ‚ΠΎΠ΄ΠΎΠΌ сворачивания Ρ†Π΅ΠΏΠΈ.

    Π­Π»Π΅ΠΌΠ΅Π½Ρ‚Ρ‹ R2 ΠΈ R3 ΠΏΠΎΠ΄ΠΊΠ»ΡŽΡ‡Π΅Π½Ρ‹ ΠΏΠ°Ρ€Π°Π»Π»Π΅Π»ΡŒΠ½ΠΎ, поэтому ΠΈΡ… ΠΌΠΎΠΆΠ½ΠΎ Π·Π°ΠΌΠ΅Π½ΠΈΡ‚ΡŒ Π½Π° R2,3, Π²Π΅Π»ΠΈΡ‡ΠΈΠ½Ρƒ ΠΊΠΎΡ‚ΠΎΡ€ΠΎΠ³ΠΎ ΠΌΠΎΠΆΠ½ΠΎ Ρ€Π°ΡΡΡ‡ΠΈΡ‚Π°Ρ‚ΡŒ ΠΏΠΎ Ρ„ΠΎΡ€ΠΌΡƒΠ»Π΅:

    R2,3= R2Β·R3 / (R2+R3).

    R4, R5 ΠΈ R6 Ρ‚Π°ΠΊΠΆΠ΅ Π²ΠΊΠ»ΡŽΡ‡Π΅Π½Ρ‹ ΠΏΠ°Ρ€Π°Π»Π»Π΅Π»ΡŒΠ½ΠΎ, ΠΈ ΠΈΡ… ΠΌΠΎΠΆΠ½ΠΎ Π·Π°ΠΌΠ΅Π½ΠΈΡ‚ΡŒ Π½Π° R4,5,6, ΠΊΠΎΡ‚ΠΎΡ€ΠΎΠ΅ вычисляСтся ΡΠ»Π΅Π΄ΡƒΡŽΡ‰ΠΈΠΌ ΠΎΠ±Ρ€Π°Π·ΠΎΠΌ:

    1/R4,5,6 = 1/R4+1/R5+1/R6.

    Π’Π°ΠΊΠΈΠΌ ΠΎΠ±Ρ€Π°Π·ΠΎΠΌ, схСму, ΠΈΠ·ΠΎΠ±Ρ€Π°ΠΆΡ‘Π½Π½ΡƒΡŽ Π½Π° ΠΊΠ°Ρ€Ρ‚ΠΈΠ½ΠΊΠ΅ Π²Ρ‹ΡˆΠ΅, ΠΌΠΎΠΆΠ½ΠΎ Π·Π°ΠΌΠ΅Π½ΠΈΡ‚ΡŒ Π½Π° ΡΠΊΠ²ΠΈΠ²Π°Π»Π΅Π½Ρ‚Π½ΡƒΡŽ, Π² ΠΊΠΎΡ‚ΠΎΡ€ΠΎΠΉ вмСсто рСзисторов R2, R3 ΠΈ R4, R5, R6 ΠΈΡΠΏΠΎΠ»ΡŒΠ·ΡƒΡŽΡ‚ΡΡ R2,3 ΠΈ R4,5,6.

    ЭквивалСнтная схСма

    Богласно ΠΊΠ°Ρ€Ρ‚ΠΈΠ½ΠΊΠ΅ Π²Ρ‹ΡˆΠ΅, Π² Ρ€Π΅Π·ΡƒΠ»ΡŒΡ‚Π°Ρ‚Π΅ ΠΏΡ€Π΅ΠΎΠ±Ρ€Π°Π·ΠΎΠ²Π°Π½ΠΈΠΉ ΠΏΠΎΠ»ΡƒΡ‡Π°Π΅ΠΌ ΠΏΠΎΡΠ»Π΅Π΄ΠΎΠ²Π°Ρ‚Π΅Π»ΡŒΠ½ΠΎΠ΅ соСдинСниС рСзисторов R1, R2,3, R4,5,6 ΠΈ R7.

    RΠΎΠ±Ρ‰ ΠΌΠΎΠΆΠ΅Ρ‚ Π±Ρ‹Ρ‚ΡŒ Π½Π°ΠΉΠ΄Π΅Π½ΠΎ ΠΏΠΎ Ρ„ΠΎΡ€ΠΌΡƒΠ»Π΅:

    RΠΎΠ±Ρ‰ = R1 + R2,3 + R4,5,6 + R7.

    ΠŸΠΎΠ΄ΡΡ‚Π°Π²Π»ΡΠ΅ΠΌ числовыС значСния ΠΈ рассчитываСм R для ΠΎΠΏΡ€Π΅Π΄Π΅Π»Ρ‘Π½Π½Ρ‹Ρ… участков:

    • R2.3 = 2Ом·3Ом / (2Ом + 3Ом) = 1,2Ом,
    • 1/R4,5,6 = 1/6Ом + 1/9Ом + 1/18Ом = 1/3Ом,
    • R4,5,6 = 3Ом,
    • Rэкв = 1Ом + 1,2Ом + 3Ом + 2,8Ом= 8Ом.

    Π’Π΅ΠΏΠ΅Ρ€ΡŒ, послС Ρ‚ΠΎΠ³ΠΎ, ΠΊΠ°ΠΊ нашли Rэкв, ΠΌΠΎΠΆΠ½ΠΎ Π²Ρ‹Ρ‡ΠΈΡΠ»ΡΡ‚ΡŒ Π·Π½Π°Ρ‡Π΅Π½ΠΈΠ΅ I:

    I = 32Π’ / 8Ом = 4А.

    ПослС Ρ‚ΠΎΠ³ΠΎ, ΠΊΠ°ΠΊ ΠΌΡ‹ ΠΏΠΎΠ»ΡƒΡ‡ΠΈΠ»ΠΈ Π²Π΅Π»ΠΈΡ‡ΠΈΠ½Ρƒ ΠΎΠ±Ρ‰Π΅Π³ΠΎ Ρ‚ΠΎΠΊΠ°, ΠΌΠΎΠΆΠ½ΠΎ Π²Ρ‹Ρ‡ΠΈΡΠ»ΠΈΡ‚ΡŒ силу Ρ‚ΠΎΠΊΠ°, ΠΏΡ€ΠΎΡ‚Π΅ΠΊΠ°ΡŽΡ‰ΡƒΡŽ Π½Π° ΠΊΠ°ΠΆΠ΄ΠΎΠΌ участкС.

    ΠŸΠΎΡΠΊΠΎΠ»ΡŒΠΊΡƒ R1, R2,3, R4,5,6 ΠΈ R7 соСдинСны ΠΏΠΎΡΠ»Π΅Π΄ΠΎΠ²Π°Ρ‚Π΅Π»ΡŒΠ½ΠΎ, Ρ‚ΠΎ:

    I1 = I2,3 = I4,5,6 = I7 = I = 4А.

    На участкС R2,3 напряТСниС Π½Π°Ρ…ΠΎΠ΄ΠΈΠΌ ΠΏΠΎ Ρ„ΠΎΡ€ΠΌΡƒΠ»Π΅:

    • U2,3 = I2,3Β·R2,3,
    • U2,3 = 4А·1,2Ом = 4,8Π’.

    ΠŸΠΎΡΠΊΠΎΠ»ΡŒΠΊΡƒ R2 ΠΈ R3 ΠΏΠΎΠ΄ΠΊΠ»ΡŽΡ‡Π΅Π½Ρ‹ ΠΏΠ°Ρ€Π°Π»Π»Π΅Π»ΡŒΠ½ΠΎ, Ρ‚ΠΎ U2,3 = U2 = U3, ΡΠ»Π΅Π΄ΠΎΠ²Π°Ρ‚Π΅Π»ΡŒΠ½ΠΎ:

    • I2 = U2 / R2,
    • I2 = 4,8Π’ / 2Ом = 2,4А,
    • I3 = U3 / R3,
    • I3 = 4,8Π’ / 3Ом = 1,6А.

    ΠŸΡ€ΠΎΠ²Π΅Ρ€ΡΠ΅ΠΌ ΠΏΡ€Π°Π²ΠΈΠ»ΡŒΠ½ΠΎΡΡ‚ΡŒ Ρ€Π΅ΡˆΠ΅Π½ΠΈΡ:

    • I2,3 = I2 + I3,
    • I2,3 = 2,4А + 1,6А = 4А.

    На участкС R4,5,Π± напряТСниС Ρ‚Π°ΠΊΠΆΠ΅ Π½Π°Ρ…ΠΎΠ΄ΠΈΠΌ, исходя ΠΈΠ· Π·Π°ΠΊΠΎΠ½Π° Ома:

    • U4,5,6 = I4,5,6Β·R4,5,6,
    • U4,5,6 = 4А·3Ом = 12Π’.

    Π’Π°ΠΊ ΠΊΠ°ΠΊ R4, R5, RΠ± ΠΏΠΎΠ΄ΠΊΠ»ΡŽΡ‡Π΅Π½Ρ‹ ΠΏΠ°Ρ€Π°Π»Π»Π΅Π»ΡŒΠ½ΠΎ Π΄Ρ€ΡƒΠ³ ΠΊ Π΄Ρ€ΡƒΠ³Ρƒ, Ρ‚ΠΎ:

    U4,5,6 = U4 = U5 = U6 = 12Π’.

    ВычисляСм I4, I5, I6:

    • I4 = U4 / R4,
    • I4 = 12Π’ / 6Ом = 2А,
    • I5 = U5 / R5,
    • I5 = 12Π’ / 9Ом Β» 1,3А,
    • I6 = U6 / R6,
    • I5 = 12Π’ / 18Ом Β» 0,7А.

    ΠŸΡ€ΠΎΠ²Π΅Ρ€ΡΠ΅ΠΌ ΠΏΡ€Π°Π²ΠΈΠ»ΡŒΠ½ΠΎΡΡ‚ΡŒ Ρ€Π΅ΡˆΠ΅Π½ΠΈΡ:

    I4,5,6 = 2А + 1,3А + 0,7А = 4А.

    Π§Ρ‚ΠΎΠ±Ρ‹ Π°Π²Ρ‚ΠΎΠΌΠ°Ρ‚ΠΈΠ·ΠΈΡ€ΠΎΠ²Π°Ρ‚ΡŒ Π²Ρ‹ΠΏΠΎΠ»Π½Π΅Π½ΠΈΠ΅ расчётов эквивалСнтных Π·Π½Π°Ρ‡Π΅Π½ΠΈΠΉ для Ρ€Π°Π·Π»ΠΈΡ‡Π½Ρ‹Ρ… участков Ρ†Π΅ΠΏΠΈ, ΠΌΠΎΠΆΠ½ΠΎ Π²ΠΎΡΠΏΠΎΠ»ΡŒΠ·ΠΎΠ²Π°Ρ‚ΡŒΡΡ сСрвисами сСти Π˜Π½Ρ‚Π΅Ρ€Π½Π΅Ρ‚, ΠΊΠΎΡ‚ΠΎΡ€Ρ‹Π΅ ΠΏΡ€Π΅Π΄Π»Π°Π³Π°ΡŽΡ‚ Π½Π° ΠΈΡ… сайтах Π²Ρ‹ΠΏΠΎΠ»Π½ΠΈΡ‚ΡŒ ΠΎΠ½Π»Π°ΠΉΠ½ вычислСния Π½ΡƒΠΆΠ½Ρ‹Ρ… элСктричСских характСристик. БСрвис ΠΎΠ±Ρ‹Ρ‡Π½ΠΎ ΠΈΠΌΠ΅Π΅Ρ‚ Π²ΡΡ‚Ρ€ΠΎΠ΅Π½Π½ΡƒΡŽ ΡΠΏΠ΅Ρ†ΠΈΠ°Π»ΡŒΠ½ΡƒΡŽ ΠΏΡ€ΠΎΠ³Ρ€Π°ΠΌΠΌΡƒ – ΠΊΠ°Π»ΡŒΠΊΡƒΠ»ΡΡ‚ΠΎΡ€, которая ΠΏΠΎΠΌΠΎΠ³Π°Π΅Ρ‚ быстро Π²Ρ‹ΠΏΠΎΠ»Π½ΠΈΡ‚ΡŒ расчСт сопротивлСния Ρ†Π΅ΠΏΠΈ любой слоТности.

    Π’Π°ΠΊΠΈΠΌ ΠΎΠ±Ρ€Π°Π·ΠΎΠΌ, использованиС ΠΌΠ΅Ρ‚ΠΎΠ΄Π° эквивалСнтных ΠΏΡ€Π΅ΠΎΠ±Ρ€Π°Π·ΠΎΠ²Π°Π½ΠΈΠΉ ΠΏΡ€ΠΈ расчётС ΡΠΌΠ΅ΡˆΠ°Π½Π½Ρ‹Ρ… соСдинСний Ρ€Π°Π·Π»ΠΈΡ‡Π½Ρ‹Ρ… устройств позволяСт ΡƒΠΏΡ€ΠΎΡΡ‚ΠΈΡ‚ΡŒ ΠΈ ΡƒΡΠΊΠΎΡ€ΠΈΡ‚ΡŒ Π²Ρ‹ΠΏΠΎΠ»Π½Π΅Π½ΠΈΠ΅ вычислСний основных элСктричСских ΠΏΠ°Ρ€Π°ΠΌΠ΅Ρ‚Ρ€ΠΎΠ².

    Π’ΠΈΠ΄Π΅ΠΎ

    ΠžΡ†Π΅Π½ΠΈΡ‚Π΅ ΡΡ‚Π°Ρ‚ΡŒΡŽ:

    ΠžΠ±Ρ‰Π΅Π΅ сопротивлСниС элСктричСской Ρ†Π΅ΠΏΠΈ, Ρ‡Π΅ΠΌΡƒ ΠΎΠ½ΠΎ Ρ€Π°Π²Π½ΠΎ ΠΈ ΠΊΠ°ΠΊ Π½Π°ΠΉΡ‚ΠΈ ΠΏΠΎ Ρ„ΠΎΡ€ΠΌΡƒΠ»Π΅.

    Β 

    Β 

    Β 

    Π’Π΅ΠΌΠ°: ΠΊΠ°ΠΊ ΡƒΠ·Π½Π°Ρ‚ΡŒ ΠΊΠ°ΠΊΠΎΠ΅ сопротивлСниС Ρƒ элСктричСской схСмы, Ρ†Π΅ΠΏΠΈ ΠΏΠΎ Ρ„ΠΎΡ€ΠΌΡƒΠ»Π΅.

    Β 

    Как извСстно Π²ΠΎ всСм Π½ΡƒΠΆΠ½Π° своя ΠΌΠ΅Ρ€Π°, которая позволяСт Π΄Π΅Π»Π°Ρ‚ΡŒ Ρ‚ΠΎΡ‡Π½Ρ‹Π΅ систСмы, устройства, ΠΌΠ΅Ρ…Π°Π½ΠΈΠ·ΠΌΡ‹, схСмы. ΠœΠ΅Ρ€Π° мноТСствСнная, ΠΈΠΌΠ΅Π΅Ρ‚ свои ΠΊΠΎΠ½ΠΊΡ€Π΅Ρ‚Π½Ρ‹Π΅ Π²Π΅Π»ΠΈΡ‡ΠΈΠ½Ρ‹. Π’ сфСрС элСктротСхники основными Π²Π΅Π»ΠΈΡ‡ΠΈΠ½Π°ΠΌΠΈ ΡΠ²Π»ΡΡŽΡ‚ΡΡ напряТСниС, Ρ‚ΠΎΠΊ, сопротивлСниС, ΠΌΠΎΡ‰Π½ΠΎΡΡ‚ΡŒ, частота (для ΠΏΠ΅Ρ€Π΅ΠΌΠ΅Π½Π½ΠΎΠ³ΠΎ ΠΈ ΠΈΠΌΠΏΡƒΠ»ΡŒΡΠ½ΠΎΠ³ΠΎ Ρ‚ΠΎΠΊΠ°). Π’Π΅Π»ΠΈΡ‡ΠΈΠ½Ρ‹ ΠΌΠ΅ΠΆΠ΄Ρƒ собой связаны ΠΎΠΏΡ€Π΅Π΄Π΅Π»Π΅Π½Π½Ρ‹ΠΌΠΈ Ρ„ΠΎΡ€ΠΌΡƒΠ»Π°ΠΌΠΈ. Π‘Π°ΠΌΠΎΠΉ Π²Π°ΠΆΠ½ΠΎΠΉ Ρ„ΠΎΡ€ΠΌΡƒΠ»ΠΎΠΉ, Π½Π°ΠΈΠ±ΠΎΠ»Π΅Π΅ ΠΈΡΠΏΠΎΠ»ΡŒΠ·ΡƒΠ΅ΠΌΠΎΠΉ элСктриками, элСктронщиками являСтся Π·Π°ΠΊΠΎΠ½ Ома ( I = U/R, Ρ‚ΠΎ Π΅ΡΡ‚ΡŒ β€” сила Ρ‚ΠΎΠΊΠ° Ρ€Π°Π²Π½Π° Π½Π°ΠΏΡ€ΡΠΆΠ΅Π½ΠΈΡŽ Π΄Π΅Π»Π΅Π½Π½ΠΎΠΌΡƒ Π½Π° сопротивлСниС). Зная Π»ΡŽΠ±Ρ‹Π΅ Π΄Π²Π΅ Π²Π΅Π»ΠΈΡ‡ΠΈΠ½Ρ‹ ΠΈΠ· этой Ρ„ΠΎΡ€ΠΌΡƒΠ»Ρ‹ всСгда ΠΌΠΎΠΆΠ½ΠΎ Π½Π°ΠΉΡ‚ΠΈ Ρ‚Ρ€Π΅Ρ‚ΡŒΡŽ.

    Β 

    ΠžΡ‚ сопротивлСния элСктричСской Ρ†Π΅ΠΏΠΈ зависит силы Ρ‚ΠΎΠΊΠ° ΠΏΡ€ΠΈ Π½Π°Π»ΠΈΡ‡ΠΈΠΈ ΠΎΠΏΡ€Π΅Π΄Π΅Π»Π΅Π½Π½ΠΎΠ³ΠΎ напряТСния. Если мСняСтся сопротивлСниС Π² цСпях схСмы, Ρ‚ΠΎ ΠΈ ΠΌΠ΅Π½ΡΡ‚ΡŒΡΡ Ρ€Π΅ΠΆΠΈΠΌΡ‹ Π΅Π΅ Ρ€Π°Π±ΠΎΡ‚Ρ‹ Π² ΠΎΡ‚Π΄Π΅Π»ΡŒΠ½Ρ‹Ρ… Π΅Π΅ участках ΠΈΠ»ΠΈ Π²ΠΎ всСй Ρ†Π΅ΠΏΠΈ. Π—Π½Π°Π½ΠΈΠ΅ Π²Π΅Π»ΠΈΡ‡ΠΈΠ½Ρ‹ сопротивлСния ΠΌΠΎΠ³ΡƒΡ‚ ΠΏΠΎΠΌΠΎΡ‡ΡŒ Π²Ρ‹ΡΠ²ΠΈΡ‚ΡŒ Π½Π΅ΠΈΡΠΏΡ€Π°Π²Π½ΠΎΡΡ‚ΡŒ, ΡƒΠ·Π½Π°Ρ‚ΡŒ (Π²Ρ‹Ρ‡ΠΈΡΠ»ΠΈΡ‚ΡŒ ΠΈΠ· Ρ„ΠΎΡ€ΠΌΡƒΠ»Ρ‹) Π΄Ρ€ΡƒΠ³ΠΈΠ΅ элСктричСскиС Π²Π΅Π»ΠΈΡ‡ΠΈΠ½Ρ‹ Π² схСмС, зависящиС ΠΎΡ‚ этого сопротивлСния.

    Β 

    Π’Π΅ΠΏΠ΅Ρ€ΡŒ Π΄Π°Π²Π°ΠΉΡ‚Π΅ посмотрим ΠΎΡ‚ Ρ‡Π΅Π³ΠΎ зависит ΠΎΠ±Ρ‰Π΅Π΅ сопротивлСниС элСктричСской Ρ†Π΅ΠΏΠΈ. ΠžΠ±Ρ‰Π΅Π΅ β€” это сумма частных. Π›ΡŽΠ±Π°Ρ элСктричСская Ρ†Π΅ΠΏΡŒ ΠΈ схСма содСрТит Π² сСбС элСктричСскиС ΠΊΠΎΠΌΠΏΠΎΠ½Π΅Π½Ρ‚Ρ‹, ΠΊΠΎΡ‚ΠΎΡ€Ρ‹Π΅ ΠΎΠ±Π»Π°Π΄Π°ΡŽΡ‚ Π²Π½ΡƒΡ‚Ρ€Π΅Π½Π½ΠΈΠΌ сопротивлСниСм. Π”Π°ΠΆΠ΅ ΠΎΠ±Ρ‹Ρ‡Π½Ρ‹ΠΉ кондСнсатор (Π΄Π²Π΅ пластины ΠΏΡ€ΠΎΠ²ΠΎΠ΄Π½ΠΈΠΊΠ°, Ρ€Π°Π·Π΄Π΅Π»Π΅Π½Π½Ρ‹Π΅ диэлСктриком, Ρ‡Ρ‚ΠΎ позволяСт Π½Π°ΠΊΠ°ΠΏΠ»ΠΈΠ²Π°Ρ‚ΡŒ элСктричСский заряд ΠΌΠ΅ΠΆΠ΄Ρƒ этими пластинами, Π½Π΅ пропуская постоянный Ρ‚ΠΎΠΊ), ΠΊΠΎΡ‚ΠΎΡ€Ρ‹ΠΉ, казалось Π±Ρ‹, ΠΏΠΎ сути своСй Π΅Π³ΠΎ Π½Π΅ Π΄ΠΎΠ»ΠΆΠ΅Π½ ΠΈΠΌΠ΅Ρ‚ΡŒ (Ρ‚ΠΎΡ‡Π½Π΅Π΅ ΠΎΠ½ΠΎ бСсконСчно большоС) ΠΎΠ±Π»Π°Π΄Π°Π΅Ρ‚ Ρ€Π΅Π°ΠΊΡ‚ΠΈΠ²Π½Ρ‹ΠΌ сопротивлСниСм.

    Β 

    Бамая простая элСктричСская Ρ†Π΅ΠΏΡŒ состоит ΠΈΠ· источника питания ΠΈ Π½Π°Π³Ρ€ΡƒΠ·ΠΊΠΈ. К ΠΏΡ€ΠΈΠΌΠ΅Ρ€Ρƒ это Π±ΡƒΠ΄Π΅Ρ‚ обычная Π±Π°Ρ‚Π°Ρ€Π΅ΠΉΠΊΠ° ΠΈ малСнькая Π»Π°ΠΌΠΏΠΎΡ‡ΠΊΠ° накаливания. И Π±Π°Ρ‚Π°Ρ€Π΅ΠΉΠΊΠ° ΠΈ Π»Π°ΠΌΠΏΠΎΡ‡ΠΊΠ° ΠΈΠΌΠ΅ΡŽΡ‚ свои сопротивлСния, ΠΊΠΎΡ‚ΠΎΡ€Ρ‹Π΅ ΡΡƒΠΌΠΌΠΈΡ€ΡƒΡŽΡ‚ΡΡ, Ρ‡Ρ‚ΠΎ опрСдСляСт силу Ρ‚ΠΎΠΊΠ°, Ρ‚Π΅ΠΊΡƒΡ‰Π΅ΠΌΡƒ ΠΏΠΎ этой ΠΏΡ€ΠΎΡΡ‚Π΅ΠΉΡˆΠ΅ΠΉ Ρ†Π΅ΠΏΠΈ (ΠΏΡ€ΠΈ ΠΎΠΏΡ€Π΅Π΄Π΅Π»Π΅Π½Π½ΠΎΠΉ Π²Π΅Π»ΠΈΡ‡ΠΈΠ½Π΅ напряТСния). Допустим ΠΊ нашСй Ρ†Π΅ΠΏΠΈ ΠΌΡ‹ Π΄ΠΎΠ±Π°Π²ΠΈΠΌ Π΅Ρ‰Π΅ ΠΎΠ΄ΠΈΠ½ элСмСнт Π½Π°Π³Ρ€ΡƒΠ·ΠΊΠΈ (Π²Ρ‚ΠΎΡ€ΡƒΡŽ Ρ‚Π°ΠΊΡƒΡŽ ΠΆΠ΅ Π»Π°ΠΌΠΏΠΎΡ‡ΠΊΡƒ). Π•Π΅ ΠΌΠΎΠΆΠ½ΠΎ ΠΏΠΎΠ΄ΠΊΠ»ΡŽΡ‡ΠΈΡ‚ΡŒ ΠΊ этой ΠΏΡ€ΠΎΡΡ‚Π΅ΠΉΡˆΠ΅ΠΉ Ρ†Π΅ΠΏΠΈ двумя способами Π»ΠΈΠ±ΠΎ ΠΏΠ°Ρ€Π°Π»Π»Π΅Π»ΡŒΠ½ΠΎ ΠΏΠ΅Ρ€Π²ΠΎΠΉ Π»Π°ΠΌΠΏΠΎΡ‡ΠΊΠΈ, Π»ΠΈΠ±ΠΎ ΠΆΠ΅ ΠΏΠΎΡΠ»Π΅Π΄ΠΎΠ²Π°Ρ‚Π΅Π»ΡŒΠ½ΠΎ Π΅ΠΉ.

    Β 

    Β 

    Β 

    Β 

    ΠŸΡ€ΠΈ ΠΏΠΎΡΠ»Π΅Π΄ΠΎΠ²Π°Ρ‚Π΅Π»ΡŒΠ½ΠΎΠΌ ΠΏΠΎΠ΄ΠΊΠ»ΡŽΡ‡Π΅Π½ΠΈΠΈ сопротивлСниС Π±ΡƒΠ΄Π΅Ρ‚ ΡΡƒΠΌΠΌΠΈΡ€ΠΎΠ²Π°Ρ‚ΡŒΡΡ:

    Β 

    Β 

    Β 

    ΠŸΡ€ΠΈ ΠΏΠ°Ρ€Π°Π»Π»Π΅Π»ΡŒΠ½ΠΎΠΌ ΠΏΠΎΠ΄ΠΊΠ»ΡŽΡ‡Π΅Π½ΠΈΠΈ ΠΎΠ±Ρ‰Π΅Π΅ сопротивлСниС ΠΌΠΎΠΆΠ½ΠΎ Π½Π°ΠΉΡ‚ΠΈ ΠΏΠΎ Ρ‚Π°ΠΊΠΈΠΌ Ρ„ΠΎΡ€ΠΌΡƒΠ»Π°ΠΌ:

    Β 

    Β 

    Π’ΠΎ Π΅ΡΡ‚ΡŒ, Π±ΠΎΠ»ΡŒΡˆΠΈΠ½ΡΡ‚Π²ΠΎ схСм Π±ΡƒΠ΄ΡƒΡ‚ ΠΈΠΌΠ΅Ρ‚ΡŒ Π² сСбС Π»ΠΈΠ±ΠΎ ΠΏΠ°Ρ€Π°Π»Π»Π΅Π»ΡŒΠ½ΠΎΠ΅ ΠΏΠΎΠ΄ΠΊΠ»ΡŽΡ‡Π΅Π½ΠΈΠ΅ сопротивлСний, Π»ΠΈΠ±ΠΎ ΠΏΠΎΡΠ»Π΅Π΄ΠΎΠ²Π°Ρ‚Π΅Π»ΡŒΠ½ΠΎΠ΅ ΠΈΠ»ΠΈ ΠΆΠ΅ смСшанноС. Π’ случаС слоТной элСктричСской Ρ†Π΅ΠΏΠΈ ΠΎΠΏΡ€Π΅Π΄Π΅Π»Π΅Π½ΠΈΠ΅ ΠΎΠ±Ρ‰Π΅Π³ΠΎ элСктричСского сопротивлСния происходит ΠΏΠΎ частям (Π³Ρ€ΡƒΠΏΠΏΠ°ΠΌ), состоящим, ΠΎΠΏΡΡ‚ΡŒ ΠΆΠ΅, ΠΈΠ· ΠΏΠ°Ρ€Π°Π»Π»Π΅Π»ΡŒΠ½Ρ‹Ρ… ΠΈ ΠΏΠΎΡΠ»Π΅Π΄ΠΎΠ²Π°Ρ‚Π΅Π»ΡŒΠ½Ρ‹Ρ… ΠΏΠΎΠ΄ΠΊΠ»ΡŽΡ‡Π΅Π½ΠΈΠΉ элСмСнтов, ΠΎΠ±Π»Π°Π΄Π°ΡŽΡ‰ΠΈΠΌΠΈ сопротивлСниСм. ΠŸΡ€Π°Π²ΠΈΠ»ΡŒΠ½Π΅Π΅ Π½Π°Ρ‡ΠΈΠ½Π°Ρ‚ΡŒ с Ρ‚ΠΎΠΉ части Ρ†Π΅ΠΏΠΈ, схСмы, которая ΠΈΠΌΠ΅Π΅Ρ‚ Π½Π°ΠΈΠ±ΠΎΠ»ΡŒΡˆΡƒΡŽ ΡƒΠ΄Π°Π»Π΅Π½Π½ΠΎΡΡ‚ΡŒ ΠΎΡ‚ Π΄Π²ΡƒΡ… ΠΊΠΎΠ½Π΅Ρ‡Π½Ρ‹Ρ… Π²Ρ‹Π²ΠΎΠ΄ΠΎΠ², рассматриваСмых ΠΊΠ°ΠΊ ΠΊΠΎΠ½Ρ‚Π°ΠΊΡ‚Ρ‹ ΠΎΠ±Ρ‰Π΅Π³ΠΎ сопротивлСния. На рисункС Π½ΠΈΠΆΠ΅ ΠΏΡ€ΠΈΠ²Π΅Π΄Π΅Π½ ΠΏΡ€ΠΈΠΌΠ΅Ρ€ ΠΏΠΎΡΠ»Π΅Π΄ΠΎΠ²Π°Ρ‚Π΅Π»ΡŒΠ½ΠΎΡΡ‚ΠΈ вычислСния ΠΎΠ±Ρ‰Π΅Π³ΠΎ сопротивлСния слоТной Ρ†Π΅ΠΏΠΈ, схСмы.

    Β 

    Β 

    Но вСдь ΡΡƒΡ‰Π΅ΡΡ‚Π²ΡƒΡŽΡ‚ элСктричСскиС Ρ†Π΅ΠΏΠΈ, Π² ΠΊΠΎΡ‚ΠΎΡ€Ρ‹Ρ… ΠΎΠ±Ρ‰Π΅Π΅ сопротивлСниС ΠΌΠΎΠΆΠ΅Ρ‚ постоянно ΠΌΠ΅Π½ΡΡ‚ΡŒΡΡ, ΠΊ ΠΏΡ€ΠΈΠΌΠ΅Ρ€Ρƒ схСма стабилизированного рСгулятора частоты вращСния постоянного элСктродвигатСля, ΠΏΠΎΠ΄ΠΊΠ»ΡŽΡ‡Π΅Π½Π½Π°Ρ ΠΊ самому Π΄Π²ΠΈΠ³Π°Ρ‚Π΅Π»ΡŽ. ΠŸΡ€ΠΈ ΠΈΠ·ΠΌΠ΅Π½Π΅Π½ΠΈΠΈ Π½Π°Π³Ρ€ΡƒΠ·ΠΊΠΈ Π½Π° Π²Π°Π»Ρƒ двигатСля Π±ΡƒΠ΄Π΅Ρ‚ ΠΌΠ΅Π½ΡΡ‚ΡŒΡΡ Π΅Π³ΠΎ Π²Π½ΡƒΡ‚Ρ€Π΅Π½Π½Π΅Π΅ сопротивлСниС, ΡΠ»Π΅Π΄ΠΎΠ²Π°Ρ‚Π΅Π»ΡŒΠ½ΠΎ ΠΌΠ΅Π½ΡΡ‚ΡŒΡΡ Π±ΡƒΠ΄Π΅Ρ‚ ΠΈ Ρ€Π΅ΠΆΠΈΠΌΡ‹ Ρ€Π°Π±ΠΎΡ‚Ρ‹ схСмы (ΠΏΠΎΠ΄Π΄Π΅Ρ€ΠΆΠΈΠ²Π°ΡŽΡ‰Π°Ρ Π½ΡƒΠΆΠ½ΡƒΡŽ частоту вращСния Π²Π°Π»Π°). Π’ Ρ‚Π°ΠΊΠΈΡ… цСпях элСктричСскоС сопротивлСниС являСтся динамичСским, ΠΈΠ·ΠΌΠ΅Π½ΡΡŽΡ‰Π΅ΠΌΡΡ. МоТно лишь Ρ€Π°ΡΡΡ‡ΠΈΡ‚Π°Ρ‚ΡŒ усрСднСнноС сопротивлСниС, ΠΊΠΎΡ‚ΠΎΡ€ΠΎΠ΅ Π½Π΅ Π±ΡƒΠ΄Π΅Ρ‚ Π°Π±ΡΠΎΠ»ΡŽΡ‚Π½ΠΎ Ρ‚ΠΎΡ‡Π½Ρ‹ΠΌ.

    Β 

    Β 

    Помимо этого, ΠΊΠ°ΠΊ Π±Ρ‹Π»ΠΎ ΠΏΠΎΠ΄ΠΌΠ΅Ρ‡Π΅Π½ΠΎ Ρ€Π°Π½Π΅Π΅, сущСствуСт Π΅Ρ‰Π΅ Ρ€Π΅Π°ΠΊΡ‚ΠΈΠ²Π½ΠΎΠ΅ сопротивлСниС, ΠΊΠΎΡ‚ΠΎΡ€ΠΎΠ΅ Π±Ρ‹Π²Π°Π΅Ρ‚ Ρƒ ΠΈΠ½Π΄ΡƒΠΊΡ‚ΠΈΠ²Π½Ρ‹Ρ… ΠΈ Смкостных элСмСнтов Ρ†Π΅ΠΏΠΈ. Оно явно сСбя проявляСт Π² схСмах, Ρ‡Ρ‚ΠΎ Ρ€Π°Π±ΠΎΡ‚Π°ΡŽΡ‚ с ΠΏΠ΅Ρ€Π΅ΠΌΠ΅Π½Π½Ρ‹ΠΌ, ΠΈΠΌΠΏΡƒΠ»ΡŒΡΠ½Ρ‹ΠΌ Ρ‚ΠΎΠΊΠΎΠΌ. Если Π² цСпях постоянного Ρ‚ΠΎΠΊΠ° кондСнсатор (стоящий ΠΏΠΎΡΠ»Π΅Π΄ΠΎΠ²Π°Ρ‚Π΅Π»ΡŒΠ½ΠΎ) Π½Π΅ Π±ΡƒΠ΄Π΅Ρ‚ ΠΏΡ€ΠΎΠ²ΠΎΠ΄ΠΈΡ‚ΡŒ Ρ‡Π΅Ρ€Π΅Π· сСбя Ρ‚ΠΎΠΊ, Ρ‚ΠΎ Π² Ρ†Π΅ΠΏΠΈ ΠΏΠ΅Ρ€Π΅ΠΌΠ΅Π½Π½ΠΎΠ³ΠΎ Ρ‚ΠΎΠΊΠ° Π±ΡƒΠ΄Π΅Ρ‚ всС ΠΈΠ½Π°Ρ‡Π΅. ΠŸΡ€ΠΈΡ‡Π΅ΠΌ Π΅Π³ΠΎ Ρ€Π΅Π°ΠΊΡ‚ΠΈΠ²Π½ΠΎΠ΅ сопротивлСниС Π±ΡƒΠ΄Π΅Ρ‚ Π·Π°Π²ΠΈΡΠ΅Ρ‚ΡŒ ΠΎΡ‚ частоты (ΠΏΡ€ΠΈ ΠΎΠ΄Π½ΠΎΠΉ ΠΈ Ρ‚ΠΎΠΉ ΠΆΠ΅ Смкости). Π’ΠΎΡ‚ Ρ„ΠΎΡ€ΠΌΡƒΠ»Ρ‹ для нахоТдСния Ρ€Π΅Π°ΠΊΡ‚ΠΈΠ²Π½ΠΎΠ³ΠΎ Смкостного ΠΈ ΠΈΠ½Π΄ΡƒΠΊΡ‚ΠΈΠ²Π½ΠΎΠ³ΠΎ сопротивлСния:

    Β 

    Β 

    P.S. ΠΎΠ±Ρ‰Π΅Π΅ сопротивлСниС ΠΌΠΎΠΆΠ½ΠΎ Π½Π°Ρ…ΠΎΠ΄ΠΈΡ‚ΡŒ ΠΈ Ρ‡Π΅Ρ€Π΅Π· использованиС Π·Π°ΠΊΠΎΠ½Π° Ома, ΠΊΠΎΡ‚ΠΎΡ€Ρ‹ΠΉ гласит, Ρ‡Ρ‚ΠΎ сопротивлСниС Ρ€Π°Π²Π½ΠΎ напряТСниС Π΄Π΅Π»Π΅Π½Π½ΠΎΠ΅ Π½Π° силу Ρ‚ΠΎΠΊΠ°. Π‘Π»Π΅Π΄ΠΎΠ²Π°Ρ‚Π΅Π»ΡŒΠ½ΠΎ, Π±Π΅Ρ€Π΅ΠΌ ΠΌΡƒΠ»ΡŒΡ‚ΠΈΠΌΠ΅Ρ‚Ρ€, измСряСм Ρ‚ΠΎΠΊ ΠΈ напряТСниС Π² Ρ‚ΠΎΠΌ мСстС Ρ†Π΅ΠΏΠΈ, Π³Π΄Π΅ Ρ…ΠΎΡ‚ΠΈΠΌ ΡƒΠ·Π½Π°Ρ‚ΡŒ сопротивлСниС. Π’ΠΎΡΠΏΠΎΠ»ΡŒΠ·ΠΎΠ²Π°Π²ΡˆΠΈΡΡŒ Ρ„ΠΎΡ€ΠΌΡƒΠ»ΠΎΠΉ Ома Π½Π°Ρ…ΠΎΠ΄ΠΈΠΌ (опрСдСляСм) элСктричСскоС сопротивлСниС Π½ΡƒΠΆΠ½ΠΎΠ³ΠΎ участка Ρ†Π΅ΠΏΠΈ. Напомню, Ρ‡Ρ‚ΠΎ ΠΏΡ€ΠΈ использовании Π·Π°ΠΊΠΎΠ½Π° ΠΎΠΌΠ° Π½ΡƒΠΆΠ½ΠΎ ΠΏΡ€ΠΈΠΌΠ΅Π½ΡΡ‚ΡŒ основныС Π΅Π΄ΠΈΠ½ΠΈΡ†Ρ‹ измСрСния β€” Ρ‚ΠΎΠΊ Π² Π°ΠΌΠΏΠ΅Ρ€Π°Ρ…, напряТСниС Π² Π²ΠΎΠ»ΡŒΡ‚Π°Ρ…, Π° сопротивлСниС Π² ΠΎΠΌΠ°Ρ….

    Β 

    ΠΊΠ°ΠΊ ΠΏΠΎΡΡ‡ΠΈΡ‚Π°Ρ‚ΡŒ ΠΈ ΠΎΠΏΡ€Π΅Π΄Π΅Π»ΠΈΡ‚ΡŒ Ρ„ΠΎΡ€ΠΌΡƒΠ»ΠΎΠΉ

    Π‘ΠΎΠΏΡ€ΠΎΡ‚ΠΈΠ²Π»Π΅Π½ΠΈΠ΅ – это физичСская элСктротСхничСская Π²Π΅Π»ΠΈΡ‡ΠΈΠ½Π°, ΠΎΡ‚Ρ€Π°ΠΆΠ°ΡŽΡ‰Π°Ρ противодСйствиС двиТСнию элСктричСского Ρ‚ΠΎΠΊΠ° Π² ΠΏΡ€ΠΎΠ²ΠΎΠ΄Π½ΠΈΠΊΠ΅ ΠΈΠ»ΠΈ Π² Ρ†Π΅ΠΏΠΈ. Π’ΠΏΠ΅Ρ€Π²Ρ‹Π΅ ΠΎΠ½Π° Π±Ρ‹Π»Π° обоснована ΠΈ Π·Π°ΠΊΡ€Π΅ΠΏΠ»Π΅Π½Π° Π² Ρ„ΡƒΠ½Π΄Π°ΠΌΠ΅Π½Ρ‚Π°Π»ΡŒΠ½ΠΎΠΉ связи с напряТСниСм ΠΈ силой Ρ‚ΠΎΠΊΠ° Π² Π·Π°ΠΊΠΎΠ½Π΅ Ома – Π½Π΅ΠΌΠ΅Ρ†ΠΊΠΎΠ³ΠΎ Ρ„ΠΈΠ·ΠΈΠΊΠ°, ΠΊΠΎΡ‚ΠΎΡ€Ρ‹ΠΉ ΠΈΠ·ΡƒΡ‡Π°Π» эту взаимосвязь. Π’ Ρ‡Π΅ΡΡ‚ΡŒ Π½Π΅Π³ΠΎ ΠΈ Π½Π°Π·Π²Π°Π½Π° Π΅Π΄ΠΈΠ½ΠΈΡ†Π° измСрСния сопротивлСния – Ом. Часто ΠΏΡ€ΠΈ Π²Ρ‹ΠΏΠΎΠ»Π½Π΅Π½ΠΈΠΈ ΠΌΠΎΠ½Ρ‚Π°ΠΆΠ° ΠΊΠ°ΠΊΠΎΠΉ-Π»ΠΈΠ±ΠΎ элСктросСти Π½Π΅ΠΎΠ±Ρ…ΠΎΠ΄ΠΈΠΌΠΎ Π½Π°ΠΉΡ‚ΠΈ ΠΎΠ±Ρ‰Π΅Π΅ сопротивлСниС Ρ†Π΅ΠΏΠΈ ΠΏΡ€ΠΈ Ρ€Π°Π·Π»ΠΈΡ‡Π½Ρ‹Ρ… способах ΠΏΠΎΠ΄ΠΊΠ»ΡŽΡ‡Π΅Π½ΠΈΡ. О Ρ‚ΠΎΠΌ, ΠΊΠ°ΠΊ это ΠΏΡ€Π°Π²ΠΈΠ»ΡŒΠ½ΠΎ ΡΠ΄Π΅Π»Π°Ρ‚ΡŒ ΠΈ расскаТСт этот ΠΌΠ°Ρ‚Π΅Ρ€ΠΈΠ°Π».

    Π§Ρ‚ΠΎ Ρ‚Π°ΠΊΠΎΠ΅ ΠΎΠ±Ρ‰Π΅Π΅ сопротивлСниС Ρ†Π΅ΠΏΠΈ

    Если Π³ΠΎΠ²ΠΎΡ€ΠΈΡ‚ΡŒ простыми словами, ΠΎΠ±Ρ‰Π΅Π΅ сопротивлСниС элСктричСской Ρ†Π΅ΠΏΠΈ – это Ρ‚Π°ΠΊΠΎΠ΅ R, ΠΊΠΎΡ‚ΠΎΡ€ΠΎΠ΅ ΠΎΠ½Π° ΠΎΠΊΠ°Π·Ρ‹Π²Π°Π΅Ρ‚ Π½Π° напряТСниС Π² Π΅Π΅ ΠΏΡ€ΠΎΠ²ΠΎΠ΄Π½ΠΈΠΊΠ°Ρ… ΠΈ ΠΏΡ€ΠΈΠ±ΠΎΡ€Π°Ρ…. БущСствуСт Π΄Π²Π° Ρ‚ΠΈΠΏΠ° напряТСния (исходя ΠΈΠ· силы Ρ‚ΠΎΠΊΠ°) – постоянноС ΠΈ ΠΏΠ΅Ρ€Π΅ΠΌΠ΅Π½Π½ΠΎΠ΅. Π’Π°ΠΊ ΠΆΠ΅ ΠΈ сопротивлСниС дСлится Π½Π° Π°ΠΊΡ‚ΠΈΠ²Π½ΠΎΠ΅ ΠΈ Ρ€Π΅Π°ΠΊΡ‚ΠΈΠ²Π½ΠΎΠ΅, ΠΊΠΎΡ‚ΠΎΡ€ΠΎΠ΅, Π² свою ΠΎΡ‡Π΅Ρ€Π΅Π΄ΡŒ, подраздСляСтся Π½Π° ΠΈΠ½Π΄ΡƒΠΊΡ‚ΠΈΠ²Π½ΠΎΠ΅ ΠΈ СмкостноС. Активный Ρ‚ΠΈΠΏ Π½Π΅ зависит ΠΎΡ‚ частот сСти. Π’Π°ΠΊΠΆΠ΅ для Π½Π΅Π³ΠΎ Π°Π±ΡΠΎΠ»ΡŽΡ‚Π½ΠΎ Π½Π΅ Π²Π°ΠΆΠ½ΠΎ, ΠΊΠ°ΠΊΠΎΠΉ Ρ‚ΠΎΠΊ ΠΏΡ€ΠΎΡ‚Π΅ΠΊΠ°Π΅Ρ‚ ΠΏΠΎ ΠΏΡ€ΠΎΠ²ΠΎΠ΄Π½ΠΈΠΊΠ°ΠΌ. Π Π΅Π°ΠΊΡ‚ΠΈΠ²Π½Ρ‹ΠΉ ΠΆΠ΅, Π½Π°ΠΎΠ±ΠΎΡ€ΠΎΡ‚, зависит ΠΎΡ‚ частоты, ΠΏΡ€ΠΈΡ‡Π΅ΠΌ Смкостная характСристика Π² кондСнсаторах ΠΈ индуктивная Π² трансформаторах Π²Π΅Π΄ΡƒΡ‚ сСбя ΠΏΠΎ-Ρ€Π°Π·Π½ΠΎΠΌΡƒ.

    Π—Π°ΠΊΠΎΠ½ Ома

    Помимо сопротивлСния ΠΏΠΎΠ΄ΠΊΠ»ΡŽΡ‡Π΅Π½Π½Ρ‹Ρ… Π² ΡΠ΅Ρ‚ΡŒ элСктроприборов, Π½Π° ΠΎΠ±Ρ‰Π΅Π΅ состояниС ΠΎΠΊΠ°Π·Ρ‹Π²Π°ΡŽΡ‚ влияниС Π΄Π°ΠΆΠ΅ ΠΏΡ€ΠΎΠΌΠ΅ΠΆΡƒΡ‚ΠΎΡ‡Π½Ρ‹Π΅ ΠΏΡ€ΠΎΠ²ΠΎΠ΄Π°, Ρ‚Π°ΠΊΠΆΠ΅ ΠΈΠΌΠ΅ΡŽΡ‰ΠΈΠ΅ ΡΠΎΠΏΡ€ΠΎΡ‚ΠΈΠ²Π»ΡΠ΅ΠΌΠΎΡΡ‚ΡŒ Π½Π°ΠΏΡ€ΡΠΆΠ΅Π½ΠΈΡŽ.

    РСзистор – основной элСмСнт сопротивляСмости Ρ†Π΅ΠΏΠΈ

    Как ΠΏΡ€Π°Π²ΠΈΠ»ΡŒΠ½ΠΎ Π½Π°ΠΉΡ‚ΠΈ ΠΈ ΠΏΠΎΡΡ‡ΠΈΡ‚Π°Ρ‚ΡŒ Ρ„ΠΎΡ€ΠΌΡƒΠ»ΠΎΠΉ сопротивлСниС Ρ†Π΅ΠΏΠΈ

    Π‘ΠΏΠ΅Ρ€Π²Π° слСдуСт Ρ€Π°Π·ΠΎΠ±Ρ€Π°Ρ‚ΡŒ понятия ΠΈ Ρ„ΠΎΡ€ΠΌΡƒΠ»Ρ‹. Π˜Π½Π΄ΡƒΠΊΡ‚ΠΈΠ²Π½Ρ‹ΠΉ Ρ‚ΠΈΠΏ считаСтся Ρ‚Π°ΠΊ: XL= Ο‰L, Π³Π΄Π΅ L – ΠΈΠ½Π΄ΡƒΠΊΡ‚ΠΈΠ²Π½ΠΎΡΡ‚ΡŒ Ρ†Π΅ΠΏΠΈ, Π° Ο‰ – круговая частота ΠΏΠ΅Ρ€Π΅ΠΌΠ΅Π½Π½ΠΎΠ³ΠΎ Ρ‚ΠΎΠΊΠ°, равная 2Ο€f (f – частота ΠΏΠ΅Ρ€Π΅ΠΌΠ΅Π½Π½ΠΎΠ³ΠΎ Ρ‚ΠΎΠΊΠ°). Π§Π΅ΠΌ большС частота сСти, Ρ‚Π΅ΠΌ большим R для Π½Π΅Π΅ становится какая-Π»ΠΈΠ±ΠΎ ΠΊΠ°Ρ‚ΡƒΡˆΠΊΠ° индуктивности.

    Емкостный Ρ‚ΠΈΠΏ ΠΌΠΎΠΆΠ½ΠΎ Ρ€Π°ΡΡΡ‡ΠΈΡ‚Π°Ρ‚ΡŒ ΠΏΠΎ Ρ„ΠΎΡ€ΠΌΡƒΠ»Π΅: Xc = 1/ Ο‰C, Π³Π΄Π΅ Π‘ – Π΅ΠΌΠΊΠΎΡΡ‚ΡŒ радиоэлСмСнта. Π—Π΄Π΅ΡΡŒ всС Π½Π°ΠΎΠ±ΠΎΡ€ΠΎΡ‚. Если происходит ΡƒΠ²Π΅Π»ΠΈΡ‡Π΅Π½ΠΈΠ΅ частоты, Ρ‚ΠΎ ΡΠΎΠΏΡ€ΠΎΡ‚ΠΈΠ²Π»ΡΠ΅ΠΌΠΎΡΡ‚ΡŒ кондСнсатора Π½Π°ΠΏΡ€ΡΠΆΠ΅Π½ΠΈΡŽ ΡƒΠΌΠ΅Π½ΡŒΡˆΠ°Π΅Ρ‚ΡΡ. Из этого исходит Ρ‚ΠΎ, Ρ‡Ρ‚ΠΎ для сСти постоянного Ρ‚ΠΎΠΊΠ° кондСнсатор – бСсконСчно большоС R.

    Π’Ρ‹ΡΡ‡ΠΈΡ‚Π°Ρ‚ΡŒ характСристику ΠΌΠΎΠΆΠ½ΠΎ ΠΈ с ΠΏΠΎΠΌΠΎΡ‰ΠΈ Π΄Ρ€ΡƒΠ³ΠΈΡ… Π²Π΅Π»ΠΈΡ‡ΠΈΠ½

    Но Π½Π΅ Ρ‚ΠΎΠ»ΡŒΠΊΠΎ Π²ΠΈΠ΄ сопротивлСния ΠΈ радиоэлСмСнты, ΠΎΠ±Π΅ΡΠΏΠ΅Ρ‡ΠΈΠ²Π°ΡŽΡ‰ΠΈΠ΅ Π΅Π³ΠΎ, Π²Π»ΠΈΡΡŽΡ‚ Π½Π° ΠΎΠ±Ρ‰Π΅Π΅ Π·Π½Π°Ρ‡Π΅Π½ΠΈΠ΅ Ρ†Π΅ΠΏΠΈ. ΠžΡΠΎΠ±ΡƒΡŽ Ρ€ΠΎΠ»ΡŒ ΠΈΠ³Ρ€Π°Π΅Ρ‚ Ρ‚Π°ΠΊΠΆΠ΅ ΠΈ способ соСдинСния элСмСнтов Π² ΡΠ»Π΅ΠΊΡ‚Ρ€ΠΎΡ†Π΅ΠΏΡŒ. БущСствуСт Π΄Π²Π° Π²Π°Ρ€ΠΈΠ°Π½Ρ‚Π°:

    • ΠŸΠΎΡΠ»Π΅Π΄ΠΎΠ²Π°Ρ‚Π΅Π»ΡŒΠ½Ρ‹ΠΉ;
    • ΠŸΠ°Ρ€Π°Π»Π»Π΅Π»ΡŒΠ½Ρ‹ΠΉ.

    Π’ ΠΏΠΎΡΠ»Π΅Π΄ΠΎΠ²Π°Ρ‚Π΅Π»ΡŒΠ½ΠΎΠΌ ΠΏΠΎΠ΄ΠΊΠ»ΡŽΡ‡Π΅Π½ΠΈΠΈ

    Π­Ρ‚ΠΎ самый простой Ρ‚ΠΈΠΏ для практичСского ΠΈ тСорСтичСского рассмотрСния. Π’ Π½Π΅ΠΌ элСмСнты рСзисторного Ρ‚ΠΈΠΏΠ° ΡΠΎΠ΅Π΄ΠΈΠ½ΡΡŽΡ‚ΡΡ, ΠΎΡ‡Π΅Π²ΠΈΠ΄Π½ΠΎ, ΠΏΠΎΡΠ»Π΅Π΄ΠΎΠ²Π°Ρ‚Π΅Π»ΡŒΠ½ΠΎ, образуя ΠΏΠΎΠ΄ΠΎΠ±ΠΈΠ΅ Β«Π·ΠΌΠ΅ΠΉΠΊΠΈΒ» послС Ρ‡Π΅Π³ΠΎ элСктричСская Ρ†Π΅ΠΏΡŒ замыкаСтся. ΠŸΠΎΡΡ‡ΠΈΡ‚Π°Ρ‚ΡŒ ΠΎΠ±Ρ‰Π΅Π΅ Π·Π½Π°Ρ‡Π΅Π½ΠΈΠ΅ Π² Ρ‚Π°ΠΊΠΎΠΌ случаС довольно просто: трСбуСтся ΠΏΠΎΡΠ»Π΅Π΄ΠΎΠ²Π°Ρ‚Π΅Π»ΡŒΠ½ΠΎ ΡΠ»ΠΎΠΆΠΈΡ‚ΡŒ всС значСния, Π²Ρ‹Π΄Π°Π²Π°Π΅ΠΌΡ‹Π΅ ΠΊΠ°ΠΆΠ΄Ρ‹ΠΌ ΠΈΠ· рСзисторов. НапримСр, Ссли ΠΏΠΎΠ΄ΠΊΠ»ΡŽΡ‡Π΅Π½ΠΎ 5 рСзисторов ΠΏΠΎ 5 Ом ΠΊΠ°ΠΆΠ΄Ρ‹ΠΉ, Ρ‚ΠΎ ΠΎΠ±Ρ‰ΠΈΠΉ ΠΏΠ°Ρ€Π°ΠΌΠ΅Ρ‚Ρ€ Π±ΡƒΠ΄Π΅Ρ‚ Ρ€Π°Π²Π΅Π½ 5 Π½Π° 5 – 25 Ом.

    Π€ΠΎΡ€ΠΌΡƒΠ»Π° ΠΏΠΎΡΠ»Π΅Π΄ΠΎΠ²Π°Ρ‚Π΅Π»ΡŒΠ½ΠΎΠΉ сСти

    Π’ ΠΏΠ°Ρ€Π°Π»Π»Π΅Π»ΡŒΠ½ΠΎΠΌ ΠΏΠΎΠ΄ΠΊΠ»ΡŽΡ‡Π΅Π½ΠΈΠΈ

    НСмного слоТнСС всС устроСно Π² ΠΏΠ°Ρ€Π°Π»Π»Π΅Π»ΡŒΠ½Ρ‹Ρ… сСтях. Если ΠΏΡ€ΠΈ ΠΏΠΎΡΠ»Π΅Π΄ΠΎΠ²Π°Ρ‚Π΅Π»ΡŒΠ½ΠΎΠΌ способС Ρ‚ΠΎΠΊΡƒ Π½ΡƒΠΆΠ½ΠΎ ΠΏΡ€ΠΎΠΉΡ‚ΠΈ всС рСзисторы, Ρ‚ΠΎ Ρ‚ΡƒΡ‚ ΠΎΠ½ Π²ΠΏΡ€Π°Π²Π΅ Π²Ρ‹Π±Ρ€Π°Ρ‚ΡŒ любой. На самом Π΄Π΅Π»Π΅ ΠΎΠ½ просто Π±ΡƒΠ΄Π΅Ρ‚ Ρ€Π°Π·Π΄Π΅Π»Π΅Π½ ΠΌΠ΅ΠΆΠ΄Ρƒ Π½ΠΈΠΌΠΈ. Π‘ΡƒΡ‚ΡŒ Π² Ρ‚ΠΎΠΌ, Ρ‡Ρ‚ΠΎ Π΅ΡΡ‚ΡŒ характСристика, схоТая для всСх радиоэлСмСнтов, Π½Π°ΠΏΡ€ΠΈΠΌΠ΅Ρ€, Π²Π΅Π»ΠΈΡ‡ΠΈΠ½Π° Π² 5 Ом ΠΎΠ·Π½Π°Ρ‡Π°Π΅Ρ‚, Ρ‡Ρ‚ΠΎ для нахоТдСния ΠΎΠ±Ρ‰Π΅Π³ΠΎ R Π½Π΅ΠΎΠ±Ρ…ΠΎΠ΄ΠΈΠΌΠΎ Ρ€Π°Π·Π΄Π΅Π»ΠΈΡ‚ΡŒ Π΅Π³ΠΎ Π½Π° количСство всСх ΠΏΠΎΠ΄ΠΊΠ»ΡŽΡ‡Π΅Π½Π½Ρ‹Ρ… рСзисторов: 5/5 = 1 Ом.

    Π’Π°ΠΆΠ½ΠΎ! Из-Π·Π° Ρ‚ΠΎΠ³ΠΎ, Ρ‡Ρ‚ΠΎ напряТСниС Π½Π° ΠΏΠ°Ρ€Π°Π»Π»Π΅Π»ΡŒΠ½Ρ‹Ρ… участках ΠΎΠ΄ΠΈΠ½Π°ΠΊΠΎΠ²ΠΎ, Π° Ρ‚ΠΎΠΊΠΈ ΡΠΊΠ»Π°Π΄Ρ‹Π²Π°ΡŽΡ‚ΡΡ, Ρ‚ΠΎ Π΅ΡΡ‚ΡŒ сумма Ρ‚ΠΎΠΊΠΎΠ² Π² участках Ρ€Π°Π²Π½Π° Π½Π΅Ρ€Π°Π·Π²Π΅Ρ‚Π²Π»Π΅Π½Π½ΠΎΠΌΡƒ Ρ‚ΠΎΠΊΡƒ, Ρ‚ΠΎ RΠΎΠ±Ρ‰ Π±ΡƒΠ΄Π΅Ρ‚ Π²Ρ‹ΡΡ‡ΠΈΡ‚Ρ‹Π²Π°Ρ‚ΡŒΡΡ Ρ„ΠΎΡ€ΠΌΡƒΠ»Π΅: 1/R = 1/R1 + 1/R2 + … + 1/Rn.

    Π€ΠΎΡ€ΠΌΡƒΠ»Π° ΠΏΠ°Ρ€Π°Π»Π»Π΅Π»ΡŒΠ½ΠΎΠΉ сСти

    Как ΠΎΠΏΡ€Π΅Π΄Π΅Π»ΠΈΡ‚ΡŒ Ρ„ΠΎΡ€ΠΌΡƒΠ»ΠΎΠΉ ΠΎΠ±Ρ‰Π΅Π΅ сопротивлСниС Ρ†Π΅ΠΏΠΈ

    Из Π·Π°ΠΊΠΎΠ½Π° Ома исходит Ρ‚ΠΎ, Ρ‡Ρ‚ΠΎ ΠΎΠ±Ρ‰Π΅Π΅ сопротивлСниС Ρ€Π°Π²Π½ΠΎ ΠΎΠ±Ρ‰Π΅ΠΌΡƒ Π½Π°ΠΏΡ€ΡΠΆΠ΅Π½ΠΈΡŽ, Π΄Π΅Π»Π΅Π½Π½ΠΎΠΌΡƒ Π½Π° ΠΎΠ±Ρ‰ΡƒΡŽ силу Ρ‚ΠΎΠΊΠ° Π² Ρ†Π΅ΠΏΠΈ. ΠŸΡ€ΠΈ ΠΏΠ°Ρ€Π°Π»Π»Π΅Π»ΡŒΠ½ΠΎΠΌ ΠΏΠΎΠ΄ΠΊΠ»ΡŽΡ‡Π΅Π½ΠΈΠΈ напряТСниС, ΠΊΠ°ΠΊ ΡƒΠΆΠ΅ Π±Ρ‹Π»ΠΎ сказано, Ρ€Π°Π²Π½ΠΎ Π²Π΅Π·Π΄Π΅, поэтому Π½Π΅ΠΎΠ±Ρ…ΠΎΠ΄ΠΈΠΌΠΎ ΡƒΠ·Π½Π°Ρ‚ΡŒ Π΅Π³ΠΎ Π·Π½Π°Ρ‡Π΅Π½ΠΈΠ΅ Π½Π° любом участкС Ρ†Π΅ΠΏΠΈ. Π‘ Ρ‚ΠΎΠΊΠΎΠΌ всС слоТнСС, Ρ‚Π°ΠΊ ΠΊΠ°ΠΊ Π½Π° ΠΊΠ°ΠΆΠ΄ΠΎΠΉ Π²Π΅Ρ‚ΠΊΠ΅ Π΅Π³ΠΎ Π·Π½Π°Ρ‡Π΅Π½ΠΈΠ΅ своС ΠΈ зависит ΠΎΡ‚ ΠΊΠΎΠ½ΠΊΡ€Π΅Ρ‚Π½ΠΎΠ³ΠΎ R.

    Π’Π°ΠΊΠΆΠ΅ Π½Π΅ΠΎΠ±Ρ…ΠΎΠ΄ΠΈΠΌΠΎ ΠΏΠΎΠΌΠ½ΠΈΡ‚ΡŒ, Ρ‡Ρ‚ΠΎ ΠΌΠΎΠ³ΡƒΡ‚ Π±Ρ‹Ρ‚ΡŒ ΠΏΠ°Ρ€Π°Π»Π»Π΅Π»ΡŒΠ½Ρ‹Π΅ ΠΏΠΎΠ΄ΠΊΠ»ΡŽΡ‡Π΅Π½ΠΈΡ с Π½ΡƒΠ»Π΅Π²Ρ‹ΠΌ Π·Π½Π°Ρ‡Π΅Π½ΠΈΠ΅ΠΌ R. Если Π² ΠΊΠ°ΠΊΠΎΠΉ-Π»ΠΈΠ±ΠΎ Π²Π΅Ρ‚ΠΊΠ΅ Π½Π΅Ρ‚ рСзистора ΠΈΠ»ΠΈ Π΄Ρ€ΡƒΠ³ΠΎΠ³ΠΎ ΠΏΠΎΠ΄ΠΎΠ±Π½ΠΎΠ³ΠΎ элСмСнта, Π½ΠΎ вСсь Ρ‚ΠΎΠΊ Π±ΡƒΠ΄Π΅Ρ‚ Ρ‚Π΅Ρ‡ΡŒ Ρ‡Π΅Ρ€Π΅Π· Π½Π΅Π΅ ΠΈ всС ΠΎΠ±Ρ‰Π΅Π΅ Π·Π½Π°Ρ‡Π΅Π½ΠΈΠ΅ для Ρ†Π΅ΠΏΠΈ станСт Π½ΡƒΠ»Π΅Π²Ρ‹ΠΌ. На ΠΏΡ€Π°ΠΊΡ‚ΠΈΠΊΠ΅ это случаСтся ΠΏΡ€ΠΈ Π²Ρ‹Ρ…ΠΎΠ΄Π΅ рСзистора ΠΈΠ· строя ΠΈΠ»ΠΈ ΠΏΡ€ΠΈ Π·Π°ΠΌΡ‹ΠΊΠ°Π½ΠΈΠΈ. Вакая ситуация ΠΌΠΎΠΆΠ΅Ρ‚ Π½Π°Π²Ρ€Π΅Π΄ΠΈΡ‚ΡŒ Π΄Ρ€ΡƒΠ³ΠΈΠΌ элСмСнтам ΠΈΠ·-Π·Π° большой силы Ρ‚ΠΎΠΊΠ°.

    Π’Π°Π±Π»ΠΈΡ†Π° ΡƒΠ΄Π΅Π»ΡŒΠ½ΠΎΠΉ Π²Π΅Π»ΠΈΡ‡ΠΈΠ½Ρ‹ для Ρ€Π°Π·Π»ΠΈΡ‡Π½Ρ‹Ρ… ΠΏΡ€ΠΎΠ²ΠΎΠ΄Π½ΠΈΠΊΠΎΠ²

    Онлайн-ΠΊΠ°Π»ΡŒΠΊΡƒΠ»ΡΡ‚ΠΎΡ€ расчСта сопротивлСниС Ρ†Π΅ΠΏΠΈ

    Для Ρ‚ΠΎΠ³ΠΎ Ρ‡Ρ‚ΠΎΠ±Ρ‹ ΡΡΠΊΠΎΠ½ΠΎΠΌΠΈΡ‚ΡŒ своС врСмя ΠΈ Π½Π΅ Π·Π°Π½ΠΈΠΌΠ°Ρ‚ΡŒΡΡ скучными пСрСсчСтами, рСкомСндуСтся ΠΏΠΎΠ»ΡŒΠ·ΠΎΠ²Π°Ρ‚ΡŒΡΡ ΠΊΠ°Π»ΡŒΠΊΡƒΠ»ΡΡ‚ΠΎΡ€Π°ΠΌΠΈ ΠΏΠΎ расчСту сопротивлСния ΠΈ ΠΌΠ½ΠΎΠ³ΠΈΡ… Π΄Ρ€ΡƒΠ³ΠΈΡ… Π²Π΅Π»ΠΈΡ‡ΠΈΠ½ Π² Ρ€Π΅ΠΆΠΈΠΌΠ΅ ΠΎΠ½Π»Π°ΠΉΠ½. Π‘ΠΎΠ»ΡŒΡˆΠΈΠ½ΡΡ‚Π²ΠΎ ΠΈΠ· Π½ΠΈΡ… бСсплатныС:

    • Π‘alc.ru (https://www.calc.ru/raschet-elektricheskikh-tsepey.html). Π’ΠΎΠ·ΠΌΠΎΠΆΠ΅Π½ расчСт Π·Π°ΠΊΠΎΠ½Π° Ома для участка Ρ†Π΅ΠΏΠΈ, Ρ€Π΅Π°ΠΊΡ‚ΠΈΠ²Π½ΠΎΠ³ΠΎ ΠΈ Π°ΠΊΡ‚ΠΈΠ²Π½ΠΎΠ³ΠΎ сопротивлСния ΠΏΡ€ΠΈ ΠΏΠΎΡΠ»Π΅Π΄ΠΎΠ²Π°Ρ‚Π΅Π»ΡŒΠ½ΠΎΠΌ ΠΈ ΠΏΠ°Ρ€Π°Π»Π»Π΅Π»ΡŒΠ½ΠΎΠΌ соСдинСнии рСзисторов;
    • Asutpp.ru (https://www.asutpp.ru/kalkulyator-rascheta-parallelnogo-soedineniya-rezistorov.html). ΠšΠ°Π»ΡŒΠΊΡƒΠ»ΡΡ‚ΠΎΡ€ для ΠΏΠ°Ρ€Π°Π»Π»Π΅Π»ΡŒΠ½ΠΎΠ³ΠΎ соСдинСния. Достаточно ΡƒΠΊΠ°Π·Π°Ρ‚ΡŒ количСство элСмСнтов ΠΈ Ом-характСристику ΠΊΠ°ΠΆΠ΄ΠΎΠ³ΠΎ ΠΈΠ· Π½ΠΈΡ…;
    • Cxem.net (https://cxem.net/calc/calc.php). ΠžΠ±Π»Π°Π΄Π°Π΅Ρ‚ Ρ‚Π°ΠΊΠΈΠΌ ΠΆΠ΅ количСством ΠΊΠ°Π»ΡŒΠΊΡƒΠ»ΡΡ‚ΠΎΡ€ΠΎΠ², ΠΊΠ°ΠΊ ΠΈ ΠΏΠ΅Ρ€Π²Ρ‹ΠΉ Π²Π°Ρ€ΠΈΠ°Π½Ρ‚, Ρ‡Ρ‚ΠΎ позволяСт Ρ€Π°Π΄ΠΈΠΎΠ»ΡŽΠ±ΠΈΡ‚Π΅Π»ΡŽ Π²Ρ‹ΠΏΠΎΠ»Π½ΠΈΡ‚ΡŒ вычислСниС Π»ΡŽΠ±Ρ‹Ρ… ΠΈΠ½Ρ‚Π΅Ρ€Π΅ΡΡƒΡŽΡ‰ΠΈΡ… ΠΏΠ°Ρ€Π°ΠΌΠ΅Ρ‚Ρ€ΠΎΠ² сСти.
    Π˜Π½Ρ‚Π΅Ρ€Ρ„Π΅ΠΉΡ ΠΎΠ΄Π½ΠΎΠ³ΠΎ ΠΈΠ· ΠΊΠ°Π»ΡŒΠΊΡƒΠ»ΡΡ‚ΠΎΡ€ΠΎΠ²

    Π’ ΡΡ‚Π°Ρ‚ΡŒΠ΅ ΠΏΠΎΠ΄Ρ€ΠΎΠ±Π½ΠΎ рассказано, ΠΊΠ°ΠΊ Π²Ρ‹Ρ‡ΠΈΡΠ»ΠΈΡ‚ΡŒ ΠΎΠ±Ρ‰Π΅Π΅ сопротивлСниС Ρ†Π΅ΠΏΠΈ. ΠŸΡ€ΠΈ Ρ€Π°Π·Π½Ρ‹Ρ… Ρ‚ΠΈΠΏΠ°Ρ… ΠΏΠΎΠ΄ΠΊΠ»ΡŽΡ‡Π΅Π½ΠΈΡ элСмСнтов ΠΎΠ½Π° считаСтся ΠΏΠΎ-Ρ€Π°Π·Π½ΠΎΠΌΡƒ, Π½ΠΎ благодаря Π΄Π°Π²Π½ΠΎ Π²Ρ‹Π²Π΅Π΄Π΅Π½Π½Ρ‹ΠΌ Ρ„ΠΎΡ€ΠΌΡƒΠ»Π°ΠΌ Π² любом случаС Π½Π΅Ρ‚ Π½ΠΈΡ‡Π΅Π³ΠΎ слоТного.

    Как Π½Π°ΠΉΡ‚ΠΈ rΠΎΠ±Ρ‰ Π² Ρ†Π΅ΠΏΠΈ

    ΠŸΠΎΡΠ»Π΅Π΄ΠΎΠ²Π°Ρ‚Π΅Π»ΡŒΠ½ΠΎΠ΅ соСдинСниС рСзисторов

    ΠŸΠΎΡΠ»Π΅Π΄ΠΎΠ²Π°Ρ‚Π΅Π»ΡŒΠ½ΠΎΠ΅ соСдинСниС – это соСдинСниС Π΄Π²ΡƒΡ… ΠΈΠ»ΠΈ Π±ΠΎΠ»Π΅Π΅ рСзисторов Π² Ρ„ΠΎΡ€ΠΌΠ΅ Ρ†Π΅ΠΏΠΈ, Π² ΠΊΠΎΡ‚ΠΎΡ€ΠΎΠΉ ΠΊΠ°ΠΆΠ΄Ρ‹ΠΉ ΠΎΡ‚Π΄Π΅Π»ΡŒΠ½Ρ‹ΠΉ рСзистор соСдиняСтся с Π΄Ρ€ΡƒΠ³ΠΈΠΌ ΠΎΡ‚Π΄Π΅Π»ΡŒΠ½Ρ‹ΠΌ рСзистором Ρ‚ΠΎΠ»ΡŒΠΊΠΎ Π² ΠΎΠ΄Π½ΠΎΠΉ Ρ‚ΠΎΡ‡ΠΊΠ΅.

    ΠžΠ±Ρ‰Π΅Π΅ сопротивлСниС R

    ΠΎΠ±Ρ‰

    ΠŸΡ€ΠΈ Ρ‚Π°ΠΊΠΎΠΌ соСдинСнии, Ρ‡Π΅Ρ€Π΅Π· всС рСзисторы ΠΏΡ€ΠΎΡ…ΠΎΠ΄ΠΈΡ‚ ΠΎΠ΄ΠΈΠ½ ΠΈ Ρ‚ΠΎΡ‚ ΠΆΠ΅ элСктричСский Ρ‚ΠΎΠΊ. Π§Π΅ΠΌ большС элСмСнтов Π½Π° Π΄Π°Π½Π½ΠΎΠΌ участкС элСктричСской Ρ†Π΅ΠΏΠΈ, Ρ‚Π΅ΠΌ Β«Ρ‚Ρ€ΡƒΠ΄Π½Π΅Π΅Β» Ρ‚ΠΎΠΊΡƒ ΠΏΡ€ΠΎΡ‚Π΅ΠΊΠ°Ρ‚ΡŒ Ρ‡Π΅Ρ€Π΅Π· Π½Π΅Π³ΠΎ. Π‘Π»Π΅Π΄ΠΎΠ²Π°Ρ‚Π΅Π»ΡŒΠ½ΠΎ, ΠΏΡ€ΠΈ ΠΏΠΎΡΠ»Π΅Π΄ΠΎΠ²Π°Ρ‚Π΅Π»ΡŒΠ½ΠΎΠΌ соСдинСнии рСзисторов ΠΈΡ… ΠΎΠ±Ρ‰Π΅Π΅ сопротивлСниС увСличиваСтся, ΠΈ ΠΎΠ½ΠΎ Ρ€Π°Π²Π½ΠΎ суммС всСх сопротивлСний.

    НапряТСниС ΠΏΡ€ΠΈ ΠΏΠΎΡΠ»Π΅Π΄ΠΎΠ²Π°Ρ‚Π΅Π»ΡŒΠ½ΠΎΠΌ соСдинСнии

    НапряТСниС ΠΏΡ€ΠΈ ΠΏΠΎΡΠ»Π΅Π΄ΠΎΠ²Π°Ρ‚Π΅Π»ΡŒΠ½ΠΎΠΌ соСдинСнии распрСдСляСтся Π½Π° ΠΊΠ°ΠΆΠ΄Ρ‹ΠΉ рСзистор согласно Π·Π°ΠΊΠΎΠ½Ρƒ Ома:

    Π’.Π΅ Ρ‡Π΅ΠΌ большСС сопротивлСниС рСзистора, Ρ‚Π΅ΠΌ большСС напряТСниС Π½Π° Π½Π΅Π³ΠΎ ΠΏΠ°Π΄Π°Π΅Ρ‚.

    ΠŸΠ°Ρ€Π°Π»Π»Π΅Π»ΡŒΠ½ΠΎΠ΅ соСдинСниС рСзисторов

    ΠŸΠ°Ρ€Π°Π»Π»Π΅Π»ΡŒΠ½ΠΎΠ΅ соСдинСниС – это соСдинСниС, ΠΏΡ€ΠΈ ΠΊΠΎΡ‚ΠΎΡ€ΠΎΠΌ рСзисторы ΡΠΎΠ΅Π΄ΠΈΠ½ΡΡŽΡ‚ΡΡ ΠΌΠ΅ΠΆΠ΄Ρƒ собой ΠΎΠ±ΠΎΠΈΠΌΠΈ ΠΊΠΎΠ½Ρ‚Π°ΠΊΡ‚Π°ΠΌΠΈ. Π’ Ρ€Π΅Π·ΡƒΠ»ΡŒΡ‚Π°Ρ‚Π΅ ΠΊ ΠΎΠ΄Π½ΠΎΠΉ Ρ‚ΠΎΡ‡ΠΊΠ΅ (элСктричСскому ΡƒΠ·Π»Ρƒ) ΠΌΠΎΠΆΠ΅Ρ‚ Π±Ρ‹Ρ‚ΡŒ присоСдинСно нСсколько рСзисторов.

    ΠžΠ±Ρ‰Π΅Π΅ сопротивлСниС R

    ΠΎΠ±Ρ‰

    ΠŸΡ€ΠΈ Ρ‚Π°ΠΊΠΎΠΌ соСдинСнии, Ρ‡Π΅Ρ€Π΅Π· ΠΊΠ°ΠΆΠ΄Ρ‹ΠΉ рСзистор ΠΏΠΎΡ‚Π΅Ρ‡Π΅Ρ‚ ΠΎΡ‚Π΄Π΅Π»ΡŒΠ½Ρ‹ΠΉ Ρ‚ΠΎΠΊ. Π‘ΠΈΠ»Π° Π΄Π°Π½Π½ΠΎΠ³ΠΎ Ρ‚ΠΎΠΊΠ° Π±ΡƒΠ΄Π΅Ρ‚ ΠΎΠ±Ρ€Π°Ρ‚Π½ΠΎ ΠΏΡ€ΠΎΠΏΠΎΡ€Ρ†ΠΈΠΎΠ½Π°Π»ΡŒΠ½Π° ΡΠΎΠΏΡ€ΠΎΡ‚ΠΈΠ²Π»Π΅Π½ΠΈΡŽ рСзистора. Π’ Ρ€Π΅Π·ΡƒΠ»ΡŒΡ‚Π°Ρ‚Π΅ общая ΠΏΡ€ΠΎΠ²ΠΎΠ΄ΠΈΠΌΠΎΡΡ‚ΡŒ Ρ‚Π°ΠΊΠΎΠ³ΠΎ участка элСктричСской Ρ†Π΅ΠΏΠΈ увСличиваСтся, Π° ΠΎΠ±Ρ‰Π΅Π΅ сопротивлСниС Π² свою ΠΎΡ‡Π΅Ρ€Π΅Π΄ΡŒ ΡƒΠΌΠ΅Π½ΡŒΡˆΠ°Π΅Ρ‚ΡΡ.

    Π’Π°ΠΊΠΈΠΌ ΠΎΠ±Ρ€Π°Π·ΠΎΠΌ, ΠΏΡ€ΠΈ ΠΏΠ°Ρ€Π°Π»Π»Π΅Π»ΡŒΠ½ΠΎΠΌ подсоСдинСнии рСзисторов с Ρ€Π°Π·Π½Ρ‹ΠΌ сопротивлСниСм, ΠΎΠ±Ρ‰Π΅Π΅ сопротивлСниС Π±ΡƒΠ΄Π΅Ρ‚ всСгда мСньшС значСния самого малСнького ΠΎΡ‚Π΄Π΅Π»ΡŒΠ½ΠΎΠ³ΠΎ рСзистора.

    Π€ΠΎΡ€ΠΌΡƒΠ»Π° ΠΎΠ±Ρ‰Π΅ΠΉ проводимости ΠΏΡ€ΠΈ ΠΏΠ°Ρ€Π°Π»Π»Π΅Π»ΡŒΠ½ΠΎΠΌ соСдинСнии рСзисторов:

    Π€ΠΎΡ€ΠΌΡƒΠ»Π° эквивалСнтного ΠΎΠ±Ρ‰Π΅Π³ΠΎ сопротивлСния ΠΏΡ€ΠΈ ΠΏΠ°Ρ€Π°Π»Π»Π΅Π»ΡŒΠ½ΠΎΠΌ соСдинСнии рСзисторов:

    Для Π΄Π²ΡƒΡ… ΠΎΠ΄ΠΈΠ½Π°ΠΊΠΎΠ²Ρ‹Ρ… рСзисторов ΠΎΠ±Ρ‰Π΅Π΅ сопротивлСниС Π±ΡƒΠ΄Π΅Ρ‚ Ρ€Π°Π²Π½ΠΎ ΠΏΠΎΠ»ΠΎΠ²ΠΈΠ½Π΅ ΠΎΠ΄Π½ΠΎΠ³ΠΎ ΠΎΡ‚Π΄Π΅Π»ΡŒΠ½ΠΎΠ³ΠΎ рСзистора:

    БоотвСтствСнно, для n ΠΎΠ΄ΠΈΠ½Π°ΠΊΠΎΠ²Ρ‹Ρ… рСзисторов ΠΎΠ±Ρ‰Π΅Π΅ сопротивлСниС Π±ΡƒΠ΄Π΅Ρ‚ Ρ€Π°Π²Π½ΠΎ Π·Π½Π°Ρ‡Π΅Π½ΠΈΡŽ ΠΎΠ΄Π½ΠΎΠ³ΠΎ рСзистора, Ρ€Π°Π·Π΄Π΅Π»Π΅Π½Π½ΠΎΠ³ΠΎ Π½Π° n.

    НапряТСниС ΠΏΡ€ΠΈ ΠΏΠ°Ρ€Π°Π»Π»Π΅Π»ΡŒΠ½ΠΎΠΌ соСдинСнии

    НапряТСниС ΠΌΠ΅ΠΆΠ΄Ρƒ Ρ‚ΠΎΡ‡ΠΊΠ°ΠΌΠΈ A ΠΈ B являСтся ΠΊΠ°ΠΊ ΠΎΠ±Ρ‰ΠΈΠΌ напряТСниСм для всСго участка Ρ†Π΅ΠΏΠΈ, Ρ‚Π°ΠΊ ΠΈ напряТСниСм, ΠΏΠ°Π΄Π°ΡŽΡ‰ΠΈΠΌ Π½Π° ΠΊΠ°ΠΆΠ΄Ρ‹ΠΉ рСзистор Π² ΠΎΡ‚Π΄Π΅Π»ΡŒΠ½ΠΎΡΡ‚ΠΈ. ΠŸΠΎΡΡ‚ΠΎΠΌΡƒ ΠΏΡ€ΠΈ ΠΏΠ°Ρ€Π°Π»Π»Π΅Π»ΡŒΠ½ΠΎΠΌ соСдинСнии Π½Π° всС рСзисторы ΡƒΠΏΠ°Π΄Π΅Ρ‚ ΠΎΠ΄ΠΈΠ½Π°ΠΊΠΎΠ²ΠΎΠ΅ напряТСниС.

    ЭлСктричСский Ρ‚ΠΎΠΊ ΠΏΡ€ΠΈ ΠΏΠ°Ρ€Π°Π»Π»Π΅Π»ΡŒΠ½ΠΎΠΌ соСдинСнии

    Π§Π΅Ρ€Π΅Π· ΠΊΠ°ΠΆΠ΄Ρ‹ΠΉ рСзистор Ρ‚Π΅Ρ‡Π΅Ρ‚ Ρ‚ΠΎΠΊ, сила ΠΊΠΎΡ‚ΠΎΡ€ΠΎΠ³ΠΎ ΠΎΠ±Ρ€Π°Ρ‚Π½ΠΎ ΠΏΡ€ΠΎΠΏΠΎΡ€Ρ†ΠΈΠΎΠ½Π°Π»ΡŒΠ½Π° ΡΠΎΠΏΡ€ΠΎΡ‚ΠΈΠ²Π»Π΅Π½ΠΈΡŽ рСзистора. Для Ρ‚ΠΎΠ³ΠΎ Ρ‡Ρ‚ΠΎΠ±Ρ‹ ΡƒΠ·Π½Π°Ρ‚ΡŒ ΠΊΠ°ΠΊΠΎΠΉ Ρ‚ΠΎΠΊ Ρ‚Π΅Ρ‡Π΅Ρ‚ Ρ‡Π΅Ρ€Π΅Π· ΠΎΠΏΡ€Π΅Π΄Π΅Π»Π΅Π½Π½Ρ‹ΠΉ рСзистор, ΠΌΠΎΠΆΠ½ΠΎ Π²ΠΎΡΠΏΠΎΠ»ΡŒΠ·ΠΎΠ²Π°Ρ‚ΡŒΡΡ Π·Π°ΠΊΠΎΠ½ΠΎΠΌ Ома:

    БмСшанноС соСдинСниС рСзисторов

    Π‘ΠΌΠ΅ΡˆΠ°Π½Π½Ρ‹ΠΌ соСдинСниСм Π½Π°Π·Ρ‹Π²Π°ΡŽΡ‚ участок Ρ†Π΅ΠΏΠΈ, Π³Π΄Π΅ Ρ‡Π°ΡΡ‚ΡŒ рСзисторов ΡΠΎΠ΅Π΄ΠΈΠ½ΡΡŽΡ‚ΡΡ ΠΌΠ΅ΠΆΠ΄Ρƒ собой ΠΏΠΎΡΠ»Π΅Π΄ΠΎΠ²Π°Ρ‚Π΅Π»ΡŒΠ½ΠΎ, Π° Ρ‡Π°ΡΡ‚ΡŒ ΠΏΠ°Ρ€Π°Π»Π»Π΅Π»ΡŒΠ½ΠΎ. Π’ свою ΠΎΡ‡Π΅Ρ€Π΅Π΄ΡŒ, смСшанноС соСдинСниС Π±Ρ‹Π²Π°Π΅Ρ‚ ΠΏΠΎΡΠ»Π΅Π΄ΠΎΠ²Π°Ρ‚Π΅Π»ΡŒΠ½ΠΎΠ³ΠΎ ΠΈ ΠΏΠ°Ρ€Π°Π»Π»Π΅Π»ΡŒΠ½ΠΎΠ³ΠΎ Ρ‚ΠΈΠΏΠΎΠ².

    ΠžΠ±Ρ‰Π΅Π΅ сопротивлСниС R

    ΠΎΠ±Ρ‰

    Для Ρ‚ΠΎΠ³ΠΎ Ρ‡Ρ‚ΠΎΠ±Ρ‹ ΠΏΠΎΡΡ‡ΠΈΡ‚Π°Ρ‚ΡŒ ΠΎΠ±Ρ‰Π΅Π΅ сопротивлСниС смСшанного соСдинСния:

    • ЦСпь Ρ€Π°Π·Π±ΠΈΠ²Π°ΡŽΡ‚ Π½Π° участки с Ρ‚ΠΎΠ»ΡŒΠΊΠΎ ΠΏΠ°Ρ€Π΅Π»Π»Π΅Π»ΡŒΠ½Ρ‹ΠΌ ΠΈΠ»ΠΈ Ρ‚ΠΎΠ»ΡŒΠΊΠΎ ΠΏΠΎΡΠ»Π΅Π΄ΠΎΠ²Π°Ρ‚Π΅Π»ΡŒΠ½Ρ‹ΠΌ соСдинСниСм.
    • Π’Ρ‹Ρ‡ΠΈΡΠ»ΡΡŽΡ‚ ΠΎΠ±Ρ‰Π΅Π΅ сопротивлСниС для ΠΊΠ°ΠΆΠ΄ΠΎΠ³ΠΎ ΠΎΡ‚Π΄Π΅Π»ΡŒΠ½ΠΎΠ³ΠΎ участка.
    • Π’Ρ‹Ρ‡ΠΈΡΠ»ΡΡŽΡ‚ ΠΎΠ±Ρ‰Π΅Π΅ сопротивлСниС для всСй Ρ†Π΅ΠΏΠΈ смСшанного соСдинСния.

    Π’Π°ΠΊ это Π±ΡƒΠ΄Π΅Ρ‚ Π²Ρ‹Π³Π»ΡΠ΄Π΅Ρ‚ΡŒ для схСмы 1:

    Π’Π°ΠΊΠΆΠ΅ сущСствуСт Π±ΠΎΠ»Π΅Π΅ быстрый способ расчСта ΠΎΠ±Ρ‰Π΅Π³ΠΎ сопротивлСния для смСшанного соСдинСния. МоТно, Π² соотвСтствии схСмС, сразу Π·Π°ΠΏΠΈΡΡ‹Π²Π°Ρ‚ΡŒ Ρ„ΠΎΡ€ΠΌΡƒΠ»Ρƒ ΡΠ»Π΅Π΄ΡƒΡŽΡ‰ΠΈΠΌ ΠΎΠ±Ρ€Π°Π·ΠΎΠΌ:

    • Если рСзисторы ΡΠΎΠ΅Π΄ΠΈΠ½ΡΡŽΡ‚ΡΡ ΠΏΠΎΡΠ»Π΅Π΄ΠΎΠ²Π°Ρ‚Π΅ΡŒΠ½ΠΎ β€” ΡΠΊΠ»Π°Π΄Ρ‹Π²Π°Ρ‚ΡŒ.
    • Если рСзисторы ΡΠΎΠ΅Π΄ΠΈΠ½ΡΡŽΡ‚ΡΡ ΠΏΠ°Ρ€Π°Π»Π»Π΅Π»ΡŒΠ½ΠΎ β€” ΠΈΡΠΏΠΎΠ»ΡŒΠ·ΠΎΠ²Π°Ρ‚ΡŒ условноС ΠΎΠ±ΠΎΠ·Π½Π°Ρ‡Π΅Π½ΠΈΠ΅ Β«||Β».
    • ΠŸΠΎΠ΄ΡΡ‚Π°Π²Π»ΡΡ‚ΡŒ Ρ„ΠΎΡ€ΠΌΡƒΠ»Ρƒ для ΠΏΠ°Ρ€Π°Π»Π»Π΅Π»ΡŒΠ½ΠΎΠ³ΠΎ соСдинСния Π³Π΄Π΅ стоит символ Β«||Β».

    Π’Π°ΠΊ это Π±ΡƒΠ΄Π΅Ρ‚ Π²Ρ‹Π³Π»ΡΠ΄Π΅Ρ‚ΡŒ для схСмы 1:

    ПослС подстановки Ρ„ΠΎΡ€ΠΌΡƒΠ»Ρ‹ ΠΏΠ°Ρ€Π°Π»Π»Π΅Π»ΡŒΠ½ΠΎΠ³ΠΎ соСдинСния вмСсто Β«||Β»:

    ΠŸΠΎΡΠ»Π΅Π΄ΠΎΠ²Π°Ρ‚Π΅Π»ΡŒΠ½ΠΎΠ΅ соСдинСниС рСзисторов

    ΠŸΠΎΡΠ»Π΅Π΄ΠΎΠ²Π°Ρ‚Π΅Π»ΡŒΠ½ΠΎΠ΅ соСдинСниС – это соСдинСниС Π΄Π²ΡƒΡ… ΠΈΠ»ΠΈ Π±ΠΎΠ»Π΅Π΅ рСзисторов Π² Ρ„ΠΎΡ€ΠΌΠ΅ Ρ†Π΅ΠΏΠΈ, Π² ΠΊΠΎΡ‚ΠΎΡ€ΠΎΠΉ ΠΊΠ°ΠΆΠ΄Ρ‹ΠΉ ΠΎΡ‚Π΄Π΅Π»ΡŒΠ½Ρ‹ΠΉ рСзистор соСдиняСтся с Π΄Ρ€ΡƒΠ³ΠΈΠΌ ΠΎΡ‚Π΄Π΅Π»ΡŒΠ½Ρ‹ΠΌ рСзистором Ρ‚ΠΎΠ»ΡŒΠΊΠΎ Π² ΠΎΠ΄Π½ΠΎΠΉ Ρ‚ΠΎΡ‡ΠΊΠ΅.

    ΠžΠ±Ρ‰Π΅Π΅ сопротивлСниС R

    ΠΎΠ±Ρ‰

    ΠŸΡ€ΠΈ Ρ‚Π°ΠΊΠΎΠΌ соСдинСнии, Ρ‡Π΅Ρ€Π΅Π· всС рСзисторы ΠΏΡ€ΠΎΡ…ΠΎΠ΄ΠΈΡ‚ ΠΎΠ΄ΠΈΠ½ ΠΈ Ρ‚ΠΎΡ‚ ΠΆΠ΅ элСктричСский Ρ‚ΠΎΠΊ. Π§Π΅ΠΌ большС элСмСнтов Π½Π° Π΄Π°Π½Π½ΠΎΠΌ участкС элСктричСской Ρ†Π΅ΠΏΠΈ, Ρ‚Π΅ΠΌ Β«Ρ‚Ρ€ΡƒΠ΄Π½Π΅Π΅Β» Ρ‚ΠΎΠΊΡƒ ΠΏΡ€ΠΎΡ‚Π΅ΠΊΠ°Ρ‚ΡŒ Ρ‡Π΅Ρ€Π΅Π· Π½Π΅Π³ΠΎ. Π‘Π»Π΅Π΄ΠΎΠ²Π°Ρ‚Π΅Π»ΡŒΠ½ΠΎ, ΠΏΡ€ΠΈ ΠΏΠΎΡΠ»Π΅Π΄ΠΎΠ²Π°Ρ‚Π΅Π»ΡŒΠ½ΠΎΠΌ соСдинСнии рСзисторов ΠΈΡ… ΠΎΠ±Ρ‰Π΅Π΅ сопротивлСниС увСличиваСтся, ΠΈ ΠΎΠ½ΠΎ Ρ€Π°Π²Π½ΠΎ суммС всСх сопротивлСний.

    НапряТСниС ΠΏΡ€ΠΈ ΠΏΠΎΡΠ»Π΅Π΄ΠΎΠ²Π°Ρ‚Π΅Π»ΡŒΠ½ΠΎΠΌ соСдинСнии

    НапряТСниС ΠΏΡ€ΠΈ ΠΏΠΎΡΠ»Π΅Π΄ΠΎΠ²Π°Ρ‚Π΅Π»ΡŒΠ½ΠΎΠΌ соСдинСнии распрСдСляСтся Π½Π° ΠΊΠ°ΠΆΠ΄Ρ‹ΠΉ рСзистор согласно Π·Π°ΠΊΠΎΠ½Ρƒ Ома:

    Π’.Π΅ Ρ‡Π΅ΠΌ большСС сопротивлСниС рСзистора, Ρ‚Π΅ΠΌ большСС напряТСниС Π½Π° Π½Π΅Π³ΠΎ ΠΏΠ°Π΄Π°Π΅Ρ‚.

    ΠŸΠ°Ρ€Π°Π»Π»Π΅Π»ΡŒΠ½ΠΎΠ΅ соСдинСниС рСзисторов

    ΠŸΠ°Ρ€Π°Π»Π»Π΅Π»ΡŒΠ½ΠΎΠ΅ соСдинСниС – это соСдинСниС, ΠΏΡ€ΠΈ ΠΊΠΎΡ‚ΠΎΡ€ΠΎΠΌ рСзисторы ΡΠΎΠ΅Π΄ΠΈΠ½ΡΡŽΡ‚ΡΡ ΠΌΠ΅ΠΆΠ΄Ρƒ собой ΠΎΠ±ΠΎΠΈΠΌΠΈ ΠΊΠΎΠ½Ρ‚Π°ΠΊΡ‚Π°ΠΌΠΈ. Π’ Ρ€Π΅Π·ΡƒΠ»ΡŒΡ‚Π°Ρ‚Π΅ ΠΊ ΠΎΠ΄Π½ΠΎΠΉ Ρ‚ΠΎΡ‡ΠΊΠ΅ (элСктричСскому ΡƒΠ·Π»Ρƒ) ΠΌΠΎΠΆΠ΅Ρ‚ Π±Ρ‹Ρ‚ΡŒ присоСдинСно нСсколько рСзисторов.

    ΠžΠ±Ρ‰Π΅Π΅ сопротивлСниС R

    ΠΎΠ±Ρ‰

    ΠŸΡ€ΠΈ Ρ‚Π°ΠΊΠΎΠΌ соСдинСнии, Ρ‡Π΅Ρ€Π΅Π· ΠΊΠ°ΠΆΠ΄Ρ‹ΠΉ рСзистор ΠΏΠΎΡ‚Π΅Ρ‡Π΅Ρ‚ ΠΎΡ‚Π΄Π΅Π»ΡŒΠ½Ρ‹ΠΉ Ρ‚ΠΎΠΊ. Π‘ΠΈΠ»Π° Π΄Π°Π½Π½ΠΎΠ³ΠΎ Ρ‚ΠΎΠΊΠ° Π±ΡƒΠ΄Π΅Ρ‚ ΠΎΠ±Ρ€Π°Ρ‚Π½ΠΎ ΠΏΡ€ΠΎΠΏΠΎΡ€Ρ†ΠΈΠΎΠ½Π°Π»ΡŒΠ½Π° ΡΠΎΠΏΡ€ΠΎΡ‚ΠΈΠ²Π»Π΅Π½ΠΈΡŽ рСзистора. Π’ Ρ€Π΅Π·ΡƒΠ»ΡŒΡ‚Π°Ρ‚Π΅ общая ΠΏΡ€ΠΎΠ²ΠΎΠ΄ΠΈΠΌΠΎΡΡ‚ΡŒ Ρ‚Π°ΠΊΠΎΠ³ΠΎ участка элСктричСской Ρ†Π΅ΠΏΠΈ увСличиваСтся, Π° ΠΎΠ±Ρ‰Π΅Π΅ сопротивлСниС Π² свою ΠΎΡ‡Π΅Ρ€Π΅Π΄ΡŒ ΡƒΠΌΠ΅Π½ΡŒΡˆΠ°Π΅Ρ‚ΡΡ.

    Π’Π°ΠΊΠΈΠΌ ΠΎΠ±Ρ€Π°Π·ΠΎΠΌ, ΠΏΡ€ΠΈ ΠΏΠ°Ρ€Π°Π»Π»Π΅Π»ΡŒΠ½ΠΎΠΌ подсоСдинСнии рСзисторов с Ρ€Π°Π·Π½Ρ‹ΠΌ сопротивлСниСм, ΠΎΠ±Ρ‰Π΅Π΅ сопротивлСниС Π±ΡƒΠ΄Π΅Ρ‚ всСгда мСньшС значСния самого малСнького ΠΎΡ‚Π΄Π΅Π»ΡŒΠ½ΠΎΠ³ΠΎ рСзистора.

    Π€ΠΎΡ€ΠΌΡƒΠ»Π° ΠΎΠ±Ρ‰Π΅ΠΉ проводимости ΠΏΡ€ΠΈ ΠΏΠ°Ρ€Π°Π»Π»Π΅Π»ΡŒΠ½ΠΎΠΌ соСдинСнии рСзисторов:

    Π€ΠΎΡ€ΠΌΡƒΠ»Π° эквивалСнтного ΠΎΠ±Ρ‰Π΅Π³ΠΎ сопротивлСния ΠΏΡ€ΠΈ ΠΏΠ°Ρ€Π°Π»Π»Π΅Π»ΡŒΠ½ΠΎΠΌ соСдинСнии рСзисторов:

    Для Π΄Π²ΡƒΡ… ΠΎΠ΄ΠΈΠ½Π°ΠΊΠΎΠ²Ρ‹Ρ… рСзисторов ΠΎΠ±Ρ‰Π΅Π΅ сопротивлСниС Π±ΡƒΠ΄Π΅Ρ‚ Ρ€Π°Π²Π½ΠΎ ΠΏΠΎΠ»ΠΎΠ²ΠΈΠ½Π΅ ΠΎΠ΄Π½ΠΎΠ³ΠΎ ΠΎΡ‚Π΄Π΅Π»ΡŒΠ½ΠΎΠ³ΠΎ рСзистора:

    БоотвСтствСнно, для n ΠΎΠ΄ΠΈΠ½Π°ΠΊΠΎΠ²Ρ‹Ρ… рСзисторов ΠΎΠ±Ρ‰Π΅Π΅ сопротивлСниС Π±ΡƒΠ΄Π΅Ρ‚ Ρ€Π°Π²Π½ΠΎ Π·Π½Π°Ρ‡Π΅Π½ΠΈΡŽ ΠΎΠ΄Π½ΠΎΠ³ΠΎ рСзистора, Ρ€Π°Π·Π΄Π΅Π»Π΅Π½Π½ΠΎΠ³ΠΎ Π½Π° n.

    НапряТСниС ΠΏΡ€ΠΈ ΠΏΠ°Ρ€Π°Π»Π»Π΅Π»ΡŒΠ½ΠΎΠΌ соСдинСнии

    НапряТСниС ΠΌΠ΅ΠΆΠ΄Ρƒ Ρ‚ΠΎΡ‡ΠΊΠ°ΠΌΠΈ A ΠΈ B являСтся ΠΊΠ°ΠΊ ΠΎΠ±Ρ‰ΠΈΠΌ напряТСниСм для всСго участка Ρ†Π΅ΠΏΠΈ, Ρ‚Π°ΠΊ ΠΈ напряТСниСм, ΠΏΠ°Π΄Π°ΡŽΡ‰ΠΈΠΌ Π½Π° ΠΊΠ°ΠΆΠ΄Ρ‹ΠΉ рСзистор Π² ΠΎΡ‚Π΄Π΅Π»ΡŒΠ½ΠΎΡΡ‚ΠΈ. ΠŸΠΎΡΡ‚ΠΎΠΌΡƒ ΠΏΡ€ΠΈ ΠΏΠ°Ρ€Π°Π»Π»Π΅Π»ΡŒΠ½ΠΎΠΌ соСдинСнии Π½Π° всС рСзисторы ΡƒΠΏΠ°Π΄Π΅Ρ‚ ΠΎΠ΄ΠΈΠ½Π°ΠΊΠΎΠ²ΠΎΠ΅ напряТСниС.

    ЭлСктричСский Ρ‚ΠΎΠΊ ΠΏΡ€ΠΈ ΠΏΠ°Ρ€Π°Π»Π»Π΅Π»ΡŒΠ½ΠΎΠΌ соСдинСнии

    Π§Π΅Ρ€Π΅Π· ΠΊΠ°ΠΆΠ΄Ρ‹ΠΉ рСзистор Ρ‚Π΅Ρ‡Π΅Ρ‚ Ρ‚ΠΎΠΊ, сила ΠΊΠΎΡ‚ΠΎΡ€ΠΎΠ³ΠΎ ΠΎΠ±Ρ€Π°Ρ‚Π½ΠΎ ΠΏΡ€ΠΎΠΏΠΎΡ€Ρ†ΠΈΠΎΠ½Π°Π»ΡŒΠ½Π° ΡΠΎΠΏΡ€ΠΎΡ‚ΠΈΠ²Π»Π΅Π½ΠΈΡŽ рСзистора. Для Ρ‚ΠΎΠ³ΠΎ Ρ‡Ρ‚ΠΎΠ±Ρ‹ ΡƒΠ·Π½Π°Ρ‚ΡŒ ΠΊΠ°ΠΊΠΎΠΉ Ρ‚ΠΎΠΊ Ρ‚Π΅Ρ‡Π΅Ρ‚ Ρ‡Π΅Ρ€Π΅Π· ΠΎΠΏΡ€Π΅Π΄Π΅Π»Π΅Π½Π½Ρ‹ΠΉ рСзистор, ΠΌΠΎΠΆΠ½ΠΎ Π²ΠΎΡΠΏΠΎΠ»ΡŒΠ·ΠΎΠ²Π°Ρ‚ΡŒΡΡ Π·Π°ΠΊΠΎΠ½ΠΎΠΌ Ома:

    БмСшанноС соСдинСниС рСзисторов

    Π‘ΠΌΠ΅ΡˆΠ°Π½Π½Ρ‹ΠΌ соСдинСниСм Π½Π°Π·Ρ‹Π²Π°ΡŽΡ‚ участок Ρ†Π΅ΠΏΠΈ, Π³Π΄Π΅ Ρ‡Π°ΡΡ‚ΡŒ рСзисторов ΡΠΎΠ΅Π΄ΠΈΠ½ΡΡŽΡ‚ΡΡ ΠΌΠ΅ΠΆΠ΄Ρƒ собой ΠΏΠΎΡΠ»Π΅Π΄ΠΎΠ²Π°Ρ‚Π΅Π»ΡŒΠ½ΠΎ, Π° Ρ‡Π°ΡΡ‚ΡŒ ΠΏΠ°Ρ€Π°Π»Π»Π΅Π»ΡŒΠ½ΠΎ. Π’ свою ΠΎΡ‡Π΅Ρ€Π΅Π΄ΡŒ, смСшанноС соСдинСниС Π±Ρ‹Π²Π°Π΅Ρ‚ ΠΏΠΎΡΠ»Π΅Π΄ΠΎΠ²Π°Ρ‚Π΅Π»ΡŒΠ½ΠΎΠ³ΠΎ ΠΈ ΠΏΠ°Ρ€Π°Π»Π»Π΅Π»ΡŒΠ½ΠΎΠ³ΠΎ Ρ‚ΠΈΠΏΠΎΠ².

    ΠžΠ±Ρ‰Π΅Π΅ сопротивлСниС R

    ΠΎΠ±Ρ‰

    Для Ρ‚ΠΎΠ³ΠΎ Ρ‡Ρ‚ΠΎΠ±Ρ‹ ΠΏΠΎΡΡ‡ΠΈΡ‚Π°Ρ‚ΡŒ ΠΎΠ±Ρ‰Π΅Π΅ сопротивлСниС смСшанного соСдинСния:

    • ЦСпь Ρ€Π°Π·Π±ΠΈΠ²Π°ΡŽΡ‚ Π½Π° участки с Ρ‚ΠΎΠ»ΡŒΠΊΠΎ ΠΏΠ°Ρ€Π΅Π»Π»Π΅Π»ΡŒΠ½Ρ‹ΠΌ ΠΈΠ»ΠΈ Ρ‚ΠΎΠ»ΡŒΠΊΠΎ ΠΏΠΎΡΠ»Π΅Π΄ΠΎΠ²Π°Ρ‚Π΅Π»ΡŒΠ½Ρ‹ΠΌ соСдинСниСм.
    • Π’Ρ‹Ρ‡ΠΈΡΠ»ΡΡŽΡ‚ ΠΎΠ±Ρ‰Π΅Π΅ сопротивлСниС для ΠΊΠ°ΠΆΠ΄ΠΎΠ³ΠΎ ΠΎΡ‚Π΄Π΅Π»ΡŒΠ½ΠΎΠ³ΠΎ участка.
    • Π’Ρ‹Ρ‡ΠΈΡΠ»ΡΡŽΡ‚ ΠΎΠ±Ρ‰Π΅Π΅ сопротивлСниС для всСй Ρ†Π΅ΠΏΠΈ смСшанного соСдинСния.

    Π’Π°ΠΊ это Π±ΡƒΠ΄Π΅Ρ‚ Π²Ρ‹Π³Π»ΡΠ΄Π΅Ρ‚ΡŒ для схСмы 1:

    Π’Π°ΠΊΠΆΠ΅ сущСствуСт Π±ΠΎΠ»Π΅Π΅ быстрый способ расчСта ΠΎΠ±Ρ‰Π΅Π³ΠΎ сопротивлСния для смСшанного соСдинСния. МоТно, Π² соотвСтствии схСмС, сразу Π·Π°ΠΏΠΈΡΡ‹Π²Π°Ρ‚ΡŒ Ρ„ΠΎΡ€ΠΌΡƒΠ»Ρƒ ΡΠ»Π΅Π΄ΡƒΡŽΡ‰ΠΈΠΌ ΠΎΠ±Ρ€Π°Π·ΠΎΠΌ:

    • Если рСзисторы ΡΠΎΠ΅Π΄ΠΈΠ½ΡΡŽΡ‚ΡΡ ΠΏΠΎΡΠ»Π΅Π΄ΠΎΠ²Π°Ρ‚Π΅ΡŒΠ½ΠΎ β€” ΡΠΊΠ»Π°Π΄Ρ‹Π²Π°Ρ‚ΡŒ.
    • Если рСзисторы ΡΠΎΠ΅Π΄ΠΈΠ½ΡΡŽΡ‚ΡΡ ΠΏΠ°Ρ€Π°Π»Π»Π΅Π»ΡŒΠ½ΠΎ β€” ΠΈΡΠΏΠΎΠ»ΡŒΠ·ΠΎΠ²Π°Ρ‚ΡŒ условноС ΠΎΠ±ΠΎΠ·Π½Π°Ρ‡Π΅Π½ΠΈΠ΅ Β«||Β».
    • ΠŸΠΎΠ΄ΡΡ‚Π°Π²Π»ΡΡ‚ΡŒ Ρ„ΠΎΡ€ΠΌΡƒΠ»Ρƒ для ΠΏΠ°Ρ€Π°Π»Π»Π΅Π»ΡŒΠ½ΠΎΠ³ΠΎ соСдинСния Π³Π΄Π΅ стоит символ Β«||Β».

    Π’Π°ΠΊ это Π±ΡƒΠ΄Π΅Ρ‚ Π²Ρ‹Π³Π»ΡΠ΄Π΅Ρ‚ΡŒ для схСмы 1:

    ПослС подстановки Ρ„ΠΎΡ€ΠΌΡƒΠ»Ρ‹ ΠΏΠ°Ρ€Π°Π»Π»Π΅Π»ΡŒΠ½ΠΎΠ³ΠΎ соСдинСния вмСсто Β«||Β»:

    Как я ΠΈ ΠΎΠ±Π΅Ρ‰Π°Π» Π² ΡΡ‚Π°Ρ‚ΡŒΠ΅ ΠΏΡ€ΠΎ ΠΏΠ΅Ρ€Π΅ΠΌΠ΅Π½Π½Ρ‹Π΅ рСзисторы (ссылка), сСгодня Ρ€Π΅Ρ‡ΡŒ ΠΏΠΎΠΉΠ΄Π΅Ρ‚ ΠΎ Π²ΠΎΠ·ΠΌΠΎΠΆΠ½Ρ‹Ρ… способах соСдинСния рСзисторов, Π² частности ΠΎ ΠΏΠΎΡΠ»Π΅Π΄ΠΎΠ²Π°Ρ‚Π΅Π»ΡŒΠ½ΠΎΠΌ соСдинСнии ΠΈ ΠΎ ΠΏΠ°Ρ€Π°Π»Π»Π΅Π»ΡŒΠ½ΠΎΠΌ.

    ΠŸΠΎΡΠ»Π΅Π΄ΠΎΠ²Π°Ρ‚Π΅Π»ΡŒΠ½ΠΎΠ΅ соСдинСниС рСзисторов.

    Π”Π°Π²Π°ΠΉΡ‚Π΅ Π½Π°Ρ‡Π½Π΅ΠΌ с рассмотрСния Ρ†Π΅ΠΏΠ΅ΠΉ, элСмСнты ΠΊΠΎΡ‚ΠΎΡ€ΠΎΠΉ соСдинСны ΠΏΠΎΡΠ»Π΅Π΄ΠΎΠ²Π°Ρ‚Π΅Π»ΡŒΠ½ΠΎ. И Ρ…ΠΎΡ‚ΡŒ ΠΌΡ‹ ΠΈ Π±ΡƒΠ΄Π΅ΠΌ Ρ€Π°ΡΡΠΌΠ°Ρ‚Ρ€ΠΈΠ²Π°Ρ‚ΡŒ Ρ‚ΠΎΠ»ΡŒΠΊΠΎ рСзисторы Π² качСствС элСмСнтов Ρ†Π΅ΠΏΠΈ Π² Π΄Π°Π½Π½ΠΎΠΉ ΡΡ‚Π°Ρ‚ΡŒΠ΅, Π½ΠΎ ΠΏΡ€Π°Π²ΠΈΠ»Π°, ΠΊΠ°ΡΠ°ΡŽΡ‰ΠΈΠ΅ΡΡ напряТСний ΠΈ Ρ‚ΠΎΠΊΠΎΠ² ΠΏΡ€ΠΈ Ρ€Π°Π·Π½Ρ‹Ρ… соСдинСниях Π±ΡƒΠ΄ΡƒΡ‚ справСдливы ΠΈ для Π΄Ρ€ΡƒΠ³ΠΈΡ… элСмСнтов. Π˜Ρ‚Π°ΠΊ, пСрвая Ρ†Π΅ΠΏΡŒ, ΠΊΠΎΡ‚ΠΎΡ€ΡƒΡŽ ΠΌΡ‹ Π±ΡƒΠ΄Π΅ΠΌ Ρ€Π°Π·Π±ΠΈΡ€Π°Ρ‚ΡŒ выглядит ΡΠ»Π΅Π΄ΡƒΡŽΡ‰ΠΈΠΌ ΠΎΠ±Ρ€Π°Π·ΠΎΠΌ:

    Π—Π΄Π΅ΡΡŒ Ρƒ нас классичСский случай ΠΏΠΎΡΠ»Π΅Π΄ΠΎΠ²Π°Ρ‚Π΅Π»ΡŒΠ½ΠΎΠ³ΠΎ соСдинСния – Π΄Π²Π° ΠΏΠΎΡΠ»Π΅Π΄ΠΎΠ²Π°Ρ‚Π΅Π»ΡŒΠ½ΠΎ Π²ΠΊΠ»ΡŽΡ‡Π΅Π½Π½Ρ‹Ρ… рСзистора. Но Π½Π΅ Π±ΡƒΠ΄Π΅ΠΌ Π·Π°Π±Π΅Π³Π°Ρ‚ΡŒ Π²ΠΏΠ΅Ρ€Π΅Π΄ ΠΈ Ρ€Π°ΡΡΡ‡ΠΈΡ‚Ρ‹Π²Π°Ρ‚ΡŒ ΠΎΠ±Ρ‰Π΅Π΅ сопротивлСниС Ρ†Π΅ΠΏΠΈ, Π° для Π½Π°Ρ‡Π°Π»Π° рассмотрим всС напряТСния ΠΈ Ρ‚ΠΎΠΊΠΈ. Π˜Ρ‚Π°ΠΊ, ΠΏΠ΅Ρ€Π²ΠΎΠ΅ ΠΏΡ€Π°Π²ΠΈΠ»ΠΎ Π·Π°ΠΊΠ»ΡŽΡ‡Π°Π΅Ρ‚ΡΡ Π² Ρ‚ΠΎΠΌ, Ρ‡Ρ‚ΠΎ ΠΏΡ€ΠΎΡ‚Π΅ΠΊΠ°ΡŽΡ‰ΠΈΠ΅ ΠΏΠΎ всСм ΠΏΡ€ΠΎΠ²ΠΎΠ΄Π½ΠΈΠΊΠ°ΠΌ Ρ‚ΠΎΠΊΠΈ ΠΏΡ€ΠΈ ΠΏΠΎΡΠ»Π΅Π΄ΠΎΠ²Π°Ρ‚Π΅Π»ΡŒΠ½ΠΎΠΌ соСдинСнии Ρ€Π°Π²Π½Ρ‹ ΠΌΠ΅ΠΆΠ΄Ρƒ собой:

    А для опрСдСлСния ΠΎΠ±Ρ‰Π΅Π³ΠΎ напряТСния ΠΏΡ€ΠΈ ΠΏΠΎΡΠ»Π΅Π΄ΠΎΠ²Π°Ρ‚Π΅Π»ΡŒΠ½ΠΎΠΌ соСдинСнии, напряТСния Π½Π° ΠΎΡ‚Π΄Π΅Π»ΡŒΠ½Ρ‹Ρ… элСмСнтах Π½Π΅ΠΎΠ±Ρ…ΠΎΠ΄ΠΈΠΌΠΎ ΠΏΡ€ΠΎΡΡƒΠΌΠΌΠΈΡ€ΠΎΠ²Π°Ρ‚ΡŒ:

    Π’ Ρ‚ΠΎ ΠΆΠ΅ врСмя, ΠΏΠΎ Π·Π°ΠΊΠΎΠ½Ρƒ Ома для напряТСний, сопротивлСний ΠΈ Ρ‚ΠΎΠΊΠΎΠ² Π² Π΄Π°Π½Π½ΠΎΠΉ Ρ†Π΅ΠΏΠΈ справСдливы ΡΠ»Π΅Π΄ΡƒΡŽΡ‰ΠΈΠ΅ ΡΠΎΠΎΡ‚Π½ΠΎΡˆΠ΅Π½ΠΈΡ:

    Π’ΠΎΠ³Π΄Π° для вычислСния ΠΎΠ±Ρ‰Π΅Π³ΠΎ напряТСния ΠΌΠΎΠΆΠ½ΠΎ Π±ΡƒΠ΄Π΅Ρ‚ ΠΈΡΠΏΠΎΠ»ΡŒΠ·ΠΎΠ²Π°Ρ‚ΡŒ ΡΠ»Π΅Π΄ΡƒΡŽΡ‰Π΅Π΅ Π²Ρ‹Ρ€Π°ΠΆΠ΅Π½ΠΈΠ΅:

    Но для ΠΎΠ±Ρ‰Π΅Π³ΠΎ напряТСниС Ρ‚Π°ΠΊΠΆΠ΅ справСдлив Π·Π°ΠΊΠΎΠ½ Ома:

    Π—Π΄Π΅ΡΡŒ – это ΠΎΠ±Ρ‰Π΅Π΅ сопротивлСниС Ρ†Π΅ΠΏΠΈ, ΠΊΠΎΡ‚ΠΎΡ€ΠΎΠ΅ исходя ΠΈΠ· Π΄Π²ΡƒΡ… Ρ„ΠΎΡ€ΠΌΡƒΠ» для ΠΎΠ±Ρ‰Π΅Π³ΠΎ напряТСния Ρ€Π°Π²Π½ΠΎ:

    Π’Π°ΠΊΠΈΠΌ ΠΎΠ±Ρ€Π°Π·ΠΎΠΌ, ΠΏΡ€ΠΈ ΠΏΠΎΡΠ»Π΅Π΄ΠΎΠ²Π°Ρ‚Π΅Π»ΡŒΠ½ΠΎΠΌ соСдинСнии рСзисторов ΠΎΠ±Ρ‰Π΅Π΅ сопротивлСниС Ρ†Π΅ΠΏΠΈ Π±ΡƒΠ΄Π΅Ρ‚ Ρ€Π°Π²Π½ΠΎ суммС сопротивлСний всСх ΠΏΡ€ΠΎΠ²ΠΎΠ΄Π½ΠΈΠΊΠΎΠ².

    НапримСр для ΡΠ»Π΅Π΄ΡƒΡŽΡ‰Π΅ΠΉ Ρ†Π΅ΠΏΠΈ:

    ΠžΠ±Ρ‰Π΅Π΅ сопротивлСниС Π±ΡƒΠ΄Π΅Ρ‚ Ρ€Π°Π²Π½ΠΎ:

    ΠšΠΎΠ»ΠΈΡ‡Π΅ΡΡ‚Π²ΠΎ элСмСнтов значСния Π½Π΅ ΠΈΠΌΠ΅Π΅Ρ‚, ΠΏΡ€Π°Π²ΠΈΠ»ΠΎ, ΠΏΠΎ ΠΊΠΎΡ‚ΠΎΡ€ΠΎΠΌΡƒ ΠΌΡ‹ опрСдСляСм ΠΎΠ±Ρ‰Π΅Π΅ сопротивлСниС Π±ΡƒΠ΄Π΅ΠΌ Ρ€Π°Π±ΠΎΡ‚Π°Ρ‚ΡŒ Π² любом случаС 🙂 А Ссли ΠΏΡ€ΠΈ ΠΏΠΎΡΠ»Π΅Π΄ΠΎΠ²Π°Ρ‚Π΅Π»ΡŒΠ½ΠΎΠΌ соСдинСнии всС сопротивлСния Ρ€Π°Π²Π½Ρ‹ (), Ρ‚ΠΎ ΠΎΠ±Ρ‰Π΅Π΅ сопротивлСниС Ρ†Π΅ΠΏΠΈ составит:

    Π² Π΄Π°Π½Π½ΠΎΠΉ Ρ„ΠΎΡ€ΠΌΡƒΠ»Π΅ Ρ€Π°Π²Π½ΠΎ количСству элСмСнтов Ρ†Π΅ΠΏΠΈ.

    Π‘ ΠΏΠΎΡΠ»Π΅Π΄ΠΎΠ²Π°Ρ‚Π΅Π»ΡŒΠ½Ρ‹ΠΌ соСдинСниСм рСзисторов ΠΌΡ‹ Ρ€Π°Π·ΠΎΠ±Ρ€Π°Π»ΠΈΡΡŒ, Π΄Π°Π²Π°ΠΉΡ‚Π΅ ΠΏΠ΅Ρ€Π΅ΠΉΠ΄Π΅ΠΌ ΠΊ ΠΏΠ°Ρ€Π°Π»Π»Π΅Π»ΡŒΠ½ΠΎΠΌΡƒ.

    ΠŸΠ°Ρ€Π°Π»Π»Π΅Π»ΡŒΠ½ΠΎΠ΅ соСдинСниС рСзисторов.

    ΠŸΡ€ΠΈ ΠΏΠ°Ρ€Π°Π»Π»Π΅Π»ΡŒΠ½ΠΎΠΌ соСдинСнии напряТСния Π½Π° ΠΏΡ€ΠΎΠ²ΠΎΠ΄Π½ΠΈΠΊΠ°Ρ… Ρ€Π°Π²Π½Ρ‹:

    А для Ρ‚ΠΎΠΊΠΎΠ² справСдливо ΡΠ»Π΅Π΄ΡƒΡŽΡ‰Π΅Π΅ Π²Ρ‹Ρ€Π°ΠΆΠ΅Π½ΠΈΠ΅:

    Π’ΠΎ Π΅ΡΡ‚ΡŒ ΠΎΠ±Ρ‰ΠΈΠΉ Ρ‚ΠΎΠΊ развСтвляСтся Π½Π° Π΄Π²Π΅ ΡΠΎΡΡ‚Π°Π²Π»ΡΡŽΡ‰ΠΈΠ΅, Π° Π΅Π³ΠΎ Π·Π½Π°Ρ‡Π΅Π½ΠΈΠ΅ Ρ€Π°Π²Π½ΠΎ суммС всСх ΡΠΎΡΡ‚Π°Π²Π»ΡΡŽΡ‰ΠΈΡ…. По Π·Π°ΠΊΠΎΠ½Ρƒ Ома:

    ΠŸΠΎΠ΄ΡΡ‚Π°Π²ΠΈΠΌ эти выраТСния Π² Ρ„ΠΎΡ€ΠΌΡƒΠ»Ρƒ ΠΎΠ±Ρ‰Π΅Π³ΠΎ Ρ‚ΠΎΠΊΠ°:

    А ΠΏΠΎ Π·Π°ΠΊΠΎΠ½Ρƒ Ома Ρ‚ΠΎΠΊ:

    ΠŸΡ€ΠΈΡ€Π°Π²Π½ΠΈΠ²Π°Π΅ΠΌ эти выраТСния ΠΈ ΠΏΠΎΠ»ΡƒΡ‡Π°Π΅ΠΌ Ρ„ΠΎΡ€ΠΌΡƒΠ»Ρƒ для ΠΎΠ±Ρ‰Π΅Π³ΠΎ сопротивлСния Ρ†Π΅ΠΏΠΈ:

    Π”Π°Π½Π½ΡƒΡŽ Ρ„ΠΎΡ€ΠΌΡƒΠ»Ρƒ ΠΌΠΎΠΆΠ½ΠΎ Π·Π°ΠΏΠΈΡΠ°Ρ‚ΡŒ ΠΈ нСсколько ΠΈΠ½Π°Ρ‡Π΅:

    Π’Π°ΠΊΠΈΠΌ ΠΎΠ±Ρ€Π°Π·ΠΎΠΌ, ΠΏΡ€ΠΈ ΠΏΠ°Ρ€Π°Π»Π»Π΅Π»ΡŒΠ½ΠΎΠΌ соСдинСнии ΠΏΡ€ΠΎΠ²ΠΎΠ΄Π½ΠΈΠΊΠΎΠ² Π²Π΅Π»ΠΈΡ‡ΠΈΠ½Π°, обратная ΠΎΠ±Ρ‰Π΅ΠΌΡƒ ΡΠΎΠΏΡ€ΠΎΡ‚ΠΈΠ²Π»Π΅Π½ΠΈΡŽ Ρ†Π΅ΠΏΠΈ, Ρ€Π°Π²Π½Π° суммС Π²Π΅Π»ΠΈΡ‡ΠΈΠ½, ΠΎΠ±Ρ€Π°Ρ‚Π½Ρ‹Ρ… сопротивлСниям ΠΏΠ°Ρ€Π°Π»Π»Π΅Π»ΡŒΠ½ΠΎ Π²ΠΊΠ»ΡŽΡ‡Π΅Π½Π½Ρ‹Ρ… ΠΏΡ€ΠΎΠ²ΠΎΠ΄Π½ΠΈΠΊΠΎΠ².

    Аналогичная ситуация Π±ΡƒΠ΄Π΅Ρ‚ Π½Π°Π±Π»ΡŽΠ΄Π°Ρ‚ΡŒΡΡ ΠΈ ΠΏΡ€ΠΈ большСм количСствС ΠΏΡ€ΠΎΠ²ΠΎΠ΄Π½ΠΈΠΊΠΎΠ², соСдинСнных ΠΏΠ°Ρ€Π°Π»Π»Π΅Π»ΡŒΠ½ΠΎ:

    БмСшанноС соСдинСниС рСзисторов.

    Помимо ΠΏΠ°Ρ€Π°Π»Π»Π΅Π»ΡŒΠ½ΠΎΠ³ΠΎ ΠΈ ΠΏΠΎΡΠ»Π΅Π΄ΠΎΠ²Π°Ρ‚Π΅Π»ΡŒΠ½ΠΎΠ³ΠΎ соСдинСний рСзисторов сущСствуСт Π΅Ρ‰Π΅ смСшанноС соСдинСниС. Из названия ΡƒΠΆΠ΅ понятно, Ρ‡Ρ‚ΠΎ ΠΏΡ€ΠΈ Ρ‚Π°ΠΊΠΎΠΌ соСдинСнии Π² Ρ†Π΅ΠΏΠΈ ΠΏΡ€ΠΈΡΡƒΡ‚ΡΡ‚Π²ΡƒΡŽΡ‚ рСзисторы, соСдинСнныС ΠΊΠ°ΠΊ ΠΏΠ°Ρ€Π°Π»Π»Π΅Π»ΡŒΠ½ΠΎ, Ρ‚Π°ΠΊ ΠΈ ΠΏΠΎΡΠ»Π΅Π΄ΠΎΠ²Π°Ρ‚Π΅Π»ΡŒΠ½ΠΎ. Π’ΠΎΡ‚ ΠΏΡ€ΠΈΠΌΠ΅Ρ€ Ρ‚Π°ΠΊΠΎΠΉ Ρ†Π΅ΠΏΠΈ:

    Π”Π°Π²Π°ΠΉΡ‚Π΅ рассчитаСм ΠΎΠ±Ρ‰Π΅Π΅ сопротивлСниС Ρ†Π΅ΠΏΠΈ. НачнСм с рСзисторов ΠΈ – ΠΎΠ½ΠΈ соСдинСны ΠΏΠ°Ρ€Π°Π»Π»Π΅Π»ΡŒΠ½ΠΎ. ΠœΡ‹ ΠΌΠΎΠΆΠ΅ΠΌ Ρ€Π°ΡΡΡ‡ΠΈΡ‚Π°Ρ‚ΡŒ ΠΎΠ±Ρ‰Π΅Π΅ сопротивлСниС для этих рСзисторов ΠΈ Π·Π°ΠΌΠ΅Π½ΠΈΡ‚ΡŒ ΠΈΡ… Π² схСмС ΠΎΠ΄Π½ΠΈΠΌ СдинствСнным рСзистором :

    Π’Π΅ΠΏΠ΅Ρ€ΡŒ Ρƒ нас ΠΎΠ±Ρ€Π°Π·ΠΎΠ²Π°Π»ΠΈΡΡŒ Π΄Π²Π΅ Π³Ρ€ΡƒΠΏΠΏΡ‹ ΠΏΠΎΡΠ»Π΅Π΄ΠΎΠ²Π°Ρ‚Π΅Π»ΡŒΠ½ΠΎ соСдинСнных рСзисторов:

    Π—Π°ΠΌΠ΅Π½ΠΈΠΌ эти Π΄Π²Π΅ Π³Ρ€ΡƒΠΏΠΏΡ‹ двумя рСзисторами, сопротивлСниС ΠΊΠΎΡ‚ΠΎΡ€Ρ‹Ρ… Ρ€Π°Π²Π½ΠΎ:

    Как Π²ΠΈΠ΄ΠΈΡ‚Π΅, схСма стала ΡƒΠΆΠ΅ совсСм простой ) Π—Π°ΠΌΠ΅Π½ΠΈΠΌ Π³Ρ€ΡƒΠΏΠΏΡƒ ΠΏΠ°Ρ€Π°Π»Π»Π΅Π»ΡŒΠ½ΠΎ соСдинСнных рСзисторов ΠΈ ΠΎΠ΄Π½ΠΈΠΌ рСзистором :

    И Π² ΠΈΡ‚ΠΎΠ³Π΅ Ρƒ нас Π½Π° схСмС ΠΎΡΡ‚Π°Π»ΠΎΡΡŒ Ρ‚ΠΎΠ»ΡŒΠΊΠΎ Π΄Π²Π° рСзистора соСдинСнных ΠΏΠΎΡΠ»Π΅Π΄ΠΎΠ²Π°Ρ‚Π΅Π»ΡŒΠ½ΠΎ:

    ΠžΠ±Ρ‰Π΅Π΅ сопротивлСниС Ρ†Π΅ΠΏΠΈ ΠΏΠΎΠ»ΡƒΡ‡ΠΈΠ»ΠΎΡΡŒ Ρ€Π°Π²Π½Ρ‹ΠΌ:

    Π’Π°ΠΊΠΈΠΌ Π²ΠΎΡ‚ ΠΎΠ±Ρ€Π°Π·ΠΎΠΌ достаточно большая схСма свСлась ΠΊ ΠΏΡ€ΠΎΡΡ‚Π΅ΠΉΡˆΠ΅ΠΌΡƒ ΠΏΠΎΡΠ»Π΅Π΄ΠΎΠ²Π°Ρ‚Π΅Π»ΡŒΠ½ΠΎΠΌΡƒ соСдинСнию Π΄Π²ΡƒΡ… рСзисторов 😉

    Π’ΡƒΡ‚ стоит ΠΎΡ‚ΠΌΠ΅Ρ‚ΠΈΡ‚ΡŒ, Ρ‡Ρ‚ΠΎ Π½Π΅ΠΊΠΎΡ‚ΠΎΡ€Ρ‹Π΅ схСмы Π½Π΅Π²ΠΎΠ·ΠΌΠΎΠΆΠ½ΠΎ Ρ‚Π°ΠΊ просто ΠΏΡ€Π΅ΠΎΠ±Ρ€Π°Π·ΠΎΠ²Π°Ρ‚ΡŒ ΠΈ ΠΎΠΏΡ€Π΅Π΄Π΅Π»ΠΈΡ‚ΡŒ ΠΎΠ±Ρ‰Π΅Π΅ сопротивлСниС – для Ρ‚Π°ΠΊΠΈΡ… схСм Π½ΡƒΠΆΠ½ΠΎ ΠΈΡΠΏΠΎΠ»ΡŒΠ·ΠΎΠ²Π°Ρ‚ΡŒ ΠΏΡ€Π°Π²ΠΈΠ»Π° ΠšΠΈΡ€Ρ…Π³ΠΎΡ„Π°, ΠΎ ΠΊΠΎΡ‚ΠΎΡ€Ρ‹Ρ… ΠΌΡ‹ ΠΎΠ±ΡΠ·Π°Ρ‚Π΅Π»ΡŒΠ½ΠΎ ΠΏΠΎΠ³ΠΎΠ²ΠΎΡ€ΠΈΠΌ Π² Π±ΡƒΠ΄ΡƒΡ‰ΠΈΡ… ΡΡ‚Π°Ρ‚ΡŒΡΡ…. А сСгодняшняя ΡΡ‚Π°Ρ‚ΡŒΡ Π½Π° этом подошла ΠΊ ΠΊΠΎΠ½Ρ†Ρƒ, Π΄ΠΎ скорых встрСч Π½Π° нашСм сайтС!

    Π‘ΠΎΠΏΡ€ΠΎΡ‚ΠΈΠ²Π»Π΅Π½ΠΈΠ΅ ΠΏΠΎΡΠ»Π΅Π΄ΠΎΠ²Π°Ρ‚Π΅Π»ΡŒΠ½ΠΎΠΉ Ρ†Π΅ΠΏΠΈ Ρ„ΠΎΡ€ΠΌΡƒΠ»Π° – ΠœΠΎΡ€ΡΠΊΠΎΠΉ Ρ„Π»ΠΎΡ‚

    ΠŸΠΎΡΠ»Π΅Π΄ΠΎΠ²Π°Ρ‚Π΅Π»ΡŒΠ½ΠΎΠ΅ соСдинСниС сопротивлСний

    Π’ΠΎΠ·ΡŒΠΌΠ΅ΠΌ Ρ‚Ρ€ΠΈ постоянных сопротивлСния R1, R2 ΠΈ R3 ΠΈ Π²ΠΊΠ»ΡŽΡ‡ΠΈΠΌ ΠΈΡ… Π² Ρ†Π΅ΠΏΡŒ Ρ‚Π°ΠΊ, Ρ‡Ρ‚ΠΎΠ±Ρ‹ ΠΊΠΎΠ½Π΅Ρ† ΠΏΠ΅Ρ€Π²ΠΎΠ³ΠΎ сопротивлСния R1 Π±Ρ‹Π» соСдинСн с Π½Π°Ρ‡Π°Π»ΠΎΠΌ Π²Ρ‚ΠΎΡ€ΠΎΠ³ΠΎ сопротивлСния R 2, ΠΊΠΎΠ½Π΅Ρ† Π²Ρ‚ΠΎΡ€ΠΎΠ³ΠΎ β€” с Π½Π°Ρ‡Π°Π»ΠΎΠΌ Ρ‚Ρ€Π΅Ρ‚ΡŒΠ΅Π³ΠΎ R 3, Π° ΠΊ Π½Π°Ρ‡Π°Π»Ρƒ ΠΏΠ΅Ρ€Π²ΠΎΠ³ΠΎ сопротивлСния ΠΈ ΠΊ ΠΊΠΎΠ½Ρ†Ρƒ Ρ‚Ρ€Π΅Ρ‚ΡŒΠ΅Π³ΠΎ ΠΏΠΎΠ΄Π²Π΅Π΄Π΅ΠΌ ΠΏΡ€ΠΎΠ²ΠΎΠ΄Π½ΠΈΠΊΠΈ ΠΎΡ‚ источника Ρ‚ΠΎΠΊΠ° (рис. 1 ).

    Π’Π°ΠΊΠΎΠ΅ соСдинСниС сопротивлСний называСтся ΠΏΠΎΡΠ»Π΅Π΄ΠΎΠ²Π°Ρ‚Π΅Π»ΡŒΠ½Ρ‹ΠΌ. ΠžΡ‡Π΅Π²ΠΈΠ΄Π½ΠΎ, Ρ‡Ρ‚ΠΎ Ρ‚ΠΎΠΊ Π² Ρ‚Π°ΠΊΠΎΠΉ Ρ†Π΅ΠΏΠΈ Π±ΡƒΠ΄Π΅Ρ‚ Π²ΠΎ всСх Π΅Π΅ Ρ‚ΠΎΡ‡ΠΊΠ°Ρ… ΠΎΠ΄ΠΈΠ½ ΠΈ Ρ‚ΠΎΡ‚ ΠΆΠ΅.

    Рис 1 . ΠŸΠΎΡΠ»Π΅Π΄ΠΎΠ²Π°Ρ‚Π΅Π»ΡŒΠ½ΠΎΠ΅ соСдинСниС сопротивлСний

    Как ΠΎΠΏΡ€Π΅Π΄Π΅Π»ΠΈΡ‚ΡŒ ΠΎΠ±Ρ‰Π΅Π΅ сопротивлСниС Ρ†Π΅ΠΏΠΈ, Ссли всС Π²ΠΊΠ»ΡŽΡ‡Π΅Π½Π½Ρ‹Π΅ Π² Π½Π΅Π΅ ΠΏΠΎΡΠ»Π΅Π΄ΠΎΠ²Π°Ρ‚Π΅Π»ΡŒΠ½ΠΎ сопротивлСния ΠΌΡ‹ ΡƒΠΆΠ΅ Π·Π½Π°Π΅ΠΌ? Π˜ΡΠΏΠΎΠ»ΡŒΠ·ΡƒΡ ΠΏΠΎΠ»ΠΎΠΆΠ΅Π½ΠΈΠ΅, Ρ‡Ρ‚ΠΎ напряТСниС U Π½Π° Π·Π°ΠΆΠΈΠΌΠ°Ρ… источника Ρ‚ΠΎΠΊΠ° Ρ€Π°Π²Π½ΠΎ суммС ΠΏΠ°Π΄Π΅Π½ΠΈΠΉ напряТСний Π½Π° участках Ρ†Π΅ΠΏΠΈ, ΠΌΡ‹ ΠΌΠΎΠΆΠ΅ΠΌ Π½Π°ΠΏΠΈΡΠ°Ρ‚ΡŒ:

    U1 = IR1 U2 = IR2 ΠΈ U3 = IR3

    IR = IR1 + IR2 + IR3

    ВынСся Π² ΠΏΡ€Π°Π²ΠΎΠΉ части равСнства I Π·Π° скобки, ΠΏΠΎΠ»ΡƒΡ‡ΠΈΠΌ IR = I(R1 + R2 + R3) .

    ПодСлив Ρ‚Π΅ΠΏΠ΅Ρ€ΡŒ ΠΎΠ±Π΅ части равСнства Π½Π° I , Π±ΡƒΠ΄Π΅ΠΌ ΠΎΠΊΠΎΠ½Ρ‡Π°Ρ‚Π΅Π»ΡŒΠ½ΠΎ ΠΈΠΌΠ΅Ρ‚ΡŒ R = R1 + R2 + R3

    Π’Π°ΠΊΠΈΠΌ ΠΎΠ±Ρ€Π°Π·ΠΎΠΌ, ΠΌΡ‹ ΠΏΡ€ΠΈΡˆΠ»ΠΈ ΠΊ Π²Ρ‹Π²ΠΎΠ΄Ρƒ, Ρ‡Ρ‚ΠΎ ΠΏΡ€ΠΈ ΠΏΠΎΡΠ»Π΅Π΄ΠΎΠ²Π°Ρ‚Π΅Π»ΡŒΠ½ΠΎΠΌ соСдинСнии сопротивлСний ΠΎΠ±Ρ‰Π΅Π΅ сопротивлСниС всСй Ρ†Π΅ΠΏΠΈ Ρ€Π°Π²Π½ΠΎ суммС сопротивлСний ΠΎΡ‚Π΄Π΅Π»ΡŒΠ½Ρ‹Ρ… участков.

    ΠŸΡ€ΠΎΠ²Π΅Ρ€ΠΈΠΌ этот Π²Ρ‹Π²ΠΎΠ΄ Π½Π° ΡΠ»Π΅Π΄ΡƒΡŽΡ‰Π΅ΠΌ ΠΏΡ€ΠΈΠΌΠ΅Ρ€Π΅. Π’ΠΎΠ·ΡŒΠΌΠ΅ΠΌ Ρ‚Ρ€ΠΈ постоянных сопротивлСния, Π²Π΅Π»ΠΈΡ‡ΠΈΠ½Ρ‹ ΠΊΠΎΡ‚ΠΎΡ€Ρ‹Ρ… извСстны (Π½Π°ΠΏΡ€ΠΈΠΌΠ΅Ρ€, R1 == 10 Ом, R 2 = 20 Ом ΠΈ R 3 = 50 Ом). Π‘ΠΎΠ΅Π΄ΠΈΠ½ΠΈΠΌ ΠΈΡ… ΠΏΠΎΡΠ»Π΅Π΄ΠΎΠ²Π°Ρ‚Π΅Π»ΡŒΠ½ΠΎ (рис. 2 ) ΠΈ ΠΏΠΎΠ΄ΠΊΠ»ΡŽΡ‡ΠΈΠΌ ΠΊ источнику Ρ‚ΠΎΠΊΠ°, Π­Π”Π‘ ΠΊΠΎΡ‚ΠΎΡ€ΠΎΠ³ΠΎ Ρ€Π°Π²Π½Π° 60 Π’ (Π²Π½ΡƒΡ‚Ρ€Π΅Π½Π½ΠΈΠΌ сопротивлСниСм источника Ρ‚ΠΎΠΊΠ° ΠΏΡ€Π΅Π½Π΅Π±Ρ€Π΅Π³Π°Π΅ΠΌ).

    Рис. 2. ΠŸΡ€ΠΈΠΌΠ΅Ρ€ ΠΏΠΎΡΠ»Π΅Π΄ΠΎΠ²Π°Ρ‚Π΅Π»ΡŒΠ½ΠΎΠ³ΠΎ соСдинСния Ρ‚Ρ€Π΅Ρ… сопротивлСний

    ΠŸΠΎΠ΄ΡΡ‡ΠΈΡ‚Π°Π΅ΠΌ, ΠΊΠ°ΠΊΠΈΠ΅ показания Π΄ΠΎΠ»ΠΆΠ½Ρ‹ Π΄Π°Ρ‚ΡŒ ΠΏΡ€ΠΈΠ±ΠΎΡ€Ρ‹, Π²ΠΊΠ»ΡŽΡ‡Π΅Π½Π½Ρ‹Π΅, ΠΊΠ°ΠΊ ΠΏΠΎΠΊΠ°Π·Π°Π½ΠΎ Π½Π° схСмС, Ссли Π·Π°ΠΌΠΊΠ½ΡƒΡ‚ΡŒ Ρ†Π΅ΠΏΡŒ. ΠžΠΏΡ€Π΅Π΄Π΅Π»ΠΈΠΌ внСшнСС сопротивлСниС Ρ†Π΅ΠΏΠΈ: R = 10 + 20 + 50 = 80 Ом.

    НайдСм Ρ‚ΠΎΠΊ Π² Ρ†Π΅ΠΏΠΈ ΠΏΠΎ Π·Π°ΠΊΠΎΠ½Ρƒ Ома: 60 / 80 = 0 ,75 А

    Зная Ρ‚ΠΎΠΊ Π² Ρ†Π΅ΠΏΠΈ ΠΈ сопротивлСния Π΅Π΅ участков, ΠΎΠΏΡ€Π΅Π΄Π΅Π»ΠΈΠΌ ΠΏΠ°Π΄Π΅Π½ΠΈΠ΅ напряТСния Π½Π° ΠΊΠ°ΠΆΠ΄ΠΎΠ΅ участкС Ρ†Π΅ΠΏΠΈ U 1 = 0,75 Ρ… 10 = 7,5 Π’, U 2 = 0,75 Ρ… 20=15 Π’, U3 = 0,75 Ρ… 50 = 37,5 Π’.

    Зная ΠΏΠ°Π΄Π΅Π½ΠΈΠ΅ напряТСний Π½Π° участках, ΠΎΠΏΡ€Π΅Π΄Π΅Π»ΠΈΠΌ ΠΎΠ±Ρ‰Π΅Π΅ ΠΏΠ°Π΄Π΅Π½ΠΈΠ΅ напряТСния Π²ΠΎ внСшнСй Ρ†Π΅ΠΏΠΈ, Ρ‚. Π΅. напряТСниС Π½Π° Π·Π°ΠΆΠΈΠΌΠ°Ρ… источника Ρ‚ΠΎΠΊΠ° U = 7,5+15 + 37,5 = 60 Π’.

    ΠœΡ‹ ΠΏΠΎΠ»ΡƒΡ‡ΠΈΠ»ΠΈ Ρ‚Π°ΠΊΠΈΠΌ ΠΎΠ±Ρ€Π°Π·ΠΎΠΌ, Ρ‡Ρ‚ΠΎ U = 60 Π’, Ρ‚. Π΅. Π½Π΅ΡΡƒΡ‰Π΅ΡΡ‚Π²ΡƒΡŽΡ‰Π΅Π΅ равСнство Π­Π”Π‘ источника Ρ‚ΠΎΠΊΠ° ΠΈ Π΅Π³ΠΎ напряТСния. ΠžΠ±ΡŠΡΡΠ½ΡΠ΅Ρ‚ΡΡ это Ρ‚Π΅ΠΌ, Ρ‡Ρ‚ΠΎ ΠΌΡ‹ ΠΏΡ€Π΅Π½Π΅Π±Ρ€Π΅Π³Π»ΠΈ Π²Π½ΡƒΡ‚Ρ€Π΅Π½Π½ΠΈΠΌ сопротивлСниСм источника Ρ‚ΠΎΠΊΠ°.

    Π—Π°ΠΌΠΊΠ½ΡƒΠ² Ρ‚Π΅ΠΏΠ΅Ρ€ΡŒ ΠΊΠ»ΡŽΡ‡ Π²Ρ‹ΠΊΠ»ΡŽΡ‡Π°Ρ‚Π΅Π»ΡŒ К, ΠΌΠΎΠΆΠ½ΠΎ ΡƒΠ±Π΅Π΄ΠΈΡ‚ΡŒΡΡ ΠΏΠΎ ΠΏΡ€ΠΈΠ±ΠΎΡ€Π°ΠΌ, Ρ‡Ρ‚ΠΎ наши подсчСты ΠΏΡ€ΠΈΠΌΠ΅Ρ€Π½ΠΎ Π²Π΅Ρ€Π½Ρ‹.

    ΠŸΠ°Ρ€Π°Π»Π»Π΅Π»ΡŒΠ½ΠΎΠ΅ соСдинСниС сопротивлСний

    Π’ΠΎΠ·ΡŒΠΌΠ΅ΠΌ Π΄Π²Π° постоянных сопротивлСния R1 ΠΈ R2 ΠΈ соСдиним ΠΈΡ… Ρ‚Π°ΠΊ, Ρ‡Ρ‚ΠΎΠ±Ρ‹ Π½Π°Ρ‡Π°Π»Π° этих сопротивлСний Π±Ρ‹Π»ΠΈ Π²ΠΊΠ»ΡŽΡ‡Π΅Π½Ρ‹ Π² ΠΎΠ΄Π½Ρƒ ΠΎΠ±Ρ‰ΡƒΡŽ Ρ‚ΠΎΡ‡ΠΊΡƒ Π°, Π° ΠΊΠΎΠ½Ρ†Ρ‹ β€” Π² Π΄Ρ€ΡƒΠ³ΡƒΡŽ ΠΎΠ±Ρ‰ΡƒΡŽ Ρ‚ΠΎΡ‡ΠΊΡƒ Π±. Π‘ΠΎΠ΅Π΄ΠΈΠ½ΠΈΠ² Π·Π°Ρ‚Π΅ΠΌ Ρ‚ΠΎΡ‡ΠΊΠΈ Π° ΠΈ Π± с источником Ρ‚ΠΎΠΊΠ°, ΠΏΠΎΠ»ΡƒΡ‡ΠΈΠΌ Π·Π°ΠΌΠΊΠ½ΡƒΡ‚ΡƒΡŽ ΡΠ»Π΅ΠΊΡ‚Ρ€ΠΈΡ‡Π΅ΡΠΊΡƒΡŽ Ρ†Π΅ΠΏΡŒ. Π’Π°ΠΊΠΎΠ΅ соСдинСниС сопротивлСний называСтся ΠΏΠ°Ρ€Π°Π»Π»Π΅Π»ΡŒΠ½Ρ‹ΠΌ соСдинСниСм.

    Рис 3. ΠŸΠ°Ρ€Π°Π»Π»Π΅Π»ΡŒΠ½ΠΎΠ΅ соСдинСниС сопротивлСний

    ΠŸΡ€ΠΎΡΠ»Π΅Π΄ΠΈΠΌ Ρ‚Π΅Ρ‡Π΅Π½ΠΈΠ΅ Ρ‚ΠΎΠΊΠ° Π² этой Ρ†Π΅ΠΏΠΈ. ΠžΡ‚ ΠΏΠΎΠ»ΠΎΠΆΠΈΡ‚Π΅Π»ΡŒΠ½ΠΎΠ³ΠΎ полюса источника Ρ‚ΠΎΠΊΠ° ΠΏΠΎ ΡΠΎΠ΅Π΄ΠΈΠ½ΠΈΡ‚Π΅Π»ΡŒΠ½ΠΎΠΌΡƒ ΠΏΡ€ΠΎΠ²ΠΎΠ΄Π½ΠΈΠΊΡƒ Ρ‚ΠΎΠΊ Π΄ΠΎΠΉΠ΄Π΅Ρ‚ Π΄ΠΎ Ρ‚ΠΎΡ‡ΠΊΠΈ Π°. Π’ Ρ‚ΠΎΡ‡ΠΊΠ΅ Π° ΠΎΠ½ развСтвится, Ρ‚Π°ΠΊ ΠΊΠ°ΠΊ здСсь сама Ρ†Π΅ΠΏΡŒ развСтвляСтся Π½Π° Π΄Π²Π΅ ΠΎΡ‚Π΄Π΅Π»ΡŒΠ½Ρ‹Π΅ Π²Π΅Ρ‚Π²ΠΈ: ΠΏΠ΅Ρ€Π²ΡƒΡŽ Π²Π΅Ρ‚Π²ΡŒ с сопротивлСниСм R1 ΠΈ Π²Ρ‚ΠΎΡ€ΡƒΡŽ β€” с сопротивлСниСм R2. ΠžΠ±ΠΎΠ·Π½Π°Ρ‡ΠΈΠΌ Ρ‚ΠΎΠΊΠΈ Π² этих вСтвях соотвСтствСнно Ρ‡Π΅Ρ€Π΅Π· I1 ΠΈ I 2. ΠšΠ°ΠΆΠ΄Ρ‹ΠΉ ΠΈΠ· этих Ρ‚ΠΎΠΊΠΎΠ² ΠΏΠΎΠΉΠ΄Π΅Ρ‚ ΠΏΠΎ своСй Π²Π΅Ρ‚Π²ΠΈ Π΄ΠΎ Ρ‚ΠΎΡ‡ΠΊΠΈ Π±. Π’ этой Ρ‚ΠΎΡ‡ΠΊΠ΅ ΠΏΡ€ΠΎΠΈΠ·ΠΎΠΉΠ΄Π΅Ρ‚ слияниС Ρ‚ΠΎΠΊΠΎΠ² Π² ΠΎΠ΄ΠΈΠ½ ΠΎΠ±Ρ‰ΠΈΠΉ Ρ‚ΠΎΠΊ, ΠΊΠΎΡ‚ΠΎΡ€Ρ‹ΠΉ ΠΈ ΠΏΡ€ΠΈΠ΄Π΅Ρ‚ ΠΊ ΠΎΡ‚Ρ€ΠΈΡ†Π°Ρ‚Π΅Π»ΡŒΠ½ΠΎΠΌΡƒ ΠΏΠΎΠ»ΡŽΡΡƒ источника Ρ‚ΠΎΠΊΠ°.

    Π’Π°ΠΊΠΈΠΌ ΠΎΠ±Ρ€Π°Π·ΠΎΠΌ, ΠΏΡ€ΠΈ ΠΏΠ°Ρ€Π°Π»Π»Π΅Π»ΡŒΠ½ΠΎΠΌ соСдинСнии сопротивлСний получаСтся развСтвлСнная Ρ†Π΅ΠΏΡŒ. ΠŸΠΎΡΠΌΠΎΡ‚Ρ€ΠΈΠΌ, ΠΊΠ°ΠΊΠΎΠ΅ ΠΆΠ΅ Π±ΡƒΠ΄Π΅Ρ‚ ΡΠΎΠΎΡ‚Π½ΠΎΡˆΠ΅Π½ΠΈΠ΅ ΠΌΠ΅ΠΆΠ΄Ρƒ Ρ‚ΠΎΠΊΠ°ΠΌΠΈ Π² составлСнной Π½Π°ΠΌΠΈ Ρ†Π΅ΠΏΠΈ.

    Π’ΠΊΠ»ΡŽΡ‡ΠΈΠΌ Π°ΠΌΠΏΠ΅Ρ€ΠΌΠ΅Ρ‚Ρ€ ΠΌΠ΅ΠΆΠ΄Ρƒ ΠΏΠΎΠ»ΠΎΠΆΠΈΡ‚Π΅Π»ΡŒΠ½Ρ‹ΠΌ полюсом источника Ρ‚ΠΎΠΊΠ° (+) ΠΈ Ρ‚ΠΎΡ‡ΠΊΠΎΠΉ Π° ΠΈ Π·Π°ΠΌΠ΅Ρ‚ΠΈΠΌ Π΅Π³ΠΎ показания. Π’ΠΊΠ»ΡŽΡ‡ΠΈΠ² Π·Π°Ρ‚Π΅ΠΌ Π°ΠΌΠΏΠ΅Ρ€ΠΌΠ΅Ρ‚Ρ€ (ΠΏΠΎΠΊΠ°Π·Π°Π½Π½Ρ‹ΠΉ Β«Π° рисункС ΠΏΡƒΠ½ΠΊΡ‚ΠΈΡ€ΠΎΠΌ) Π² ΠΏΡ€ΠΎΠ²ΠΎΠ΄, ΡΠΎΠ΅Π΄ΠΈΠ½ΡΡŽΡ‰ΠΈΠΉ Ρ‚ΠΎΡ‡ΠΊΡƒ Π± с ΠΎΡ‚Ρ€ΠΈΡ†Π°Ρ‚Π΅Π»ΡŒΠ½Ρ‹ΠΌ полюсом источника Ρ‚ΠΎΠΊΠ° (β€”), Π·Π°ΠΌΠ΅Ρ‚ΠΈΠΌ, Ρ‡Ρ‚ΠΎ ΠΏΡ€ΠΈΠ±ΠΎΡ€ ΠΏΠΎΠΊΠ°ΠΆΠ΅Ρ‚ Ρ‚Ρƒ ΠΆΠ΅ Π²Π΅Π»ΠΈΡ‡ΠΈΠ½Ρƒ силы Ρ‚ΠΎΠΊΠ°.

    Π—Π½Π°Ρ‡ΠΈΡ‚, сила Ρ‚ΠΎΠΊΠ° Π² Ρ†Π΅ΠΏΠΈ Π΄ΠΎ Π΅Π΅ развСтвлСния (Π΄ΠΎ Ρ‚ΠΎΡ‡ΠΊΠΈ Π°) Ρ€Π°Π²Π½Π° силС Ρ‚ΠΎΠΊΠ° послС развСтвлСния Ρ†Π΅ΠΏΠΈ (послС Ρ‚ΠΎΡ‡ΠΊΠΈ Π±).

    Π‘ΡƒΠ΄Π΅ΠΌ Ρ‚Π΅ΠΏΠ΅Ρ€ΡŒ Π²ΠΊΠ»ΡŽΡ‡Π°Ρ‚ΡŒ Π°ΠΌΠΏΠ΅Ρ€ΠΌΠ΅Ρ‚Ρ€ ΠΏΠΎΠΎΡ‡Π΅Ρ€Π΅Π΄Π½ΠΎ Π² ΠΊΠ°ΠΆΠ΄ΡƒΡŽ Π²Π΅Ρ‚Π²ΡŒ Ρ†Π΅ΠΏΠΈ, запоминая показания ΠΏΡ€ΠΈΠ±ΠΎΡ€Π°. ΠŸΡƒΡΡ‚ΡŒ Π² ΠΏΠ΅Ρ€Π²ΠΎΠΉ Π²Π΅Ρ‚Π²ΠΈ Π°ΠΌΠΏΠ΅Ρ€ΠΌΠ΅Ρ‚Ρ€ ΠΏΠΎΠΊΠ°ΠΆΠ΅Ρ‚ силу Ρ‚ΠΎΠΊΠ° I1 , Π° Π²ΠΎ Π²Ρ‚ΠΎΡ€ΠΎΠΉ β€” I 2. Π‘Π»ΠΎΠΆΠΈΠ² эти Π΄Π²Π° показания Π°ΠΌΠΏΠ΅Ρ€ΠΌΠ΅Ρ‚Ρ€Π°, ΠΌΡ‹ ΠΏΠΎΠ»ΡƒΡ‡ΠΈΠΌ суммарный Ρ‚ΠΎΠΊ, ΠΏΠΎ Π²Π΅Π»ΠΈΡ‡ΠΈΠ½Π΅ Ρ€Π°Π²Π½Ρ‹ΠΉ Ρ‚ΠΎΠΊΡƒ I Π΄ΠΎ развСтвлСния (Π΄ΠΎ Ρ‚ΠΎΡ‡ΠΊΠΈ Π°).

    Π‘Π»Π΅Π΄ΠΎΠ²Π°Ρ‚Π΅Π»ΡŒΠ½ΠΎ, сила Ρ‚ΠΎΠΊΠ°, ΠΏΡ€ΠΎΡ‚Π΅ΠΊΠ°ΡŽΡ‰Π΅Π³ΠΎ Π΄ΠΎ Ρ‚ΠΎΡ‡ΠΊΠΈ развСтвлСния, Ρ€Π°Π²Π½Π° суммС сил Ρ‚ΠΎΠΊΠΎΠ², ΡƒΡ‚Π΅ΠΊΠ°ΡŽΡ‰ΠΈΡ… ΠΎΡ‚ этой Ρ‚ΠΎΡ‡ΠΊΠΈ. I = I1 + I2 ВыраТая это Ρ„ΠΎΡ€ΠΌΡƒΠ»ΠΎΠΉ, ΠΏΠΎΠ»ΡƒΡ‡ΠΈΠΌ

    Π­Ρ‚ΠΎ ΡΠΎΠΎΡ‚Π½ΠΎΡˆΠ΅Π½ΠΈΠ΅, ΠΈΠΌΠ΅ΡŽΡ‰Π΅Π΅ большоС практичСскоС Π·Π½Π°Ρ‡Π΅Π½ΠΈΠ΅, носит Π½Π°Π·Π²Π°Π½ΠΈΠ΅ Π·Π°ΠΊΠΎΠ½Π° Ρ€Π°Π·Π²Π΅Ρ‚Π²Π»Π΅Π½Π½ΠΎΠΉ Ρ†Π΅ΠΏΠΈ .

    Рассмотрим Ρ‚Π΅ΠΏΠ΅Ρ€ΡŒ, ΠΊΠ°ΠΊΠΎΠ²ΠΎ Π±ΡƒΠ΄Π΅Ρ‚ ΡΠΎΠΎΡ‚Π½ΠΎΡˆΠ΅Π½ΠΈΠ΅ ΠΌΠ΅ΠΆΠ΄Ρƒ Ρ‚ΠΎΠΊΠ°ΠΌΠΈ Π² вСтвях.

    Π’ΠΊΠ»ΡŽΡ‡ΠΈΠΌ ΠΌΠ΅ΠΆΠ΄Ρƒ Ρ‚ΠΎΡ‡ΠΊΠ°ΠΌΠΈ Π° ΠΈ Π± Π²ΠΎΠ»ΡŒΡ‚ΠΌΠ΅Ρ‚Ρ€ ΠΈ посмотрим, Ρ‡Ρ‚ΠΎ ΠΎΠ½ Π½Π°ΠΌ ΠΏΠΎΠΊΠ°ΠΆΠ΅Ρ‚. Π’ΠΎ-ΠΏΠ΅Ρ€Π²Ρ‹Ρ…, Π²ΠΎΠ»ΡŒΡ‚ΠΌΠ΅Ρ‚Ρ€ ΠΏΠΎΠΊΠ°ΠΆΠ΅Ρ‚ напряТСниС источника Ρ‚ΠΎΠΊΠ°, Ρ‚Π°ΠΊ ΠΊΠ°ΠΊ ΠΎΠ½ ΠΏΠΎΠ΄ΠΊΠ»ΡŽΡ‡Π΅Π½, ΠΊΠ°ΠΊ это Π²ΠΈΠ΄Π½ΠΎ ΠΈΠ· рис. 3 , нСпосрСдствСнно ΠΊ Π·Π°ΠΆΠΈΠΌΠ°ΠΌ источника Ρ‚ΠΎΠΊΠ°. Π’ΠΎ-Π²Ρ‚ΠΎΡ€Ρ‹Ρ…, Π²ΠΎΠ»ΡŒΡ‚ΠΌΠ΅Ρ‚Ρ€ ΠΏΠΎΠΊΠ°ΠΆΠ΅Ρ‚ падСния напряТСний U1 ΠΈ U2 Π½Π° сопротивлСниях R 1 ΠΈ R2, Ρ‚Π°ΠΊ ΠΊΠ°ΠΊ ΠΎΠ½ соСдинСн с Π½Π°Ρ‡Π°Π»ΠΎΠΌ ΠΈ ΠΊΠΎΠ½Ρ†ΠΎΠΌ ΠΊΠ°ΠΆΠ΄ΠΎΠ³ΠΎ сопротивлСния.

    Π‘Π»Π΅Π΄ΠΎΠ²Π°Ρ‚Π΅Π»ΡŒΠ½ΠΎ, ΠΏΡ€ΠΈ ΠΏΠ°Ρ€Π°Π»Π»Π΅Π»ΡŒΠ½ΠΎΠΌ соСдинСнии сопротивлСний напряТСниС Π½Π° Π·Π°ΠΆΠΈΠΌΠ°Ρ… источника Ρ‚ΠΎΠΊΠ° Ρ€Π°Π²Π½ΠΎ падСнию напряТСния Π½Π° ΠΊΠ°ΠΆΠ΄ΠΎΠΌ сопротивлСнии.

    Π­Ρ‚ΠΎ Π΄Π°Π΅Ρ‚ Π½Π°ΠΌ ΠΏΡ€Π°Π²ΠΎ Π½Π°ΠΏΠΈΡΠ°Ρ‚ΡŒ, Ρ‡Ρ‚ΠΎ U = U1 = U2 ,

    Π³Π΄Π΅ U β€” напряТСниС Π½Π° Π·Π°ΠΆΠΈΠΌΠ°Ρ… источника Ρ‚ΠΎΠΊΠ°; U 1 β€” ΠΏΠ°Π΄Π΅Π½ΠΈΠ΅ напряТСния Π½Π° сопротивлСнии R 1 , U2 β€” ΠΏΠ°Π΄Π΅Π½ΠΈΠ΅ напряТСния Π½Π° сопротивлСнии R2. Вспомним, Ρ‡Ρ‚ΠΎ ΠΏΠ°Π΄Π΅Π½ΠΈΠ΅ напряТСния Π½Π° участкС Ρ†Π΅ΠΏΠΈ числСнно Ρ€Π°Π²Π½ΠΎ ΠΏΡ€ΠΎΠΈΠ·Π²Π΅Π΄Π΅Π½ΠΈΡŽ силы Ρ‚ΠΎΠΊΠ°, ΠΏΡ€ΠΎΡ‚Π΅ΠΊΠ°ΡŽΡ‰Π΅Π³ΠΎ Ρ‡Π΅Ρ€Π΅Π· этот участок, Π½Π° сопротивлСниС участка U = IR .

    ΠŸΠΎΡΡ‚ΠΎΠΌΡƒ для ΠΊΠ°ΠΆΠ΄ΠΎΠΉ Π²Π΅Ρ‚Π²ΠΈ ΠΌΠΎΠΆΠ½ΠΎ Π½Π°ΠΏΠΈΡΠ°Ρ‚ΡŒ: U1 = I1R1 ΠΈ U2 = I2R2 , Π½ΠΎ Ρ‚Π°ΠΊ ΠΊΠ°ΠΊ U 1 = U2, Ρ‚ΠΎ ΠΈ I1R1 = I2R2 .

    ΠŸΡ€ΠΈΠΌΠ΅Π½ΡΡ ΠΊ этому Π²Ρ‹Ρ€Π°ΠΆΠ΅Π½ΠΈΡŽ ΠΏΡ€Π°Π²ΠΈΠ»ΠΎ ΠΏΡ€ΠΎΠΏΠΎΡ€Ρ†ΠΈΠΈ, ΠΏΠΎΠ»ΡƒΡ‡ΠΈΠΌ I1/ I2 = U2 / U1 Ρ‚. Π΅. Ρ‚ΠΎΠΊ Π² ΠΏΠ΅Ρ€Π²ΠΎΠΉ Π²Π΅Ρ‚Π²ΠΈ Π±ΡƒΠ΄Π΅Ρ‚ Π²ΠΎ ΡΡ‚ΠΎΠ»ΡŒΠΊΠΎ Ρ€Π°Π· большС (ΠΈΠ»ΠΈ мСньшС) Ρ‚ΠΎΠΊΠ° Π²ΠΎ Π²Ρ‚ΠΎΡ€ΠΎΠΉ Π²Π΅Ρ‚Π²ΠΈ, Π²ΠΎ сколько Ρ€Π°Π· сопротивлСниС ΠΏΠ΅Ρ€Π²ΠΎΠΉ Π²Π΅Ρ‚Π²ΠΈ мСньшС (ΠΈΠ»ΠΈ большС) сопротивлСния Π²Ρ‚ΠΎΡ€ΠΎΠΉ Π²Π΅Ρ‚Π²ΠΈ.

    Π˜Ρ‚Π°ΠΊ, ΠΌΡ‹ ΠΏΡ€ΠΈΡˆΠ»ΠΈ ΠΊ Π²Π°ΠΆΠ½ΠΎΠΌΡƒ Π²Ρ‹Π²ΠΎΠ΄Ρƒ, Π·Π°ΠΊΠ»ΡŽΡ‡Π°ΡŽΡ‰Π΅ΠΌΡƒΡΡ Π² Ρ‚ΠΎΠΌ, Ρ‡Ρ‚ΠΎ ΠΏΡ€ΠΈ ΠΏΠ°Ρ€Π°Π»Π»Π΅Π»ΡŒΠ½ΠΎΠΌ соСдинСнии сопротивлСний ΠΎΠ±Ρ‰ΠΈΠΉ Ρ‚ΠΎΠΊ Ρ†Π΅ΠΏΠΈ развСтвляСтся Π½Π° Ρ‚ΠΎΠΊΠΈ, ΠΎΠ±Ρ€Π°Ρ‚Π½ΠΎ ΠΏΡ€ΠΎΠΏΠΎΡ€Ρ†ΠΈΠΎΠ½Π°Π»ΡŒΠ½Ρ‹Π΅ Π²Π΅Π»ΠΈΡ‡ΠΈΠ½Π°ΠΌ сопротивлСнии ΠΏΠ°Ρ€Π°Π»Π»Π΅Π»ΡŒΠ½Ρ‹Ρ… Π²Π΅Ρ‚Π²Π΅ΠΉ. Π˜Π½Π°Ρ‡Π΅ говоря, Ρ‡Π΅ΠΌ большС сопротивлСниС Π²Π΅Ρ‚Π²ΠΈ, Ρ‚Π΅ΠΌ мСньший Ρ‚ΠΎΠΊ ΠΏΠΎΡ‚Π΅Ρ‡Π΅Ρ‚ Ρ‡Π΅Ρ€Π΅Π· Π½Π΅Π΅, ΠΈ, Π½Π°ΠΎΠ±ΠΎΡ€ΠΎΡ‚, Ρ‡Π΅ΠΌ мСньшС сопротивлСниС Π²Π΅Ρ‚Π²ΠΈ, Ρ‚Π΅ΠΌ больший Ρ‚ΠΎΠΊ ΠΏΠΎΡ‚Π΅Ρ‡Π΅Ρ‚ Ρ‡Π΅Ρ€Π΅Π· эту Π²Π΅Ρ‚Π²ΡŒ.

    УбСдимся Π² ΠΏΡ€Π°Π²ΠΈΠ»ΡŒΠ½ΠΎΡΡ‚ΠΈ этой зависимости Π½Π° ΡΠ»Π΅Π΄ΡƒΡŽΡ‰Π΅ΠΌ ΠΏΡ€ΠΈΠΌΠ΅Ρ€Π΅. Π‘ΠΎΠ±Π΅Ρ€Π΅ΠΌ схСму, ΡΠΎΡΡ‚ΠΎΡΡ‰ΡƒΡŽ ΠΈΠ· Π΄Π²ΡƒΡ… ΠΏΠ°Ρ€Π°Π»Π»Π΅Π»ΡŒΠ½ΠΎ соСдинСнных сопротивлСний R1 ΠΈ R 2, ΠΏΠΎΠ΄ΠΊΠ»ΡŽΡ‡Π΅Π½Π½Ρ‹Ρ… ΠΊ источнику Ρ‚ΠΎΠΊΠ°. ΠŸΡƒΡΡ‚ΡŒ R1 = 10 Ом, R2 = 20 Ом ΠΈ U = 3 Π’.

    ΠŸΠΎΠ΄ΡΡ‡ΠΈΡ‚Π°Π΅ΠΌ сначала, Ρ‡Ρ‚ΠΎ ΠΏΠΎΠΊΠ°ΠΆΠ΅Ρ‚ Π½Π°ΠΌ Π°ΠΌΠΏΠ΅Ρ€ΠΌΠ΅Ρ‚Ρ€, Π²ΠΊΠ»ΡŽΡ‡Π΅Π½Π½Ρ‹ΠΉ Π² ΠΊΠ°ΠΆΠ΄ΡƒΡŽ Π²Π΅Ρ‚Π²ΡŒ:

    I1 = U / R1 = 3 / 10 = 0 ,3 А = 300 мА

    I 2 = U / R 2 = 3 / 20 = 0,15 А = 150 мА

    ΠžΠ±Ρ‰ΠΈΠΉ Ρ‚ΠΎΠΊ Π² Ρ†Π΅ΠΏΠΈ I = I1 + I2 = 300 + 150 = 450 мА

    ΠŸΡ€ΠΎΠ΄Π΅Π»Π°Π½Π½Ρ‹ΠΉ Π½Π°ΠΌΠΈ расчСт ΠΏΠΎΠ΄Ρ‚Π²Π΅Ρ€ΠΆΠ΄Π°Π΅Ρ‚, Ρ‡Ρ‚ΠΎ ΠΏΡ€ΠΈ ΠΏΠ°Ρ€Π°Π»Π»Π΅Π»ΡŒΠ½ΠΎΠΌ соСдинСнии сопротивлСний Ρ‚ΠΎΠΊ Π² Ρ†Π΅ΠΏΠΈ развСтвляСтся ΠΎΠ±Ρ€Π°Ρ‚Π½ΠΎ ΠΏΡ€ΠΎΠΏΠΎΡ€Ρ†ΠΈΠΎΠ½Π°Π»ΡŒΠ½ΠΎ сопротивлСниям.

    Π”Π΅ΠΉΡΡ‚Π²ΠΈΡ‚Π΅Π»ΡŒΠ½ΠΎ, R1 == 10 Ом Π²Π΄Π²ΠΎΠ΅ мСньшС R 2 = 20 Ом, ΠΏΡ€ΠΈ этом I1 = 300 мА Π²Π΄Π²ΠΎΠ΅ большС I2 = 150 мА. ΠžΠ±Ρ‰ΠΈΠΉ Ρ‚ΠΎΠΊ Π² Ρ†Π΅ΠΏΠΈ I = 450 мА развСтвился Π½Π° Π΄Π²Π΅ части Ρ‚Π°ΠΊ, Ρ‡Ρ‚ΠΎ большая Π΅Π³ΠΎ Ρ‡Π°ΡΡ‚ΡŒ ( I1 = 300 мА) пошла Ρ‡Π΅Ρ€Π΅Π· мСньшСС сопротивлСниС ( R1 = 10 Ом), Π° мСньшая Ρ‡Π°ΡΡ‚ΡŒ ( R2 = 150 мА) β€” Ρ‡Π΅Ρ€Π΅Π· большСС сопротивлСниС ( R 2 = 20 Ом).

    Π’Π°ΠΊΠΎΠ΅ Ρ€Π°Π·Π²Π΅Ρ‚Π²Π»Π΅Π½ΠΈΠ΅ Ρ‚ΠΎΠΊΠ° Π² ΠΏΠ°Ρ€Π°Π»Π»Π΅Π»ΡŒΠ½Ρ‹Ρ… вСтвях сходно с Ρ‚Π΅Ρ‡Π΅Π½ΠΈΠ΅ΠΌ Тидкости ΠΏΠΎ Ρ‚Ρ€ΡƒΠ±Π°ΠΌ. ΠŸΡ€Π΅Π΄ΡΡ‚Π°Π²ΡŒΡ‚Π΅ сСбС Ρ‚Ρ€ΡƒΠ±Ρƒ А, которая Π² ΠΊΠ°ΠΊΠΎΠΌ-Ρ‚ΠΎ мСстС развСтвляСтся Π½Π° Π΄Π²Π΅ Ρ‚Ρ€ΡƒΠ±Ρ‹ Π‘ ΠΈ Π’ Ρ€Π°Π·Π»ΠΈΡ‡Π½ΠΎΠ³ΠΎ Π΄ΠΈΠ°ΠΌΠ΅Ρ‚Ρ€Π° (рис. 4). Π’Π°ΠΊ ΠΊΠ°ΠΊ Π΄ΠΈΠ°ΠΌΠ΅Ρ‚Ρ€ Ρ‚Ρ€ΡƒΠ±Ρ‹ Π‘ большС Π΄ΠΈΠ°ΠΌΠ΅Ρ‚Ρ€Π° Ρ‚Ρ€ΡƒΠ±ΠΎΠΊ Π’, Ρ‚ΠΎ Ρ‡Π΅Ρ€Π΅Π· Ρ‚Ρ€ΡƒΠ±Ρƒ Π‘ Π² ΠΎΠ΄Π½ΠΎ ΠΈ Ρ‚ΠΎ ΠΆΠ΅ врСмя ΠΏΡ€ΠΎΠΉΠ΄Π΅Ρ‚ большС Π²ΠΎΠ΄Ρ‹, Ρ‡Π΅ΠΌ Ρ‡Π΅Ρ€Π΅Π· Ρ‚Ρ€ΡƒΠ±Ρƒ Π’, которая ΠΎΠΊΠ°Π·Ρ‹Π²Π°Π΅Ρ‚ ΠΏΠΎΡ‚ΠΎΠΊΡƒ Π²ΠΎΠ΄Ρ‹ большСС сопротивлСниС.

    Рис. 4 . Π§Π΅Ρ€Π΅Π· Ρ‚ΠΎΠ½ΠΊΡƒΡŽ Ρ‚Ρ€ΡƒΠ±Ρƒ Π² ΠΎΠ΄ΠΈΠ½ ΠΈ Ρ‚ΠΎΡ‚ ΠΆΠ΅ ΠΏΡ€ΠΎΠΌΠ΅ΠΆΡƒΡ‚ΠΎΠΊ Π²Ρ€Π΅ΠΌΠ΅Π½ΠΈ ΠΏΡ€ΠΎΠΉΠ΄Π΅Ρ‚ Π²ΠΎΠ΄Ρ‹ мСньшС, Ρ‡Π΅ΠΌ Ρ‡Π΅Ρ€Π΅Π· Ρ‚ΠΎΠ»ΡΡ‚ΡƒΡŽ

    Рассмотрим Ρ‚Π΅ΠΏΠ΅Ρ€ΡŒ, Ρ‡Π΅ΠΌΡƒ Π±ΡƒΠ΄Π΅Ρ‚ Ρ€Π°Π²Π½ΠΎ ΠΎΠ±Ρ‰Π΅Π΅ сопротивлСниС внСшнСй Ρ†Π΅ΠΏΠΈ, состоящСй ΠΈΠ· Π΄Π²ΡƒΡ… ΠΏΠ°Ρ€Π°Π»Π»Π΅Π»ΡŒΠ½ΠΎ соСдинСнных сопротивлСний.

    Под этим ΠΎΠ±Ρ‰ΠΈΠΌ сопротивлСниСм внСшнСй Ρ†Π΅ΠΏΠΈ Π½Π°Π΄ΠΎ ΠΏΠΎΠ½ΠΈΠΌΠ°Ρ‚ΡŒ Ρ‚Π°ΠΊΠΎΠ΅ сопротивлСниС, ΠΊΠΎΡ‚ΠΎΡ€Ρ‹ΠΌ ΠΌΠΎΠΆΠ½ΠΎ Π±Ρ‹Π»ΠΎ Π±Ρ‹ Π·Π°ΠΌΠ΅Π½ΠΈΡ‚ΡŒ ΠΏΡ€ΠΈ Π΄Π°Π½Π½ΠΎΠΌ напряТСнии Ρ†Π΅ΠΏΠΈ ΠΎΠ±Π° ΠΏΠ°Ρ€Π°Π»Π»Π΅Π»ΡŒΠ½ΠΎ Π²ΠΊΠ»ΡŽΡ‡Π΅Π½Π½Ρ‹Ρ… сопротивлСния, Π½Π΅ измСняя ΠΏΡ€ΠΈ этом Ρ‚ΠΎΠΊΠ° Π΄ΠΎ развСтвлСния. Π’Π°ΠΊΠΎΠ΅ сопротивлСниС называСтся эквивалСнтным сопротивлСниСм.

    ВСрнСмся ΠΊ Ρ†Π΅ΠΏΠΈ, ΠΏΠΎΠΊΠ°Π·Π°Π½Π½ΠΎΠΉ Π½Π° рис. 3, ΠΈ посмотрим, Ρ‡Π΅ΠΌΡƒ Π±ΡƒΠ΄Π΅Ρ‚ Ρ€Π°Π²Π½ΠΎ эквивалСнтноС сопротивлСниС Π΄Π²ΡƒΡ… ΠΏΠ°Ρ€Π°Π»Π»Π΅Π»ΡŒΠ½ΠΎ соСдинСнных сопротивлСний. ΠŸΡ€ΠΈΠΌΠ΅Π½ΡΡ ΠΊ этой Ρ†Π΅ΠΏΠΈ Π·Π°ΠΊΠΎΠ½ Ома, ΠΌΡ‹ ΠΌΠΎΠΆΠ΅ΠΌ Π½Π°ΠΏΠΈΡΠ°Ρ‚ΡŒ: I = U/R , Π³Π΄Π΅ I β€” Ρ‚ΠΎΠΊ Π²ΠΎ внСшнСй Ρ†Π΅ΠΏΠΈ (Π΄ΠΎ Ρ‚ΠΎΡ‡ΠΊΠΈ развСтвлСния), U β€” напряТСниС внСшнСй Ρ†Π΅ΠΏΠΈ, R β€” сопротивлСниС внСшнСй Ρ†Π΅ΠΏΠΈ, Ρ‚. Π΅. эквивалСнтноС сопротивлСниС.

    Π’ΠΎΡ‡Π½ΠΎ Ρ‚Π°ΠΊ ΠΆΠ΅ для ΠΊΠ°ΠΆΠ΄ΠΎΠΉ Π²Π΅Ρ‚Π²ΠΈ I1 = U1 / R1 , I2 = U2 / R2 , Π³Π΄Π΅ I1 ΠΈ I 2 β€” Ρ‚ΠΎΠΊΠΈ Π² вСтвях; U 1 ΠΈ U2 β€” напряТСниС Π½Π° вСтвях; R1 ΠΈ R2 β€” сопротивлСния Π²Π΅Ρ‚Π²Π΅ΠΉ.

    По Π·Π°ΠΊΠΎΠ½Ρƒ Ρ€Π°Π·Π²Π΅Ρ‚Π²Π»Π΅Π½Π½ΠΎΠΉ Ρ†Π΅ΠΏΠΈ: I = I1 + I2

    ΠŸΠΎΠ΄ΡΡ‚Π°Π²Π»ΡΡ значСния Ρ‚ΠΎΠΊΠΎΠ², ΠΏΠΎΠ»ΡƒΡ‡ΠΈΠΌ U / R = U1 / R1 + U2 / R2

    Π’Π°ΠΊ ΠΊΠ°ΠΊ ΠΏΡ€ΠΈ ΠΏΠ°Ρ€Π°Π»Π»Π΅Π»ΡŒΠ½ΠΎΠΌ соСдинСнии U = U1 = U2 , Ρ‚ΠΎ ΠΌΠΎΠΆΠ΅ΠΌ Π½Π°ΠΏΠΈΡΠ°Ρ‚ΡŒ U / R = U / R1 + U / R2

    ВынСся U Π² ΠΏΡ€Π°Π²ΠΎΠΉ части равСнства Π·Π° скобки, ΠΏΠΎΠ»ΡƒΡ‡ΠΈΠΌ U / R = U (1 / R1 + 1 / R2 )

    Π Π°Π·Π΄Π΅Π»ΠΈΠ² Ρ‚Π΅ΠΏΠ΅Ρ€ΡŒ ΠΎΠ±Π΅ части равСнства Π½Π° U , Π±ΡƒΠ΄Π΅ΠΌ ΠΎΠΊΠΎΠ½Ρ‡Π°Ρ‚Π΅Π»ΡŒΠ½ΠΎ ΠΈΠΌΠ΅Ρ‚ΡŒ 1 / R = 1 / R1 + 1 / R2

    Помня, Ρ‡Ρ‚ΠΎ ΠΏΡ€ΠΎΠ²ΠΎΠ΄ΠΈΠΌΠΎΡΡ‚ΡŒΡŽ называСтся Π²Π΅Π»ΠΈΡ‡ΠΈΠ½Π°, обратная ΡΠΎΠΏΡ€ΠΎΡ‚ΠΈΠ²Π»Π΅Π½ΠΈΡŽ , ΠΌΡ‹ ΠΌΠΎΠΆΠ΅ΠΌ ΡΠΊΠ°Π·Π°Ρ‚ΡŒ, Ρ‡Ρ‚ΠΎ Π² ΠΏΠΎΠ»ΡƒΡ‡Π΅Π½Π½ΠΎΠΉ Ρ„ΠΎΡ€ΠΌΡƒΠ»Π΅ 1 / R – ΠΏΡ€ΠΎΠ²ΠΎΠ΄ΠΈΠΌΠΎΡΡ‚ΡŒ внСшнСй Ρ†Π΅ΠΏΠΈ; 1 / R1 ΠΏΡ€ΠΎΠ²ΠΎΠ΄ΠΈΠΌΠΎΡΡ‚ΡŒ ΠΏΠ΅Ρ€Π²ΠΎΠΉ Π²Π΅Ρ‚Π²ΠΈ; 1 / R2- ΠΏΡ€ΠΎΠ²ΠΎΠ΄ΠΈΠΌΠΎΡΡ‚ΡŒ Π²Ρ‚ΠΎΡ€ΠΎΠΉ Π²Π΅Ρ‚Π²ΠΈ.

    На основании этой Ρ„ΠΎΡ€ΠΌΡƒΠ»Ρ‹ Π΄Π΅Π»Π°Π΅ΠΌ Π²Ρ‹Π²ΠΎΠ΄: ΠΏΡ€ΠΈ ΠΏΠ°Ρ€Π°Π»Π»Π΅Π»ΡŒΠ½ΠΎΠΌ соСдинСнии ΠΏΡ€ΠΎΠ²ΠΎΠ΄ΠΈΠΌΠΎΡΡ‚ΡŒ внСшнСй Ρ†Π΅ΠΏΠΈ Ρ€Π°Π²Π½Π° суммС проводимостСй ΠΎΡ‚Π΄Π΅Π»ΡŒΠ½Ρ‹Ρ… Π²Π΅Ρ‚Π²Π΅ΠΉ.

    Π‘Π»Π΅Π΄ΠΎΠ²Π°Ρ‚Π΅Π»ΡŒΠ½ΠΎ, Ρ‡Ρ‚ΠΎΠ±Ρ‹ ΠΎΠΏΡ€Π΅Π΄Π΅Π»ΠΈΡ‚ΡŒ эквивалСнтноС сопротивлСниС Π²ΠΊΠ»ΡŽΡ‡Π΅Π½Π½Ρ‹Ρ… ΠΏΠ°Ρ€Π°Π»Π»Π΅Π»ΡŒΠ½ΠΎ сопротивлСний, Π½Π°Π΄ΠΎ ΠΎΠΏΡ€Π΅Π΄Π΅Π»ΠΈΡ‚ΡŒ ΠΏΡ€ΠΎΠ²ΠΎΠ΄ΠΈΠΌΠΎΡΡ‚ΡŒ Ρ†Π΅ΠΏΠΈ ΠΈ Π²Π·ΡΡ‚ΡŒ Π²Π΅Π»ΠΈΡ‡ΠΈΠ½Ρƒ, Π΅ΠΉ ΠΎΠ±Ρ€Π°Ρ‚Π½ΡƒΡŽ.

    Из Ρ„ΠΎΡ€ΠΌΡƒΠ»Ρ‹ Ρ‚Π°ΠΊΠΆΠ΅ слСдуСт, Ρ‡Ρ‚ΠΎ ΠΏΡ€ΠΎΠ²ΠΎΠ΄ΠΈΠΌΠΎΡΡ‚ΡŒ Ρ†Π΅ΠΏΠΈ большС проводимости ΠΊΠ°ΠΆΠ΄ΠΎΠΉ Π²Π΅Ρ‚Π²ΠΈ, Π° это Π·Π½Π°Ρ‡ΠΈΡ‚, Ρ‡Ρ‚ΠΎ эквивалСнтноС сопротивлСниС внСшнСй Ρ†Π΅ΠΏΠΈ мСньшС наимСньшСго ΠΈΠ· Π²ΠΊΠ»ΡŽΡ‡Π΅Π½Π½Ρ‹Ρ… ΠΏΠ°Ρ€Π°Π»Π»Π΅Π»ΡŒΠ½ΠΎ сопротивлСний.

    Рассматривая случай ΠΏΠ°Ρ€Π°Π»Π»Π΅Π»ΡŒΠ½ΠΎΠ³ΠΎ соСдинСния сопротивлСний, ΠΌΡ‹ взяли Π½Π°ΠΈΠ±ΠΎΠ»Π΅Π΅ ΠΏΡ€ΠΎΡΡ‚ΡƒΡŽ Ρ†Π΅ΠΏΡŒ, ΡΠΎΡΡ‚ΠΎΡΡ‰ΡƒΡŽ ΠΈΠ· Π΄Π²ΡƒΡ… Π²Π΅Ρ‚Π²Π΅ΠΉ. Однако Π½Π° ΠΏΡ€Π°ΠΊΡ‚ΠΈΠΊΠ΅ ΠΌΠΎΠ³ΡƒΡ‚ Π²ΡΡ‚Ρ€Π΅Ρ‚ΠΈΡ‚ΡŒΡΡ случаи, ΠΊΠΎΠ³Π΄Π° Ρ†Π΅ΠΏΡŒ состоит ΠΈΠ· Ρ‚Ρ€Π΅Ρ… ΠΈ Π±ΠΎΠ»Π΅Π΅ ΠΏΠ°Ρ€Π°Π»Π»Π΅Π»ΡŒΠ½Ρ‹Ρ… Π²Π΅Ρ‚Π²Π΅ΠΉ. Как ΠΆΠ΅ ΠΏΠΎΡΡ‚ΡƒΠΏΠ°Ρ‚ΡŒ Π² этих случаях?

    ΠžΠΊΠ°Π·Ρ‹Π²Π°Π΅Ρ‚ΡΡ, всС ΠΏΠΎΠ»ΡƒΡ‡Π΅Π½Π½Ρ‹Π΅ Π½Π°ΠΌΠΈ ΡΠΎΠΎΡ‚Π½ΠΎΡˆΠ΅Π½ΠΈΡ ΠΎΡΡ‚Π°ΡŽΡ‚ΡΡ справСдливыми ΠΈ для Ρ†Π΅ΠΏΠΈ, состоящСй ΠΈΠ· любого числа ΠΏΠ°Ρ€Π°Π»Π»Π΅Π»ΡŒΠ½ΠΎ соСдинСнных сопротивлСний.

    Π§Ρ‚ΠΎΠ±Ρ‹ ΡƒΠ±Π΅Π΄ΠΈΡ‚ΡŒΡΡ Π² этом, рассмотрим ΡΠ»Π΅Π΄ΡƒΡŽΡ‰ΠΈΠΉ ΠΏΡ€ΠΈΠΌΠ΅Ρ€.

    Π’ΠΎΠ·ΡŒΠΌΠ΅ΠΌ Ρ‚Ρ€ΠΈ сопротивлСния R1 = 10 Ом, R2 = 20 Ом ΠΈ R3 = 60 Ом ΠΈ соСдиним ΠΈΡ… ΠΏΠ°Ρ€Π°Π»Π»Π΅Π»ΡŒΠ½ΠΎ. ΠžΠΏΡ€Π΅Π΄Π΅Π»ΠΈΠΌ эквивалСнтноС сопротивлСниС Ρ†Π΅ΠΏΠΈ (рис. 5 ).

    Рис. 5. ЦСпь с трСмя ΠΏΠ°Ρ€Π°Π»Π»Π΅Π»ΡŒΠ½ΠΎ соСдинСнными сопротивлСниями

    ΠŸΡ€ΠΈΠΌΠ΅Π½ΡΡ для этой Ρ†Π΅ΠΏΠΈ Ρ„ΠΎΡ€ΠΌΡƒΠ»Ρƒ 1 / R = 1 / R1 + 1 / R2 , ΠΌΠΎΠΆΠ΅ΠΌ Π½Π°ΠΏΠΈΡΠ°Ρ‚ΡŒ 1 / R = 1 / R1 + 1 / R2 + 1 / R3 ΠΈ, подставляя извСстныС Π²Π΅Π»ΠΈΡ‡ΠΈΠ½Ρ‹, ΠΏΠΎΠ»ΡƒΡ‡ΠΈΠΌ 1 / R = 1 / 10 + 1 / 20 + 1 / 60

    Π‘Π»ΠΎΠΆΠΈΠΌ эта Π΄Ρ€ΠΎΠ±ΠΈ: 1/R = 10 / 60 = 1 / 6, Ρ‚. Π΅.. ΠΏΡ€ΠΎΠ²ΠΎΠ΄ΠΈΠΌΠΎΡΡ‚ΡŒ Ρ†Π΅ΠΏΠΈ 1 / R = 1 / 6 Π‘Π»Π΅Π΄ΠΎΠ²Π°Ρ‚Π΅Π»ΡŒΠ½ΠΎ, эквивалСнтноС сопротивлСниС R = 6 Ом.

    Π’Π°ΠΊΠΈΠΌ ΠΎΠ±Ρ€Π°Π·ΠΎΠΌ, эквивалСнтноС сопротивлСниС мСньшС наимСньшСго ΠΈΠ· Π²ΠΊΠ»ΡŽΡ‡Π΅Π½Π½Ρ‹Ρ… ΠΏΠ°Ρ€Π°Π»Π»Π΅Π»ΡŒΠ½ΠΎ Π² Ρ†Π΅ΠΏΡŒ сопротивлСний , Ρ‚. Π΅. мСньшС сопротивлСния R1.

    ΠŸΠΎΡΠΌΠΎΡ‚Ρ€ΠΈΠΌ Ρ‚Π΅ΠΏΠ΅Ρ€ΡŒ, Π΄Π΅ΠΉΡΡ‚Π²ΠΈΡ‚Π΅Π»ΡŒΠ½ΠΎ Π»ΠΈ это сопротивлСниС являСтся эквивалСнтным, Ρ‚. Π΅. Ρ‚Π°ΠΊΠΈΠΌ, ΠΊΠΎΡ‚ΠΎΡ€ΠΎΠ΅ ΠΌΠΎΠ³Π»ΠΎ Π±Ρ‹ Π·Π°ΠΌΠ΅Π½ΠΈΡ‚ΡŒ Π²ΠΊΠ»ΡŽΡ‡Π΅Π½Π½Ρ‹Π΅ ΠΏΠ°Ρ€Π°Π»Π»Π΅Π»ΡŒΠ½ΠΎ сопротивлСния Π² 10, 20 ΠΈ 60 Ом, Π½Π΅ измСняя ΠΏΡ€ΠΈ этом силы Ρ‚ΠΎΠΊΠ° Π΄ΠΎ развСтвлСния Ρ†Π΅ΠΏΠΈ.

    Допустим, Ρ‡Ρ‚ΠΎ напряТСниС внСшнСй Ρ†Π΅ΠΏΠΈ, Π° ΡΠ»Π΅Π΄ΠΎΠ²Π°Ρ‚Π΅Π»ΡŒΠ½ΠΎ, ΠΈ напряТСниС Π½Π° сопротивлСниях R1, R2, R3 Ρ€Π°Π²Π½ΠΎ 12 Π’. Π’ΠΎΠ³Π΄Π° сила Ρ‚ΠΎΠΊΠΎΠ² Π² вСтвях Π±ΡƒΠ΄Π΅Ρ‚: I1 = U/R1 = 12 / 10 = 1 ,2 А I 2 = U/R 2 = 12 / 20 = 1 ,6 А I 3 = U/R1 = 12 / 60 = 0, 2 А

    ΠžΠ±Ρ‰ΠΈΠΉ Ρ‚ΠΎΠΊ Π² Ρ†Π΅ΠΏΠΈ ΠΏΠΎΠ»ΡƒΡ‡ΠΈΠΌ, ΠΏΠΎΠ»ΡŒΠ·ΡƒΡΡΡŒ Ρ„ΠΎΡ€ΠΌΡƒΠ»ΠΎΠΉ I = I1 + I2 + I3 = 1,2 + 0,6 + 0,2 = 2 А.

    ΠŸΡ€ΠΎΠ²Π΅Ρ€ΠΈΠΌ ΠΏΠΎ Ρ„ΠΎΡ€ΠΌΡƒΠ»Π΅ Π·Π°ΠΊΠΎΠ½Π° Ома, получится Π»ΠΈ Π² Ρ†Π΅ΠΏΠΈ Ρ‚ΠΎΠΊ силой 2 А, Ссли вмСсто Ρ‚Ρ€Π΅Ρ… ΠΏΠ°Ρ€Π°Π»Π»Π΅Π»ΡŒΠ½ΠΎ Π²ΠΊΠ»ΡŽΡ‡Π΅Π½Π½Ρ‹Ρ… извСстных Π½Π°ΠΌ сопротивлСний Π²ΠΊΠ»ΡŽΡ‡Π΅Π½ΠΎ ΠΎΠ΄Π½ΠΎ эквивалСнтноС ΠΈΠΌ сопротивлСниС 6 Ом.

    I = U / R = 12 / 6 = 2 А

    Как Π²ΠΈΠ΄ΠΈΠΌ, Π½Π°ΠΉΠ΄Π΅Π½Π½ΠΎΠ΅ Π½Π°ΠΌΠΈ сопротивлСниС R = 6 Ом Π΄Π΅ΠΉΡΡ‚Π²ΠΈΡ‚Π΅Π»ΡŒΠ½ΠΎ являСтся для Π΄Π°Π½Π½ΠΎΠΉ Ρ†Π΅ΠΏΠΈ эквивалСнтным.

    Π’ этом ΠΌΠΎΠΆΠ½ΠΎ ΡƒΠ±Π΅Π΄ΠΈΡ‚ΡŒΡΡ ΠΈ Π½Π° ΠΈΠ·ΠΌΠ΅Ρ€ΠΈΡ‚Π΅Π»ΡŒΠ½Ρ‹Ρ… ΠΏΡ€ΠΈΠ±ΠΎΡ€Π°Ρ…, Ссли ΡΠΎΠ±Ρ€Π°Ρ‚ΡŒ схСму с взятыми Π½Π°ΠΌΠΈ сопротивлСниями, ΠΈΠ·ΠΌΠ΅Ρ€ΠΈΡ‚ΡŒ Ρ‚ΠΎΠΊ Π²ΠΎ внСшнСй Ρ†Π΅ΠΏΠΈ (Π΄ΠΎ развСтвлСния), Π·Π°Ρ‚Π΅ΠΌ Π·Π°ΠΌΠ΅Π½ΠΈΡ‚ΡŒ ΠΏΠ°Ρ€Π°Π»Π»Π΅Π»ΡŒΠ½ΠΎ Π²ΠΊΠ»ΡŽΡ‡Π΅Π½Π½Ρ‹Π΅ сопротивлСния ΠΎΠ΄Π½ΠΈΠΌ сопротивлСниСм 6 Ом ΠΈ снова ΠΈΠ·ΠΌΠ΅Ρ€ΠΈΡ‚ΡŒ Ρ‚ΠΎΠΊ. Показания Π°ΠΌΠΏΠ΅Ρ€ΠΌΠ΅Ρ‚Ρ€Π° ΠΈ Π² Ρ‚ΠΎΠΌ ΠΈ Π² Π΄Ρ€ΡƒΠ³ΠΎΠΌ случаС Π±ΡƒΠ΄ΡƒΡ‚ ΠΏΡ€ΠΈΠΌΠ΅Ρ€Π½ΠΎ ΠΎΠ΄ΠΈΠ½Π°ΠΊΠΎΠ²Ρ‹ΠΌΠΈ.

    На ΠΏΡ€Π°ΠΊΡ‚ΠΈΠΊΠ΅ ΠΌΠΎΠ³ΡƒΡ‚ Π²ΡΡ‚Ρ€Π΅Ρ‚ΠΈΡ‚ΡŒΡΡ Ρ‚Π°ΠΊΠΆΠ΅ ΠΏΠ°Ρ€Π°Π»Π»Π΅Π»ΡŒΠ½Ρ‹Π΅ соСдинСния, для ΠΊΠΎΡ‚ΠΎΡ€Ρ‹Ρ… Ρ€Π°ΡΡΡ‡ΠΈΡ‚Π°Ρ‚ΡŒ эквивалСнтноС сопротивлСниС ΠΌΠΎΠΆΠ½ΠΎ ΠΏΡ€ΠΎΡ‰Π΅, Ρ‚. Π΅. Π½Π΅ опрСдСляя ΠΏΡ€Π΅Π΄Π²Π°Ρ€ΠΈΡ‚Π΅Π»ΡŒΠ½ΠΎ проводимостСй, сразу Π½Π°ΠΉΡ‚ΠΈ сопротивлСниС.

    НапримСр, Ссли соСдинСны ΠΏΠ°Ρ€Π°Π»Π»Π΅Π»ΡŒΠ½ΠΎ Π΄Π²Π° сопротивлСния R1 ΠΈ R2 , Ρ‚ΠΎ Ρ„ΠΎΡ€ΠΌΡƒΠ»Ρƒ 1 / R = 1 / R1 + 1 / R2 ΠΌΠΎΠΆΠ½ΠΎ ΠΏΡ€Π΅ΠΎΠ±Ρ€Π°Π·ΠΎΠ²Π°Ρ‚ΡŒ Ρ‚Π°ΠΊ: 1/R = (R2 + R1) / R1 R2 ΠΈ, Ρ€Π΅ΡˆΠ°Ρ равСнство ΠΎΡ‚Π½ΠΎΡΠΈΡ‚Π΅Π»ΡŒΠ½ΠΎ R, ΠΏΠΎΠ»ΡƒΡ‡ΠΈΡ‚ΡŒ R = R1 Ρ… R2 / ( R1 + R2 ), Ρ‚. Π΅. ΠΏΡ€ΠΈ ΠΏΠ°Ρ€Π°Π»Π»Π΅Π»ΡŒΠ½ΠΎΠΌ соСдинСнии Π΄Π²ΡƒΡ… сопротивлСний эквивалСнтноС сопротивлСниС Ρ†Π΅ΠΏΠΈ Ρ€Π°Π²Π½ΠΎ ΠΏΡ€ΠΎΠΈΠ·Π²Π΅Π΄Π΅Π½ΠΈΡŽ Π²ΠΊΠ»ΡŽΡ‡Π΅Π½Π½Ρ‹Ρ… ΠΏΠ°Ρ€Π°Π»Π»Π΅Π»ΡŒΠ½ΠΎ сопротивлСний, Π΄Π΅Π»Π΅Π½Π½ΠΎΠΌΡƒ Π½Π° ΠΈΡ… сумму.

    Π’ элСктричСских цСпях элСмСнты ΠΌΠΎΠ³ΡƒΡ‚ ΡΠΎΠ΅Π΄ΠΈΠ½ΡΡ‚ΡŒΡΡ ΠΏΠΎ Ρ€Π°Π·Π»ΠΈΡ‡Π½Ρ‹ΠΌ схСмам, Π² Ρ‚ΠΎΠΌ числС ΠΎΠ½ΠΈ ΠΈΠΌΠ΅ΡŽΡ‚ ΠΏΠΎΡΠ»Π΅Π΄ΠΎΠ²Π°Ρ‚Π΅Π»ΡŒΠ½ΠΎΠ΅ ΠΈ ΠΏΠ°Ρ€Π°Π»Π»Π΅Π»ΡŒΠ½ΠΎΠ΅ соСдинСниС.

    ΠŸΠΎΡΠ»Π΅Π΄ΠΎΠ²Π°Ρ‚Π΅Π»ΡŒΠ½ΠΎΠ΅ соСдинСниС

    ΠŸΡ€ΠΈ Ρ‚Π°ΠΊΠΎΠΌ соСдинСнии ΠΏΡ€ΠΎΠ²ΠΎΠ΄Π½ΠΈΠΊΠΈ ΡΠΎΠ΅Π΄ΠΈΠ½ΡΡŽΡ‚ΡΡ Π΄Ρ€ΡƒΠ³ с Π΄Ρ€ΡƒΠ³ΠΎΠΌ ΠΏΠΎΡΠ»Π΅Π΄ΠΎΠ²Π°Ρ‚Π΅Π»ΡŒΠ½ΠΎ, Ρ‚ΠΎ Π΅ΡΡ‚ΡŒ, Π½Π°Ρ‡Π°Π»ΠΎ ΠΎΠ΄Π½ΠΎΠ³ΠΎ ΠΏΡ€ΠΎΠ²ΠΎΠ΄Π½ΠΈΠΊΠ° Π±ΡƒΠ΄Π΅Ρ‚ ΡΠΎΠ΅Π΄ΠΈΠ½ΡΡ‚ΡŒΡΡ с ΠΊΠΎΠ½Ρ†ΠΎΠΌ Π΄Ρ€ΡƒΠ³ΠΎΠ³ΠΎ. Основная ΠΎΡΠΎΠ±Π΅Π½Π½ΠΎΡΡ‚ΡŒ Π΄Π°Π½Π½ΠΎΠ³ΠΎ соСдинСния Π·Π°ΠΊΠ»ΡŽΡ‡Π°Π΅Ρ‚ΡΡ Π² Ρ‚ΠΎΠΌ, Ρ‡Ρ‚ΠΎ всС ΠΏΡ€ΠΎΠ²ΠΎΠ΄Π½ΠΈΠΊΠΈ ΠΏΡ€ΠΈΠ½Π°Π΄Π»Π΅ΠΆΠ°Ρ‚ ΠΎΠ΄Π½ΠΎΠΌΡƒ ΠΏΡ€ΠΎΠ²ΠΎΠ΄Ρƒ, Π½Π΅Ρ‚ Π½ΠΈΠΊΠ°ΠΊΠΈΡ… Ρ€Π°Π·Π²Π΅Ρ‚Π²Π»Π΅Π½ΠΈΠΉ. Π§Π΅Ρ€Π΅Π· ΠΊΠ°ΠΆΠ΄Ρ‹ΠΉ ΠΈΠ· ΠΏΡ€ΠΎΠ²ΠΎΠ΄Π½ΠΈΠΊΠΎΠ² Π±ΡƒΠ΄Π΅Ρ‚ ΠΏΡ€ΠΎΡ‚Π΅ΠΊΠ°Ρ‚ΡŒ ΠΎΠ΄ΠΈΠ½ ΠΈ Ρ‚ΠΎΡ‚ ΠΆΠ΅ элСктричСский Ρ‚ΠΎΠΊ. Но суммарноС напряТСниС Π½Π° ΠΏΡ€ΠΎΠ²ΠΎΠ΄Π½ΠΈΠΊΠ°Ρ… Π±ΡƒΠ΄Π΅Ρ‚ Ρ€Π°Π²Π½ΡΡ‚ΡŒΡΡ вмСстС взятым напряТСниям Π½Π° ΠΊΠ°ΠΆΠ΄ΠΎΠΌ ΠΈΠ· Π½ΠΈΡ….

    Рассмотрим Π½Π΅ΠΊΠΎΡ‚ΠΎΡ€ΠΎΠ΅ количСство рСзисторов, соСдинСнных ΠΏΠΎΡΠ»Π΅Π΄ΠΎΠ²Π°Ρ‚Π΅Π»ΡŒΠ½ΠΎ. Π’Π°ΠΊ ΠΊΠ°ΠΊ Π½Π΅Ρ‚ Ρ€Π°Π·Π²Π΅Ρ‚Π²Π»Π΅Π½ΠΈΠΉ, Ρ‚ΠΎ количСство проходящСго заряда Ρ‡Π΅Ρ€Π΅Π· ΠΎΠ΄ΠΈΠ½ ΠΏΡ€ΠΎΠ²ΠΎΠ΄Π½ΠΈΠΊ, Π±ΡƒΠ΄Π΅Ρ‚ Ρ€Π°Π²Π½ΠΎ количСству заряда, ΠΏΡ€ΠΎΡˆΠ΅Π΄ΡˆΠ΅Π³ΠΎ Ρ‡Π΅Ρ€Π΅Π· Π΄Ρ€ΡƒΠ³ΠΎΠΉ ΠΏΡ€ΠΎΠ²ΠΎΠ΄Π½ΠΈΠΊ. Π‘ΠΈΠ»Ρ‹ Ρ‚ΠΎΠΊΠ° Π½Π° всСх ΠΏΡ€ΠΎΠ²ΠΎΠ΄Π½ΠΈΠΊΠ°Ρ… Π±ΡƒΠ΄ΡƒΡ‚ ΠΎΠ΄ΠΈΠ½Π°ΠΊΠΎΠ²Ρ‹ΠΌΠΈ. Π­Ρ‚ΠΎ основная ΠΎΡΠΎΠ±Π΅Π½Π½ΠΎΡΡ‚ΡŒ Π΄Π°Π½Π½ΠΎΠ³ΠΎ соСдинСния.

    Π­Ρ‚ΠΎ соСдинСниС ΠΌΠΎΠΆΠ½ΠΎ Ρ€Π°ΡΡΠΌΠΎΡ‚Ρ€Π΅Ρ‚ΡŒ ΠΈΠ½Π°Ρ‡Π΅. ВсС рСзисторы ΠΌΠΎΠΆΠ½ΠΎ Π·Π°ΠΌΠ΅Π½ΠΈΡ‚ΡŒ ΠΎΠ΄Π½ΠΈΠΌ эквивалСнтным рСзистором.

    Π’ΠΎΠΊ Π½Π° эквивалСнтном рСзисторС Π±ΡƒΠ΄Π΅Ρ‚ ΡΠΎΠ²ΠΏΠ°Π΄Π°Ρ‚ΡŒ с ΠΎΠ±Ρ‰ΠΈΠΌ Ρ‚ΠΎΠΊΠΎΠΌ, ΠΏΡ€ΠΎΡ‚Π΅ΠΊΠ°ΡŽΡ‰ΠΈΠΌ Ρ‡Π΅Ρ€Π΅Π· всС рСзисторы. Π­ΠΊΠ²ΠΈΠ²Π°Π»Π΅Π½Ρ‚Π½ΠΎΠ΅ ΠΎΠ±Ρ‰Π΅Π΅ напряТСниС Π±ΡƒΠ΄Π΅Ρ‚ ΡΠΊΠ»Π°Π΄Ρ‹Π²Π°Ρ‚ΡŒΡΡ ΠΈΠ· напряТСний Π½Π° ΠΊΠ°ΠΆΠ΄ΠΎΠΌ рСзисторС. Π­Ρ‚ΠΎ являСтся Ρ€Π°Π·Π½ΠΎΡΡ‚ΡŒΡŽ ΠΏΠΎΡ‚Π΅Π½Ρ†ΠΈΠ°Π»ΠΎΠ² Π½Π° рСзисторС.

    Если Π²ΠΎΡΠΏΠΎΠ»ΡŒΠ·ΠΎΠ²Π°Ρ‚ΡŒΡΡ этими ΠΏΡ€Π°Π²ΠΈΠ»Π°ΠΌΠΈ ΠΈ Π·Π°ΠΊΠΎΠ½ΠΎΠΌ Ома, ΠΊΠΎΡ‚ΠΎΡ€Ρ‹ΠΉ ΠΏΠΎΠ΄Ρ…ΠΎΠ΄ΠΈΡ‚ для ΠΊΠ°ΠΆΠ΄ΠΎΠ³ΠΎ рСзистора, ΠΌΠΎΠΆΠ½ΠΎ Π΄ΠΎΠΊΠ°Π·Π°Ρ‚ΡŒ, Ρ‡Ρ‚ΠΎ сопротивлСниС эквивалСнтного ΠΎΠ±Ρ‰Π΅Π³ΠΎ рСзистора Π±ΡƒΠ΄Π΅Ρ‚ Ρ€Π°Π²Π½ΠΎ суммС сопротивлСний. БлСдствиСм ΠΏΠ΅Ρ€Π²Ρ‹Ρ… Π΄Π²ΡƒΡ… ΠΏΡ€Π°Π²ΠΈΠ» Π±ΡƒΠ΄Π΅Ρ‚ ΡΠ²Π»ΡΡ‚ΡŒΡΡ Ρ‚Ρ€Π΅Ρ‚ΡŒΠ΅ ΠΏΡ€Π°Π²ΠΈΠ»ΠΎ.

    ΠŸΡ€ΠΈΠΌΠ΅Π½Π΅Π½ΠΈΠ΅

    ΠŸΠΎΡΠ»Π΅Π΄ΠΎΠ²Π°Ρ‚Π΅Π»ΡŒΠ½ΠΎΠ΅ соСдинСниС ΠΈΡΠΏΠΎΠ»ΡŒΠ·ΡƒΠ΅Ρ‚ΡΡ, ΠΊΠΎΠ³Π΄Π° Π½ΡƒΠΆΠ½ΠΎ Ρ†Π΅Π»Π΅Π½Π°ΠΏΡ€Π°Π²Π»Π΅Π½Π½ΠΎ Π²ΠΊΠ»ΡŽΡ‡Π°Ρ‚ΡŒ ΠΈΠ»ΠΈ Π²Ρ‹ΠΊΠ»ΡŽΡ‡Π°Ρ‚ΡŒ ΠΊΠ°ΠΊΠΎΠΉ-Π»ΠΈΠ±ΠΎ ΠΏΡ€ΠΈΠ±ΠΎΡ€, Π²Ρ‹ΠΊΠ»ΡŽΡ‡Π°Ρ‚Π΅Π»ΡŒ ΡΠΎΠ΅Π΄ΠΈΠ½ΡΡŽΡ‚ с Π½ΠΈΠΌ ΠΏΠΎ ΠΏΠΎΡΠ»Π΅Π΄ΠΎΠ²Π°Ρ‚Π΅Π»ΡŒΠ½ΠΎΠΉ схСмС. НапримСр, элСктричСский Π·Π²ΠΎΠ½ΠΎΠΊ Π±ΡƒΠ΄Π΅Ρ‚ Π·Π²Π΅Π½Π΅Ρ‚ΡŒ Ρ‚ΠΎΠ»ΡŒΠΊΠΎ Ρ‚ΠΎΠ³Π΄Π°, ΠΊΠΎΠ³Π΄Π° ΠΎΠ½ Π±ΡƒΠ΄Π΅Ρ‚ ΠΏΠΎΡΠ»Π΅Π΄ΠΎΠ²Π°Ρ‚Π΅Π»ΡŒΠ½ΠΎ соСдинСн с источником ΠΈ ΠΊΠ½ΠΎΠΏΠΊΠΎΠΉ. Богласно ΠΏΠ΅Ρ€Π²ΠΎΠΌΡƒ ΠΏΡ€Π°Π²ΠΈΠ»Ρƒ, Ссли элСктричСский Ρ‚ΠΎΠΊ отсутствуСт хотя Π±Ρ‹ Π½Π° ΠΎΠ΄Π½ΠΎΠΌ ΠΈΠ· ΠΏΡ€ΠΎΠ²ΠΎΠ΄Π½ΠΈΠΊΠΎΠ², Ρ‚ΠΎ Π΅Π³ΠΎ Π½Π΅ Π±ΡƒΠ΄Π΅Ρ‚ ΠΈ Π½Π° Π΄Ρ€ΡƒΠ³ΠΈΡ… ΠΏΡ€ΠΎΠ²ΠΎΠ΄Π½ΠΈΠΊΠ°Ρ…. И Π½Π°ΠΎΠ±ΠΎΡ€ΠΎΡ‚, Ссли Ρ‚ΠΎΠΊ имССтся хотя Π±Ρ‹ Π½Π° ΠΎΠ΄Π½ΠΎΠΌ ΠΏΡ€ΠΎΠ²ΠΎΠ΄Π½ΠΈΠΊΠ΅, Ρ‚ΠΎ ΠΎΠ½ Π±ΡƒΠ΄Π΅Ρ‚ ΠΈ Π½Π° всСх Π΄Ρ€ΡƒΠ³ΠΈΡ… ΠΏΡ€ΠΎΠ²ΠΎΠ΄Π½ΠΈΠΊΠ°Ρ…. Π’Π°ΠΊΠΆΠ΅ Ρ€Π°Π±ΠΎΡ‚Π°Π΅Ρ‚ ΠΊΠ°Ρ€ΠΌΠ°Π½Π½Ρ‹ΠΉ Ρ„ΠΎΠ½Π°Ρ€ΠΈΠΊ, Π² ΠΊΠΎΡ‚ΠΎΡ€ΠΎΠΌ Π΅ΡΡ‚ΡŒ ΠΊΠ½ΠΎΠΏΠΊΠ°, Π±Π°Ρ‚Π°Ρ€Π΅ΠΉΠΊΠ° ΠΈ Π»Π°ΠΌΠΏΠΎΡ‡ΠΊΠ°. ВсС эти элСмСнты Π½Π΅ΠΎΠ±Ρ…ΠΎΠ΄ΠΈΠΌΠΎ ΡΠΎΠ΅Π΄ΠΈΠ½ΠΈΡ‚ΡŒ ΠΏΠΎΡΠ»Π΅Π΄ΠΎΠ²Π°Ρ‚Π΅Π»ΡŒΠ½ΠΎ, Ρ‚Π°ΠΊ ΠΊΠ°ΠΊ Π½ΡƒΠΆΠ½ΠΎ, Ρ‡Ρ‚ΠΎΠ±Ρ‹ Ρ„ΠΎΠ½Π°Ρ€ΠΈΠΊ свСтил, ΠΊΠΎΠ³Π΄Π° Π±ΡƒΠ΄Π΅Ρ‚ Π½Π°ΠΆΠ°Ρ‚Π° ΠΊΠ½ΠΎΠΏΠΊΠ°.

    Иногда ΠΏΠΎΡΠ»Π΅Π΄ΠΎΠ²Π°Ρ‚Π΅Π»ΡŒΠ½ΠΎΠ΅ соСдинСниС Π½Π΅ ΠΏΡ€ΠΈΠ²ΠΎΠ΄ΠΈΡ‚ ΠΊ Π½ΡƒΠΆΠ½Ρ‹ΠΌ цСлям. НапримСр, Π² ΠΊΠ²Π°Ρ€Ρ‚ΠΈΡ€Π΅, Π³Π΄Π΅ ΠΌΠ½ΠΎΠ³ΠΎ Π»ΡŽΡΡ‚Ρ€, Π»Π°ΠΌΠΏΠΎΡ‡Π΅ΠΊ ΠΈ Π΄Ρ€ΡƒΠ³ΠΈΡ… устройств, Π½Π΅ слСдуСт всС Π»Π°ΠΌΠΏΡ‹ ΠΈ устройства ΡΠΎΠ΅Π΄ΠΈΠ½ΡΡ‚ΡŒ ΠΏΠΎΡΠ»Π΅Π΄ΠΎΠ²Π°Ρ‚Π΅Π»ΡŒΠ½ΠΎ, Ρ‚Π°ΠΊ ΠΊΠ°ΠΊ Π½ΠΈΠΊΠΎΠ³Π΄Π° Π½Π΅ трСбуСтся ΠΎΠ΄Π½ΠΎΠ²Ρ€Π΅ΠΌΠ΅Π½Π½ΠΎ Π²ΠΊΠ»ΡŽΡ‡Π°Ρ‚ΡŒ свСт Π² ΠΊΠ°ΠΆΠ΄ΠΎΠΉ ΠΈΠ· ΠΊΠΎΠΌΠ½Π°Ρ‚ ΠΊΠ²Π°Ρ€Ρ‚ΠΈΡ€Ρ‹. Для этого ΠΏΠΎΡΠ»Π΅Π΄ΠΎΠ²Π°Ρ‚Π΅Π»ΡŒΠ½ΠΎΠ΅ ΠΈ ΠΏΠ°Ρ€Π°Π»Π»Π΅Π»ΡŒΠ½ΠΎΠ΅ соСдинСниС Ρ€Π°ΡΡΠΌΠ°Ρ‚Ρ€ΠΈΠ²Π°ΡŽΡ‚ ΠΎΡ‚Π΄Π΅Π»ΡŒΠ½ΠΎ, ΠΈ для ΠΏΠΎΠ΄ΠΊΠ»ΡŽΡ‡Π΅Π½ΠΈΡ ΠΎΡΠ²Π΅Ρ‚ΠΈΡ‚Π΅Π»ΡŒΠ½Ρ‹Ρ… ΠΏΡ€ΠΈΠ±ΠΎΡ€ΠΎΠ² Π² ΠΊΠ²Π°Ρ€Ρ‚ΠΈΡ€Π΅ ΠΏΡ€ΠΈΠΌΠ΅Π½ΡΡŽΡ‚ ΠΏΠ°Ρ€Π°Π»Π»Π΅Π»ΡŒΠ½Ρ‹ΠΉ Π²ΠΈΠ΄ схСмы.

    ΠŸΠ°Ρ€Π°Π»Π»Π΅Π»ΡŒΠ½ΠΎΠ΅ соСдинСниС

    Π’ этом Π²ΠΈΠ΄Π΅ схСмы всС ΠΏΡ€ΠΎΠ²ΠΎΠ΄Π½ΠΈΠΊΠΈ ΡΠΎΠ΅Π΄ΠΈΠ½ΡΡŽΡ‚ΡΡ ΠΏΠ°Ρ€Π°Π»Π»Π΅Π»ΡŒΠ½ΠΎ Π΄Ρ€ΡƒΠ³ с Π΄Ρ€ΡƒΠ³ΠΎΠΌ. ВсС Π½Π°Ρ‡Π°Π»Π° ΠΏΡ€ΠΎΠ²ΠΎΠ΄Π½ΠΈΠΊΠΎΠ² ΠΎΠ±ΡŠΠ΅Π΄ΠΈΠ½Π΅Π½Ρ‹ Π² ΠΎΠ΄Π½Ρƒ Ρ‚ΠΎΡ‡ΠΊΡƒ, ΠΈ всС ΠΊΠΎΠ½Ρ†Ρ‹ Ρ‚Π°ΠΊΠΆΠ΅ соСдинСны вмСстС. Рассмотрим Π½Π΅ΠΊΠΎΡ‚ΠΎΡ€ΠΎΠ΅ количСство ΠΎΠ΄Π½ΠΎΡ€ΠΎΠ΄Π½Ρ‹Ρ… ΠΏΡ€ΠΎΠ²ΠΎΠ΄Π½ΠΈΠΊΠΎΠ² (рСзисторов), соСдинСнных ΠΏΠΎ ΠΏΠ°Ρ€Π°Π»Π»Π΅Π»ΡŒΠ½ΠΎΠΉ схСмС.

    Π­Ρ‚ΠΎΡ‚ Π²ΠΈΠ΄ соСдинСния являСтся Ρ€Π°Π·Π²Π΅Ρ‚Π²Π»Π΅Π½Π½Ρ‹ΠΌ. Π’ ΠΊΠ°ΠΆΠ΄ΠΎΠΉ Π²Π΅Ρ‚Π²ΠΈ содСрТится ΠΏΠΎ ΠΎΠ΄Π½ΠΎΠΌΡƒ рСзистору. ЭлСктричСский Ρ‚ΠΎΠΊ, дойдя Π΄ΠΎ Ρ‚ΠΎΡ‡ΠΊΠΈ развСтвлСния, раздСляСтся Π½Π° ΠΊΠ°ΠΆΠ΄Ρ‹ΠΉ рСзистор, ΠΈ Π±ΡƒΠ΄Π΅Ρ‚ Ρ€Π°Π²Π½ΡΡ‚ΡŒΡΡ суммС Ρ‚ΠΎΠΊΠΎΠ² Π½Π° всСх сопротивлСниях. НапряТСниС Π½Π° всСх элСмСнтах, соСдинСнных ΠΏΠ°Ρ€Π°Π»Π»Π΅Π»ΡŒΠ½ΠΎ, являСтся ΠΎΠ΄ΠΈΠ½Π°ΠΊΠΎΠ²Ρ‹ΠΌ.

    ВсС рСзисторы ΠΌΠΎΠΆΠ½ΠΎ Π·Π°ΠΌΠ΅Π½ΠΈΡ‚ΡŒ ΠΎΠ΄Π½ΠΈΠΌ эквивалСнтным рСзистором. Если Π²ΠΎΡΠΏΠΎΠ»ΡŒΠ·ΠΎΠ²Π°Ρ‚ΡŒΡΡ Π·Π°ΠΊΠΎΠ½ΠΎΠΌ Ома, ΠΌΠΎΠΆΠ½ΠΎ ΠΏΠΎΠ»ΡƒΡ‡ΠΈΡ‚ΡŒ Π²Ρ‹Ρ€Π°ΠΆΠ΅Π½ΠΈΠ΅ сопротивлСния. Если ΠΏΡ€ΠΈ ΠΏΠΎΡΠ»Π΅Π΄ΠΎΠ²Π°Ρ‚Π΅Π»ΡŒΠ½ΠΎΠΌ соСдинСнии сопротивлСния ΡΠΊΠ»Π°Π΄Ρ‹Π²Π°Π»ΠΈΡΡŒ, Ρ‚ΠΎ ΠΏΡ€ΠΈ ΠΏΠ°Ρ€Π°Π»Π»Π΅Π»ΡŒΠ½ΠΎΠΌ Π±ΡƒΠ΄ΡƒΡ‚ ΡΠΊΠ»Π°Π΄Ρ‹Π²Π°Ρ‚ΡŒΡΡ Π²Π΅Π»ΠΈΡ‡ΠΈΠ½Ρ‹ ΠΎΠ±Ρ€Π°Ρ‚Π½Ρ‹Π΅ ΠΈΠΌ, ΠΊΠ°ΠΊ записано Π² Ρ„ΠΎΡ€ΠΌΡƒΠ»Π΅ Π²Ρ‹ΡˆΠ΅.

    ΠŸΡ€ΠΈΠΌΠ΅Π½Π΅Π½ΠΈΠ΅

    Если Ρ€Π°ΡΡΠΌΠ°Ρ‚Ρ€ΠΈΠ²Π°Ρ‚ΡŒ соСдинСния Π² Π±Ρ‹Ρ‚ΠΎΠ²Ρ‹Ρ… условиях, Ρ‚ΠΎ Π² ΠΊΠ²Π°Ρ€Ρ‚ΠΈΡ€Π΅ Π»Π°ΠΌΠΏΡ‹ освСщСния, Π»ΡŽΡΡ‚Ρ€Ρ‹ Π΄ΠΎΠ»ΠΆΠ½Ρ‹ Π±Ρ‹Ρ‚ΡŒ соСдинСны ΠΏΠ°Ρ€Π°Π»Π»Π΅Π»ΡŒΠ½ΠΎ. Если ΠΈΡ… ΡΠΎΠ΅Π΄ΠΈΠ½ΠΈΡ‚ΡŒ ΠΏΠΎΡΠ»Π΅Π΄ΠΎΠ²Π°Ρ‚Π΅Π»ΡŒΠ½ΠΎ, Ρ‚ΠΎ ΠΏΡ€ΠΈ Π²ΠΊΠ»ΡŽΡ‡Π΅Π½ΠΈΠΈ ΠΎΠ΄Π½ΠΎΠΉ Π»Π°ΠΌΠΏΠΎΡ‡ΠΊΠΈ ΠΌΡ‹ Π²ΠΊΠ»ΡŽΡ‡ΠΈΠΌ всС ΠΎΡΡ‚Π°Π»ΡŒΠ½Ρ‹Π΅. ΠŸΡ€ΠΈ ΠΏΠ°Ρ€Π°Π»Π»Π΅Π»ΡŒΠ½ΠΎΠΌ ΠΆΠ΅ соСдинСнии ΠΌΡ‹ ΠΌΠΎΠΆΠ΅ΠΌ, добавляя ΡΠΎΠΎΡ‚Π²Π΅Ρ‚ΡΡ‚Π²ΡƒΡŽΡ‰ΠΈΠΉ Π²Ρ‹ΠΊΠ»ΡŽΡ‡Π°Ρ‚Π΅Π»ΡŒ Π² ΠΊΠ°ΠΆΠ΄ΡƒΡŽ ΠΈΠ· Π²Π΅Ρ‚Π²Π΅ΠΉ, Π²ΠΊΠ»ΡŽΡ‡Π°Ρ‚ΡŒ ΡΠΎΠΎΡ‚Π²Π΅Ρ‚ΡΡ‚Π²ΡƒΡŽΡ‰ΡƒΡŽ Π»Π°ΠΌΠΏΠΎΡ‡ΠΊΡƒ ΠΏΠΎ ΠΌΠ΅Ρ€Π΅ ТСлания. ΠŸΡ€ΠΈ этом Ρ‚Π°ΠΊΠΎΠ΅ Π²ΠΊΠ»ΡŽΡ‡Π΅Π½ΠΈΠ΅ ΠΎΠ΄Π½ΠΎΠΉ Π»Π°ΠΌΠΏΡ‹ Π½Π΅ влияСт Π½Π° ΠΎΡΡ‚Π°Π»ΡŒΠ½Ρ‹Π΅ Π»Π°ΠΌΠΏΡ‹.

    ВсС элСктричСскиС Π±Ρ‹Ρ‚ΠΎΠ²Ρ‹Π΅ устройства Π² ΠΊΠ²Π°Ρ€Ρ‚ΠΈΡ€Π΅ соСдинСны ΠΏΠ°Ρ€Π°Π»Π»Π΅Π»ΡŒΠ½ΠΎ Π² ΡΠ΅Ρ‚ΡŒ с напряТСниСм 220 Π’, ΠΈ ΠΏΠΎΠ΄ΠΊΠ»ΡŽΡ‡Π΅Π½Ρ‹ ΠΊ Ρ€Π°ΡΠΏΡ€Π΅Π΄Π΅Π»ΠΈΡ‚Π΅Π»ΡŒΠ½ΠΎΠΌΡƒ Ρ‰ΠΈΡ‚ΠΊΡƒ. Π”Ρ€ΡƒΠ³ΠΈΠΌΠΈ словами, ΠΏΠ°Ρ€Π°Π»Π»Π΅Π»ΡŒΠ½ΠΎΠ΅ соСдинСниС ΠΈΡΠΏΠΎΠ»ΡŒΠ·ΡƒΠ΅Ρ‚ΡΡ ΠΏΡ€ΠΈ нСобходимости ΠΏΠΎΠ΄ΠΊΠ»ΡŽΡ‡Π΅Π½ΠΈΡ элСктричСских устройств нСзависимо Π΄Ρ€ΡƒΠ³ ΠΎΡ‚ Π΄Ρ€ΡƒΠ³Π°. ΠŸΠΎΡΠ»Π΅Π΄ΠΎΠ²Π°Ρ‚Π΅Π»ΡŒΠ½ΠΎΠ΅ ΠΈ ΠΏΠ°Ρ€Π°Π»Π»Π΅Π»ΡŒΠ½ΠΎΠ΅ соСдинСниС ΠΈΠΌΠ΅ΡŽΡ‚ свои особСнности. Π‘ΡƒΡ‰Π΅ΡΡ‚Π²ΡƒΡŽΡ‚ Ρ‚Π°ΠΊΠΆΠ΅ ΡΠΌΠ΅ΡˆΠ°Π½Π½Ρ‹Π΅ соСдинСния.

    Π Π°Π±ΠΎΡ‚Π° Ρ‚ΠΎΠΊΠ°

    ΠŸΠΎΡΠ»Π΅Π΄ΠΎΠ²Π°Ρ‚Π΅Π»ΡŒΠ½ΠΎΠ΅ ΠΈ ΠΏΠ°Ρ€Π°Π»Π»Π΅Π»ΡŒΠ½ΠΎΠ΅ соСдинСниС, рассмотрСнноС Ρ€Π°Π½Π΅Π΅, Π±Ρ‹Π»ΠΎ справСдливо для Π²Π΅Π»ΠΈΡ‡ΠΈΠ½ напряТСния, сопротивлСния ΠΈ силы Ρ‚ΠΎΠΊΠ°, ΡΠ²Π»ΡΡŽΡ‰ΠΈΡ…ΡΡ основными. Π Π°Π±ΠΎΡ‚Π° Ρ‚ΠΎΠΊΠ° опрСдСляСтся ΠΏΠΎ Ρ„ΠΎΡ€ΠΌΡƒΠ»Π΅:

    А = I Ρ… U Ρ… t, Π³Π΄Π΅ А – Ρ€Π°Π±ΠΎΡ‚Π° Ρ‚ΠΎΠΊΠ°, t – врСмя тСчСния ΠΏΠΎ ΠΏΡ€ΠΎΠ²ΠΎΠ΄Π½ΠΈΠΊΡƒ.

    Для опрСдСлСния Ρ€Π°Π±ΠΎΡ‚Ρ‹ ΠΏΡ€ΠΈ ΠΏΠΎΡΠ»Π΅Π΄ΠΎΠ²Π°Ρ‚Π΅Π»ΡŒΠ½ΠΎΠΉ схСмС соСдинСния, Π½Π΅ΠΎΠ±Ρ…ΠΎΠ΄ΠΈΠΌΠΎ Π·Π°ΠΌΠ΅Π½ΠΈΡ‚ΡŒ Π² ΠΏΠ΅Ρ€Π²ΠΎΠ½Π°Ρ‡Π°Π»ΡŒΠ½ΠΎΠΌ Π²Ρ‹Ρ€Π°ΠΆΠ΅Π½ΠΈΠΈ напряТСниС. ΠŸΠΎΠ»ΡƒΡ‡Π°Π΅ΠΌ:

    А=I Ρ… (U1 + U2) Ρ… t

    РаскрываСм скобки ΠΈ ΠΏΠΎΠ»ΡƒΡ‡Π°Π΅ΠΌ, Ρ‡Ρ‚ΠΎ Π½Π° всСй схСмС Ρ€Π°Π±ΠΎΡ‚Π° опрСдСляСтся суммой Π½Π° ΠΊΠ°ΠΆΠ΄ΠΎΠΉ Π½Π°Π³Ρ€ΡƒΠ·ΠΊΠ΅.

    Π’ΠΎΡ‡Π½ΠΎ Ρ‚Π°ΠΊΠΆΠ΅ рассматриваСм ΠΏΠ°Ρ€Π°Π»Π»Π΅Π»ΡŒΠ½ΡƒΡŽ схСму соСдинСния. Волько мСняСм ΡƒΠΆΠ΅ Π½Π΅ напряТСниС, Π° силу Ρ‚ΠΎΠΊΠ°. ΠŸΠΎΠ»ΡƒΡ‡Π°Π΅Ρ‚ΡΡ Ρ€Π΅Π·ΡƒΠ»ΡŒΡ‚Π°Ρ‚:

    А = А1+А2

    ΠœΠΎΡ‰Π½ΠΎΡΡ‚ΡŒ Ρ‚ΠΎΠΊΠ°

    ΠŸΡ€ΠΈ рассмотрСнии Ρ„ΠΎΡ€ΠΌΡƒΠ»Ρ‹ мощности участка Ρ†Π΅ΠΏΠΈ снова Π½Π΅ΠΎΠ±Ρ…ΠΎΠ΄ΠΈΠΌΠΎ ΠΏΠΎΠ»ΡŒΠ·ΠΎΠ²Π°Ρ‚ΡŒΡΡ Ρ„ΠΎΡ€ΠΌΡƒΠ»ΠΎΠΉ:

    Π =U Ρ… I

    ПослС Π°Π½Π°Π»ΠΎΠ³ΠΈΡ‡Π½Ρ‹Ρ… рассуТдСний Π²Ρ‹Ρ…ΠΎΠ΄ΠΈΡ‚ Ρ€Π΅Π·ΡƒΠ»ΡŒΡ‚Π°Ρ‚, Ρ‡Ρ‚ΠΎ ΠΏΠΎΡΠ»Π΅Π΄ΠΎΠ²Π°Ρ‚Π΅Π»ΡŒΠ½ΠΎΠ΅ ΠΈ ΠΏΠ°Ρ€Π°Π»Π»Π΅Π»ΡŒΠ½ΠΎΠ΅ соСдинСниС ΠΌΠΎΠΆΠ½ΠΎ ΠΎΠΏΡ€Π΅Π΄Π΅Π»ΠΈΡ‚ΡŒ ΡΠ»Π΅Π΄ΡƒΡŽΡ‰Π΅ΠΉ Ρ„ΠΎΡ€ΠΌΡƒΠ»ΠΎΠΉ мощности:

    Π =Π 1 + Π 2

    Π”Ρ€ΡƒΠ³ΠΈΠΌΠΈ словами, ΠΏΡ€ΠΈ Π»ΡŽΠ±Ρ‹Ρ… схСмах общая ΠΌΠΎΡ‰Π½ΠΎΡΡ‚ΡŒ Ρ€Π°Π²Π½Π° суммС всСх мощностСй Π² схСмС. Π­Ρ‚ΠΈΠΌ ΠΌΠΎΠΆΠ½ΠΎ ΠΎΠ±ΡŠΡΡΠ½ΠΈΡ‚ΡŒ, Ρ‡Ρ‚ΠΎ Π½Π΅ рСкомСндуСтся Π²ΠΊΠ»ΡŽΡ‡Π°Ρ‚ΡŒ Π² ΠΊΠ²Π°Ρ€Ρ‚ΠΈΡ€Π΅ сразу нСсколько ΠΌΠΎΡ‰Π½Ρ‹Ρ… элСктричСских устройств, Ρ‚Π°ΠΊ ΠΊΠ°ΠΊ ΠΏΡ€ΠΎΠ²ΠΎΠ΄ΠΊΠ° ΠΌΠΎΠΆΠ΅Ρ‚ Π½Π΅ Π²Ρ‹Π΄Π΅Ρ€ΠΆΠ°Ρ‚ΡŒ Ρ‚Π°ΠΊΠΎΠΉ мощности.

    ВлияниС схСмы соСдинСния Π½Π° новогоднюю гирлянду

    ПослС пСрСгорания ΠΎΠ΄Π½ΠΎΠΉ Π»Π°ΠΌΠΏΡ‹ Π² гирляндС ΠΌΠΎΠΆΠ½ΠΎ ΠΎΠΏΡ€Π΅Π΄Π΅Π»ΠΈΡ‚ΡŒ Π²ΠΈΠ΄ схСмы соСдинСния. Если схСма ΠΏΠΎΡΠ»Π΅Π΄ΠΎΠ²Π°Ρ‚Π΅Π»ΡŒΠ½Π°Ρ, Ρ‚ΠΎ Π½Π΅ Π±ΡƒΠ΄Π΅Ρ‚ Π³ΠΎΡ€Π΅Ρ‚ΡŒ Π½ΠΈ ΠΎΠ΄Π½ΠΎΠΉ Π»Π°ΠΌΠΏΠΎΡ‡ΠΊΠΈ, Ρ‚Π°ΠΊ ΠΊΠ°ΠΊ ΡΠ³ΠΎΡ€Π΅Π²ΡˆΠ°Ρ Π»Π°ΠΌΠΏΠΎΡ‡ΠΊΠ° Ρ€Π°Π·Ρ€Ρ‹Π²Π°Π΅Ρ‚ ΠΎΠ±Ρ‰ΡƒΡŽ Ρ†Π΅ΠΏΡŒ. Π§Ρ‚ΠΎΠ±Ρ‹ Π²Ρ‹ΡΡΠ½ΠΈΡ‚ΡŒ, какая ΠΈΠΌΠ΅Π½Π½ΠΎ Π»Π°ΠΌΠΏΠΎΡ‡ΠΊΠ° сгорСла, Π½ΡƒΠΆΠ½ΠΎ ΠΏΡ€ΠΎΠ²Π΅Ρ€ΡΡ‚ΡŒ всС подряд. Π”Π°Π»Π΅Π΅, Π·Π°ΠΌΠ΅Π½ΠΈΡ‚ΡŒ Π½Π΅ΠΈΡΠΏΡ€Π°Π²Π½ΡƒΡŽ Π»Π°ΠΌΠΏΡƒ, гирлянда Π±ΡƒΠ΄Π΅Ρ‚ Ρ„ΡƒΠ½ΠΊΡ†ΠΈΠΎΠ½ΠΈΡ€ΠΎΠ²Π°Ρ‚ΡŒ.

    ΠŸΡ€ΠΈ ΠΏΡ€ΠΈΠΌΠ΅Π½Π΅Π½ΠΈΠΈ ΠΏΠ°Ρ€Π°Π»Π»Π΅Π»ΡŒΠ½ΠΎΠΉ схСмы соСдинСния гирлянда Π±ΡƒΠ΄Π΅Ρ‚ ΠΏΡ€ΠΎΠ΄ΠΎΠ»ΠΆΠ°Ρ‚ΡŒ Ρ€Π°Π±ΠΎΡ‚Π°Ρ‚ΡŒ, Π΄Π°ΠΆΠ΅ Ссли ΠΎΠ΄Π½Π° ΠΈΠ»ΠΈ нСсколько Π»Π°ΠΌΠΏ сгорСли, Ρ‚Π°ΠΊ ΠΊΠ°ΠΊ Ρ†Π΅ΠΏΡŒ Π½Π΅ Ρ€Π°Π·ΠΎΡ€Π²Π°Π½Π° ΠΏΠΎΠ»Π½ΠΎΡΡ‚ΡŒΡŽ, Π° Ρ‚ΠΎΠ»ΡŒΠΊΠΎ ΠΎΠ΄ΠΈΠ½ нСбольшой ΠΏΠ°Ρ€Π°Π»Π»Π΅Π»ΡŒΠ½Ρ‹ΠΉ участок. Для восстановлСния Ρ‚Π°ΠΊΠΎΠΉ гирлянды достаточно ΡƒΠ²ΠΈΠ΄Π΅Ρ‚ΡŒ, ΠΊΠ°ΠΊΠΈΠ΅ Π»Π°ΠΌΠΏΡ‹ Π½Π΅ горят, ΠΈ Π·Π°ΠΌΠ΅Π½ΠΈΡ‚ΡŒ ΠΈΡ….

    ΠŸΠΎΡΠ»Π΅Π΄ΠΎΠ²Π°Ρ‚Π΅Π»ΡŒΠ½ΠΎΠ΅ ΠΈ ΠΏΠ°Ρ€Π°Π»Π»Π΅Π»ΡŒΠ½ΠΎΠ΅ соСдинСниС для кондСнсаторов

    ΠŸΡ€ΠΈ ΠΏΠΎΡΠ»Π΅Π΄ΠΎΠ²Π°Ρ‚Π΅Π»ΡŒΠ½ΠΎΠΉ схСмС Π²ΠΎΠ·Π½ΠΈΠΊΠ°Π΅Ρ‚ такая ΠΊΠ°Ρ€Ρ‚ΠΈΠ½Π°: заряды ΠΎΡ‚ ΠΏΠΎΠ»ΠΎΠΆΠΈΡ‚Π΅Π»ΡŒΠ½ΠΎΠ³ΠΎ полюса источника питания ΠΈΠ΄ΡƒΡ‚ Ρ‚ΠΎΠ»ΡŒΠΊΠΎ Π½Π° Π½Π°Ρ€ΡƒΠΆΠ½Ρ‹Π΅ пластины ΠΊΡ€Π°ΠΉΠ½ΠΈΡ… кондСнсаторов. ΠšΠΎΠ½Π΄Π΅Π½ΡΠ°Ρ‚ΠΎΡ€Ρ‹, находящиСся ΠΌΠ΅ΠΆΠ΄Ρƒ Π½ΠΈΠΌΠΈ, ΠΏΠ΅Ρ€Π΅Π΄Π°ΡŽΡ‚ заряд ΠΏΠΎ Ρ†Π΅ΠΏΠΈ. Π­Ρ‚ΠΈΠΌ ΠΎΠ±ΡŠΡΡΠ½ΡΠ΅Ρ‚ΡΡ появлСниС Π½Π° всСх пластинах Ρ€Π°Π²Π½Ρ‹Ρ… зарядов с Ρ€Π°Π·Π½Ρ‹ΠΌΠΈ Π·Π½Π°ΠΊΠ°ΠΌΠΈ. Π˜ΡΡ…ΠΎΠ΄Ρ ΠΈΠ· этого, заряд любого кондСнсатора, соСдинСнного ΠΏΠΎ ΠΏΠΎΡΠ»Π΅Π΄ΠΎΠ²Π°Ρ‚Π΅Π»ΡŒΠ½ΠΎΠΉ схСмС, ΠΌΠΎΠΆΠ½ΠΎ Π²Ρ‹Ρ€Π°Π·ΠΈΡ‚ΡŒ Ρ‚Π°ΠΊΠΎΠΉ Ρ„ΠΎΡ€ΠΌΡƒΠ»ΠΎΠΉ:

    qΠΎΠ±Ρ‰= q1 = q2 = q3

    Для опрСдСлСния напряТСния Π½Π° любом кондСнсаторС, Π½Π΅ΠΎΠ±Ρ…ΠΎΠ΄ΠΈΠΌΠ° Ρ„ΠΎΡ€ΠΌΡƒΠ»Π°:

    U= q/Π‘

    Π“Π΄Π΅ Π‘ β€” Π΅ΠΌΠΊΠΎΡΡ‚ΡŒ. Π‘ΡƒΠΌΠΌΠ°Ρ€Π½ΠΎΠ΅ напряТСниС выраТаСтся Ρ‚Π°ΠΊΠΈΠΌ ΠΆΠ΅ Π·Π°ΠΊΠΎΠ½ΠΎΠΌ, ΠΊΠΎΡ‚ΠΎΡ€Ρ‹ΠΉ ΠΏΠΎΠ΄Ρ…ΠΎΠ΄ΠΈΡ‚ для сопротивлСний. ΠŸΠΎΡΡ‚ΠΎΠΌΡƒ ΠΏΠΎΠ»ΡƒΡ‡Π°Π΅ΠΌ Ρ„ΠΎΡ€ΠΌΡƒΠ»Ρƒ Смкости:

    Π‘= q/(U1 + U2 + U3)

    Π§Ρ‚ΠΎΠ±Ρ‹ ΡΠ΄Π΅Π»Π°Ρ‚ΡŒ эту Ρ„ΠΎΡ€ΠΌΡƒΠ»Ρƒ ΠΏΡ€ΠΎΡ‰Π΅, ΠΌΠΎΠΆΠ½ΠΎ ΠΏΠ΅Ρ€Π΅Π²Π΅Ρ€Π½ΡƒΡ‚ΡŒ Π΄Ρ€ΠΎΠ±ΠΈ ΠΈ Π·Π°ΠΌΠ΅Π½ΠΈΡ‚ΡŒ ΠΎΡ‚Π½ΠΎΡˆΠ΅Π½ΠΈΠ΅ разности ΠΏΠΎΡ‚Π΅Π½Ρ†ΠΈΠ°Π»ΠΎΠ² ΠΊ заряду Смкости. Π’ Ρ€Π΅Π·ΡƒΠ»ΡŒΡ‚Π°Ρ‚Π΅ ΠΏΠΎΠ»ΡƒΡ‡Π°Π΅ΠΌ:

    1/Π‘= 1/Π‘1 + 1/Π‘2 + 1/C3

    НСмного ΠΈΠ½Π°Ρ‡Π΅ рассчитываСтся ΠΏΠ°Ρ€Π°Π»Π»Π΅Π»ΡŒΠ½ΠΎΠ΅ соСдинСниС кондСнсаторов.

    ΠžΠ±Ρ‰ΠΈΠΉ заряд вычисляСтся ΠΊΠ°ΠΊ сумма всСх зарядов, Π½Π°ΠΊΠΎΠΏΠΈΠ²ΡˆΠΈΡ…ΡΡ Π½Π° пластинах всСх кондСнсаторов. А Π²Π΅Π»ΠΈΡ‡ΠΈΠ½Π° напряТСния Ρ‚Π°ΠΊΠΆΠ΅ вычисляСтся ΠΏΠΎ ΠΎΠ±Ρ‰ΠΈΠΌ Π·Π°ΠΊΠΎΠ½Π°ΠΌ. Π’ связи с этим Ρ„ΠΎΡ€ΠΌΡƒΠ»Π° суммарной Смкости ΠΏΡ€ΠΈ ΠΏΠ°Ρ€Π°Π»Π»Π΅Π»ΡŒΠ½ΠΎΠΉ схСмС соСдинСния выглядит Ρ‚Π°ΠΊ:

    Π‘= (q1 + q2 + q3)/U

    Π­Ρ‚ΠΎ Π·Π½Π°Ρ‡Π΅Π½ΠΈΠ΅ рассчитываСтся ΠΊΠ°ΠΊ сумма ΠΊΠ°ΠΆΠ΄ΠΎΠ³ΠΎ ΠΏΡ€ΠΈΠ±ΠΎΡ€Π° Π² схСмС:

    Π‘=Π‘1 + Π‘2 + Π‘3

    БмСшанноС соСдинСниС ΠΏΡ€ΠΎΠ²ΠΎΠ΄Π½ΠΈΠΊΠΎΠ²

    Π’ элСктричСской схСмС участки Ρ†Π΅ΠΏΠΈ ΠΌΠΎΠ³ΡƒΡ‚ ΠΈΠΌΠ΅Ρ‚ΡŒ ΠΈ ΠΏΠΎΡΠ»Π΅Π΄ΠΎΠ²Π°Ρ‚Π΅Π»ΡŒΠ½ΠΎΠ΅ ΠΈ ΠΏΠ°Ρ€Π°Π»Π»Π΅Π»ΡŒΠ½ΠΎΠ΅ соСдинСниС, ΠΏΠ΅Ρ€Π΅ΠΏΠ»Π΅Ρ‚Π°ΡŽΡ‰ΠΈΡ…ΡΡ ΠΌΠ΅ΠΆΠ΄Ρƒ собой. Но всС Π·Π°ΠΊΠΎΠ½Ρ‹, рассмотрСнныС Π²Ρ‹ΡˆΠ΅ для ΠΎΡ‚Π΄Π΅Π»ΡŒΠ½Ρ‹Ρ… Π²ΠΈΠ΄ΠΎΠ² соСдинСний, справСдливы ΠΏΠΎ-ΠΏΡ€Π΅ΠΆΠ½Π΅ΠΌΡƒ, ΠΈ ΠΈΡΠΏΠΎΠ»ΡŒΠ·ΡƒΡŽΡ‚ΡΡ ΠΏΠΎ этапам.

    Π‘Π½Π°Ρ‡Π°Π»Π° Π½ΡƒΠΆΠ½ΠΎ мыслСнно Ρ€Π°Π·Π»ΠΎΠΆΠΈΡ‚ΡŒ схСму Π½Π° ΠΎΡ‚Π΄Π΅Π»ΡŒΠ½Ρ‹Π΅ части. Для Π»ΡƒΡ‡ΡˆΠ΅Π³ΠΎ прСдставлСния Π΅Π΅ Ρ€ΠΈΡΡƒΡŽΡ‚ Π½Π° Π±ΡƒΠΌΠ°Π³Π΅. Рассмотрим наш ΠΏΡ€ΠΈΠΌΠ΅Ρ€ ΠΏΠΎ ΠΈΠ·ΠΎΠ±Ρ€Π°ΠΆΠ΅Π½Π½ΠΎΠΉ Π²Ρ‹ΡˆΠ΅ схСмС.

    Π£Π΄ΠΎΠ±Π½Π΅Π΅ всСго Π΅Π΅ ΠΈΠ·ΠΎΠ±Ρ€Π°Π·ΠΈΡ‚ΡŒ, начиная с Ρ‚ΠΎΡ‡Π΅ΠΊ Π‘ ΠΈ Π’. Они Ρ€Π°ΡΡΡ‚Π°Π²Π»ΡΡŽΡ‚ΡΡ Π½Π° Π½Π΅ΠΊΠΎΡ‚ΠΎΡ€ΠΎΠΌ расстоянии ΠΌΠ΅ΠΆΠ΄Ρƒ собой ΠΈ ΠΎΡ‚ края листа Π±ΡƒΠΌΠ°Π³ΠΈ. Π‘ Π»Π΅Π²ΠΎΠΉ стороны ΠΊ Ρ‚ΠΎΡ‡ΠΊΠ΅ Π‘ ΠΏΠΎΠ΄ΠΊΠ»ΡŽΡ‡Π°Π΅Ρ‚ΡΡ ΠΎΠ΄ΠΈΠ½ ΠΏΡ€ΠΎΠ²ΠΎΠ΄, Π° справа отходят Π΄Π²Π° ΠΏΡ€ΠΎΠ²ΠΎΠ΄Π°. Π’ΠΎΡ‡ΠΊΠ° Π’ Π½Π°ΠΎΠ±ΠΎΡ€ΠΎΡ‚, слСва ΠΈΠΌΠ΅Π΅Ρ‚ Π΄Π²Π΅ Π²Π΅Ρ‚ΠΊΠΈ, Π° послС Ρ‚ΠΎΡ‡ΠΊΠΈ ΠΎΡ‚Ρ…ΠΎΠ΄ΠΈΡ‚ ΠΎΠ΄ΠΈΠ½ ΠΏΡ€ΠΎΠ²ΠΎΠ΄.

    Π”Π°Π»Π΅Π΅ Π½ΡƒΠΆΠ½ΠΎ ΠΈΠ·ΠΎΠ±Ρ€Π°Π·ΠΈΡ‚ΡŒ пространство ΠΌΠ΅ΠΆΠ΄Ρƒ Ρ‚ΠΎΡ‡ΠΊΠ°ΠΌΠΈ. По Π²Π΅Ρ€Ρ…Π½Π΅ΠΌΡƒ ΠΏΡ€ΠΎΠ²ΠΎΠ΄Π½ΠΈΠΊΡƒ располоТСны 3 сопротивлСния с условными значСниями 2, 3, 4. Π‘Π½ΠΈΠ·Ρƒ Π±ΡƒΠ΄Π΅Ρ‚ ΠΈΠ΄Ρ‚ΠΈ Ρ‚ΠΎΠΊ с индСксом 5. ΠŸΠ΅Ρ€Π²Ρ‹Π΅ 3 сопротивлСния Π²ΠΊΠ»ΡŽΡ‡Π΅Π½Ρ‹ Π² схСму ΠΏΠΎΡΠ»Π΅Π΄ΠΎΠ²Π°Ρ‚Π΅Π»ΡŒΠ½ΠΎ, Π° пятый рСзистор ΠΏΠΎΠ΄ΠΊΠ»ΡŽΡ‡Π΅Π½ ΠΏΠ°Ρ€Π°Π»Π»Π΅Π»ΡŒΠ½ΠΎ.

    ΠžΡΡ‚Π°Π»ΡŒΠ½Ρ‹Π΅ Π΄Π²Π° сопротивлСния (ΠΏΠ΅Ρ€Π²Ρ‹ΠΉ ΠΈ ΡˆΠ΅ΡΡ‚ΠΎΠΉ) ΠΏΠΎΠ΄ΠΊΠ»ΡŽΡ‡Π΅Π½Ρ‹ ΠΏΠΎΡΠ»Π΅Π΄ΠΎΠ²Π°Ρ‚Π΅Π»ΡŒΠ½ΠΎ с рассматриваСмым Π½Π°ΠΌΠΈ участком Π‘-Π’. ΠŸΠΎΡΡ‚ΠΎΠΌΡƒ схСму дополняСм 2-мя ΠΏΡ€ΡΠΌΠΎΡƒΠ³ΠΎΠ»ΡŒΠ½ΠΈΠΊΠ°ΠΌΠΈ ΠΏΠΎ сторонам ΠΎΡ‚ Π²Ρ‹Π±Ρ€Π°Π½Π½Ρ‹Ρ… Ρ‚ΠΎΡ‡Π΅ΠΊ.

    Π’Π΅ΠΏΠ΅Ρ€ΡŒ ΠΈΡΠΏΠΎΠ»ΡŒΠ·ΡƒΠ΅ΠΌ Ρ„ΠΎΡ€ΠΌΡƒΠ»Ρƒ расчСта сопротивлСния:

    • ΠŸΠ΅Ρ€Π²Π°Ρ Ρ„ΠΎΡ€ΠΌΡƒΠ»Π° для ΠΏΠΎΡΠ»Π΅Π΄ΠΎΠ²Π°Ρ‚Π΅Π»ΡŒΠ½ΠΎΠ³ΠΎ Π²ΠΈΠ΄Π° соСдинСния.
    • Π”Π°Π»Π΅Π΅, для ΠΏΠ°Ρ€Π°Π»Π»Π΅Π»ΡŒΠ½ΠΎΠΉ схСмы.
    • И ΠΎΠΊΠΎΠ½Ρ‡Π°Ρ‚Π΅Π»ΡŒΠ½ΠΎ для ΠΏΠΎΡΠ»Π΅Π΄ΠΎΠ²Π°Ρ‚Π΅Π»ΡŒΠ½ΠΎΠΉ схСмы.

    Аналогичным ΠΎΠ±Ρ€Π°Π·ΠΎΠΌ ΠΌΠΎΠΆΠ½ΠΎ Ρ€Π°Π·Π»ΠΎΠΆΠΈΡ‚ΡŒ Π½Π° ΠΎΡ‚Π΄Π΅Π»ΡŒΠ½Ρ‹Π΅ схСмы Π»ΡŽΠ±ΡƒΡŽ ΡΠ»ΠΎΠΆΠ½ΡƒΡŽ схСму, Π²ΠΊΠ»ΡŽΡ‡Π°Ρ соСдинСния Π½Π΅ Ρ‚ΠΎΠ»ΡŒΠΊΠΎ ΠΏΡ€ΠΎΠ²ΠΎΠ΄Π½ΠΈΠΊΠΎΠ² Π² Π²ΠΈΠ΄Π΅ сопротивлСний, Π½ΠΎ ΠΈ кондСнсаторов. Π§Ρ‚ΠΎΠ±Ρ‹ Π½Π°ΡƒΡ‡ΠΈΡ‚ΡŒΡΡ Π²Π»Π°Π΄Π΅Ρ‚ΡŒ ΠΏΡ€ΠΈΠ΅ΠΌΠ°ΠΌΠΈ расчСта ΠΏΠΎ Ρ€Π°Π·Π½Ρ‹ΠΌ Π²ΠΈΠ΄Π°ΠΌ схСм, Π½Π΅ΠΎΠ±Ρ…ΠΎΠ΄ΠΈΠΌΠΎ ΠΏΠΎΡ‚Ρ€Π΅Π½ΠΈΡ€ΠΎΠ²Π°Ρ‚ΡŒΡΡ Π½Π° ΠΏΡ€Π°ΠΊΡ‚ΠΈΠΊΠ΅, Π²Ρ‹ΠΏΠΎΠ»Π½ΠΈΠ² нСсколько Π·Π°Π΄Π°Π½ΠΈΠΉ.

    ΠŸΡ€ΠΎΠ²ΠΎΠ΄Π½ΠΈΠΊΠΈ Π² элСктричСских цСпях ΠΌΠΎΠ³ΡƒΡ‚ ΡΠΎΠ΅Π΄ΠΈΠ½ΡΡ‚ΡŒΡΡ ΠΏΠΎΡΠ»Π΅Π΄ΠΎΠ²Π°Ρ‚Π΅Π»ΡŒΠ½ΠΎ ΠΈ ΠΏΠ°Ρ€Π°Π»Π»Π΅Π»ΡŒΠ½ΠΎ.

    ΠŸΡ€ΠΈ ΠΏΠΎΡΠ»Π΅Π΄ΠΎΠ²Π°Ρ‚Π΅Π»ΡŒΠ½ΠΎΠΌ соСдинСнии ΠΏΡ€ΠΎΠ²ΠΎΠ΄Π½ΠΈΠΊΠΎΠ² (рис. 1.9.1) сила Ρ‚ΠΎΠΊΠ° Π²ΠΎ всСх ΠΏΡ€ΠΎΠ²ΠΎΠ΄Π½ΠΈΠΊΠ°Ρ… ΠΎΠ΄ΠΈΠ½Π°ΠΊΠΎΠ²Π°:

    Рисунок 1.9.1.

    По Π·Π°ΠΊΠΎΠ½Ρƒ Ома, напряТСния ΠΈ Π½Π° ΠΏΡ€ΠΎΠ²ΠΎΠ΄Π½ΠΈΠΊΠ°Ρ… Ρ€Π°Π²Π½Ρ‹

    ΠžΠ±Ρ‰Π΅Π΅ напряТСниС Π½Π° ΠΎΠ±ΠΎΠΈΡ… ΠΏΡ€ΠΎΠ²ΠΎΠ΄Π½ΠΈΠΊΠ°Ρ… Ρ€Π°Π²Π½ΠΎ суммС напряТСний 1 ΠΈ 2:

    Π³Π΄Π΅ – элСктричСскоС сопротивлСниС всСй Ρ†Π΅ΠΏΠΈ. ΠžΡ‚ΡΡŽΠ΄Π° слСдуСт:

    ΠŸΡ€ΠΈ ΠΏΠΎΡΠ»Π΅Π΄ΠΎΠ²Π°Ρ‚Π΅Π»ΡŒΠ½ΠΎΠΌ соСдинСнии ΠΏΠΎΠ»Π½ΠΎΠ΅ сопротивлСниС Ρ†Π΅ΠΏΠΈ Ρ€Π°Π²Π½ΠΎ суммС сопротивлСний ΠΎΡ‚Π΄Π΅Π»ΡŒΠ½Ρ‹Ρ… ΠΏΡ€ΠΎΠ²ΠΎΠ΄Π½ΠΈΠΊΠΎΠ².

    Π­Ρ‚ΠΎΡ‚ Ρ€Π΅Π·ΡƒΠ»ΡŒΡ‚Π°Ρ‚ справСдлив для любого числа ΠΏΠΎΡΠ»Π΅Π΄ΠΎΠ²Π°Ρ‚Π΅Π»ΡŒΠ½ΠΎ соСдинСнных ΠΏΡ€ΠΎΠ²ΠΎΠ΄Π½ΠΈΠΊΠΎΠ².

    ΠŸΡ€ΠΈ ΠΏΠ°Ρ€Π°Π»Π»Π΅Π»ΡŒΠ½ΠΎΠΌ соСдинСнии (рис. 1.9.2) напряТСния 1 ΠΈ 2 Π½Π° ΠΎΠ±ΠΎΠΈΡ… ΠΏΡ€ΠΎΠ²ΠΎΠ΄Π½ΠΈΠΊΠ°Ρ… ΠΎΠ΄ΠΈΠ½Π°ΠΊΠΎΠ²Ρ‹:

    Π‘ΡƒΠΌΠΌΠ° Ρ‚ΠΎΠΊΠΎΠ² 1 + 2, ΠΏΡ€ΠΎΡ‚Π΅ΠΊΠ°ΡŽΡ‰ΠΈΡ… ΠΏΠΎ ΠΎΠ±ΠΎΠΈΠΌ ΠΏΡ€ΠΎΠ²ΠΎΠ΄Π½ΠΈΠΊΠ°ΠΌ, Ρ€Π°Π²Π½Π° Ρ‚ΠΎΠΊΡƒ Π² Π½Π΅Ρ€Π°Π·Π²Π΅Ρ‚Π²Π»Π΅Π½Π½ΠΎΠΉ Ρ†Π΅ΠΏΠΈ:

    Π­Ρ‚ΠΎΡ‚ Ρ€Π΅Π·ΡƒΠ»ΡŒΡ‚Π°Ρ‚ слСдуСт ΠΈΠ· Ρ‚ΠΎΠ³ΠΎ, Ρ‡Ρ‚ΠΎ Π² Ρ‚ΠΎΡ‡ΠΊΠ°Ρ… развСтвлСния Ρ‚ΠΎΠΊΠΎΠ² (ΡƒΠ·Π»Ρ‹ ΠΈ ) Π² Ρ†Π΅ΠΏΠΈ постоянного Ρ‚ΠΎΠΊΠ° Π½Π΅ ΠΌΠΎΠ³ΡƒΡ‚ Π½Π°ΠΊΠ°ΠΏΠ»ΠΈΠ²Π°Ρ‚ΡŒΡΡ заряды. НапримСр, ΠΊ ΡƒΠ·Π»Ρƒ Π·Π° врСмя Ξ” ΠΏΠΎΠ΄Ρ‚Π΅ΠΊΠ°Π΅Ρ‚ заряд Ξ”, Π° ΡƒΡ‚Π΅ΠΊΠ°Π΅Ρ‚ ΠΎΡ‚ ΡƒΠ·Π»Π° Π·Π° Ρ‚ΠΎ ΠΆΠ΅ врСмя заряд 1Ξ” + 2Ξ”. Π‘Π»Π΅Π΄ΠΎΠ²Π°Ρ‚Π΅Π»ΡŒΠ½ΠΎ, = 1 + 2.

    Рисунок 1.9.2.

    Записывая Π½Π° основании Π·Π°ΠΊΠΎΠ½Π° Ома

    Π³Π΄Π΅ – элСктричСскоС сопротивлСниС всСй Ρ†Π΅ΠΏΠΈ, ΠΏΠΎΠ»ΡƒΡ‡ΠΈΠΌ

    ΠŸΡ€ΠΈ ΠΏΠ°Ρ€Π°Π»Π»Π΅Π»ΡŒΠ½ΠΎΠΌ соСдинСнии ΠΏΡ€ΠΎΠ²ΠΎΠ΄Π½ΠΈΠΊΠΎΠ² Π²Π΅Π»ΠΈΡ‡ΠΈΠ½Π°, обратная ΠΎΠ±Ρ‰Π΅ΠΌΡƒ ΡΠΎΠΏΡ€ΠΎΡ‚ΠΈΠ²Π»Π΅Π½ΠΈΡŽ Ρ†Π΅ΠΏΠΈ, Ρ€Π°Π²Π½Π° суммС Π²Π΅Π»ΠΈΡ‡ΠΈΠ½, ΠΎΠ±Ρ€Π°Ρ‚Π½Ρ‹Ρ… сопротивлСниям ΠΏΠ°Ρ€Π°Π»Π»Π΅Π»ΡŒΠ½ΠΎ Π²ΠΊΠ»ΡŽΡ‡Π΅Π½Π½Ρ‹Ρ… ΠΏΡ€ΠΎΠ²ΠΎΠ΄Π½ΠΈΠΊΠΎΠ².

    Π­Ρ‚ΠΎΡ‚ Ρ€Π΅Π·ΡƒΠ»ΡŒΡ‚Π°Ρ‚ справСдлив для любого числа ΠΏΠ°Ρ€Π°Π»Π»Π΅Π»ΡŒΠ½ΠΎ Π²ΠΊΠ»ΡŽΡ‡Π΅Π½Π½Ρ‹Ρ… ΠΏΡ€ΠΎΠ²ΠΎΠ΄Π½ΠΈΠΊΠΎΠ².

    Π€ΠΎΡ€ΠΌΡƒΠ»Ρ‹ для ΠΏΠΎΡΠ»Π΅Π΄ΠΎΠ²Π°Ρ‚Π΅Π»ΡŒΠ½ΠΎΠ³ΠΎ ΠΈ ΠΏΠ°Ρ€Π°Π»Π»Π΅Π»ΡŒΠ½ΠΎΠ³ΠΎ соСдинСния ΠΏΡ€ΠΎΠ²ΠΎΠ΄Π½ΠΈΠΊΠΎΠ² ΠΏΠΎΠ·Π²ΠΎΠ»ΡΡŽΡ‚ Π²ΠΎ ΠΌΠ½ΠΎΠ³ΠΈΡ… случаях Ρ€Π°ΡΡΡ‡ΠΈΡ‚Ρ‹Π²Π°Ρ‚ΡŒ сопротивлСниС слоТной Ρ†Π΅ΠΏΠΈ, состоящСй ΠΈΠ· ΠΌΠ½ΠΎΠ³ΠΈΡ… рСзисторов. На рис. 1.9.3 ΠΏΡ€ΠΈΠ²Π΅Π΄Π΅Π½ ΠΏΡ€ΠΈΠΌΠ΅Ρ€ Ρ‚Π°ΠΊΠΎΠΉ слоТной Ρ†Π΅ΠΏΠΈ ΠΈ ΡƒΠΊΠ°Π·Π°Π½Π° ΠΏΠΎΡΠ»Π΅Π΄ΠΎΠ²Π°Ρ‚Π΅Π»ΡŒΠ½ΠΎΡΡ‚ΡŒ вычислСний.

    Рисунок 1.9.3.

    Π‘Π»Π΅Π΄ΡƒΠ΅Ρ‚ ΠΎΡ‚ΠΌΠ΅Ρ‚ΠΈΡ‚ΡŒ, Ρ‡Ρ‚ΠΎ Π΄Π°Π»Π΅ΠΊΠΎ Π½Π΅ всС слоТныС Ρ†Π΅ΠΏΠΈ, состоящиС ΠΈΠ· ΠΏΡ€ΠΎΠ²ΠΎΠ΄Π½ΠΈΠΊΠΎΠ² с Ρ€Π°Π·Π»ΠΈΡ‡Π½Ρ‹ΠΌΠΈ сопротивлСниями, ΠΌΠΎΠ³ΡƒΡ‚ Π±Ρ‹Ρ‚ΡŒ рассчитаны с ΠΏΠΎΠΌΠΎΡ‰ΡŒΡŽ Ρ„ΠΎΡ€ΠΌΡƒΠ» для ΠΏΠΎΡΠ»Π΅Π΄ΠΎΠ²Π°Ρ‚Π΅Π»ΡŒΠ½ΠΎΠ³ΠΎ ΠΈ ΠΏΠ°Ρ€Π°Π»Π»Π΅Π»ΡŒΠ½ΠΎΠ³ΠΎ соСдинСния. На рис. 1.9.4 ΠΏΡ€ΠΈΠ²Π΅Π΄Π΅Π½ ΠΏΡ€ΠΈΠΌΠ΅Ρ€ элСктричСской Ρ†Π΅ΠΏΠΈ, ΠΊΠΎΡ‚ΠΎΡ€ΡƒΡŽ нСльзя Ρ€Π°ΡΡΡ‡ΠΈΡ‚Π°Ρ‚ΡŒ ΡƒΠΊΠ°Π·Π°Π½Π½Ρ‹ΠΌ Π²Ρ‹ΡˆΠ΅ ΠΌΠ΅Ρ‚ΠΎΠ΄ΠΎΠΌ.

    Рисунок 1.9.4.

    Π¦Π΅ΠΏΠΈ, ΠΏΠΎΠ΄ΠΎΠ±Π½Ρ‹Π΅ ΠΈΠ·ΠΎΠ±Ρ€Π°ΠΆΠ΅Π½Π½ΠΎΠΉ Π½Π° рис. 1.9.4, Π° Ρ‚Π°ΠΊΠΆΠ΅ Ρ†Π΅ΠΏΠΈ с развСтвлСниями, содСрТащиС нСсколько источников, Ρ€Π°ΡΡΡ‡ΠΈΡ‚Ρ‹Π²Π°ΡŽΡ‚ΡΡ с ΠΏΠΎΠΌΠΎΡ‰ΡŒΡŽ ΠΏΡ€Π°Π²ΠΈΠ» ΠšΠΈΡ€Ρ…Π³ΠΎΡ„Π°.

    Π£Ρ€ΠΎΠΊ 29. Π·Π°ΠΊΠΎΠ½ ΠΎΠΌΠ° для участка Ρ†Π΅ΠΏΠΈ. соСдинСния ΠΏΡ€ΠΎΠ²ΠΎΠ΄Π½ΠΈΠΊΠΎΠ² – Π€ΠΈΠ·ΠΈΠΊΠ° – 10 класс

    Ѐизика, 10 класс

    Π£Ρ€ΠΎΠΊ 29. Π—Π°ΠΊΠΎΠ½ Ома для участка Ρ†Π΅ΠΏΠΈ. БоСдинСния ΠΏΡ€ΠΎΠ²ΠΎΠ΄Π½ΠΈΠΊΠΎΠ²

    ΠŸΠ΅Ρ€Π΅Ρ‡Π΅Π½ΡŒ вопросов, рассматриваСмых Π½Π° ΡƒΡ€ΠΎΠΊΠ΅:

    1. условия, Π½Π΅ΠΎΠ±Ρ…ΠΎΠ΄ΠΈΠΌΡ‹Π΅ для сущСствования элСктричСского Ρ‚ΠΎΠΊΠ°;
    2. постоянный элСктричСский Ρ‚ΠΎΠΊ;
    3. Π·Π°ΠΊΠΎΠ½ Ома для участка Ρ†Π΅ΠΏΠΈ;
    4. Ρ„ΠΎΡ€ΠΌΡƒΠ»Π° расчСта сопротивлСния ΠΏΡ€ΠΎΠ²ΠΎΠ΄Π½ΠΈΠΊΠ° с ΡƒΡ‡Π΅Ρ‚ΠΎΠΌ свойств ΠΌΠ°Ρ‚Π΅Ρ€ΠΈΠ°Π»Π° ΠΏΡ€ΠΎΠ²ΠΎΠ΄Π½ΠΈΠΊΠ° ΠΈ Π΅Π³ΠΎ гСомСтричСских Ρ€Π°Π·ΠΌΠ΅Ρ€ΠΎΠ²;
    5. Ρ‚ΠΈΠΏΡ‹ соСдинСний ΠΏΡ€ΠΎΠ²ΠΎΠ΄Π½ΠΈΠΊΠΎΠ² ΠΈ Ρ„ΠΎΡ€ΠΌΡƒΠ»Ρ‹ расчСта ΠΏΠ°Ρ€Π°ΠΌΠ΅Ρ‚Ρ€ΠΎΠ² элСктричСской Ρ†Π΅ΠΏΠΈ для ΠΊΠ°ΠΆΠ΄ΠΎΠ³ΠΎ Ρ‚ΠΈΠΏΠ°.

    Глоссарий ΠΏΠΎ Ρ‚Π΅ΠΌΠ΅.

    Π‘ΠΈΠ»Π° Ρ‚ΠΎΠΊΠ° I – скалярная Π²Π΅Π»ΠΈΡ‡ΠΈΠ½Π°, равная ΠΎΡ‚Π½ΠΎΡˆΠ΅Π½ΠΈΡŽ заряда q, ΠΏΡ€ΠΎΡˆΠ΅Π΄ΡˆΠ΅Π³ΠΎ Ρ‡Π΅Ρ€Π΅Π· ΠΏΠΎΠΏΠ΅Ρ€Π΅Ρ‡Π½ΠΎΠ΅ сСчСниС ΠΏΡ€ΠΎΠ²ΠΎΠ΄Π½ΠΈΠΊΠ°, ΠΊ ΠΏΡ€ΠΎΠΌΠ΅ΠΆΡƒΡ‚ΠΊΡƒ Π²Ρ€Π΅ΠΌΠ΅Π½ΠΈ t, Π² Ρ‚Π΅Ρ‡Π΅Π½ΠΈΠ΅ ΠΊΠΎΡ‚ΠΎΡ€ΠΎΠ³ΠΎ ΡˆΡ‘Π» Ρ‚ΠΎΠΊ.

    ΠŸΠΎΡΡ‚ΠΎΡΠ½Π½Ρ‹ΠΉ Ρ‚ΠΎΠΊ – элСктричСский Ρ‚ΠΎΠΊ, Π½Π΅ ΠΈΠ·ΠΌΠ΅Π½ΡΡŽΡ‰ΠΈΠΉΡΡ со Π²Ρ€Π΅ΠΌΠ΅Π½Π΅ΠΌ.

    ΠŸΠΎΡΠ»Π΅Π΄ΠΎΠ²Π°Ρ‚Π΅Π»ΡŒΠ½ΠΎΠ΅ соСдинСниС ΠΏΡ€ΠΎΠ²ΠΎΠ΄Π½ΠΈΠΊΠΎΠ². ΠŸΡ€ΠΈ ΠΏΠΎΡΠ»Π΅Π΄ΠΎΠ²Π°Ρ‚Π΅Π»ΡŒΠ½ΠΎΠΌ соСдинСнии элСктричСская Ρ†Π΅ΠΏΡŒ Π½Π΅ ΠΈΠΌΠ΅Π΅Ρ‚ Ρ€Π°Π·Π²Π΅Ρ‚Π²Π»Π΅Π½ΠΈΠΉ. ВсС ΠΏΡ€ΠΎΠ²ΠΎΠ΄Π½ΠΈΠΊΠΈ Π²ΠΊΠ»ΡŽΡ‡Π°ΡŽΡ‚ Π² Ρ†Π΅ΠΏΡŒ ΠΏΠΎΠΎΡ‡Π΅Ρ€Π΅Π΄Π½ΠΎ Π΄Ρ€ΡƒΠ³ Π·Π° Π΄Ρ€ΡƒΠ³ΠΎΠΌ.

    ΠŸΠ°Ρ€Π°Π»Π»Π΅Π»ΡŒΠ½ΠΎΠ΅ соСдинСниС ΠΏΡ€ΠΎΠ²ΠΎΠ΄Π½ΠΈΠΊΠΎΠ². ΠŸΡ€ΠΈ ΠΏΠ°Ρ€Π°Π»Π»Π΅Π»ΡŒΠ½ΠΎΠΌ соСдинСнии ΠΊΠΎΠ½Ρ†Ρ‹ ΠΏΡ€ΠΎΠ²ΠΎΠ΄Π½ΠΈΠΊΠΎΠ² присоСдинСны ΠΊ ΠΎΠ΄Π½ΠΎΠΉ ΠΈ Ρ‚ΠΎΠΉ ΠΆΠ΅ ΠΏΠ°Ρ€Π΅ Ρ‚ΠΎΡ‡Π΅ΠΊ.

    БмСшанноС соСдинСниС проводниковэто Ρ‚Π°ΠΊΠΎΠ΅ соСдинСниС, ΠΊΠΎΠ³Π΄Π° Π² Ρ†Π΅ΠΏΠΈ присутствуСт ΠΈ ΠΏΠΎΡΠ»Π΅Π΄ΠΎΠ²Π°Ρ‚Π΅Π»ΡŒΠ½ΠΎΠ΅, ΠΈ ΠΏΠ°Ρ€Π°Π»Π»Π΅Π»ΡŒΠ½ΠΎΠ΅ соСдинСниС.

    Π£Π·Π΅Π» – это Ρ‚ΠΎΡ‡ΠΊΠ° элСктричСской Ρ†Π΅ΠΏΠΈ, Π³Π΄Π΅ сходится Π½Π΅ ΠΌΠ΅Π½Π΅Π΅ Ρ‚Ρ€Π΅Ρ… Π²Π΅Ρ‚Π²Π΅ΠΉ.

    Бвойство ΠΏΡ€ΠΎΠ²ΠΎΠ΄Π½ΠΈΠΊΠ° ΠΎΠ³Ρ€Π°Π½ΠΈΡ‡ΠΈΠ²Π°Ρ‚ΡŒ силу Ρ‚ΠΎΠΊΠ° Π² Ρ†Π΅ΠΏΠΈ, Ρ‚ΠΎ Π΅ΡΡ‚ΡŒ ΠΏΡ€ΠΎΡ‚ΠΈΠ²ΠΎΠ΄Π΅ΠΉΡΡ‚Π²ΠΎΠ²Π°Ρ‚ΡŒ элСктричСскому Ρ‚ΠΎΠΊΡƒ, Π½Π°Π·Ρ‹Π²Π°ΡŽΡ‚ элСктричСским сопротивлСниСм ΠΏΡ€ΠΎΠ²ΠΎΠ΄Π½ΠΈΠΊΠ°.

    РСзистор ΠΈΠ»ΠΈ ΠΏΡ€ΠΎΠ²ΠΎΠ΄Π½ΠΈΠΊ элСмСнт элСктричСских Ρ†Π΅ΠΏΠ΅ΠΉ, ΠΎΠ±Π»Π°Π΄Π°ΡŽΡ‰ΠΈΠΉ ΠΎΠΏΡ€Π΅Π΄Π΅Π»Ρ‘Π½Π½Ρ‹ΠΌ ΠΈΠ»ΠΈ ΠΏΠ΅Ρ€Π΅ΠΌΠ΅Π½Π½Ρ‹ΠΌ Π·Π½Π°Ρ‡Π΅Π½ΠΈΠ΅ΠΌ элСктричСского сопротивлСния.

    Основная ΠΈ Π΄ΠΎΠΏΠΎΠ»Π½ΠΈΡ‚Π΅Π»ΡŒΠ½Π°Ρ Π»ΠΈΡ‚Π΅Ρ€Π°Ρ‚ΡƒΡ€Π° ΠΏΠΎ Ρ‚Π΅ΠΌΠ΅ ΡƒΡ€ΠΎΠΊΠ°:

    1. МякишСв Π“.Π―., Π‘ΡƒΡ…ΠΎΠ²Ρ†Π΅Π² Π‘.Π‘., Ботский Н.Н. Π€ΠΈΠ·ΠΈΠΊΠ°.10 класс. Π£Ρ‡Π΅Π±Π½ΠΈΠΊ для ΠΎΠ±Ρ‰Π΅ΠΎΠ±Ρ€Π°Π·ΠΎΠ²Π°Ρ‚Π΅Π»ΡŒΠ½Ρ‹Ρ… ΠΎΡ€Π³Π°Π½ΠΈΠ·Π°Ρ†ΠΈΠΉ М.: ΠŸΡ€ΠΎΡΠ²Π΅Ρ‰Π΅Π½ΠΈΠ΅, 2017. – Π‘. 335 – 340.

    2. Π Ρ‹ΠΌΠΊΠ΅Π²ΠΈΡ‡ А.П. Π‘Π±ΠΎΡ€Π½ΠΈΠΊ Π·Π°Π΄Π°Ρ‡ ΠΏΠΎ Ρ„ΠΈΠ·ΠΈΠΊΠ΅. 10-11 класс. – М.: Π”Ρ€ΠΎΡ„Π°, 2009. – Π‘. 105 – 109.

    3. Π­Π»Π΅ΠΌΠ΅Π½Ρ‚Π°Ρ€Π½Ρ‹ΠΉ ΡƒΡ‡Π΅Π±Π½ΠΈΠΊ Ρ„ΠΈΠ·ΠΈΠΊΠΈ. Π£Ρ‡Π΅Π±Π½ΠΎΠ΅ пособиС Π² 3 Ρ‚ΠΎΠΌΠ°Ρ… ΠΏΠΎΠ΄ Ρ€Π΅Π΄Π°ΠΊΡ†ΠΈΠ΅ΠΉ Π°ΠΊΠ°Π΄Π΅ΠΌΠΈΠΊΠ° ЛандсбСрга Π“.Π‘.: Π’.2. ЭлСктричСство ΠΈ ΠΌΠ°Π³Π½Π΅Ρ‚ΠΈΠ·ΠΌ. – 12-Π΅ ΠΈΠ·Π΄. – М.: Π€Π˜Π—ΠœΠΠ’Π›Π˜Π’, 2001. Π‘. 110 – 115.

    4. Π’ΡƒΠ»ΡŒΡ‡ΠΈΠ½ΡΠΊΠΈΠΉ М.Π•. ΠšΠ°Ρ‡Π΅ΡΡ‚Π²Π΅Π½Π½Ρ‹Π΅ Π·Π°Π΄Π°Ρ‡ΠΈ ΠΏΠΎ Ρ„ΠΈΠ·ΠΈΠΊΠ΅ Π² срСднСй школС. ПособиС для ΡƒΡ‡ΠΈΡ‚Π΅Π»Π΅ΠΉ. Изд. 4-Π΅, ΠΏΠ΅Ρ€Π΅Ρ€Π°Π±ΠΎΡ‚. ΠΈ Π΄ΠΎΠΏ. М. Β«ΠŸΡ€ΠΎΡΠ²Π΅Ρ‰Π΅Π½ΠΈΠ΅Β», 1972. Π‘. 83 – 87.

    5. БавСльСв И.Π’. ΠšΡƒΡ€Ρ ΠΎΠ±Ρ‰Π΅ΠΉ Ρ„ΠΈΠ·ΠΈΠΊΠΈ, Ρ‚ΠΎΠΌ II. ЭлСктричСство. М.: Изд. «Наука», 1970 Π³. Π‘. 108.

    ΠžΡ‚ΠΊΡ€Ρ‹Ρ‚Ρ‹Π΅ элСктронныС рСсурсы:

    http://kvant.mccme.ru/1979/02/elektrichestvo_ie_temperatura.htm

    ВСорСтичСский ΠΌΠ°Ρ‚Π΅Ρ€ΠΈΠ°Π» для Π΄ΠΎΠΏΠΎΠ»Π½ΠΈΡ‚Π΅Π»ΡŒΠ½ΠΎΠ³ΠΎ изучСния

    Π‘Π»ΠΎΠΆΠ½ΠΎ ΠΏΡ€Π΅Π΄ΡΡ‚Π°Π²ΠΈΡ‚ΡŒ Π½Π°ΡˆΡƒ Тизнь Π±Π΅Π· элСктричСского Ρ‚ΠΎΠΊΠ°. ΠšΠ°ΠΆΠ΄Ρ‹ΠΉ дСнь, Π½Π΅ Π·Π°Π΄ΡƒΠΌΡ‹Π²Π°ΡΡΡŒ, ΠΌΡ‹ ΠΈΡΠΏΠΎΠ»ΡŒΠ·ΡƒΠ΅ΠΌ Ρ€Π°Π·Π»ΠΈΡ‡Π½Ρ‹Π΅ элСктричСскиС ΠΏΡ€ΠΈΠ±ΠΎΡ€Ρ‹, Π² основС Ρ€Π°Π±ΠΎΡ‚Ρ‹ ΠΊΠΎΡ‚ΠΎΡ€Ρ‹Ρ… Π»Π΅ΠΆΠ°Ρ‚ простыС ΠΈ слоТныС элСктричСскиС Ρ†Π΅ΠΏΠΈ. ΠšΠ°ΠΊΠΎΠΌΡƒ Π·Π°ΠΊΠΎΠ½Ρƒ ΠΏΠΎΠ΄Ρ‡ΠΈΠ½ΡΡŽΡ‚ΡΡ основныС ΠΏΠ°Ρ€Π°ΠΌΠ΅Ρ‚Ρ€Ρ‹ элСктричСских Ρ†Π΅ΠΏΠ΅ΠΉ? Как Ρ€Π°ΡΡΡ‡ΠΈΡ‚Π°Ρ‚ΡŒ эти Ρ†Π΅ΠΏΠΈ, Ρ‡Ρ‚ΠΎΠ±Ρ‹ ΠΏΡ€ΠΈΠ±ΠΎΡ€Ρ‹ Ρ€Π°Π±ΠΎΡ‚Π°Π»ΠΈ исправно?

    Π’Ρ‹ ΡƒΠΆΠ΅ Π·Π½Π°Π΅Ρ‚Π΅, элСктричСским Ρ‚ΠΎΠΊΠΎΠΌ Π½Π°Π·Ρ‹Π²Π°ΡŽΡ‚ упорядочСнноС (Π½Π°ΠΏΡ€Π°Π²Π»Π΅Π½Π½ΠΎΠ΅) Π΄Π²ΠΈΠΆΠ΅Π½ΠΈΠ΅ заряТСнных частиц.

    Для возникновСния ΠΈ сущСствования элСктричСского Ρ‚ΠΎΠΊΠ° Π² ΠΏΡ€ΠΎΠ²ΠΎΠ΄Π½ΠΈΠΊΠ΅ Π½Π΅ΠΎΠ±Ρ…ΠΎΠ΄ΠΈΠΌΠΎ:

    1. Π½Π°Π»ΠΈΡ‡ΠΈΠ΅ свободных заряТСнных частиц;
    2. сила, Π΄Π΅ΠΉΡΡ‚Π²ΡƒΡŽΡ‰Π°Ρ Π½Π° Π½ΠΈΡ… Π² ΠΎΠΏΡ€Π΅Π΄Π΅Π»Ρ‘Π½Π½ΠΎΠΌ Π½Π°ΠΏΡ€Π°Π²Π»Π΅Π½ΠΈΠΈ, Ρ‚ΠΎ Π΅ΡΡ‚ΡŒ Π½Π°Π»ΠΈΡ‡ΠΈΠ΅ элСктричСского поля Π² ΠΏΡ€ΠΎΠ²ΠΎΠ΄Π½ΠΈΠΊΠ΅.

    Π Π°Π·Π»ΠΈΡ‡Π°ΡŽΡ‚ ΡΠ»Π΅Π΄ΡƒΡŽΡ‰ΠΈΠ΅ дСйствия элСктричСского Ρ‚ΠΎΠΊΠ°:

    1. Ρ‚Π΅ΠΏΠ»ΠΎΠ²ΠΎΠ΅ ;
    2. химичСскоС ;
    3. ΠΌΠ°Π³Π½ΠΈΡ‚Π½ΠΎΠ΅ .

    ΠŸΠΎΡΡ‚ΠΎΡΠ½Π½Ρ‹ΠΉ Ρ‚ΠΎΠΊ β€” элСктричСский Ρ‚ΠΎΠΊ, Ρƒ ΠΊΠΎΡ‚ΠΎΡ€ΠΎΠ³ΠΎ сила Ρ‚ΠΎΠΊΠ° ΠΈ Π½Π°ΠΏΡ€Π°Π²Π»Π΅Π½ΠΈΠ΅ Π½Π΅ ΠΈΠ·ΠΌΠ΅Π½ΡΡŽΡ‚ΡΡ со Π²Ρ€Π΅ΠΌΠ΅Π½Π΅ΠΌ.

    Π‘ΠΈΠ»Π° Ρ‚ΠΎΠΊΠ° I Ρ€Π°Π²Π½Π° ΠΎΡ‚Π½ΠΎΡˆΠ΅Π½ΠΈΡŽ элСктричСского заряда q, ΠΏΡ€ΠΎΡˆΠ΅Π΄ΡˆΠ΅Π³ΠΎ Ρ‡Π΅Ρ€Π΅Π· ΠΏΠΎΠΏΠ΅Ρ€Π΅Ρ‡Π½ΠΎΠ΅ сСчСниС ΠΏΡ€ΠΎΠ²ΠΎΠ΄Π½ΠΈΠΊΠ°, ΠΊΠΎ Π²Ρ€Π΅ΠΌΠ΅Π½ΠΈ Π΅Π³ΠΎ прохоТдСния t:

    Π—Π° Π½Π°ΠΏΡ€Π°Π²Π»Π΅Π½ΠΈΠ΅ элСктричСского Ρ‚ΠΎΠΊΠ° условно Π²Ρ‹Π±Ρ€Π°Π½ΠΎ Π½Π°ΠΏΡ€Π°Π²Π»Π΅Π½ΠΈΠ΅ двиТСния ΠΏΠΎΠ»ΠΎΠΆΠΈΡ‚Π΅Π»ΡŒΠ½ΠΎ заряТСнных частиц, Ρ‚ΠΎ Π΅ΡΡ‚ΡŒ Π² сторону, ΠΏΡ€ΠΎΡ‚ΠΈΠ²ΠΎΠΏΠΎΠ»ΠΎΠΆΠ½ΡƒΡŽ Π½Π°ΠΏΡ€Π°Π²Π»Π΅Π½ΠΈΡŽ двиТСния элСктронов.

    Для ΠΊΠ°ΠΆΠ΄ΠΎΠ³ΠΎ ΠΏΡ€ΠΎΠ²ΠΎΠ΄Π½ΠΈΠΊΠ° – Ρ‚Π²Π΅Ρ€Π΄ΠΎΠ³ΠΎ, ΠΆΠΈΠ΄ΠΊΠΎΠ³ΠΎ ΠΈ Π³Π°Π·ΠΎΠΎΠ±Ρ€Π°Π·Π½ΠΎΠ³ΠΎ – сущСствуСт опрСдСлённая Π·Π°Π²ΠΈΡΠΈΠΌΠΎΡΡ‚ΡŒ силы Ρ‚ΠΎΠΊΠ° ΠΎΡ‚ ΠΏΡ€ΠΈΠ»ΠΎΠΆΠ΅Π½Π½ΠΎΠΉ разности ΠΏΠΎΡ‚Π΅Π½Ρ†ΠΈΠ°Π»ΠΎΠ² (напряТСния) Π½Π° ΠΊΠΎΠ½Ρ†Π°Ρ… ΠΏΡ€ΠΎΠ²ΠΎΠ΄Π½ΠΈΠΊΠ°. Π­Ρ‚Ρƒ Π·Π°Π²ΠΈΡΠΈΠΌΠΎΡΡ‚ΡŒ Π²Ρ‹Ρ€Π°ΠΆΠ°Π΅Ρ‚, Ρ‚Π°ΠΊ называСмая, Π²ΠΎΠ»ΡŒΡ‚-ампСрная характСристика ΠΏΡ€ΠΎΠ²ΠΎΠ΄Π½ΠΈΠΊΠ°.

    Для ΡˆΠΈΡ€ΠΎΠΊΠΎΠ³ΠΎ класса ΠΏΡ€ΠΎΠ²ΠΎΠ΄Π½ΠΈΠΊΠΎΠ² (Π² Ρ‚. Ρ‡. ΠΌΠ΅Ρ‚Π°Π»Π»ΠΎΠ² ) ΠΏΡ€ΠΈ Π½Π΅ΠΈΠ·ΠΌΠ΅Π½Π½ΠΎΠΉ Ρ‚Π΅ΠΌΠΏΠ΅Ρ€Π°Ρ‚ΡƒΡ€Π΅ справСдлив Π·Π°ΠΊΠΎΠ½ Ома для участка Ρ†Π΅ΠΏΠΈ:

    Π‘ΠΈΠ»Π° Ρ‚ΠΎΠΊΠ° Π½Π° участкС Ρ†Π΅ΠΏΠΈ прямо ΠΏΡ€ΠΎΠΏΠΎΡ€Ρ†ΠΈΠΎΠ½Π°Π»ΡŒΠ½Π° ΠΏΡ€ΠΈΠ»ΠΎΠΆΠ΅Π½Π½ΠΎΠΌΡƒ Π½Π°ΠΏΡ€ΡΠΆΠ΅Π½ΠΈΡŽ U ΠΈ ΠΎΠ±Ρ€Π°Ρ‚Π½ΠΎ ΠΏΡ€ΠΎΠΏΠΎΡ€Ρ†ΠΈΠΎΠ½Π°Π»ΡŒΠ½Π° ΡΠΎΠΏΡ€ΠΎΡ‚ΠΈΠ²Π»Π΅Π½ΠΈΡŽ этого участка Ρ†Π΅ΠΏΠΈ:

    Π—Π°ΠΊΠΎΠ½ ΠΈΠΌΠ΅Π΅Ρ‚ ΠΏΡ€ΠΎΡΡ‚ΡƒΡŽ Ρ„ΠΎΡ€ΠΌΡƒ, Π½ΠΎ Π΄ΠΎΠΊΠ°Π·Π°Ρ‚ΡŒ ΡΠΊΡΠΏΠ΅Ρ€ΠΈΠΌΠ΅Π½Ρ‚Π°Π»ΡŒΠ½ΠΎ Π΅Π³ΠΎ ΡΠΏΡ€Π°Π²Π΅Π΄Π»ΠΈΠ²ΠΎΡΡ‚ΡŒ довольно Ρ‚Ρ€ΡƒΠ΄Π½ΠΎ.

    Π—Π°ΠΊΠΎΠ½ Ома являСтся основой всСй элСктротСхники постоянных Ρ‚ΠΎΠΊΠΎΠ². Из Π·Π°ΠΊΠΎΠ½Π° Ома Π²Ρ‹Ρ‚Π΅ΠΊΠ°Π΅Ρ‚, Ρ‡Ρ‚ΠΎ Π·Π°ΠΌΡ‹ΠΊΠ°Ρ‚ΡŒ ΠΎΠ±Ρ‹Ρ‡Π½ΡƒΡŽ ΠΎΡΠ²Π΅Ρ‚ΠΈΡ‚Π΅Π»ΡŒΠ½ΡƒΡŽ ΡΠ΅Ρ‚ΡŒ ΠΏΡ€ΠΎΠ²ΠΎΠ΄Π½ΠΈΠΊΠΎΠΌ ΠΌΠ°Π»ΠΎΠ³ΠΎ сопротивлСния опасно.

    Основная элСктричСская характСристика ΠΏΡ€ΠΎΠ²ΠΎΠ΄Π½ΠΈΠΊΠ° – сопротивлСниС. ΠžΡ‚ этой Π²Π΅Π»ΠΈΡ‡ΠΈΠ½Ρ‹ зависит сила Ρ‚ΠΎΠΊΠ° Π² ΠΏΡ€ΠΎΠ²ΠΎΠ΄Π½ΠΈΠΊΠ΅ ΠΏΡ€ΠΈ Π·Π°Π΄Π°Π½Π½ΠΎΠΌ напряТСнии. ΠŸΡ€ΠΈΡ‡ΠΈΠ½ΠΎΠΉ элСктричСского сопротивлСния являСтся взаимодСйствиС элСктронов ΠΏΡ€ΠΈ ΠΈΡ… Π΄Π²ΠΈΠΆΠ΅Π½ΠΈΠΈ ΠΏΠΎ ΠΏΡ€ΠΎΠ²ΠΎΠ΄Π½ΠΈΠΊΡƒ с ΠΈΠΎΠ½Π°ΠΌΠΈ кристалличСской Ρ€Π΅ΡˆΠ΅Ρ‚ΠΊΠΈ. Π‘ΠΎΠΏΡ€ΠΎΡ‚ΠΈΠ²Π»Π΅Π½ΠΈΠ΅ ΠΏΡ€ΠΎΠ²ΠΎΠ΄Π½ΠΈΠΊΠ° зависит ΠΎΡ‚ свойств ΠΌΠ°Ρ‚Π΅Ρ€ΠΈΠ°Π»Π° ΠΏΡ€ΠΎΠ²ΠΎΠ΄Π½ΠΈΠΊΠ° ΠΈ Π΅Π³ΠΎ гСомСтричСских Ρ€Π°Π·ΠΌΠ΅Ρ€ΠΎΠ².

    ЭлСктричСскоС сопротивлСниС ΠΌΠ΅Ρ‚Π°Π»Π»ΠΎΠ² прямо ΠΏΡ€ΠΎΠΏΠΎΡ€Ρ†ΠΈΠΎΠ½Π°Π»ΡŒΠ½ΠΎ Π΄Π»ΠΈΠ½Π΅ ΠΏΡ€ΠΎΠ²ΠΎΠ΄Π½ΠΈΠΊΠ° ΠΈ ΠΎΠ±Ρ€Π°Ρ‚Π½ΠΎ ΠΏΡ€ΠΎΠΏΠΎΡ€Ρ†ΠΈΠΎΠ½Π°Π»ΡŒΠ½ΠΎ ΠΏΠ»ΠΎΡ‰Π°Π΄ΠΈ Π΅Π³ΠΎ ΠΏΠΎΠΏΠ΅Ρ€Π΅Ρ‡Π½ΠΎΠ³ΠΎ сСчСния:

    Π³Π΄Π΅ Π²Π΅Π»ΠΈΡ‡ΠΈΠ½Π° ρ – ΡƒΠ΄Π΅Π»ΡŒΠ½ΠΎΠ΅ сопротивлСниС ΠΏΡ€ΠΎΠ²ΠΎΠ΄Π½ΠΈΠΊΠ° – Π²Π΅Π»ΠΈΡ‡ΠΈΠ½Π°, зависящая ΠΎΡ‚ Ρ€ΠΎΠ΄Π° вСщСства ΠΈ Π΅Π³ΠΎ состояния (ΠΎΡ‚ Ρ‚Π΅ΠΌΠΏΠ΅Ρ€Π°Ρ‚ΡƒΡ€Ρ‹ Π² ΠΏΠ΅Ρ€Π²ΡƒΡŽ ΠΎΡ‡Π΅Ρ€Π΅Π΄ΡŒ). УдСльноС сопротивлСниС вСщСств приводятся Π² справочных Ρ‚Π°Π±Π»ΠΈΡ†Π°Ρ….

    ΠžΠΌΠΌΠ΅Ρ‚Ρ€ – ΠΏΡ€ΠΈΠ±ΠΎΡ€ для измСрСния сопротивлСния.

    ΠžΡ‚ источника Ρ‚ΠΎΠΊΠ° энСргия ΠΌΠΎΠΆΠ΅Ρ‚ Π±Ρ‹Ρ‚ΡŒ ΠΏΠ΅Ρ€Π΅Π΄Π°Π½Π° ΠΏΠΎ ΠΏΡ€ΠΎΠ²ΠΎΠ΄Π°ΠΌ ΠΊ устройствам, ΠΏΠΎΡ‚Ρ€Π΅Π±Π»ΡΡŽΡ‰ΠΈΠΌ ΡΠ½Π΅Ρ€Π³ΠΈΡŽ. Для этого ΡΠΎΡΡ‚Π°Π²Π»ΡΡŽΡ‚ элСктричСскиС Ρ†Π΅ΠΏΠΈ Ρ€Π°Π·Π»ΠΈΡ‡Π½ΠΎΠΉ слоТности. Π Π°Π·Π»ΠΈΡ‡Π°ΡŽΡ‚ ΠΏΠΎΡΠ»Π΅Π΄ΠΎΠ²Π°Ρ‚Π΅Π»ΡŒΠ½ΠΎΠ΅, ΠΏΠ°Ρ€Π°Π»Π»Π΅Π»ΡŒΠ½ΠΎΠ΅, смСшанноС соСдинСния ΠΏΡ€ΠΎΠ²ΠΎΠ΄Π½ΠΈΠΊΠΎΠ².

    ΠŸΠΎΡΠ»Π΅Π΄ΠΎΠ²Π°Ρ‚Π΅Π»ΡŒΠ½ΠΎΠ΅ соСдинСниС ΠΏΡ€ΠΎΠ²ΠΎΠ΄Π½ΠΈΠΊΠΎΠ². ΠŸΡ€ΠΈ ΠΏΠΎΡΠ»Π΅Π΄ΠΎΠ²Π°Ρ‚Π΅Π»ΡŒΠ½ΠΎΠΌ соСдинСнии элСктричСская Ρ†Π΅ΠΏΡŒ Π½Π΅ ΠΈΠΌΠ΅Π΅Ρ‚ Ρ€Π°Π·Π²Π΅Ρ‚Π²Π»Π΅Π½ΠΈΠΉ. ВсС ΠΏΡ€ΠΎΠ²ΠΎΠ΄Π½ΠΈΠΊΠΈ Π²ΠΊΠ»ΡŽΡ‡Π°ΡŽΡ‚ Π² Ρ†Π΅ΠΏΡŒ ΠΏΠΎΠΎΡ‡Π΅Ρ€Π΅Π΄Π½ΠΎ Π΄Ρ€ΡƒΠ³ Π·Π° Π΄Ρ€ΡƒΠ³ΠΎΠΌ. Главная ΠΎΡΠΎΠ±Π΅Π½Π½ΠΎΡΡ‚ΡŒ ΠΏΠΎΡΠ»Π΅Π΄ΠΎΠ²Π°Ρ‚Π΅Π»ΡŒΠ½ΠΎΠ³ΠΎ соСдинСния Π·Π°ΠΊΠ»ΡŽΡ‡Π°Π΅Ρ‚ΡΡ Π² Ρ‚ΠΎΠΌ, Ρ‡Ρ‚ΠΎ Ρ‡Π΅Ρ€Π΅Π· всС ΠΏΡ€ΠΎΠ²ΠΎΠ΄Π½ΠΈΠΊΠΈ ΠΏΡ€ΠΎΡ‚Π΅ΠΊΠ°Π΅Ρ‚ ΠΎΠ΄ΠΈΠ½Π°ΠΊΠΎΠ²Ρ‹ΠΉ Ρ‚ΠΎΠΊ. Если Ρ‡Π΅Ρ€Π΅Π· ΠΎΠ΄ΠΈΠ½ ΠΏΡ€ΠΎΠ²ΠΎΠ΄Π½ΠΈΠΊ ΠΏΡ€ΠΎΡ‚Π΅ΠΊΠ°Π΅Ρ‚ Ρ‚ΠΎΠΊ ΠΎΠΏΡ€Π΅Π΄Π΅Π»Π΅Π½Π½ΠΎΠΉ Π²Π΅Π»ΠΈΡ‡ΠΈΠ½Ρ‹, Ρ‚ΠΎ Ρ‚Π°ΠΊΠΎΠΉ ΠΆΠ΅ Ρ‚ΠΎΠΊ ΠΏΡ€ΠΎΡ‚Π΅ΠΊΠ°Π΅Ρ‚ ΠΈ Ρ‡Π΅Ρ€Π΅Π· всС ΠΎΡΡ‚Π°Π»ΡŒΠ½Ρ‹Π΅. Если хотя Π±Ρ‹ Π² ΠΎΠ΄Π½ΠΎΠΌ ΠΏΡ€ΠΎΠ²ΠΎΠ΄Π½ΠΈΠΊΠ΅ отсутствуСт Ρ‚ΠΎΠΊ, Ρ‚ΠΎ ΠΎΠ½ ΠΎΠ±ΡΠ·Π°Ρ‚Π΅Π»ΡŒΠ½ΠΎ отсутствуСт ΠΈ Π²ΠΎ всСх ΠΎΡΡ‚Π°Π»ΡŒΠ½Ρ‹Ρ…. НапряТСниС Π½Π° ΠΊΠΎΠ½Ρ†Π°Ρ… ΠΏΠΎΡΠ»Π΅Π΄ΠΎΠ²Π°Ρ‚Π΅Π»ΡŒΠ½ΠΎ соСдинСнных ΠΏΡ€ΠΎΠ²ΠΎΠ΄Π½ΠΈΠΊΠΎΠ² складываСтся. ПолноС сопротивлСниС всСго участка Ρ†Π΅ΠΏΠΈ ΠΏΡ€ΠΈ ΠΏΠΎΡΠ»Π΅Π΄ΠΎΠ²Π°Ρ‚Π΅Π»ΡŒΠ½ΠΎΠΌ соСдинСнии Ρ€Π°Π²Π½ΠΎ суммС сопротивлСний всСх ΠΏΡ€ΠΎΠ²ΠΎΠ΄Π½ΠΈΠΊΠΎΠ².

    ΠŸΠΎΡΠ»Π΅Π΄ΠΎΠ²Π°Ρ‚Π΅Π»ΡŒΠ½ΠΎΠ΅ соСдинСниС

    ЀизичСская Π²Π΅Π»ΠΈΡ‡ΠΈΠ½Π°

    Π€ΠΎΡ€ΠΌΡƒΠ»Π°

    Π‘ΠΈΠ»Π° Ρ‚ΠΎΠΊΠ°

    I = I1 = I2

    НапряТСниС

    U = U1 + U2

    Π‘ΠΎΠΏΡ€ΠΎΡ‚ΠΈΠ²Π»Π΅Π½ΠΈΠ΅

    R = R1 + R2

    ΠŸΠ°Ρ€Π°Π»Π»Π΅Π»ΡŒΠ½ΠΎΠ΅ соСдинСниС ΠΏΡ€ΠΎΠ²ΠΎΠ΄Π½ΠΈΠΊΠΎΠ². ΠŸΡ€ΠΈ ΠΏΠ°Ρ€Π°Π»Π»Π΅Π»ΡŒΠ½ΠΎΠΌ соСдинСнии ΠΊΠΎΠ½Ρ†Ρ‹ ΠΏΡ€ΠΎΠ²ΠΎΠ΄Π½ΠΈΠΊΠΎΠ² присоСдинСны ΠΊ ΠΎΠ΄Π½ΠΎΠΉ ΠΈ Ρ‚ΠΎΠΉ ΠΆΠ΅ ΠΏΠ°Ρ€Π΅ Ρ‚ΠΎΡ‡Π΅ΠΊ.

    ΠŸΠ°Ρ€Π°Π»Π»Π΅Π»ΡŒΠ½ΠΎΠ΅ соСдинСниС

    ЀизичСская Π²Π΅Π»ΠΈΡ‡ΠΈΠ½Π°

    Π€ΠΎΡ€ΠΌΡƒΠ»Π°

    Π‘ΠΈΠ»Π° Ρ‚ΠΎΠΊΠ°

    I = I1 + I2

    НапряТСниС

    U = U1 = U2

    Π‘ΠΎΠΏΡ€ΠΎΡ‚ΠΈΠ²Π»Π΅Π½ΠΈΠ΅

    Π£Π·Π΅Π» – это Ρ‚ΠΎΡ‡ΠΊΠ° элСктричСской Ρ†Π΅ΠΏΠΈ, Π³Π΄Π΅ сходится Π½Π΅ ΠΌΠ΅Π½Π΅Π΅ Ρ‚Ρ€Π΅Ρ… Π²Π΅Ρ‚Π²Π΅ΠΉ.

    Π£Π·Π΅Π» обозначаСтся Π½Π° схСмС ΠΆΠΈΡ€Π½ΠΎΠΉ Ρ‚ΠΎΡ‡ΠΊΠΎΠΉ Π² Ρ‚ΠΎΠΌ мСстС, Π³Π΄Π΅ Π²Π΅Ρ‚Π²ΠΈ ΡΠΎΠ΅Π΄ΠΈΠ½ΡΡŽΡ‚ΡΡ ΠΌΠ΅ΠΆΠ΄Ρƒ собой.

    БмСшанноС соСдинСниС ΠΏΡ€ΠΎΠ²ΠΎΠ΄Π½ΠΈΠΊΠΎΠ².

    Π‘ΠΌΠ΅ΡˆΠ°Π½Π½Ρ‹ΠΌ соСдинСниСм ΠΏΡ€ΠΎΠ²ΠΎΠ΄Π½ΠΈΠΊΠΎΠ² Π½Π°Π·Ρ‹Π²Π°ΡŽΡ‚ Ρ‚Π°ΠΊΠΎΠ΅ соСдинСниС, ΠΏΡ€ΠΈ ΠΊΠΎΡ‚ΠΎΡ€ΠΎΠΌ Π² Ρ†Π΅ΠΏΠΈ присутствуСт ΠΈ ΠΏΠΎΡΠ»Π΅Π΄ΠΎΠ²Π°Ρ‚Π΅Π»ΡŒΠ½ΠΎΠ΅, ΠΈ ΠΏΠ°Ρ€Π°Π»Π»Π΅Π»ΡŒΠ½ΠΎΠ΅ соСдинСниС.

    ΠœΠ΅Ρ‚ΠΎΠ΄ эквивалСнтных ΠΏΡ€Π΅ΠΎΠ±Ρ€Π°Π·ΠΎΠ²Π°Π½ΠΈΠΉ Π·Π°ΠΊΠ»ΡŽΡ‡Π°Π΅Ρ‚ΡΡ Π² Ρ‚ΠΎΠΌ, Ρ‡Ρ‚ΠΎ ΡΠ»Π΅ΠΊΡ‚Ρ€ΠΈΡ‡Π΅ΡΠΊΡƒΡŽ Ρ†Π΅ΠΏΡŒ ΠΈΠ»ΠΈ Π΅Π΅ Ρ‡Π°ΡΡ‚ΡŒ Π·Π°ΠΌΠ΅Π½ΡΡŽΡ‚ Π±ΠΎΠ»Π΅Π΅ простой ΠΏΠΎ структурС элСктричСской Ρ†Π΅ΠΏΡŒΡŽ. ΠŸΡ€ΠΈ этом Ρ‚ΠΎΠΊΠΈ ΠΈ напряТСния Π² Π½Π΅ΠΏΡ€Π΅ΠΎΠ±Ρ€Π°Π·ΠΎΠ²Π°Π½Π½ΠΎΠΉ части Ρ†Π΅ΠΏΠΈ Π΄ΠΎΠ»ΠΆΠ½Ρ‹ ΠΎΡΡ‚Π°Π²Π°Ρ‚ΡŒΡΡ Π½Π΅ΠΈΠ·ΠΌΠ΅Π½Π½Ρ‹ΠΌΠΈ, Ρ‚.Π΅. Ρ‚Π°ΠΊΠΈΠΌΠΈ, ΠΊΠ°ΠΊΠΈΠΌΠΈ ΠΎΠ½ΠΈ Π±Ρ‹Π»ΠΈ Π΄ΠΎ прСобразования. Π’ Ρ€Π΅Π·ΡƒΠ»ΡŒΡ‚Π°Ρ‚Π΅ ΠΏΡ€Π΅ΠΎΠ±Ρ€Π°Π·ΠΎΠ²Π°Π½ΠΈΠΉ расчСт Ρ†Π΅ΠΏΠΈ упрощаСтся ΠΈ часто сводится ΠΊ элСмСнтарным арифмСтичСским опСрациям.

    РасчСт сопротивлСния слоТной Ρ†Π΅ΠΏΠΈ:

    РСзи́стор ΠΈΠ»ΠΈ ΠΏΡ€ΠΎΠ²ΠΎΠ΄Π½ΠΈΠΊ – пассивный элСмСнт элСктричСских Ρ†Π΅ΠΏΠ΅ΠΉ, ΠΎΠ±Π»Π°Π΄Π°ΡŽΡ‰ΠΈΠΉ ΠΎΠΏΡ€Π΅Π΄Π΅Π»Ρ‘Π½Π½Ρ‹ΠΌ ΠΈΠ»ΠΈ ΠΏΠ΅Ρ€Π΅ΠΌΠ΅Π½Π½Ρ‹ΠΌ Π·Π½Π°Ρ‡Π΅Π½ΠΈΠ΅ΠΌ элСктричСского сопротивлСния.

    ΠŸΡ€ΠΈΠΌΠ΅Ρ€Ρ‹ ΠΈ Ρ€Π°Π·Π±ΠΎΡ€ Ρ€Π΅ΡˆΠ΅Π½ΠΈΡ Π·Π°Π΄Π°Π½ΠΈΠΉ

    1. Π’Ρ‹Π±Π΅Ρ€ΠΈΡ‚Π΅ ΠΎΠ΄ΠΈΠ½ ΠΈΠ· 3 Π²Π°Ρ€ΠΈΠ°Π½Ρ‚ΠΎΠ² ΠΎΡ‚Π²Π΅Ρ‚Π°:

    ΠŸΡ€ΠΈ ΠΏΠ°Ρ€Π°Π»Π»Π΅Π»ΡŒΠ½ΠΎΠΌ соСдинСнии ΠΏΡ€ΠΎΠ²ΠΎΠ΄Π½ΠΈΠΊΠΎΠ²…

    1) напряТСниС зависит ΠΎΡ‚ сопротивлСния Π½Π° Π΄Π°Π½Π½ΠΎΠΌ участкС Ρ†Π΅ΠΏΠΈ

    2) напряТСниС Π²Π΅Π·Π΄Π΅ Ρ€Π°Π·Π½ΠΎΠ΅

    3) напряТСниС Π²Π΅Π·Π΄Π΅ ΠΎΠ΄ΠΈΠ½Π°ΠΊΠΎΠ²ΠΎΠ΅

    ΠžΡ‚Π²Π΅Ρ‚: 3) напряТСниС Π²Π΅Π·Π΄Π΅ ΠΎΠ΄ΠΈΠ½Π°ΠΊΠΎΠ²ΠΎΠ΅.

    2. На участкС Ρ†Π΅ΠΏΠΈ, ΠΈΠ·ΠΎΠ±Ρ€Π°ΠΆΠ΅Π½Π½ΠΎΠΌ Π½Π° рисункС, сопротивлСниС ΠΊΠ°ΠΆΠ΄ΠΎΠ³ΠΎ ΠΈΠ· рСзисторов Ρ€Π°Π²Π½ΠΎ 24 Ом. Π§Π΅ΠΌΡƒ Ρ€Π°Π²Π½ΠΎ ΠΏΠΎΠ»Π½ΠΎΠ΅ сопротивлСниС участка ΠΏΡ€ΠΈ Π·Π°ΠΌΠΊΠ½ΡƒΡ‚ΠΎΠΌ ΠΊΠ»ΡŽΡ‡Π΅ К?

    РСшСниС.

    ПослС замыкания ΠΊΠ»ΡŽΡ‡Π° схСма Π±ΡƒΠ΄Π΅Ρ‚ ΠΏΡ€Π΅Π΄ΡΡ‚Π°Π²Π»ΡΡ‚ΡŒ собой ΠΏΠ°Ρ€Π°Π»Π»Π΅Π»ΡŒΠ½ΠΎΠ΅ соСдинСниС рСзистора с двумя ΠΏΠΎΡΠ»Π΅Π΄ΠΎΠ²Π°Ρ‚Π΅Π»ΡŒΠ½ΠΎ соСдинСнными рСзисторами.

    ПолноС сопротивлСниС участка ΠΏΡ€ΠΈ Π·Π°ΠΌΠΊΠ½ΡƒΡ‚ΠΎΠΌ ΠΊΠ»ΡŽΡ‡Π΅ Ρ€Π°Π²Π½ΠΎ

    (R+R)R/((R+R) + R) = 2R/3 = 16 Ом.

    ΠžΡ‚Π²Π΅Ρ‚: 16 Ом.

    Π§Ρ‚ΠΎ Ρ‚Π°ΠΊΠΎΠ΅ сопротивлСниС? – ΠžΡΠ½ΠΎΠ²Ρ‹ схСмотСхники

    Π’ ΠΏΡ€Π΅Π΄Ρ‹Π΄ΡƒΡ‰ΠΈΡ… ΡΡ‚Π°Ρ‚ΡŒΡΡ… ΠΌΡ‹ обсуТдали напряТСниС ΠΈ Ρ‚ΠΎΠΊ. На этот Ρ€Π°Π· ΠΌΡ‹ ΠΏΠΎΠ³ΠΎΠ²ΠΎΡ€ΠΈΠΌ ΠΎ Ρ‚Ρ€Π΅Ρ‚ΡŒΠ΅ΠΌ Ρ„ΡƒΠ½Π΄Π°ΠΌΠ΅Π½Ρ‚Π°Π»ΡŒΠ½ΠΎΠΌ понятии Π² элСктроникС – сопротивлСнии. Π’ самом простом ΠΎΠΏΡ€Π΅Π΄Π΅Π»Π΅Π½ΠΈΠΈ сопротивлСниС – это ΠΌΠ΅Ρ€Π° сопротивлСния Ρ‚ΠΎΠΊΡƒ Π² элСктричСской Ρ†Π΅ΠΏΠΈ. Но Π΄Π°Π²Π°ΠΉΡ‚Π΅ ΠΏΠΎΠ΄Ρ€ΠΎΠ±Π½Π΅Π΅ рассмотрим, Ρ‡Ρ‚ΠΎ это Π·Π½Π°Ρ‡ΠΈΡ‚!

    НСмного истории

    Π’ 1827 Π³ΠΎΠ΄Ρƒ Π”ΠΆΠΎΡ€Π΄ΠΆ Ом ΠΎΡ‚ΠΊΡ€Ρ‹Π» ΠΈ Π²Π²Π΅Π» Ρ‚Π΅Ρ€ΠΌΠΈΠ½ элСктричСскоС сопротивлСниС.Π­Ρ‚Π° концСпция ΠΈΠΌΠ΅Π΅Ρ‚ сходныС ΠΏΠ°Ρ€Π°Π»Π»Π΅Π»ΠΈ с мСханичСским Ρ‚Π΅Ρ€ΠΌΠΈΠ½ΠΎΠΌ Β«Ρ‚Ρ€Π΅Π½ΠΈΠ΅Β». ПослС Ρ‚ΠΎΠ³ΠΎ, ΠΊΠ°ΠΊ АлСссандро Π’ΠΎΠ»ΡŒΡ‚Π° ΠΈΠ·ΠΎΠ±Ρ€Π΅Π» ΠΏΠ΅Ρ€Π²ΡƒΡŽ ΡΠ»Π΅ΠΊΡ‚Ρ€ΠΎΡ…ΠΈΠΌΠΈΡ‡Π΅ΡΠΊΡƒΡŽ Π±Π°Ρ‚Π°Ρ€Π΅ΡŽ, Ом использовал Π΅Π΅ Π² качСствС основы для ΠΌΠ½ΠΎΠ³ΠΈΡ… своих экспСримСнтов, ΠΊΠΎΡ‚ΠΎΡ€Ρ‹Π΅ Π²ΠΊΠ»ΡŽΡ‡Π°Π»ΠΈ установлСниС взаимосвязи ΠΌΠ΅ΠΆΠ΄Ρƒ Ρ€Π°Π·Π½ΠΎΡΡ‚ΡŒΡŽ ΠΏΠΎΡ‚Π΅Π½Ρ†ΠΈΠ°Π»ΠΎΠ² ΠΈ Ρ‚ΠΎΠΊΠΎΠΌ. Он ΠΎΠ±Π½Π°Ρ€ΡƒΠΆΠΈΠ», Ρ‡Ρ‚ΠΎ Ρ‚ΠΎΠΊ ΠΈ напряТСниС прямо ΠΏΡ€ΠΎΠΏΠΎΡ€Ρ†ΠΈΠΎΠ½Π°Π»ΡŒΠ½Ρ‹, ΠΈ это ΡΠΎΠΎΡ‚Π½ΠΎΡˆΠ΅Π½ΠΈΠ΅ Π±Ρ‹Π»ΠΎ Π½Π°Π·Π²Π°Π½ΠΎ Π·Π°ΠΊΠΎΠ½ΠΎΠΌ Ома. Он ΠΎΠ±Π½Π°Ρ€ΡƒΠΆΠΈΠ», Ρ‡Ρ‚ΠΎ сопротивлСниС – это ΡΠΎΠΎΡ‚Π½ΠΎΡˆΠ΅Π½ΠΈΠ΅ ΠΌΠ΅ΠΆΠ΄Ρƒ напряТСниСм ΠΈ Ρ‚ΠΎΠΊΠΎΠΌ, ΠΊΠ°ΠΊ ΠΏΠΎΠΊΠ°Π·Π°Π½ΠΎ Π² Π΅Π³ΠΎ ΡƒΡ€Π°Π²Π½Π΅Π½ΠΈΠΈ Π½ΠΈΠΆΠ΅:

    Π€Π°ΠΊΡ‚ΠΎΡ€Ρ‹, ΠΎΠΏΡ€Π΅Π΄Π΅Π»ΡΡŽΡ‰ΠΈΠ΅ сопротивлСниС

    Π‘ΠΎΠΏΡ€ΠΎΡ‚ΠΈΠ²Π»Π΅Π½ΠΈΠ΅ Π²ΠΎΠ·Π½ΠΈΠΊΠ°Π΅Ρ‚, ΠΊΠΎΠ³Π΄Π° элСктроны Π½Π΅ ΠΌΠΎΠ³ΡƒΡ‚ свободно ΠΏΠ΅Ρ€Π΅ΠΌΠ΅Ρ‰Π°Ρ‚ΡŒΡΡ ΠΏΠΎ ΠΏΡ€ΠΎΠ²ΠΎΠ΄Π½ΠΈΠΊΡƒ.ΠžΠ±Ρ‹Ρ‡Π½ΠΎ это происходит ΠΈΠ·-Π·Π° отсутствия свободных Π²Π°Π»Π΅Π½Ρ‚Π½Ρ‹Ρ… элСктронов Π²ΠΎ ΠΌΠ½ΠΎΠ³ΠΈΡ… структурах. Π­Ρ‚ΠΎ ΠΏΡ€ΠΈΠ²ΠΎΠ΄ΠΈΡ‚ ΠΊ ΡƒΠ²Π΅Π»ΠΈΡ‡Π΅Π½ΠΈΡŽ столкновСний ΠΌΠ΅ΠΆΠ΄Ρƒ элСктронами ΠΈ ΠΈΠΎΠ½Π°ΠΌΠΈ Π² ΠΌΠ°Ρ‚Π΅Ρ€ΠΈΠ°Π»Π΅. Когда происходят эти столкновСния, кинСтичСская энСргия элСктронов прСобразуСтся Π² Ρ‚Π΅ΠΏΠ»ΠΎΠ²ΡƒΡŽ, поэтому, ΠΊΠΎΠ³Π΄Π° большиС Ρ‚ΠΎΠΊΠΈ ΡΡ‚Π°Π»ΠΊΠΈΠ²Π°ΡŽΡ‚ΡΡ с высоким сопротивлСниСм, выдСляСтся ΠΌΠ½ΠΎΠ³ΠΎ Ρ‚Π΅ΠΏΠ»Π°.

    На сопротивлСниС ΠΏΡ€ΠΎΠ²ΠΎΠ΄Π½ΠΈΠΊΠ° Π²Π»ΠΈΡΡŽΡ‚ Ρ‚Ρ€ΠΈ Ρ„Π°ΠΊΡ‚ΠΎΡ€Π°:

    • Π”Π»ΠΈΠ½Π° ΠΏΡ€ΠΎΠ²ΠΎΠ΄Π½ΠΈΠΊΠ° (L)
    • ΠŸΠ»ΠΎΡ‰Π°Π΄ΡŒ ΠΏΠΎΠΏΠ΅Ρ€Π΅Ρ‡Π½ΠΎΠ³ΠΎ сСчСния ΠΏΡ€ΠΎΠ²ΠΎΠ΄Π½ΠΈΠΊΠ° (A)
    • УдСльноС сопротивлСниС ΠΌΠ°Ρ‚Π΅Ρ€ΠΈΠ°Π»Π° ΠΏΡ€ΠΎΠ²ΠΎΠ΄Π½ΠΈΠΊΠ° (ρ)

    Π­Ρ‚ΠΎ ΡƒΡ€Π°Π²Π½Π΅Π½ΠΈΠ΅ Π½ΠΈΠΆΠ΅ ΠΏΠΎΠΊΠ°Π·Ρ‹Π²Π°Π΅Ρ‚ взаимосвязь ΠΌΠ΅ΠΆΠ΄Ρƒ этими Ρ„Π°ΠΊΡ‚ΠΎΡ€Π°ΠΌΠΈ:

    Π”Π»ΠΈΠ½Π°

    Π”Π»ΠΈΠ½Π° ΠΏΡ€ΠΎΠ²ΠΎΠ΄Π½ΠΈΠΊΠ° влияСт Π½Π° Π·Π½Π°Ρ‡Π΅Π½ΠΈΠ΅ Π΅Π³ΠΎ сопротивлСния.Π§Π΅ΠΌ Π΄Π»ΠΈΠ½Π½Π΅Π΅ ΠΏΡ€ΠΎΠ²ΠΎΠ΄Π½ΠΈΠΊ, Ρ‚Π΅ΠΌ большС сопротивлСниС. Π­Ρ‚ΠΎ ΠΏΠΎΡ‚ΠΎΠΌΡƒ, Ρ‡Ρ‚ΠΎ элСктроны ΡΡ‚Π°Π»ΠΊΠΈΠ²Π°ΡŽΡ‚ΡΡ с большим количСством ΠΈΠΎΠ½ΠΎΠ² ΠΏΠΎ ΠΌΠ΅Ρ€Π΅ ΠΈΡ… прохоТдСния. Π‘Π»Π΅Π΄ΠΎΠ²Π°Ρ‚Π΅Π»ΡŒΠ½ΠΎ, Π΄Π»ΠΈΠ½Π° ΠΏΡ€ΠΎΠ²ΠΎΠ΄Π½ΠΈΠΊΠ° ΠΏΡ€ΠΎΠΏΠΎΡ€Ρ†ΠΈΠΎΠ½Π°Π»ΡŒΠ½Π° ΡΠΎΠΏΡ€ΠΎΡ‚ΠΈΠ²Π»Π΅Π½ΠΈΡŽ ΠΏΡ€ΠΎΠ²ΠΎΠ΄Π½ΠΈΠΊΠ°.

    ΠŸΠ»ΠΎΡ‰Π°Π΄ΡŒ ΠΏΠΎΠΏΠ΅Ρ€Π΅Ρ‡Π½ΠΎΠ³ΠΎ сСчСния

    Π”ΠΈΠ°ΠΌΠ΅Ρ‚Ρ€ ΠΈΠ»ΠΈ ΠΏΠ»ΠΎΡ‰Π°Π΄ΡŒ ΠΏΠΎΠΏΠ΅Ρ€Π΅Ρ‡Π½ΠΎΠ³ΠΎ сСчСния ΠΏΡ€ΠΎΠ²ΠΎΠ΄Π½ΠΈΠΊΠ° Ρ‚Π°ΠΊΠΆΠ΅ влияСт Π½Π° Π·Π½Π°Ρ‡Π΅Π½ΠΈΠ΅ Π΅Π³ΠΎ сопротивлСния. Π§Π΅ΠΌ большС Π΄ΠΈΠ°ΠΌΠ΅Ρ‚Ρ€ ΠΏΡ€ΠΎΠ²ΠΎΠ΄Π° ΠΈΠ»ΠΈ CSA ΠΏΡ€ΠΎΠ²ΠΎΠ΄Π½ΠΈΠΊΠ°, Ρ‚Π΅ΠΌ мСньшС сопротивлСниС ΠΏΡ€ΠΎΠ²ΠΎΠ΄Π½ΠΈΠΊΠ°. Π‘ΠΎΠΏΡ€ΠΎΡ‚ΠΈΠ²Π»Π΅Π½ΠΈΠ΅ Π²ΠΎΠ·Π½ΠΈΠΊΠ°Π΅Ρ‚ ΠΈΠ·-Π·Π° столкновСния ΠΈΠΎΠ½ΠΎΠ² / элСктронов, ΠΈ Ссли CSA ΠΏΡ€ΠΎΠ²ΠΎΠ΄Π½ΠΈΠΊΠ° увСличиваСтся, Π·Π°Π·ΠΎΡ€ ΠΌΠ΅ΠΆΠ΄Ρƒ элСктронами Ρ‚Π°ΠΊΠΆΠ΅ увСличиваСтся.Π’Π΅ΠΏΠ΅Ρ€ΡŒ это ΡƒΠΌΠ΅Π½ΡŒΡˆΠ°Π΅Ρ‚ количСство происходящих столкновСний, Ρ‚Π΅ΠΌ самым ΡƒΠΌΠ΅Π½ΡŒΡˆΠ°Ρ сопротивлСниС ΠΏΡ€ΠΎΠ²ΠΎΠ΄Π½ΠΈΠΊΠ°.

    УдСльноС сопротивлСниС

    Π’Ρ€Π΅Ρ‚ΠΈΠΉ Ρ„Π°ΠΊΡ‚ΠΎΡ€, Π²Π»ΠΈΡΡŽΡ‰ΠΈΠΉ Π½Π° сопротивлСниС ΠΏΡ€ΠΎΠ²ΠΎΠ΄Π½ΠΈΠΊΠ°, – это ΡƒΠ΄Π΅Π»ΡŒΠ½ΠΎΠ΅ сопротивлСниС ΠΌΠ°Ρ‚Π΅Ρ€ΠΈΠ°Π»Π° ΠΏΡ€ΠΈ ΠΏΡ€ΠΎΡ…ΠΎΠΆΠ΄Π΅Π½ΠΈΠΈ Ρ‚ΠΎΠΊΠ° (ΠΏΡ€ΠΎΠ²ΠΎΠ΄Π½ΠΈΠΊΠ°). Π£ Ρ€Π°Π·Π½Ρ‹Ρ… ΠΌΠ°Ρ‚Π΅Ρ€ΠΈΠ°Π»ΠΎΠ² Ρ€Π°Π·Π½Ρ‹Π΅ значСния ΡƒΠ΄Π΅Π»ΡŒΠ½ΠΎΠ³ΠΎ сопротивлСния. Как ΠΏΠΎΠΊΠ°Π·Π°Π½ΠΎ Π² ΠΏΡ€ΠΈΠ²Π΅Π΄Π΅Π½Π½ΠΎΠΌ Π²Ρ‹ΡˆΠ΅ ΡƒΡ€Π°Π²Π½Π΅Π½ΠΈΠΈ, сопротивлСниС прямо ΠΏΡ€ΠΎΠΏΠΎΡ€Ρ†ΠΈΠΎΠ½Π°Π»ΡŒΠ½ΠΎ ΡƒΠ΄Π΅Π»ΡŒΠ½ΠΎΠΌΡƒ ΡΠΎΠΏΡ€ΠΎΡ‚ΠΈΠ²Π»Π΅Π½ΠΈΡŽ.

    РСзистор с Π’ΠΈΠΏΠΈΡ‡Π½Ρ‹ΠΉ рСзистор, ΠΈΡΠΏΠΎΠ»ΡŒΠ·ΡƒΠ΅ΠΌΡ‹ΠΉ Π² схСмах DIY

    РСзистор – это пассивный элСктричСский ΠΊΠΎΠΌΠΏΠΎΠ½Π΅Π½Ρ‚, ΠΊΠΎΡ‚ΠΎΡ€Ρ‹ΠΉ добавляСт ΠΎΠΏΡ€Π΅Π΄Π΅Π»Π΅Π½Π½ΠΎΠ΅ Π·Π½Π°Ρ‡Π΅Π½ΠΈΠ΅ сопротивлСния элСктричСской Ρ†Π΅ΠΏΠΈ.РСзисторы ΠΈΡΠΏΠΎΠ»ΡŒΠ·ΡƒΡŽΡ‚ΡΡ, срСди ΠΏΡ€ΠΎΡ‡Π΅Π³ΠΎ, для ΡƒΠΌΠ΅Π½ΡŒΡˆΠ΅Π½ΠΈΡ протСкания Ρ‚ΠΎΠΊΠ°, Ρ€Π΅Π³ΡƒΠ»ΠΈΡ€ΠΎΠ²ΠΊΠΈ напряТСний. РСзистор ΠΏΡ€Π΅ΠΎΠ±Ρ€Π°Π·ΡƒΠ΅Ρ‚ ΡΠ»Π΅ΠΊΡ‚Ρ€ΠΈΡ‡Π΅ΡΠΊΡƒΡŽ ΡΠ½Π΅Ρ€Π³ΠΈΡŽ Π² Ρ‚Π΅ΠΏΠ»ΠΎ ΠΈ ΠΎΠ±Ρ‹Ρ‡Π½ΠΎ состоит ΠΈΠ· Π½Π΅ΡΠΊΠΎΠ»ΡŒΠΊΠΈΡ… ΠΌΠ΅Π΄Π½Ρ‹Ρ… Π²ΠΈΡ‚ΠΊΠΎΠ². Π’ΠΎΠ»Ρ‰ΠΈΠ½Π° ΠΈ Π΄Π»ΠΈΠ½Π° этой ΠΌΠ΅Π΄Π½ΠΎΠΉ ΠΊΠ°Ρ‚ΡƒΡˆΠΊΠΈ ΠΎΠΏΡ€Π΅Π΄Π΅Π»ΡΡŽΡ‚ фактичСскоС Π·Π½Π°Ρ‡Π΅Π½ΠΈΠ΅ сопротивлСния. ΠŸΠΎΡΡ‚ΠΎΠΌΡƒ рСзисторы ΠΈΡΠΏΠΎΠ»ΡŒΠ·ΡƒΡŽΡ‚ΡΡ ΠΏΠΎΡ‡Ρ‚ΠΈ Π²ΠΎ всСх элСктронных устройствах ΠΈ Π³Π°Π΄ΠΆΠ΅Ρ‚Π°Ρ…, ΠΏΠΎΡΠΊΠΎΠ»ΡŒΠΊΡƒ ΠΎΠ½ΠΈ слуТат ΠΎΠ΄Π½ΠΈΠΌ ΠΈΠ· самых Ρ„ΡƒΠ½Π΄Π°ΠΌΠ΅Π½Ρ‚Π°Π»ΡŒΠ½Ρ‹Ρ… ΠΊΠΎΠΌΠΏΠΎΠ½Π΅Π½Ρ‚ΠΎΠ² элСктричСских Ρ†Π΅ΠΏΠ΅ΠΉ.

    РассСиваСмая ΠΌΠΎΡ‰Π½ΠΎΡΡ‚ΡŒ

    Как ΡƒΠΏΠΎΠΌΠΈΠ½Π°Π»ΠΎΡΡŒ Ρ€Π°Π½Π΅Π΅, рСзисторы Ρ€Π°Π±ΠΎΡ‚Π°ΡŽΡ‚ Π·Π° счСт рассСивания мощности ΠΏΡƒΡ‚Π΅ΠΌ прСобразования элСктричСской энСргии Π² Ρ‚Π΅ΠΏΠ»ΠΎΠ²ΡƒΡŽ.Π˜ΡΠΏΠΎΠ»ΡŒΠ·ΡƒΠΉΡ‚Π΅ ΡΠ»Π΅Π΄ΡƒΡŽΡ‰Π΅Π΅ ΡƒΡ€Π°Π²Π½Π΅Π½ΠΈΠ΅ для расчСта ΠΏΠΎΡ‚Π΅Ρ€ΡŒ мощности:

    ΠžΠΏΡ€Π΅Π΄Π΅Π»Π΅Π½ΠΈΠ΅ ΠΏΠΎΡ‚Π΅Ρ€ΡŒ мощности рСзистора Ρ‚Π°ΠΊΠΆΠ΅ Π²Π°ΠΆΠ½ΠΎ, ΠΏΠΎΡΠΊΠΎΠ»ΡŒΠΊΡƒ Ρ€Π°Π·Π½Ρ‹Π΅ рСзисторы ΠΈΠΌΠ΅ΡŽΡ‚ Ρ€Π°Π·Π½ΡƒΡŽ Π½ΠΎΠΌΠΈΠ½Π°Π»ΡŒΠ½ΡƒΡŽ ΠΌΠΎΡ‰Π½ΠΎΡΡ‚ΡŒ. Если расчСтная потСря мощности рСзистора Π² Ρ†Π΅ΠΏΠΈ ΠΏΡ€Π΅Π²Ρ‹ΡˆΠ°Π΅Ρ‚ Π½ΠΎΠΌΠΈΠ½Π°Π»ΡŒΠ½ΡƒΡŽ ΠΌΠΎΡ‰Π½ΠΎΡΡ‚ΡŒ рСзистора, рСзистор, скорСС всСго, Π²Ρ‹ΠΉΠ΄Π΅Ρ‚ ΠΈΠ· строя ΠΈΠ·-Π·Π° ΠΏΠ΅Ρ€Π΅Π³Ρ€Π΅Π²Π°.

    ΠŸΡ€ΠΈΠΌΠ΅Ρ€ Π·Π°Π΄Π°Ρ‡ΠΈ

    Π§Ρ‚ΠΎΠ±Ρ‹ Ρ€Π°ΡΡΡ‡ΠΈΡ‚Π°Ρ‚ΡŒ ΠΌΠΎΡ‰Π½ΠΎΡΡ‚ΡŒ, Ρ€Π°ΡΡΠ΅ΠΈΠ²Π°Π΅ΠΌΡƒΡŽ рСзистором Π² ΠΏΡ€ΠΈΠ²Π΅Π΄Π΅Π½Π½ΠΎΠΉ Π²Ρ‹ΡˆΠ΅ схСмС, Π½Π°ΠΌ Π½ΡƒΠΆΠ½ΠΎ ΠΎΠΏΡ€Π΅Π΄Π΅Π»ΠΈΡ‚ΡŒ Π²Π΅Π»ΠΈΡ‡ΠΈΠ½Ρƒ Ρ‚ΠΎΠΊΠ°, ΠΏΡ€ΠΎΡ‚Π΅ΠΊΠ°ΡŽΡ‰Π΅Π³ΠΎ ΠΏΠΎ Ρ†Π΅ΠΏΠΈ.Π­Ρ‚ΠΎ ΠΌΠΎΠΆΠ½ΠΎ Ρ€Π°ΡΡΡ‡ΠΈΡ‚Π°Ρ‚ΡŒ с ΠΏΠΎΠΌΠΎΡ‰ΡŒΡŽ Π·Π°ΠΊΠΎΠ½Π° Ома, Π³Π΄Π΅ I = V / R. Π‘Π»Π΅Π΄ΠΎΠ²Π°Ρ‚Π΅Π»ΡŒΠ½ΠΎ, 9/50 = 0,18 А. Π˜ΡΠΏΠΎΠ»ΡŒΠ·ΡƒΡ ΡƒΡ€Π°Π²Π½Π΅Π½ΠΈΠ΅ P = IV, ΠΌΡ‹ ΠΏΠΎΠ»ΡƒΡ‡Π°Π΅ΠΌ, Ρ‡Ρ‚ΠΎ ΠΌΠΎΡ‰Π½ΠΎΡΡ‚ΡŒ, рассСиваСмая ΡƒΠΊΠ°Π·Π°Π½Π½Ρ‹ΠΌ Π²Ρ‹ΡˆΠ΅ рСзистором, составляСт 1,62 Π’Ρ‚. Π­Ρ‚ΠΎ ΠΎΠ·Π½Π°Ρ‡Π°Π΅Ρ‚, Ρ‡Ρ‚ΠΎ номинальная ΠΌΠΎΡ‰Π½ΠΎΡΡ‚ΡŒ рСзистора Π΄ΠΎΠ»ΠΆΠ½Π° Π±Ρ‹Ρ‚ΡŒ большС 1,62 Π’Ρ‚, Ρ‡Ρ‚ΠΎΠ±Ρ‹ ΠΈΠ·Π±Π΅ΠΆΠ°Ρ‚ΡŒ ΠΏΠ΅Ρ€Π΅Π³Ρ€Π΅Π²Π°.

    РСзисторы Π² схСмах

    Когда Π΄Π΅Π»ΠΎ Π΄ΠΎΡ…ΠΎΠ΄ΠΈΡ‚ Π΄ΠΎ установки рСзисторов Π² Ρ†Π΅ΠΏΠΈ, сущСствуСт Π΄Π²Π΅ основныС ΠΊΠΎΠ½Ρ„ΠΈΠ³ΡƒΡ€Π°Ρ†ΠΈΠΈ: ΠΏΠΎΡΠ»Π΅Π΄ΠΎΠ²Π°Ρ‚Π΅Π»ΡŒΠ½ΠΎ ΠΈΠ»ΠΈ ΠΏΠ°Ρ€Π°Π»Π»Π΅Π»ΡŒΠ½ΠΎ.

    ЦСпь сСрии

    Π’ ΠΏΠΎΡΠ»Π΅Π΄ΠΎΠ²Π°Ρ‚Π΅Π»ΡŒΠ½ΠΎΠΉ Ρ†Π΅ΠΏΠΈ рСзисторы выстроСны ΠΎΠ΄ΠΈΠ½ Π·Π° Π΄Ρ€ΡƒΠ³ΠΈΠΌ.Π’ этой ΠΊΠΎΠ½Ρ„ΠΈΠ³ΡƒΡ€Π°Ρ†ΠΈΠΈ Ρ‚ΠΎΠΊ ΠΏΠΎ всСй Ρ†Π΅ΠΏΠΈ остаСтся постоянным. Однако Ρ€Π°Π·Π½ΠΎΡΡ‚ΡŒ ΠΏΠΎΡ‚Π΅Π½Ρ†ΠΈΠ°Π»ΠΎΠ² ΠΌΠ΅ΠΆΠ΄Ρƒ ΠΊΠ°ΠΆΠ΄Ρ‹ΠΌ рСзистором ΠΌΠΎΠΆΠ΅Ρ‚ Π²Π°Ρ€ΡŒΠΈΡ€ΠΎΠ²Π°Ρ‚ΡŒΡΡ Π² зависимости ΠΎΡ‚ Π½ΠΎΠΌΠΈΠ½Π°Π»Π° ΠΊΠ°ΠΆΠ΄ΠΎΠ³ΠΎ рСзистора.

    Π˜ΡΠΏΠΎΠ»ΡŒΠ·ΡƒΠΉΡ‚Π΅ это ΡƒΡ€Π°Π²Π½Π΅Π½ΠΈΠ΅, Ρ‡Ρ‚ΠΎΠ±Ρ‹ ΠΏΠΎΠ»ΡƒΡ‡ΠΈΡ‚ΡŒ ΠΏΠΎΠ»Π½ΠΎΠ΅ сопротивлСниС Π² ΠΏΠΎΡΠ»Π΅Π΄ΠΎΠ²Π°Ρ‚Π΅Π»ΡŒΠ½ΠΎΠΉ Ρ†Π΅ΠΏΠΈ:

    ΠŸΠ°Ρ€Π°Π»Π»Π΅Π»ΡŒΠ½Ρ‹Π΅ схСмы

    Π’ ΠΏΠ°Ρ€Π°Π»Π»Π΅Π»ΡŒΠ½ΠΎΠΉ Ρ†Π΅ΠΏΠΈ рСзисторы выстроСны Β«ΠΏΠ°Ρ€Π°Π»Π»Π΅Π»ΡŒΠ½ΠΎΒ» ΠΎΠ΄ΠΈΠ½ Π·Π° Π΄Ρ€ΡƒΠ³ΠΈΠΌ. Π’ этой ΠΊΠΎΠ½Ρ„ΠΈΠ³ΡƒΡ€Π°Ρ†ΠΈΠΈ ΠΏΠ°Π΄Π΅Π½ΠΈΠ΅ напряТСния Π½Π° ΠΊΠ°ΠΆΠ΄ΠΎΠΌ рСзисторС остаСтся постоянным.Однако Ρ‚ΠΎΠΊ Π½Π° ΠΊΠ°ΠΆΠ΄ΠΎΠΌ рСзисторС ΠΌΠΎΠΆΠ΅Ρ‚ Π²Π°Ρ€ΡŒΠΈΡ€ΠΎΠ²Π°Ρ‚ΡŒΡΡ Π² зависимости ΠΎΡ‚ Π½ΠΎΠΌΠΈΠ½Π°Π»Π° ΠΊΠ°ΠΆΠ΄ΠΎΠ³ΠΎ рСзистора.

    Π˜ΡΠΏΠΎΠ»ΡŒΠ·ΡƒΠΉΡ‚Π΅ это ΡƒΡ€Π°Π²Π½Π΅Π½ΠΈΠ΅, Ρ‡Ρ‚ΠΎΠ±Ρ‹ ΠΏΠΎΠ»ΡƒΡ‡ΠΈΡ‚ΡŒ ΠΏΠΎΠ»Π½ΠΎΠ΅ сопротивлСниС Π² ΠΏΠ°Ρ€Π°Π»Π»Π΅Π»ΡŒΠ½ΠΎΠΉ Ρ†Π΅ΠΏΠΈ:


    Π‘ΠΎΠΏΡ€ΠΎΡ‚ΠΈΠ²Π»Π΅Π½ΠΈΠ΅ Π² ΠΏΠ°Ρ€Π°Π»Π»Π΅Π»ΡŒΠ½ΠΎΠΉ Ρ†Π΅ΠΏΠΈ

    Π‘ΠΎΠΏΡ€ΠΎΡ‚ΠΈΠ²Π»Π΅Π½ΠΈΠ΅ Π² ΠΏΠ°Ρ€Π°Π»Π»Π΅Π»ΡŒΠ½ΠΎΠΉ Ρ†Π΅ΠΏΠΈ
    На ΠΏΡ€ΠΈΠΌΠ΅Ρ€Π½ΠΎΠΉ схСмС, рис. 3-44, Π΄Π²Π° рСзистора соСдинСны ΠΏΠ°Ρ€Π°Π»Π»Π΅Π»ΡŒΠ½ΠΎ Ρ‡Π΅Ρ€Π΅Π· 5-Π²ΠΎΠ»ΡŒΡ‚ΠΎΠ²ΡƒΡŽ Π±Π°Ρ‚Π°Ρ€Π΅ΡŽ.ΠšΠ°ΠΆΠ΄Ρ‹ΠΉ ΠΈΠΌΠ΅Π΅Ρ‚ Π·Π½Π°Ρ‡Π΅Π½ΠΈΠ΅ сопротивлСния 10 Ом. ЀормируСтся полная Ρ†Π΅ΠΏΡŒ, состоящая ΠΈΠ· Π΄Π²ΡƒΡ… ΠΏΠ°Ρ€Π°Π»Π»Π΅Π»ΡŒΠ½Ρ‹Ρ… ΠΏΡƒΡ‚Π΅ΠΉ, ΠΈ Ρ‚ΠΎΠΊ Ρ‚Π΅Ρ‡Π΅Ρ‚, ΠΊΠ°ΠΊ ΠΏΠΎΠΊΠ°Π·Π°Π½ΠΎ.

    Рисунок 3-44. – Π”Π²Π° Ρ€Π°Π²Π½Ρ‹Ρ… рСзистора, Π²ΠΊΠ»ΡŽΡ‡Π΅Π½Π½Ρ‹Ρ… ΠΏΠ°Ρ€Π°Π»Π»Π΅Π»ΡŒΠ½ΠΎ.

    ВычислСниС ΠΎΡ‚Π΄Π΅Π»ΡŒΠ½Ρ‹Ρ… Ρ‚ΠΎΠΊΠΎΠ² ΠΏΠΎΠΊΠ°Π·Ρ‹Π²Π°Π΅Ρ‚, Ρ‡Ρ‚ΠΎ Ρ‡Π΅Ρ€Π΅Π· ΠΊΠ°ΠΆΠ΄ΠΎΠ΅ сопротивлСниС ΠΏΡ€ΠΎΡ…ΠΎΠ΄ΠΈΡ‚ ΠΏΠΎΠ»ΠΎΠ²ΠΈΠ½Π° Π°ΠΌΠΏΠ΅Ρ€Π°. ΠŸΠΎΠ»Π½Ρ‹ΠΉ Ρ‚ΠΎΠΊ, ΠΏΡ€ΠΎΡ‚Π΅ΠΊΠ°ΡŽΡ‰ΠΈΠΉ ΠΎΡ‚ Π±Π°Ρ‚Π°Ρ€Π΅ΠΈ ΠΊ ΠΏΠ΅Ρ€Π΅Ρ…ΠΎΠ΄Ρƒ рСзисторов ΠΈ Π²ΠΎΠ·Π²Ρ€Π°Ρ‰Π°ΡŽΡ‰ΠΈΠΉΡΡ ΠΎΡ‚ рСзисторов ΠΊ Π±Π°Ρ‚Π°Ρ€Π΅Π΅, Ρ€Π°Π²Π΅Π½ 1 Π°ΠΌΠΏΠ΅Ρ€Π°ΠΌ.

    ΠžΠ±Ρ‰Π΅Π΅ сопротивлСниС Ρ†Π΅ΠΏΠΈ ΠΌΠΎΠΆΠ½ΠΎ Ρ€Π°ΡΡΡ‡ΠΈΡ‚Π°Ρ‚ΡŒ, ΠΈΡΠΏΠΎΠ»ΡŒΠ·ΡƒΡ
    значСния ΠΏΠΎΠ»Π½ΠΎΠ³ΠΎ напряТСния (E T ) ΠΈ ΠΏΠΎΠ»Π½ΠΎΠ³ΠΎ Ρ‚ΠΎΠΊΠ° (I T ).

    ΠŸΠ Π˜ΠœΠ•Π§ΠΠΠ˜Π•: Π‘ этого ΠΌΠΎΠΌΠ΅Π½Ρ‚Π° Π² ΠΏΡ€ΠΈΠΌΠ΅Ρ€Π°Ρ… Π·Π°Π΄Π°Ρ‡ Π±ΡƒΠ΄ΡƒΡ‚ ΠΈΡΠΏΠΎΠ»ΡŒΠ·ΠΎΠ²Π°Ρ‚ΡŒΡΡ сокращСния ΠΈ символы для элСктричСских Π²Π΅Π»ΠΈΡ‡ΠΈΠ½.

    Π”Π°Π½ΠΎ:


    РСшСниС:


    Π­Ρ‚ΠΎ вычислСниС ΠΏΠΎΠΊΠ°Π·Ρ‹Π²Π°Π΅Ρ‚, Ρ‡Ρ‚ΠΎ ΠΏΠΎΠ»Π½ΠΎΠ΅ сопротивлСниС составляСт 5 Ом; ΠΏΠΎΠ»ΠΎΠ²ΠΈΠ½Π° Π½ΠΎΠΌΠΈΠ½Π°Π»Π° любого ΠΈΠ· Π΄Π²ΡƒΡ… рСзисторов.

    ΠŸΠΎΡΠΊΠΎΠ»ΡŒΠΊΡƒ ΠΎΠ±Ρ‰Π΅Π΅ сопротивлСниС ΠΏΠ°Ρ€Π°Π»Π»Π΅Π»ΡŒΠ½ΠΎΠΉ Ρ†Π΅ΠΏΠΈ мСньшС, Ρ‡Π΅ΠΌ Ρƒ любого ΠΈΠ· ΠΎΡ‚Π΄Π΅Π»ΡŒΠ½Ρ‹Ρ… рСзисторов, ΠΎΠ±Ρ‰Π΅Π΅ сопротивлСниС ΠΏΠ°Ρ€Π°Π»Π»Π΅Π»ΡŒΠ½ΠΎΠΉ Ρ†Π΅ΠΏΠΈ Π½Π΅ являСтся суммой Π·Π½Π°Ρ‡Π΅Π½ΠΈΠΉ ΠΎΡ‚Π΄Π΅Π»ΡŒΠ½Ρ‹Ρ… рСзисторов, ΠΊΠ°ΠΊ это Π±Ρ‹Π»ΠΎ Π² случаС ΠΏΠΎΡΠ»Π΅Π΄ΠΎΠ²Π°Ρ‚Π΅Π»ΡŒΠ½ΠΎΠΉ Ρ†Π΅ΠΏΠΈ.ΠžΠ±Ρ‰Π΅Π΅ сопротивлСниС ΠΏΠ°Ρ€Π°Π»Π»Π΅Π»ΡŒΠ½ΠΎ Π²ΠΊΠ»ΡŽΡ‡Π΅Π½Π½Ρ‹Ρ… рСзисторов Ρ‚Π°ΠΊΠΆΠ΅ называСтся Π­ΠšΠ’Π˜Π’ΠΠ›Π•ΠΠ’ΠΠ«Πœ Π‘ΠžΠŸΠ ΠžΠ’Π˜Π’Π›Π•ΠΠ˜Π•Πœ. ( рэндов экв. ). Π’Π΅Ρ€ΠΌΠΈΠ½Ρ‹ ΠΏΠΎΠ»Π½ΠΎΠ΅ сопротивлСниС ΠΈ эквивалСнтноС сопротивлСниС ΠΈΡΠΏΠΎΠ»ΡŒΠ·ΡƒΡŽΡ‚ΡΡ ΠΊΠ°ΠΊ синонимы.

    БущСствуСт нСсколько ΠΌΠ΅Ρ‚ΠΎΠ΄ΠΎΠ² опрСдСлСния эквивалСнтного сопротивлСния ΠΏΠ°Ρ€Π°Π»Π»Π΅Π»ΡŒΠ½Ρ‹Ρ… Ρ†Π΅ΠΏΠ΅ΠΉ. Π›ΡƒΡ‡ΡˆΠΈΠΉ ΠΌΠ΅Ρ‚ΠΎΠ΄ для Π΄Π°Π½Π½ΠΎΠΉ схСмы зависит ΠΎΡ‚ количСства ΠΈ Π½ΠΎΠΌΠΈΠ½Π°Π»Π° рСзисторов. Для схСмы, описанной Π²Ρ‹ΡˆΠ΅, Π³Π΄Π΅ всС рСзисторы ΠΈΠΌΠ΅ΡŽΡ‚ ΠΎΠ΄ΠΈΠ½Π°ΠΊΠΎΠ²ΠΎΠ΅ Π·Π½Π°Ρ‡Π΅Π½ΠΈΠ΅, ΠΈΡΠΏΠΎΠ»ΡŒΠ·ΡƒΠ΅Ρ‚ΡΡ ΡΠ»Π΅Π΄ΡƒΡŽΡ‰Π΅Π΅ простоС ΡƒΡ€Π°Π²Π½Π΅Π½ΠΈΠ΅:


    Π­Ρ‚ΠΎ ΡƒΡ€Π°Π²Π½Π΅Π½ΠΈΠ΅ Π΄Π΅ΠΉΡΡ‚Π²ΠΈΡ‚Π΅Π»ΡŒΠ½ΠΎ для любого количСства ΠΏΠ°Ρ€Π°Π»Π»Π΅Π»ΡŒΠ½Ρ‹Ρ… рСзисторов с Π ΠΠ’ΠΠ«Πœ Π—ΠΠΠ§Π•ΠΠ˜Π•Πœ.

    ΠŸΡ€ΠΈΠΌΠ΅Ρ€. ΠŸΠ°Ρ€Π°Π»Π»Π΅Π»ΡŒΠ½ΠΎ ΠΏΠΎΠ΄ΠΊΠ»ΡŽΡ‡Π΅Π½Ρ‹ Ρ‡Π΅Ρ‚Ρ‹Ρ€Π΅ рСзистора Π½Π° 40 Ом. Каково ΠΈΡ… эквивалСнтноС сопротивлСниС?

    Π”Π°Π½ΠΎ:


    РСшСниС:


    На рис. 3-45 ΠΏΠΎΠΊΠ°Π·Π°Π½Ρ‹ Π΄Π²Π° рСзистора Ρ€Π°Π·Π½ΠΎΠ³ΠΎ Π½ΠΎΠΌΠΈΠ½Π°Π»Π°, Π²ΠΊΠ»ΡŽΡ‡Π΅Π½Π½Ρ‹Π΅ ΠΏΠ°Ρ€Π°Π»Π»Π΅Π»ΡŒΠ½ΠΎ. ΠŸΠΎΡΠΊΠΎΠ»ΡŒΠΊΡƒ ΠΏΠΎΠΊΠ°Π·Π°Π½ ΠΏΠΎΠ»Π½Ρ‹ΠΉ Ρ‚ΠΎΠΊ, ΠΌΠΎΠΆΠ½ΠΎ Ρ€Π°ΡΡΡ‡ΠΈΡ‚Π°Ρ‚ΡŒ эквивалСнтноС сопротивлСниС.

    Рисунок 3-45. – ΠŸΡ€ΠΈΠΌΠ΅Ρ€ схСмы с Π½Π΅Ρ€Π°Π²Π½ΠΎΠΏΠ°Ρ€Π°Π»Π»Π΅Π»ΡŒΠ½Ρ‹ΠΌΠΈ рСзисторами.


    Π”Π°Π½ΠΎ:


    РСшСниС:


    Π­ΠΊΠ²ΠΈΠ²Π°Π»Π΅Π½Ρ‚Π½ΠΎΠ΅ сопротивлСниС Ρ†Π΅ΠΏΠΈ, ΠΏΠΎΠΊΠ°Π·Π°Π½Π½ΠΎΠΉ Π½Π° рисункС 3-45, мСньшС, Ρ‡Π΅ΠΌ Ρƒ любого ΠΈΠ· Π΄Π²ΡƒΡ… рСзисторов (R 1 , R 2 ).Π’Π°ΠΆΠ½ΠΎ ΠΏΠΎΠΌΠ½ΠΈΡ‚ΡŒ, Ρ‡Ρ‚ΠΎ эквивалСнтноС сопротивлСниС ΠΏΠ°Ρ€Π°Π»Π»Π΅Π»ΡŒΠ½ΠΎΠΉ Ρ†Π΅ΠΏΠΈ всСгда мСньшС, Ρ‡Π΅ΠΌ сопротивлСниС любой Π²Π΅Ρ‚Π²ΠΈ.

    Π­ΠΊΠ²ΠΈΠ²Π°Π»Π΅Π½Ρ‚Π½ΠΎΠ΅ сопротивлСниС ΠΌΠΎΠΆΠ½ΠΎ Π½Π°ΠΉΡ‚ΠΈ, зная ΠΎΡ‚Π΄Π΅Π»ΡŒΠ½Ρ‹Π΅ значСния сопротивлСния ΠΈ напряТСниС источника. Вычисляя Ρ‚ΠΎΠΊ ΠΊΠ°ΠΆΠ΄ΠΎΠΉ Π²Π΅Ρ‚Π²ΠΈ, складывая Ρ‚ΠΎΠΊΠΈ Π²Π΅Ρ‚Π²ΠΈ для вычислСния ΠΎΠ±Ρ‰Π΅Π³ΠΎ Ρ‚ΠΎΠΊΠ° ΠΈ Ρ€Π°Π·Π΄Π΅Π»ΠΈΠ² напряТСниС источника Π½Π° ΠΎΠ±Ρ‰ΠΈΠΉ Ρ‚ΠΎΠΊ, ΠΌΠΎΠΆΠ½ΠΎ Π½Π°ΠΉΡ‚ΠΈ ΠΎΠ±Ρ‰ΠΈΠΉ Ρ‚ΠΎΠΊ. Π­Ρ‚ΠΎΡ‚ ΠΌΠ΅Ρ‚ΠΎΠ΄ Ρ…ΠΎΡ‚ΡŒ ΠΈ эффСктивСн, Π½ΠΎ довольно Π΄Π»ΠΈΡ‚Π΅Π»ΡŒΠ½Ρ‹ΠΉ. Π‘ΠΎΠ»Π΅Π΅ быстрый способ Π½Π°ΠΉΡ‚ΠΈ эквивалСнтноС сопротивлСниС – ΠΈΡΠΏΠΎΠ»ΡŒΠ·ΠΎΠ²Π°Ρ‚ΡŒ ΠΎΠ±Ρ‰ΡƒΡŽ Ρ„ΠΎΡ€ΠΌΡƒΠ»Ρƒ для ΠΏΠ°Ρ€Π°Π»Π»Π΅Π»ΡŒΠ½Ρ‹Ρ… рСзисторов:


    Если Π²Ρ‹ ΠΏΡ€ΠΈΠΌΠ΅Π½ΠΈΡ‚Π΅ ΠΎΠ±Ρ‰ΡƒΡŽ Ρ„ΠΎΡ€ΠΌΡƒΠ»Ρƒ ΠΊ схСмС, ΠΏΠΎΠΊΠ°Π·Π°Π½Π½ΠΎΠΉ Π½Π° рисункС 3-45, Π²Ρ‹ ΠΏΠΎΠ»ΡƒΡ‡ΠΈΡ‚Π΅ Ρ‚ΠΎ ΠΆΠ΅ Π·Π½Π°Ρ‡Π΅Π½ΠΈΠ΅ для эквивалСнтного сопротивлСния (2 Ом), Ρ‡Ρ‚ΠΎ ΠΈ Π±Ρ‹Π»ΠΎ. ΠΏΠΎΠ»ΡƒΡ‡Π΅Π½Π½Ρ‹ΠΉ Π² ΠΏΡ€Π΅Π΄Ρ‹Π΄ΡƒΡ‰Π΅ΠΌ расчСтС, Π² ΠΊΠΎΡ‚ΠΎΡ€ΠΎΠΌ использовались напряТСниС источника ΠΈ ΠΏΠΎΠ»Π½Ρ‹ΠΉ Ρ‚ΠΎΠΊ.

    Π”Π°Π½ΠΎ:


    РСшСниС:


    ΠŸΡ€Π΅ΠΎΠ±Ρ€Π°Π·ΡƒΠΉΡ‚Π΅ Π΄Ρ€ΠΎΠ±ΠΈ ΠΊ ΠΎΠ±Ρ‰Π΅ΠΌΡƒ Π·Π½Π°ΠΌΠ΅Π½Π°Ρ‚Π΅Π»ΡŽ.


    ΠŸΠΎΡΠΊΠΎΠ»ΡŒΠΊΡƒ ΠΎΠ±Π΅ стороны ΡΠ²Π»ΡΡŽΡ‚ΡΡ Π²Π·Π°ΠΈΠΌΠ½Ρ‹ΠΌΠΈ (Ρ€Π°Π·Π΄Π΅Π»Π΅Π½Π½Ρ‹ΠΌΠΈ Π½Π° ΠΎΠ΄Π½Ρƒ), ΠΈΠ³Π½ΠΎΡ€ΠΈΡ€ΡƒΠΉΡ‚Π΅ ΠΎΠ±Ρ€Π°Ρ‚Π½ΡƒΡŽ Ρ„ΡƒΠ½ΠΊΡ†ΠΈΡŽ.


    Π€ΠΎΡ€ΠΌΡƒΠ»Π°, ΠΊΠΎΡ‚ΠΎΡ€ΡƒΡŽ Π²Ρ‹ Π΄Π°Π»ΠΈ для ΠΎΠ΄ΠΈΠ½Π°ΠΊΠΎΠ²Ρ‹Ρ… рСзисторов, Π²ΠΊΠ»ΡŽΡ‡Π΅Π½Π½Ρ‹Ρ… ΠΏΠ°Ρ€Π°Π»Π»Π΅Π»ΡŒΠ½ΠΎ


    , являСтся ΡƒΠΏΡ€ΠΎΡ‰Π΅Π½ΠΈΠ΅ΠΌ ΠΎΠ±Ρ‰Π΅ΠΉ Ρ„ΠΎΡ€ΠΌΡƒΠ»Ρ‹ для ΠΏΠ°Ρ€Π°Π»Π»Π΅Π»ΡŒΠ½Ρ‹Ρ… рСзисторов


    Π‘ΡƒΡ‰Π΅ΡΡ‚Π²ΡƒΡŽΡ‚ ΠΈ Π΄Ρ€ΡƒΠ³ΠΈΠ΅ упрощСния ΠΎΠ±Ρ‰Π΅ΠΉ Ρ„ΠΎΡ€ΠΌΡƒΠ»Ρ‹ для ΠΏΠ°Ρ€Π°Π»Π»Π΅Π»ΡŒΠ½Ρ‹Ρ… рСзисторов, ΠΊΠΎΡ‚ΠΎΡ€Ρ‹Π΅ ΠΌΠΎΠΆΠ½ΠΎ ΠΈΡΠΏΠΎΠ»ΡŒΠ·ΠΎΠ²Π°Ρ‚ΡŒ для расчСта ΠΎΠ±Ρ‰Π΅ΠΉ ΠΈΠ»ΠΈ эквивалСнтноС сопротивлСниС Π² ΠΏΠ°Ρ€Π°Π»Π»Π΅Π»ΡŒΠ½ΠΎΠΉ Ρ†Π΅ΠΏΠΈ.

    Π’Π—ΠΠ˜ΠœΠΠ«Π™ ΠœΠ•Π’ΠžΠ”. – Π­Ρ‚ΠΎΡ‚ ΠΌΠ΅Ρ‚ΠΎΠ΄ основан Π½Π° взятии ΠΎΠ±Ρ€Π°Ρ‚Π½ΠΎΠΉ Π²Π΅Π»ΠΈΡ‡ΠΈΠ½Ρ‹ для ΠΊΠ°ΠΆΠ΄ΠΎΠΉ стороны уравнСния. Π­Ρ‚ΠΎ прСдставляСт собой ΠΎΠ±Ρ‰ΡƒΡŽ Ρ„ΠΎΡ€ΠΌΡƒΠ»Ρƒ для рСзисторов, Π²ΠΊΠ»ΡŽΡ‡Π΅Π½Π½Ρ‹Ρ… ΠΏΠ°Ρ€Π°Π»Π»Π΅Π»ΡŒΠ½ΠΎ:


    Π­Ρ‚Π° Ρ„ΠΎΡ€ΠΌΡƒΠ»Π° ΠΈΡΠΏΠΎΠ»ΡŒΠ·ΡƒΠ΅Ρ‚ΡΡ для опрСдСлСния эквивалСнтного сопротивлСния ряда Π½Π΅Ρ€Π°Π²Π½Ρ‹Ρ… ΠΏΠ°Ρ€Π°Π»Π»Π΅Π»ΡŒΠ½Ρ‹Ρ… рСзисторов. ΠŸΡ€ΠΈ Ρ€Π΅ΡˆΠ΅Π½ΠΈΠΈ этих Π·Π°Π΄Π°Ρ‡ Π²Ρ‹ Π΄ΠΎΠ»ΠΆΠ½Ρ‹ Π½Π°ΠΉΡ‚ΠΈ наимСньший ΠΎΠ±Ρ‰ΠΈΠΉ Π·Π½Π°ΠΌΠ΅Π½Π°Ρ‚Π΅Π»ΡŒ. Если Π²Ρ‹ Π½Π΅ Π·Π½Π°Π΅Ρ‚Π΅, ΠΊΠ°ΠΊ Π½Π°ΠΉΡ‚ΠΈ наимСньший ΠΎΠ±Ρ‰ΠΈΠΉ Π·Π½Π°ΠΌΠ΅Π½Π°Ρ‚Π΅Π»ΡŒ, освСТитС Π΅Π³ΠΎ Π² ΠΌΠ°Ρ‚Π΅ΠΌΠ°Ρ‚ΠΈΠΊΠ΅, Ρ‚ΠΎΠΌ 1, NAVEDTRA 10069 (сСрия).

    ΠŸΡ€ΠΈΠΌΠ΅Ρ€: Ρ‚Ρ€ΠΈ рСзистора ΠΏΠΎΠ΄ΠΊΠ»ΡŽΡ‡Π΅Π½Ρ‹ ΠΏΠ°Ρ€Π°Π»Π»Π΅Π»ΡŒΠ½ΠΎ, ΠΊΠ°ΠΊ ΠΏΠΎΠΊΠ°Π·Π°Π½ΠΎ Π½Π° рисункС 3-46. ЗначСния рСзистора: R 1 = 20 Ом, R 2 = 30 Ом, R 3 = 40 Ом. КакоС эквивалСнтноС сопротивлСниС? (Π˜ΡΠΏΠΎΠ»ΡŒΠ·ΡƒΠΉΡ‚Π΅ ΠΎΠ±Ρ€Π°Ρ‚Π½Ρ‹ΠΉ ΠΌΠ΅Ρ‚ΠΎΠ΄.)

    Рисунок 3-46. – ΠŸΡ€ΠΈΠΌΠ΅Ρ€ ΠΏΠ°Ρ€Π°Π»Π»Π΅Π»ΡŒΠ½ΠΎΠΉ схСмы с Ρ€Π°Π·Π½Ρ‹ΠΌΠΈ рСзисторами отвСтвлСния.


    Π”Π°Π½ΠΎ:


    РСшСниС:

    ΠŸΠ ΠžΠ”Π£ΠšΠ’ ΠŸΠ Π•Π’Π«Π¨ΠΠ•Π’ БУММУ. – Π£Π΄ΠΎΠ±Π½Ρ‹ΠΉ ΠΌΠ΅Ρ‚ΠΎΠ΄ опрСдСлСния эквивалСнтного ΠΈΠ»ΠΈ ΠΏΠΎΠ»Π½ΠΎΠ³ΠΎ сопротивлСния Π΄Π²ΡƒΡ… ΠΏΠ°Ρ€Π°Π»Π»Π΅Π»ΡŒΠ½Ρ‹Ρ… рСзисторов – ΠΈΡΠΏΠΎΠ»ΡŒΠ·ΠΎΠ²Π°Ρ‚ΡŒ ΡΠ»Π΅Π΄ΡƒΡŽΡ‰ΡƒΡŽ Ρ„ΠΎΡ€ΠΌΡƒΠ»Ρƒ.


    Π­Ρ‚ΠΎ ΡƒΡ€Π°Π²Π½Π΅Π½ΠΈΠ΅, Π½Π°Π·Ρ‹Π²Π°Π΅ΠΌΠΎΠ΅ Ρ„ΠΎΡ€ΠΌΡƒΠ»ΠΎΠΉ произвСдСния Π½Π° сумму, ΠΈΡΠΏΠΎΠ»ΡŒΠ·ΡƒΠ΅Ρ‚ΡΡ Π½Π°ΡΡ‚ΠΎΠ»ΡŒΠΊΠΎ часто, Ρ‡Ρ‚ΠΎ Π΅Π³ΠΎ слСдуСт ΡΠΎΡ…Ρ€Π°Π½ΠΈΡ‚ΡŒ Π² памяти.

    ΠŸΡ€ΠΈΠΌΠ΅Ρ€. КакоС эквивалСнтноС сопротивлСниС рСзисторов Π½Π° 20 Ом ΠΈ 30 Ом, ΠΏΠΎΠ΄ΠΊΠ»ΡŽΡ‡Π΅Π½Π½Ρ‹Ρ… ΠΏΠ°Ρ€Π°Π»Π»Π΅Π»ΡŒΠ½ΠΎ, ΠΊΠ°ΠΊ ΠΏΠΎΠΊΠ°Π·Π°Π½ΠΎ Π½Π° рисункС 3-47?

    Рисунок 3-47. – ΠŸΠ°Ρ€Π°Π»Π»Π΅Π»ΡŒΠ½Π°Ρ схСма с двумя Π½Π΅Ρ€Π°Π²Π½Ρ‹ΠΌΠΈ рСзисторами.

    Π”Π°Π½ΠΎ:


    РСшСниС:


    Π§Π΅Ρ‚Ρ‹Ρ€Π΅ Ρ€Π°Π²Π½Ρ‹Ρ… рСзистора ΠΏΠΎΠ΄ΠΊΠ»ΡŽΡ‡Π΅Π½Ρ‹ ΠΏΠ°Ρ€Π°Π»Π»Π΅Π»ΡŒΠ½ΠΎ, ΠΊΠ°ΠΆΠ΄Ρ‹ΠΉ рСзистор ΠΈΠΌΠ΅Π΅Ρ‚ омичСскоС сопротивлСниС 100 Ом, ΠΊΠ°ΠΊΠΎΠ²ΠΎ эквивалСнтноС сопротивлСниС?

    Π’Ρ€ΠΈ ΠΏΠ°Ρ€Π°Π»Π»Π΅Π»ΡŒΠ½ΠΎ соСдинСнных рСзистора ΠΈΠΌΠ΅ΡŽΡ‚ Π½ΠΎΠΌΠΈΠ½Π°Π»Ρ‹ 12 кОм, 20 кОм ΠΈ 30 кОм.КакоС эквивалСнтноС сопротивлСниС?

    Π”Π²Π° рСзистора, соСдинСнных ΠΏΠ°Ρ€Π°Π»Π»Π΅Π»ΡŒΠ½ΠΎ, ΠΈΠΌΠ΅ΡŽΡ‚ Π½ΠΎΠΌΠΈΠ½Π°Π»Ρ‹ 10 кОм ΠΈ 30 кОм. КакоС эквивалСнтноС сопротивлСниС?

    Π‘ΠΎΠΏΡ€ΠΎΡ‚ΠΈΠ²Π»Π΅Π½ΠΈΠ΅ ΠΏΠ°Ρ€Π°Π»Π»Π΅Π»ΡŒΠ½ΠΎΠΉ Ρ†Π΅ΠΏΠΈ

    – HVAC School

    Π’ Ρ†Π΅ΠΏΠΈ сСрии (Π½Π°Π³Ρ€ΡƒΠ·ΠΊΠΈ, ΠΏΠΎΠ΄ΠΊΠ»ΡŽΡ‡Π΅Π½Π½Ρ‹Π΅ Π² ряд, встык), Π»Π΅Π³ΠΊΠΎ Ρ€Π°ΡΡΡ‡ΠΈΡ‚Π°Ρ‚ΡŒ ΠΎΠ±Ρ‰Π΅Π΅ сопротивлСниС Ρ†Π΅ΠΏΠΈ, ΠΏΠΎΡ‚ΠΎΠΌΡƒ Ρ‡Ρ‚ΠΎ Π²Ρ‹ просто складываСтС всС сопротивлСния, Ρ‡Ρ‚ΠΎΠ±Ρ‹ ΠΏΠΎΠ»ΡƒΡ‡ΠΈΡ‚ΡŒ ΠΎΠ±Ρ‰Π΅Π΅.

    Π’ Ρ†Π΅ΠΏΠΈ , ΠΏΠ°Ρ€Π°Π»Π»Π΅Π»ΡŒΠ½ΠΎΠΉ , напряТСниС ΠΎΠ΄ΠΈΠ½Π°ΠΊΠΎΠ²ΠΎ Π½Π° всСх Π½Π°Π³Ρ€ΡƒΠ·ΠΊΠ°Ρ…; сила Ρ‚ΠΎΠΊΠ° просто складываСтся, Π½ΠΎ сопротивлСниС Π½Π΅ΠΌΠ½ΠΎΠ³ΠΎ слоТнСС.

    Π­Ρ‚ΠΎ слоТно ΠΏΡ€Π΅Π΄ΡΡ‚Π°Π²ΠΈΡ‚ΡŒ, ΠΏΠΎΡ‚ΠΎΠΌΡƒ Ρ‡Ρ‚ΠΎ ΠΎΠ±Ρ‰Π΅Π΅ сопротивлСниС Ρ†Π΅ΠΏΠΈ ΠΏΠ°Ρ€Π°Π»Π»Π΅Π»ΡŒΠ½Ρ‹Ρ… Π½Π°Π³Ρ€ΡƒΠ·ΠΎΠΊ ΡƒΠΌΠ΅Π½ΡŒΡˆΠ°Π΅Ρ‚ΡΡ с ΡƒΠ²Π΅Π»ΠΈΡ‡Π΅Π½ΠΈΠ΅ΠΌ количСства Π΄ΠΎΠ±Π°Π²Π»Π΅Π½Π½Ρ‹Ρ… Π½Π°Π³Ρ€ΡƒΠ·ΠΎΠΊ.

    НапримСр, Ссли Ρƒ вас Π΅ΡΡ‚ΡŒ ΠΎΠ΄Π½Π° Π»Π°ΠΌΠΏΠΎΡ‡ΠΊΠ°, ΠΏΠΎΠ΄ΠΊΠ»ΡŽΡ‡Π΅Π½Π½Π°Ρ ΠΊ источнику питания, ΠΏΠΎΠ»Π½ΠΎΠ΅ сопротивлСниС Ρ†Π΅ΠΏΠΈ Ρ€Π°Π²Π½ΠΎ ΡΠΎΠΏΡ€ΠΎΡ‚ΠΈΠ²Π»Π΅Π½ΠΈΡŽ Π»Π°ΠΌΠΏΡ‹.

    Π”ΠΎΠ±Π°Π²ΡŒΡ‚Π΅ Π΅Ρ‰Π΅ ΠΎΠ΄Π½Ρƒ Π»Π°ΠΌΠΏΠΎΡ‡ΠΊΡƒ Π² ΠŸΠΠ ΠΠ›Π›Π•Π›Π¬ΠΠž , ΠΈ сопротивлСниС Ρ†Π΅ΠΏΠΈ станСт Π½Π° ΠΠ˜Π–Π• .

    Когда Π²Ρ‹ вычисляСтС ΠΏΠΎΠ»Π½ΠΎΠ΅ сопротивлСниС ΠΏΠ°Ρ€Π°Π»Π»Π΅Π»ΡŒΠ½ΠΎΠΉ Ρ†Π΅ΠΏΠΈ, Π²Ρ‹ Π±Π΅Ρ€Π΅Ρ‚Π΅ ΠΊΠ°ΠΆΠ΄ΠΎΠ΅ ΠΎΡ‚Π΄Π΅Π»ΡŒΠ½ΠΎΠ΅ сопротивлСниС ΠΈ Π΄Π΅Π»ΠΈΡ‚Π΅ Π΅Π³ΠΎ Π½Π° (Π½Π΅ Π½Π°) ΠΎΠ΄Π½ΠΎ.Π—Π°Ρ‚Π΅ΠΌ Π²Ρ‹ складываСтС всС сопротивлСния, ΠΊΠΎΡ‚ΠΎΡ€Ρ‹Π΅ Π±Ρ‹Π»ΠΈ Ρ€Π°Π·Π΄Π΅Π»Π΅Π½Ρ‹ Π½Π° ΠΎΠ΄Π½ΠΎ, ΠΈ Π΄Π΅Π»ΠΈΡ‚Π΅ эту сумму Π½Π° ΠΎΠ΄Π½ΠΎ. Π€ΠΎΡ€ΠΌΡƒΠ»Π° для Π΄ΠΈΠ°Π³Ρ€Π°ΠΌΠΌΡ‹ Π²Π²Π΅Ρ€Ρ…Ρƒ ΡΡ‚Π°Ρ‚ΡŒΠΈ выглядит Ρ‚Π°ΠΊ:

    1 Γ· Rt (ΠΏΠΎΠ»Π½ΠΎΠ΅ сопротивлСниС) = 1 Γ· R1 + 1 Γ· R2 + 1 Γ· R3

    Для этого ΠΊΠΎΠ½ΠΊΡ€Π΅Ρ‚Π½ΠΎΠ³ΠΎ прилоТСния, ΠΊΠ°ΠΊ ΠΏΠΎΠΊΠ°Π·Π°Π½ΠΎ Π²Ρ‹ΡˆΠ΅, Π±ΡƒΠ΄Π΅Ρ‚:

    1 Γ· Rt (ΠΎΠ±Ρ‰Π΅Π΅ сопротивлСниС) = 1 Γ· 120 + 1 Γ· 45 + 1 Γ· 360

    Π˜Ρ‚Π°ΠΊ, 1 Γ· 120 = 0,0083 + 1 Γ· 45 = 0,022 + 1 Γ· 360 = 0,0028

    Π—Π°Ρ‚Π΅ΠΌ складываСм ΠΈΡ… всС:

    0.0083 + 0,022 + 0,0028 = 0,0331

    Π—Π°Ρ‚Π΅ΠΌ, Ρ‡Ρ‚ΠΎΠ±Ρ‹ Π½Π°ΠΉΡ‚ΠΈ сумму, Π²Ρ‹ Ρ€Π°Π·Π΄Π΅Π»ΠΈΡ‚Π΅ Π΅Π΄ΠΈΠ½ΠΈΡ†Ρƒ Π½Π° сумму:

    1 ÷ 0,0331 = 30,21 Ом всСго

    Как Π²Ρ‹ Π·Π°ΠΌΠ΅Ρ‚ΠΈΡ‚Π΅, 30,21 Ом мСньшС наимСньшСго сопротивлСния Π² Ρ†Π΅ΠΏΠΈ. Π­Ρ‚ΠΎ ΠΈΠΌΠ΅Π΅Ρ‚ смысл, Ссли ΠΏΠΎΠ΄ΡƒΠΌΠ°Ρ‚ΡŒ ΠΎ Π·Π°ΠΊΠΎΠ½Π΅ Ома.

    Π§Π΅ΠΌ Π½ΠΈΠΆΠ΅ сопротивлСниС, Ρ‚Π΅ΠΌ Π²Ρ‹ΡˆΠ΅ Ρ‚ΠΎΠΊ. Π”ΠΎΠ±Π°Π²Π»Π΅Π½ΠΈΠ΅ Π΄ΠΎΠΏΠΎΠ»Π½ΠΈΡ‚Π΅Π»ΡŒΠ½Ρ‹Ρ… ΠΏΠ°Ρ€Π°Π»Π»Π΅Π»ΡŒΠ½Ρ‹Ρ… Π½Π°Π³Ρ€ΡƒΠ·ΠΎΠΊ Π£Π’Π•Π›Π˜Π§Π˜Π’ΠΠ•Π’ силу Ρ‚ΠΎΠΊΠ° Π² Ρ†Π΅ΠΏΠΈ. ΠœΡ‹ Π²ΠΈΠ΄ΠΈΠΌ это ΠΊΠ°ΠΆΠ΄Ρ‹ΠΉ дСнь, ΠΊΠΎΠ³Π΄Π° Π·Π°ΠΌΠ΅Ρ‡Π°Π΅ΠΌ, Ρ‡Ρ‚ΠΎ усилитСли компрСссора ΠΈ усилитСли вСнтилятора кондСнсатора, слоТСнныС вмСстС, Ρ€Π°Π²Π½Ρ‹ ΠΎΠ±Ρ‰ΠΈΠΌ усилитСлям кондСнсатора.

    Π˜Ρ‚Π°ΠΊ, ΠΎΡ‡Π΅Π²ΠΈΠ΄Π½ΠΎ, Ρ‡Ρ‚ΠΎ Ссли Π±ΠΎΠ»Π΅Π΅ Π½ΠΈΠ·ΠΊΠΎΠ΅ сопротивлСниС равняСтся Π±ΠΎΠ»Π΅Π΅ высокому Ρ‚ΠΎΠΊΡƒ, ΠΈ Π΄ΠΎΠ±Π°Π²Π»Π΅Π½ΠΈΠ΅ большСго количСства ΠΏΠ°Ρ€Π°Π»Π»Π΅Π»ΡŒΠ½Ρ‹Ρ… Π½Π°Π³Ρ€ΡƒΠ·ΠΎΠΊ ΡƒΠ²Π΅Π»ΠΈΡ‡ΠΈΠ²Π°Π΅Ρ‚ Π°ΠΌΠΏΠ΅Ρ€, Ρ‚ΠΎ Π΄ΠΎΠ±Π°Π²Π»Π΅Π½ΠΈΠ΅ большСго количСства ΠΏΠ°Ρ€Π°Π»Π»Π΅Π»ΡŒΠ½Ρ‹Ρ… Π½Π°Π³Ρ€ΡƒΠ·ΠΎΠΊ сниТаСт сопротивлСниС.

    Π•Ρ‰Π΅ ΠΎΠ΄ΠΈΠ½ ΠΌΠΈΡ„, ΠΊΠΎΡ‚ΠΎΡ€Ρ‹ΠΉ этот ΠΎΠΏΡ€ΠΎΠ²Π΅Ρ€Π³Π°Π΅Ρ‚, – это идСя ΠΎ Ρ‚ΠΎΠΌ, Ρ‡Ρ‚ΠΎ элСктричСство Π’ΠžΠ›Π¬ΠšΠž ΠΈΠ΄Π΅Ρ‚ ΠΏΠΎ ΠΏΡƒΡ‚ΠΈ наимСньшСго сопротивлСния. ЭлСктричСство фактичСски ΠΏΡ€ΠΎΡ…ΠΎΠ΄ΠΈΡ‚ всС ΠΏΡƒΡ‚ΠΈ ΠΌΠ΅ΠΆΠ΄Ρƒ ΠΏΠΎΠ»ΠΎΠΆΠΈΡ‚Π΅Π»ΡŒΠ½Ρ‹ΠΌΠΈ ΠΈ ΠΎΡ‚Ρ€ΠΈΡ†Π°Ρ‚Π΅Π»ΡŒΠ½Ρ‹ΠΌΠΈ зарядами, ΠΈ ΠΊΠ°ΠΆΠ΄Ρ‹ΠΉ Π΄ΠΎΠΏΠΎΠ»Π½ΠΈΡ‚Π΅Π»ΡŒΠ½Ρ‹ΠΉ ΠΏΡƒΡ‚ΡŒ (ΠΏΠ°Ρ€Π°Π»Π»Π΅Π»ΡŒΠ½Π°Ρ Ρ†Π΅ΠΏΡŒ) просто ΡƒΠΌΠ΅Π½ΡŒΡˆΠ°Π΅Ρ‚ сопротивлСниС ΠΌΠ΅ΠΆΠ΄Ρƒ двумя Ρ‚ΠΎΡ‡ΠΊΠ°ΠΌΠΈ разности ΠΏΠΎΡ‚Π΅Π½Ρ†ΠΈΠ°Π»ΠΎΠ². Π­Ρ‚ΠΎ явлСниС ΡƒΠ²Π΅Π»ΠΈΡ‡ΠΈΠ²Π°Π΅Ρ‚ ΠΎΠ±Ρ‰ΡƒΡŽ силу Ρ‚ΠΎΠΊΠ° Π² Ρ†Π΅ΠΏΠΈ, поэтому Π²Ρ‹ΠΊΠ»ΡŽΡ‡Π°Ρ‚Π΅Π»ΡŒ срабатываСт, ΠΊΠΎΠ³Π΄Π° Π²Ρ‹ ΠΏΡ‹Ρ‚Π°Π΅Ρ‚Π΅ΡΡŒ Π²ΠΊΠ»ΡŽΡ‡ΠΈΡ‚ΡŒ Π΄Π²Π° Ρ„Π΅Π½Π° Π² ΠΎΠ΄Π½ΠΎΠΉ Ρ†Π΅ΠΏΠΈ 15 А.Π”Π²Π° Ρ„Π΅Π½Π°, ΠΏΠΎΠ΄ΠΊΠ»ΡŽΡ‡Π΅Π½Π½Ρ‹Ρ… ΠΏΠ°Ρ€Π°Π»Π»Π΅Π»ΡŒΠ½ΠΎ = мСньшСС ΠΎΠ±Ρ‰Π΅Π΅ сопротивлСниС Ρ†Π΅ΠΏΠΈ = Π±ΠΎΠ»Π΅Π΅ высокий Ρ‚ΠΎΠΊ.

    НС Ρ‚ΠΎ Ρ‡Ρ‚ΠΎΠ±Ρ‹ я пользовалась двумя фСнами… ΠΌΠΎΠΆΠ΅Ρ‚, поэтому я ΠΏΠΎΡ‡Ρ‚ΠΈ лысая.

    β€”Bryan

    БвязанныС

    Π¦Π΅ΠΏΠΈ – Ρ‚ΠΎΠΊ, Ρ€Π°Π·Π½ΠΎΡΡ‚ΡŒ ΠΏΠΎΡ‚Π΅Π½Ρ†ΠΈΠ°Π»ΠΎΠ², сопротивлСниС ΠΈ элСмСнты Π² ΠΏΠΎΡΠ»Π΅Π΄ΠΎΠ²Π°Ρ‚Π΅Π»ΡŒΠ½ΠΎΠΉ ΠΈ ΠΏΠ°Ρ€Π°Π»Π»Π΅Π»ΡŒΠ½ΠΎΠΉ цСпях, сохранСниС заряда

    Π―Ρ‡Π΅ΠΉΠΊΠΈ ΠΏΠΎΡΠ»Π΅Π΄ΠΎΠ²Π°Ρ‚Π΅Π»ΡŒΠ½ΠΎ ΠΈ ΠΏΠ°Ρ€Π°Π»Π»Π΅Π»ΡŒΠ½ΠΎ

    Π―Ρ‡Π΅ΠΉΠΊΠΈ Π² сСрии

    Когда элСмСнты соСдинСны ΠΏΠΎΡΠ»Π΅Π΄ΠΎΠ²Π°Ρ‚Π΅Π»ΡŒΠ½ΠΎ Π΄Ρ€ΡƒΠ³ с Π΄Ρ€ΡƒΠ³ΠΎΠΌ, ΠΈ всС ΠΎΠ½ΠΈ ΠΏΠΎΠ΄ΠΊΠ»ΡŽΡ‡Π΅Π½Ρ‹ Π² ΠΎΠ΄Π½ΠΎΠΌ Π½Π°ΠΏΡ€Π°Π²Π»Π΅Π½ΠΈΠΈ, общая Ρ€Π°Π·Π½ΠΎΡΡ‚ΡŒ ΠΏΠΎΡ‚Π΅Π½Ρ†ΠΈΠ°Π»ΠΎΠ², подаваСмая Π² Ρ†Π΅ΠΏΡŒ, прСдставляСт собой слоТСнныС вмСстС ΠΈΠ½Π΄ΠΈΠ²ΠΈΠ΄ΡƒΠ°Π»ΡŒΠ½Ρ‹Π΅ разности ΠΏΠΎΡ‚Π΅Π½Ρ†ΠΈΠ°Π»ΠΎΠ².

    V всСго = V 1 + V 2 + V 3

    Π˜Π΄Π΅Π½Ρ‚ΠΈΡ‡Π½Ρ‹Π΅ ячСйки ΠΏΠ°Ρ€Π°Π»Π»Π΅Π»ΡŒΠ½ΠΎ Π΄Ρ€ΡƒΠ³ Π΄Ρ€ΡƒΠ³Ρƒ

    Когда ΠΈΠ΄Π΅Π½Ρ‚ΠΈΡ‡Π½Ρ‹Π΅ элСмСнты ΠΏΠ°Ρ€Π°Π»Π»Π΅Π»ΡŒΠ½Ρ‹ Π΄Ρ€ΡƒΠ³ Π΄Ρ€ΡƒΠ³Ρƒ, полная Ρ€Π°Π·Π½ΠΎΡΡ‚ΡŒ ΠΏΠΎΡ‚Π΅Π½Ρ†ΠΈΠ°Π»ΠΎΠ², подаваСмая Π² Ρ†Π΅ΠΏΡŒ, Ρ€Π°Π²Π½Π° разности ΠΏΠΎΡ‚Π΅Π½Ρ†ΠΈΠ°Π»ΠΎΠ² Ρ‚ΠΎΠ»ΡŒΠΊΠΎ ΠΎΠ΄Π½ΠΎΠΉ ΠΈΠ· ячССк.

    V всСго = V 1 = V 2 = V 3

    Π˜Ρ‚Π°ΠΊ, Ссли Ρ‚Ρ€ΠΈ ячСйки 2 Π’ соСдинСны ΠΏΠ°Ρ€Π°Π»Π»Π΅Π»ΡŒΠ½ΠΎ Π΄Ρ€ΡƒΠ³ с Π΄Ρ€ΡƒΠ³ΠΎΠΌ, Ρ€Π°Π·Π½ΠΎΡΡ‚ΡŒ ΠΏΠΎΡ‚Π΅Π½Ρ†ΠΈΠ°Π»ΠΎΠ², подаваСмая Π² Ρ†Π΅ΠΏΡŒ, составляСт 2 Π’.

    РСзисторы ΠΏΠΎΡΠ»Π΅Π΄ΠΎΠ²Π°Ρ‚Π΅Π»ΡŒΠ½ΠΎ ΠΈ ΠΏΠ°Ρ€Π°Π»Π»Π΅Π»ΡŒΠ½ΠΎ

    РСзисторы сСрии

    Когда рСзисторы соСдинСны ΠΏΠΎΡΠ»Π΅Π΄ΠΎΠ²Π°Ρ‚Π΅Π»ΡŒΠ½ΠΎ Π΄Ρ€ΡƒΠ³ с Π΄Ρ€ΡƒΠ³ΠΎΠΌ, ΠΎΠ±Ρ‰Π΅Π΅ сопротивлСниС складываСтся ΠΈΠ· ΠΎΡ‚Π΄Π΅Π»ΡŒΠ½Ρ‹Ρ… сопротивлСний.

    РСзисторы, Π²ΠΊΠ»ΡŽΡ‡Π΅Π½Π½Ρ‹Π΅ ΠΏΠ°Ρ€Π°Π»Π»Π΅Π»ΡŒΠ½ΠΎ

    Когда рСзисторов соСдинСны ΠΏΠ°Ρ€Π°Π»Π»Π΅Π»ΡŒΠ½ΠΎ Π΄Ρ€ΡƒΠ³ с Π΄Ρ€ΡƒΠ³ΠΎΠΌ, ΠΎΠ±Ρ‰Π΅Π΅ сопротивлСниС опрСдСляСтся с ΠΏΠΎΠΌΠΎΡ‰ΡŒΡŽ ΡΠ»Π΅Π΄ΡƒΡŽΡ‰Π΅Π³ΠΎ уравнСния.

    Π’ΠΎΠΊ Π² ΠΏΠΎΡΠ»Π΅Π΄ΠΎΠ²Π°Ρ‚Π΅Π»ΡŒΠ½ΠΎΠΉ ΠΈ ΠΏΠ°Ρ€Π°Π»Π»Π΅Π»ΡŒΠ½ΠΎΠΉ цСпях

    Π‘ΠΎΡ…Ρ€Π°Π½Π΅Π½ΠΈΠ΅ заряда – Β«ΠΎΠ±Ρ‰ΠΈΠΉ заряд, ΠΏΡ€ΠΎΡ‚Π΅ΠΊΠ°ΡŽΡ‰ΠΈΠΉ Π² стык ΠΏΡ€ΠΎΠ²ΠΎΠ΄ΠΎΠ², Π΄ΠΎΠ»ΠΆΠ΅Π½ Ρ€Π°Π²Π½ΡΡ‚ΡŒΡΡ ΠΎΠ±Ρ‰Π΅ΠΌΡƒ заряду, Π²Ρ‹Ρ‚Π΅ΠΊΠ°ΡŽΡ‰Π΅ΠΌΡƒ ΠΈΠ· стыка».

    ΠŸΠ΅Ρ€Π²Ρ‹ΠΉ Π·Π°ΠΊΠΎΠ½ ΠšΠΈΡ€Ρ…Π³ΠΎΡ„Π° – «сумма Ρ‚ΠΎΠΊΠΎΠ², ΠΏΡ€ΠΎΡ‚Π΅ΠΊΠ°ΡŽΡ‰ΠΈΡ… Π² мСстС соСдинСния ΠΏΡ€ΠΎΠ²ΠΎΠ΄ΠΎΠ², Π΄ΠΎΠ»ΠΆΠ½Π° Ρ€Π°Π²Π½ΡΡ‚ΡŒΡΡ суммС Ρ‚ΠΎΠΊΠΎΠ², исходящих ΠΎΡ‚ мСста соСдинСния ΠΏΡ€ΠΎΠ²ΠΎΠ΄ΠΎΠ²Β».

    Π’ΠΎΠΊ Π² ΠΏΠΎΡΠ»Π΅Π΄ΠΎΠ²Π°Ρ‚Π΅Π»ΡŒΠ½Ρ‹Ρ… цСпях .

    Когда Π²Ρ‹ ΠΏΠΎΠ΄ΠΊΠ»ΡŽΡ‡Π°Π΅Ρ‚Π΅ Π°ΠΌΠΏΠ΅Ρ€ΠΌΠ΅Ρ‚Ρ€ ΠΊ ΠΏΠΎΡΠ»Π΅Π΄ΠΎΠ²Π°Ρ‚Π΅Π»ΡŒΠ½ΠΎΠΉ Ρ†Π΅ΠΏΠΈ, Ρ‚ΠΎΠΊ остаСтся Π½Π΅ΠΈΠ·ΠΌΠ΅Π½Π½Ρ‹ΠΌ, ΠΊΡƒΠ΄Π° Π±Ρ‹ Π²Ρ‹ Π½ΠΈ вставили Π°ΠΌΠΏΠ΅Ρ€ΠΌΠ΅Ρ‚Ρ€.

    Π’ΠΎΠΊ Π² ΠΏΠ°Ρ€Π°Π»Π»Π΅Π»ΡŒΠ½Ρ‹Ρ… цСпях .

    ΠŸΠΎΠ»Π½Ρ‹ΠΉ Ρ‚ΠΎΠΊ, ΠΏΡ€ΠΎΡ‚Π΅ΠΊΠ°ΡŽΡ‰ΠΈΠΉ ΠΎΡ‚ ячСйки ΠΊ вСтвям Π² Ρ†Π΅ΠΏΠΈ, всСгда Π΄ΠΎΠ»ΠΆΠ΅Π½ Π±Ρ‹Ρ‚ΡŒ Ρ€Π°Π²Π΅Π½ Ρ‚ΠΎΠΊΡƒ, ΠΏΡ€ΠΎΡ‚Π΅ΠΊΠ°ΡŽΡ‰Π΅ΠΌΡƒ Ρ‡Π΅Ρ€Π΅Π· ΠΊΠ°ΠΆΠ΄Ρ‹ΠΉ ΠΊΠΎΠΌΠΏΠΎΠ½Π΅Π½Ρ‚ Π² вСтвях Ρ†Π΅ΠΏΠΈ, ΠΊΠΎΠ³Π΄Π° ΠΎΠ½ΠΈ ΡΠΊΠ»Π°Π΄Ρ‹Π²Π°ΡŽΡ‚ΡΡ.

    Если ΠΊΠΎΠΌΠΏΠΎΠ½Π΅Π½Ρ‚Ρ‹ ΠΈΠΌΠ΅ΡŽΡ‚ Ρ€Π°Π·Π½ΠΎΠ΅ сопротивлСниС, Ρ‚ΠΎΠ³Π΄Π° Ρ‚ΠΎΠΊ Ρ‡Π΅Ρ€Π΅Π· ΠΊΠ°ΠΆΠ΄Ρ‹ΠΉ ΠΊΠΎΠΌΠΏΠΎΠ½Π΅Π½Ρ‚ ΠΌΠΎΠΆΠ΅Ρ‚ Π±Ρ‹Ρ‚ΡŒ Ρ€Π°Π·Π½Ρ‹ΠΌ, Π½ΠΎ ΠΊΠΎΠ³Π΄Π° Π²Ρ‹ складываСтС ΠΈΡ… вмСстС, ΠΎΠ½ΠΈ Π΄ΠΎΠ»ΠΆΠ½Ρ‹ ΡΠΎΡΡ‚Π°Π²Π»ΡΡ‚ΡŒ ΠΎΠ±Ρ‰ΡƒΡŽ сумму Ρ‚ΠΎΠΊΠ°, выходящСго ΠΈΠ· ячСйки.

    Π Π°Π·Π½ΠΈΡ†Π° ΠΏΠΎΡ‚Π΅Π½Ρ†ΠΈΠ°Π»ΠΎΠ² Π² ΠΏΠΎΡΠ»Π΅Π΄ΠΎΠ²Π°Ρ‚Π΅Π»ΡŒΠ½ΠΎΠΉ ΠΈ ΠΏΠ°Ρ€Π°Π»Π»Π΅Π»ΡŒΠ½ΠΎΠΉ цСпях

    Π’Ρ‚ΠΎΡ€ΠΎΠΉ Π·Π°ΠΊΠΎΠ½ ΠšΠΈΡ€Ρ…Π³ΠΎΡ„Π° – «сумма Π­Π”Π‘ Π² любом Π·Π°ΠΌΠΊΠ½ΡƒΡ‚ΠΎΠΌ ΠΊΠΎΠ½Ρ‚ΡƒΡ€Π΅ Π² Ρ†Π΅ΠΏΠΈ Π΄ΠΎΠ»ΠΆΠ½Π° Π±Ρ‹Ρ‚ΡŒ Ρ€Π°Π²Π½Π° суммС разностСй ΠΏΠΎΡ‚Π΅Π½Ρ†ΠΈΠ°Π»ΠΎΠ² Π² Π·Π°ΠΌΠΊΠ½ΡƒΡ‚ΠΎΠΌ ΠΊΠΎΠ½Ρ‚ΡƒΡ€Π΅ Π² Ρ†Π΅ΠΏΠΈΒ».

    Π Π°Π·Π½ΠΎΡΡ‚ΡŒ ΠΏΠΎΡ‚Π΅Π½Ρ†ΠΈΠ°Π»ΠΎΠ² Π² ΠΏΠΎΡΠ»Π΅Π΄ΠΎΠ²Π°Ρ‚Π΅Π»ΡŒΠ½ΠΎΠΉ Ρ†Π΅ΠΏΠΈ .

    Полная Ρ€Π°Π·Π½ΠΎΡΡ‚ΡŒ ΠΏΠΎΡ‚Π΅Π½Ρ†ΠΈΠ°Π»ΠΎΠ², подаваСмая ячСйкой, дСлится ΠΌΠ΅ΠΆΠ΄Ρƒ ΠΊΠΎΠΌΠΏΠΎΠ½Π΅Π½Ρ‚Π°ΠΌΠΈ. Если всС ΠΊΠΎΠΌΠΏΠΎΠ½Π΅Π½Ρ‚Ρ‹ ΠΈΠΌΠ΅ΡŽΡ‚ ΠΎΠ΄ΠΈΠ½Π°ΠΊΠΎΠ²ΠΎΠ΅ сопротивлСниС, ΠΌΠ΅ΠΆΠ΄Ρƒ Π½ΠΈΠΌΠΈ Π±ΡƒΠ΄Π΅Ρ‚ равная Ρ€Π°Π·Π½ΠΎΡΡ‚ΡŒ ΠΏΠΎΡ‚Π΅Π½Ρ†ΠΈΠ°Π»ΠΎΠ².

    Если сопротивлСния Π½Π΅ Ρ€Π°Π²Π½Ρ‹, ΠΎΠ½ΠΈ ΠΌΠΎΠ³ΡƒΡ‚ ΠΈΠΌΠ΅Ρ‚ΡŒ Ρ€Π°Π·Π½ΡƒΡŽ Π²Π΅Π»ΠΈΡ‡ΠΈΠ½Ρƒ разности ΠΏΠΎΡ‚Π΅Π½Ρ†ΠΈΠ°Π»ΠΎΠ² Π½Π° Π½ΠΈΡ…, Π½ΠΎ ΠΏΡ€ΠΈ суммировании ΠΎΠ½ΠΈ всСгда Π΄ΠΎΠ»ΠΆΠ½Ρ‹ Ρ€Π°Π²Π½ΡΡ‚ΡŒΡΡ p.d. поставляСтся ячСйкой.

    Π Π°Π·Π½ΠΎΡΡ‚ΡŒ ΠΏΠΎΡ‚Π΅Π½Ρ†ΠΈΠ°Π»ΠΎΠ² Π² ΠΏΠ°Ρ€Π°Π»Π»Π΅Π»ΡŒΠ½Ρ‹Ρ… цСпях .

    Π Π°Π·Π½ΠΎΡΡ‚ΡŒ ΠΏΠΎΡ‚Π΅Π½Ρ†ΠΈΠ°Π»ΠΎΠ², подаваСмая элСмСнтом, Ρ€Π°Π²Π½Π° разности ΠΏΠΎΡ‚Π΅Π½Ρ†ΠΈΠ°Π»ΠΎΠ² Π½Π° ΠΊΠ°ΠΆΠ΄ΠΎΠΌ ΠΊΠΎΠΌΠΏΠΎΠ½Π΅Π½Ρ‚Π΅ ΠΏΠ°Ρ€Π°Π»Π»Π΅Π»ΡŒΠ½ΠΎΠΉ Ρ†Π΅ΠΏΠΈ.

    ΠΠ»ΡŒΡ‚Π΅Ρ€Π½Π°Ρ‚ΠΈΠ²Π½Ρ‹ΠΉ тСкст Electrotech

    НиТС ΠΏΡ€ΠΈΠ²Π΅Π΄Π΅Π½Ρ‹ информация ΠΈ вопросы ΠΏΠΎ ΠΏΠ°Ρ€Π°Π»Π»Π΅Π»ΡŒΠ½Ρ‹ΠΌ цСпям.

    Π˜Π½ΡΡ‚Ρ€ΡƒΠΊΡ†ΠΈΠΈ:

    Для вопросов с нСсколькими Π²Π°Ρ€ΠΈΠ°Π½Ρ‚Π°ΠΌΠΈ ΠΎΡ‚Π²Π΅Ρ‚ΠΎΠ²:

    • Π²Ρ‹Π±Π΅Ρ€ΠΈΡ‚Π΅ ΠΏΡ€Π°Π²ΠΈΠ»ΡŒΠ½Ρ‹ΠΉ ΠΎΡ‚Π²Π΅Ρ‚
    • Π·Π°ΠΏΠΈΡˆΠΈΡ‚Π΅ свой ΠΎΡ‚Π²Π΅Ρ‚.

    По вопросам расчСта:

    • Π²Ρ‹Ρ‡ΠΈΡΠ»ΠΈΡ‚ΡŒ нСизвСстноС Π·Π½Π°Ρ‡Π΅Π½ΠΈΠ΅
    • ΡƒΠΊΠ°ΠΆΠΈΡ‚Π΅ ΡΠΎΠΎΡ‚Π²Π΅Ρ‚ΡΡ‚Π²ΡƒΡŽΡ‰ΠΈΠΉ Π±Π»ΠΎΠΊ
    • Π·Π°ΠΏΠΈΡˆΠΈΡ‚Π΅ свой ΠΎΡ‚Π²Π΅Ρ‚.

    ΠŸΡ€ΠΈΠΌΠ΅Ρ‡Π°Π½ΠΈΠ΅:

    • Π’Ρ‹ ΠΌΠΎΠΆΠ΅Ρ‚Π΅ Ρ€Π°ΡΠΏΠ΅Ρ‡Π°Ρ‚Π°Ρ‚ΡŒ эту страницу ΠΈ Π½Π°ΠΏΠΈΡΠ°Ρ‚ΡŒ Π½Π° Π½Π΅ΠΉ свои ΠΎΡ‚Π²Π΅Ρ‚Ρ‹.
    • Π’Ρ‹ ΠΌΠΎΠΆΠ΅Ρ‚Π΅ ΠΈΡΠΏΠΎΠ»ΡŒΠ·ΠΎΠ²Π°Ρ‚ΡŒ ΠΊΠ°Π»ΡŒΠΊΡƒΠ»ΡΡ‚ΠΎΡ€.

    1. КакоС напряТСниС ΠΏΡ€ΠΈΠ»ΠΎΠΆΠ΅Π½ΠΎ ΠΊ ΠΊΠ°ΠΆΠ΄ΠΎΠΌΡƒ ΠΈΠ· рСзисторов Π² ΠΏΠ°Ρ€Π°Π»Π»Π΅Π»ΡŒΠ½ΠΎΠΉ Ρ†Π΅ΠΏΠΈ?

    1. На рСзистор 1 (R1) подаСтся наибольшСС напряТСниС.
    2. НаибольшСС напряТСниС ΠΏΡ€ΠΈΠ»ΠΎΠΆΠ΅Π½ΠΎ ΠΊ рСзистору 2 (R2).
    3. На рСзистор 1 ΠΈ рСзистор 2 подаСтся ΠΎΠ΄ΠΈΠ½Π°ΠΊΠΎΠ²ΠΎΠ΅ напряТСниС.

    Π’ ΠΏΠ°Ρ€Π°Π»Π»Π΅Π»ΡŒΠ½ΠΎΠΉ схСмС ΠΊΠ°ΠΆΠ΄Ρ‹ΠΉ рСзистор ΠΏΠΎΠ΄ΠΊΠ»ΡŽΡ‡Π΅Π½ нСпосрСдствСнно ΠΊ источнику напряТСния, поэтому напряТСниС Π½Π° ΠΊΠ°ΠΆΠ΄ΠΎΠΌ рСзисторС ΠΎΠ΄ΠΈΠ½Π°ΠΊΠΎΠ²ΠΎΠ΅, ΠΊΠ°ΠΊ ΠΏΠΎΠΊΠ°Π·Π°Π½ΠΎ Π½Π° ΡΠ»Π΅Π΄ΡƒΡŽΡ‰Π΅ΠΉ Π΄ΠΈΠ°Π³Ρ€Π°ΠΌΠΌΠ΅.

    2. КакоС ΠΈΠ· ΡΠ»Π΅Π΄ΡƒΡŽΡ‰ΠΈΡ… ΡƒΡ‚Π²Π΅Ρ€ΠΆΠ΄Π΅Π½ΠΈΠΉ ΠΎΡ‚Ρ€Π°ΠΆΠ°Π΅Ρ‚ Π΄Π΅ΠΉΡΡ‚Π²ΡƒΡŽΡ‰ΠΈΠΉ Π·Π°ΠΊΠΎΠ½ ΠšΠΈΡ€Ρ…Π³ΠΎΡ„Π°?

    1. Π’ΠΎΠΊ Π½Π° Π²Ρ‹Ρ…ΠΎΠ΄Π΅ ΠΈΠ· источника питания большС, Ρ‡Π΅ΠΌ Ρ‚ΠΎΠΊ Π²ΠΎΠ·Π²Ρ€Π°Ρ‚Π°.
    2. Π’ΠΎΠΊ Π½Π° Π²Ρ‹Ρ…ΠΎΠ΄Π΅ ΠΈΠ· источника питания мСньшС ΠΎΠ±Ρ€Π°Ρ‚Π½ΠΎΠ³ΠΎ.
    3. Π’ΠΎΠΊ Π½Π° Π²Ρ‹Ρ…ΠΎΠ΄Π΅ ΠΈΠ· источника Ρ€Π°Π²Π΅Π½ ΠΎΠ±Ρ€Π°Ρ‚Π½ΠΎΠΌΡƒ Ρ‚ΠΎΠΊΡƒ.

    Π’ΠΎΠΊ, ΠΏΡ€ΠΎΡ‚Π΅ΠΊΠ°ΡŽΡ‰ΠΈΠΉ ΠΎΡ‚ источника напряТСния, всСгда Ρ€Π°Π²Π΅Π½ Π²Π΅Π»ΠΈΡ‡ΠΈΠ½Π΅ Ρ‚ΠΎΠΊΠ°, Π²ΠΎΠ·Π²Ρ€Π°Ρ‰Π°ΡŽΡ‰Π΅Π³ΠΎΡΡ ΠΊ источнику напряТСния, ΠΊΠ°ΠΊ ΠΏΠΎΠΊΠ°Π·Π°Π½ΠΎ Π½Π° ΡΠ»Π΅Π΄ΡƒΡŽΡ‰Π΅ΠΉ Π΄ΠΈΠ°Π³Ρ€Π°ΠΌΠΌΠ΅.

    3. Как Ρ‚ΠΎΠΊ, ΠΏΡ€ΠΎΡ‚Π΅ΠΊΠ°ΡŽΡ‰ΠΈΠΉ ΠΎΡ‚ источника напряТСния, Π±ΡƒΠ΄Π΅Ρ‚ ΠΏΡ€ΠΎΡ…ΠΎΠ΄ΠΈΡ‚ΡŒ Ρ‡Π΅Ρ€Π΅Π· ΠΊΠ°ΠΆΠ΄Ρ‹ΠΉ ΠΈΠ· рСзисторов?

    1. Π‘ΠΎΠ»ΡŒΡˆΠ°Ρ Ρ‡Π°ΡΡ‚ΡŒ Ρ‚ΠΎΠΊΠ° Π±ΡƒΠ΄Π΅Ρ‚ ΠΏΡ€ΠΎΡ…ΠΎΠ΄ΠΈΡ‚ΡŒ Ρ‡Π΅Ρ€Π΅Π· рСзистор 1 (R1).
    2. Наибольший Ρ‚ΠΎΠΊ Π±ΡƒΠ΄Π΅Ρ‚ ΠΏΡ€ΠΎΡ…ΠΎΠ΄ΠΈΡ‚ΡŒ Ρ‡Π΅Ρ€Π΅Π· рСзистор 2 (R2).
    3. Π’Π°ΠΊΠΎΠ΅ ΠΆΠ΅ количСство ΠΏΡ€ΠΎΡ‚Π΅ΠΊΠ°Π΅Ρ‚ Ρ‡Π΅Ρ€Π΅Π· рСзистор 1 ΠΈ рСзистор 2.

    Π’ΠΎΠΊ ΠΎΡ‚ источника напряТСния раздСляСтся ΠΈ ΠΏΡ€ΠΎΡ…ΠΎΠ΄ΠΈΡ‚ Ρ‡Π΅Ρ€Π΅Π· ΠΊΠ°ΠΆΠ΄Ρ‹ΠΉ ΠΈΠ· рСзисторов.

    Наибольший Ρ‚ΠΎΠΊ Π±ΡƒΠ΄Π΅Ρ‚ ΠΏΡ€ΠΎΡ…ΠΎΠ΄ΠΈΡ‚ΡŒ Ρ‡Π΅Ρ€Π΅Π· R1. Π­Ρ‚ΠΎ связано с Ρ‚Π΅ΠΌ, Ρ‡Ρ‚ΠΎ R1 ΠΈΠΌΠ΅Π΅Ρ‚ наимСньшСС Π·Π½Π°Ρ‡Π΅Π½ΠΈΠ΅ сопротивлСния ΠΈ, ΡΠ»Π΅Π΄ΠΎΠ²Π°Ρ‚Π΅Π»ΡŒΠ½ΠΎ, наимСньшСС сопротивлСниС ΠΏΡ€ΠΎΡ‚Π΅ΠΊΠ°Π½ΠΈΡŽ Ρ‚ΠΎΠΊΠ°, ΠΊΠ°ΠΊ ΠΏΠΎΠΊΠ°Π·Π°Π½ΠΎ Π½Π° ΡΠ»Π΅Π΄ΡƒΡŽΡ‰Π΅ΠΉ Π΄ΠΈΠ°Π³Ρ€Π°ΠΌΠΌΠ΅.

    4.РассчитайтС Ρ‚ΠΎΠΊ, ΠΏΡ€ΠΎΡ‚Π΅ΠΊΠ°ΡŽΡ‰ΠΈΠΉ Ρ‡Π΅Ρ€Π΅Π· R3 Π² этой Ρ†Π΅ΠΏΠΈ.

    ΠžΠ±Ρ‰ΠΈΠΉ Ρ‚ΠΎΠΊ, ΠΏΡ€ΠΎΡ‚Π΅ΠΊΠ°ΡŽΡ‰ΠΈΠΉ Π² ΠΏΠ°Ρ€Π°Π»Π»Π΅Π»ΡŒΠ½ΠΎΠΉ Ρ†Π΅ΠΏΠΈ, Ρ€Π°Π²Π΅Π½ суммС Ρ‚ΠΎΠΊΠΎΠ², ΠΏΡ€ΠΎΡ‚Π΅ΠΊΠ°ΡŽΡ‰ΠΈΡ… Ρ‡Π΅Ρ€Π΅Π· ΠΊΠ°ΠΆΠ΄Ρ‹ΠΉ рСзистор. Π­Ρ‚ΠΎ ΠΌΠΎΠΆΠ½ΠΎ ΠΏΡ€Π΅Π΄ΡΡ‚Π°Π²ΠΈΡ‚ΡŒ Ρ„ΠΎΡ€ΠΌΡƒΠ»ΠΎΠΉ:

    5. Π§Ρ‚ΠΎ ΠΏΡ€ΠΎΠΈΠ·ΠΎΠΉΠ΄Π΅Ρ‚ с ΠΎΠ±Ρ‰ΠΈΠΌ сопротивлСниСм ΠΏΠ°Ρ€Π°Π»Π»Π΅Π»ΡŒΠ½ΠΎΠΉ Ρ†Π΅ΠΏΠΈ, Ссли ΠΏΠ°Ρ€Π°Π»Π»Π΅Π»ΡŒΠ½ΠΎ Π΄ΠΎΠ±Π°Π²ΠΈΡ‚ΡŒ Π΄ΠΎΠΏΠΎΠ»Π½ΠΈΡ‚Π΅Π»ΡŒΠ½Ρ‹ΠΉ рСзистор?

    1. ΠžΠ±Ρ‰Π΅Π΅ сопротивлСниС увСличиваСтся.
    2. ΠžΠ±Ρ‰Π΅Π΅ сопротивлСниС ΡƒΠΌΠ΅Π½ΡŒΡˆΠ°Π΅Ρ‚ΡΡ.
    3. ΠžΠ±Ρ‰Π΅Π΅ сопротивлСниС ΠΎΡΡ‚Π°Π»ΠΎΡΡŒ ΠΏΡ€Π΅ΠΆΠ½ΠΈΠΌ.

    Добавляя Π΅Ρ‰Π΅ ΠΎΠ΄ΠΈΠ½ рСзистор ΠΏΠ°Ρ€Π°Π»Π»Π΅Π»ΡŒΠ½ΠΎ, ΠΌΡ‹ обСспСчиваСм Π΄ΠΎΠΏΠΎΠ»Π½ΠΈΡ‚Π΅Π»ΡŒΠ½Ρ‹ΠΉ ΠΏΡƒΡ‚ΡŒ для прохоТдСния Ρ‚ΠΎΠΊΠ°. Π‘Π»Π΅Π΄ΠΎΠ²Π°Ρ‚Π΅Π»ΡŒΠ½ΠΎ, Ρ‚ΠΎΠΊ, ΠΏΡ€ΠΎΡ‚Π΅ΠΊΠ°ΡŽΡ‰ΠΈΠΉ ΠΎΡ‚ источника напряТСния, увСличится. Π£Π²Π΅Π»ΠΈΡ‡Π΅Π½ΠΈΠ΅ Ρ‚ΠΎΠΊΠ° ΠΎΡ‚ источника ΠΌΠΎΠΆΠ΅Ρ‚ ΠΏΡ€ΠΎΠΈΠ·ΠΎΠΉΡ‚ΠΈ Ρ‚ΠΎΠ»ΡŒΠΊΠΎ ΠΏΠΎΡ‚ΠΎΠΌΡƒ, Ρ‡Ρ‚ΠΎ ΠΎΠ±Ρ‰Π΅Π΅ сопротивлСниС ΡƒΠΌΠ΅Π½ΡŒΡˆΠΈΠ»ΠΎΡΡŒ, ΠΊΠ°ΠΊ ΠΏΠΎΠΊΠ°Π·Π°Π½ΠΎ Π½Π° ΡΠ»Π΅Π΄ΡƒΡŽΡ‰Π΅ΠΉ Π΄ΠΈΠ°Π³Ρ€Π°ΠΌΠΌΠ΅.

    6. По ΠΊΠ°ΠΊΠΎΠΉ Ρ„ΠΎΡ€ΠΌΡƒΠ»Π΅ опрСдСляСтся ΠΏΠΎΠ»Π½ΠΎΠ΅ сопротивлСниС ΠΏΠ°Ρ€Π°Π»Π»Π΅Π»ΡŒΠ½ΠΎΠΉ Ρ†Π΅ΠΏΠΈ?

    1. Π Π’ = R1 + R2 + R3
    2. 1 / RT = 1 / R1 + 1 / R2 + 1 / R3
    3. RT = R1 x R2 x R3
    4. 1 / RT = 1 / R1x 1 / R2 x 1 / R3

    ΠŸΡ€ΠΈΠ²Π΅Π΄Π΅Π½Π½Π°Ρ Π½ΠΈΠΆΠ΅ Ρ„ΠΎΡ€ΠΌΡƒΠ»Π° являСтся ΠΎΠ΄Π½ΠΎΠΉ ΠΈΠ· Π½Π΅ΡΠΊΠΎΠ»ΡŒΠΊΠΈΡ… Ρ„ΠΎΡ€ΠΌΡƒΠ», ΠΊΠΎΡ‚ΠΎΡ€Ρ‹Π΅ ΠΌΠΎΠΆΠ½ΠΎ ΠΈΡΠΏΠΎΠ»ΡŒΠ·ΠΎΠ²Π°Ρ‚ΡŒ для расчСта ΠΎΠ±Ρ‰Π΅Π³ΠΎ сопротивлСния ΠΏΠ°Ρ€Π°Π»Π»Π΅Π»ΡŒΠ½ΠΎΠΉ Ρ†Π΅ΠΏΠΈ.Π­Ρ‚Ρƒ Ρ„ΠΎΡ€ΠΌΡƒΠ»Ρƒ ΠΌΠΎΠΆΠ½ΠΎ ΠΈΡΠΏΠΎΠ»ΡŒΠ·ΠΎΠ²Π°Ρ‚ΡŒ для любого количСства рСзисторов, ΠΈ ΠΎΠ½Π° Π΄Π΅ΠΉΡΡ‚Π²ΠΈΡ‚Π΅Π»ΡŒΠ½Π° нСзависимо ΠΎΡ‚ Ρ‚ΠΎΠ³ΠΎ, ΠΎΠ΄ΠΈΠ½Π°ΠΊΠΎΠ²Ρ‹ ΠΈΠ»ΠΈ Ρ€Π°Π·Π»ΠΈΡ‡Π°ΡŽΡ‚ΡΡ ΠΊΠ°ΠΆΠ΄ΠΎΠ΅ ΠΎΡ‚Π΄Π΅Π»ΡŒΠ½ΠΎΠ΅ Π·Π½Π°Ρ‡Π΅Π½ΠΈΠ΅ сопротивлСния.

    7. РассчитайтС ΠΏΠΎΠ»Π½ΠΎΠ΅ сопротивлСниС этой Ρ†Π΅ΠΏΠΈ

    На ΡΠ»Π΅Π΄ΡƒΡŽΡ‰Π΅ΠΉ схСмС ΠΏΠΎΠΊΠ°Π·Π°Π½ ΠΏΡ€ΠΈΠΌΠ΅Ρ€ расчСта ΠΏΠΎΠ»Π½ΠΎΠ³ΠΎ сопротивлСния Π² Ρ†Π΅ΠΏΠΈ с двумя рСзисторами, ΠΏΠΎΠ΄ΠΊΠ»ΡŽΡ‡Π΅Π½Π½Ρ‹ΠΌΠΈ ΠΏΠ°Ρ€Π°Π»Π»Π΅Π»ΡŒΠ½ΠΎ.

    Π‘Π²ΠΎΠ΄ΠΊΠ° ΠΏΠ°Ρ€Π°Π»Π»Π΅Π»ΡŒΠ½Ρ‹Ρ… Ρ†Π΅ΠΏΠ΅ΠΉ:

    • напряТСниС Π½Π° ΠΊΠ°ΠΆΠ΄ΠΎΠΌ рСзисторС ΠΎΠ΄ΠΈΠ½Π°ΠΊΠΎΠ²ΠΎΠ΅.
    • Π’ΠΎΠΊ, ΠΏΡ€ΠΎΡ‚Π΅ΠΊΠ°ΡŽΡ‰ΠΈΠΉ ΠΎΡ‚ источника напряТСния, Ρ€Π°Π²Π΅Π½ Ρ‚ΠΎΠΊΡƒ, Π²ΠΎΠ·Π²Ρ€Π°Ρ‰Π°ΡŽΡ‰Π΅ΠΌΡƒΡΡ ΠΊ источнику напряТСния.
    • Π²Π΅Π»ΠΈΡ‡ΠΈΠ½Π° Ρ‚ΠΎΠΊΠ°, ΠΏΡ€ΠΎΡ‚Π΅ΠΊΠ°ΡŽΡ‰Π΅Π³ΠΎ Ρ‡Π΅Ρ€Π΅Π· ΠΊΠ°ΠΆΠ΄Ρ‹ΠΉ рСзистор, зависит ΠΎΡ‚ Π½ΠΎΠΌΠΈΠ½Π°Π»Π° рСзистора.
    • сумма Ρ‚ΠΎΠΊΠ°, ΠΏΡ€ΠΎΡ‚Π΅ΠΊΠ°ΡŽΡ‰Π΅Π³ΠΎ Ρ‡Π΅Ρ€Π΅Π· ΠΊΠ°ΠΆΠ΄Ρ‹ΠΉ рСзистор, Ρ€Π°Π²Π½Π° Ρ‚ΠΎΠΊΡƒ питания.
    • Π΄ΠΎΠ±Π°Π²Π»Π΅Π½ΠΈΠ΅ рСзистора Π² ΠΏΠ°Ρ€Π°Π»Π»Π΅Π»ΡŒΠ½ΡƒΡŽ Π²Π΅Ρ‚Π²ΡŒ сниТаСт ΠΎΠ±Ρ‰Π΅Π΅ сопротивлСниС.
    • ΠΎΠ±Ρ‰Π΅Π΅ сопротивлСниС ΠΏΠ°Ρ€Π°Π»Π»Π΅Π»ΡŒΠ½ΠΎΠΉ Ρ†Π΅ΠΏΠΈ ΠΌΠΎΠΆΠ½ΠΎ Ρ€Π°ΡΡΡ‡ΠΈΡ‚Π°Ρ‚ΡŒ ΠΏΠΎ ΡΠ»Π΅Π΄ΡƒΡŽΡ‰Π΅ΠΉ Ρ„ΠΎΡ€ΠΌΡƒΠ»Π΅:

    Π—Π½Π°Π½ΠΈΠ΅ ΠΏΠ°Ρ€Π°Π»Π»Π΅Π»ΡŒΠ½Ρ‹Ρ… Ρ†Π΅ΠΏΠ΅ΠΉ ΠΈ Π·Π°ΠΊΠΎΠ½Π° Ома ΠΏΠΎΠ·Π²ΠΎΠ»ΠΈΡ‚ Π²Π°ΠΌ ΠΎΡ‚Π²Π΅Ρ‚ΠΈΡ‚ΡŒ Π½Π° ΡΠ»Π΅Π΄ΡƒΡŽΡ‰ΠΈΠ΅ вопросы.

    8. РассчитайтС ΠΏΠΎΠ»Π½ΠΎΠ΅ сопротивлСниС этой Ρ†Π΅ΠΏΠΈ

    9. РассчитайтС напряТСниС Π½Π° R3 Π² этой Ρ†Π΅ΠΏΠΈ.

    10. РассчитайтС Ρ‚ΠΎΠΊ, ΠΏΡ€ΠΎΡ‚Π΅ΠΊΠ°ΡŽΡ‰ΠΈΠΉ Ρ‡Π΅Ρ€Π΅Π· R1 Π² этой Ρ†Π΅ΠΏΠΈ.

    11. РассчитайтС Ρ‚ΠΎΠΊ, ΠΏΡ€ΠΎΡ‚Π΅ΠΊΠ°ΡŽΡ‰ΠΈΠΉ Ρ‡Π΅Ρ€Π΅Π· R2 Π² этой Ρ†Π΅ΠΏΠΈ.

    12.РассчитайтС Ρ‚ΠΎΠΊ, ΠΏΡ€ΠΎΡ‚Π΅ΠΊΠ°ΡŽΡ‰ΠΈΠΉ Ρ‡Π΅Ρ€Π΅Π· R3 Π² этой Ρ†Π΅ΠΏΠΈ.

    13. РассчитайтС ΠΎΠ±Ρ‰ΠΈΠΉ Ρ‚ΠΎΠΊ, ΠΏΡ€ΠΎΡ‚Π΅ΠΊΠ°ΡŽΡ‰ΠΈΠΉ Π² этой Ρ†Π΅ΠΏΠΈ.

    14. ВычислитС Π·Π½Π°Ρ‡Π΅Π½ΠΈΠ΅ сопротивлСния R2 Π² этой Ρ†Π΅ΠΏΠΈ.

    Если Ρƒ вас Π²ΠΎΠ·Π½ΠΈΠΊΠ»ΠΈ ΠΏΡ€ΠΎΠ±Π»Π΅ΠΌΡ‹ с ΠΎΡ‚Π²Π΅Ρ‚ΠΎΠΌ Π½Π° вопросы ΠΎ ΠΏΠ°Ρ€Π°Π»Π»Π΅Π»ΡŒΠ½Ρ‹Ρ… цСпях, Π΄ΠΎΠΏΠΎΠ»Π½ΠΈΡ‚Π΅Π»ΡŒΠ½ΡƒΡŽ ΠΈΠ½Ρ„ΠΎΡ€ΠΌΠ°Ρ†ΠΈΡŽ ΠΌΠΎΠΆΠ½ΠΎ ΠΏΠΎΠ»ΡƒΡ‡ΠΈΡ‚ΡŒ ΠΏΠΎ Ρ‚Π΅Π»Π΅Ρ„ΠΎΠ½Ρƒ:

    ЭлСктричСскиС ΠΏΡ€ΠΈΠ½Ρ†ΠΈΠΏΡ‹ Ρ€Π°Π·Π΄Π΅Π» Π² РСсурсы

    • ΡƒΡ‡Π΅Π±Π½ΠΈΠΊΠΈ
    • ΠΈΠ½Ρ‚Π΅Ρ€Π½Π΅Ρ‚-сайтов
    • ваш Ρ‚Ρ€Π΅Π½Π΅Ρ€.

    ΠŸΠΎΠ²Ρ‚ΠΎΡ€ΠΈΡ‚Π΅ это дСйствиС Β«Π£ΠΊΡ€Π΅ΠΏΠ»Π΅Π½ΠΈΠ΅Β» ΠΈΠ»ΠΈ ΠΏΠ΅Ρ€Π΅ΠΉΠ΄ΠΈΡ‚Π΅ ΠΊ Π΄Π΅ΠΉΡΡ‚Π²ΠΈΡŽ с условиями Ρ†Π΅ΠΏΠΈ.

    Когда Π²Ρ‹ Π·Π°ΠΊΠΎΠ½Ρ‡ΠΈΡ‚Π΅:

    • ΠΏΠΎΠΊΠ°ΠΆΠΈ свои ΠΎΡ‚Π²Π΅Ρ‚Ρ‹ Ρ‚Ρ€Π΅Π½Π΅Ρ€Ρƒ
    • Π·Π°ΠΊΡ€ΠΎΠΉΡ‚Π΅ это ΠΎΠΊΠ½ΠΎ ΠΈ Π²Ρ‹Π±Π΅Ρ€ΠΈΡ‚Π΅ Π½ΠΎΠ²ΠΎΠ΅ дСйствиС Π² мСню.

    Как Ρ€Π°ΡΡΡ‡ΠΈΡ‚Π°Ρ‚ΡŒ ΠΏΠΎΠ»Π½ΠΎΠ΅ сопротивлСниС ΠΏΠΎΡΠ»Π΅Π΄ΠΎΠ²Π°Ρ‚Π΅Π»ΡŒΠ½ΠΎΠΉ Ρ†Π΅ΠΏΠΈ?

    ΠΎΡ‚ Veerendra

    Как Ρ€Π°ΡΡΡ‡ΠΈΡ‚Π°Ρ‚ΡŒ ΠΏΠΎΠ»Π½ΠΎΠ΅ сопротивлСниС ΠΏΠΎΡΠ»Π΅Π΄ΠΎΠ²Π°Ρ‚Π΅Π»ΡŒΠ½ΠΎΠΉ Ρ†Π΅ΠΏΠΈ?

    Π­Ρ„Ρ„Π΅ΠΊΡ‚ΠΈΠ²Π½ΠΎΠ΅ сопротивлСниС рСзисторов, ΠΏΠΎΠ΄ΠΊΠ»ΡŽΡ‡Π΅Π½Π½Ρ‹Ρ… ΠΏΠΎΡΠ»Π΅Π΄ΠΎΠ²Π°Ρ‚Π΅Π»ΡŒΠ½ΠΎ

    1. Π’ Ρ†Π΅ΠΏΠΈ сСрии Π΅ΡΡ‚ΡŒ Ρ‚Ρ€ΠΈ Π²Π°ΠΆΠ½Ρ‹Ρ… характСристики:
      (a) Π’ΠΎΠΊ, проходящий Ρ‡Π΅Ρ€Π΅Π· ΠΊΠ°ΠΆΠ΄Ρ‹ΠΉ рСзистор, ΠΎΠ΄ΠΈΠ½Π°ΠΊΠΎΠ².
      (b) Π Π°Π·Π½ΠΎΡΡ‚ΡŒ ΠΏΠΎΡ‚Π΅Π½Ρ†ΠΈΠ°Π»ΠΎΠ² Π½Π° ΠΊΠ°ΠΆΠ΄ΠΎΠΌ рСзисторС Π½Π°ΠΏΡ€ΡΠΌΡƒΡŽ зависит ΠΎΡ‚ Π΅Π³ΠΎ сопротивлСния.
      (c) Π‘ΡƒΠΌΠΌΠ° разности ΠΏΠΎΡ‚Π΅Π½Ρ†ΠΈΠ°Π»ΠΎΠ² Π½Π° ΠΊΠ°ΠΆΠ΄ΠΎΠΌ рСзисторС Ρ€Π°Π²Π½Π° ΠΎΠ±Ρ‰Π΅ΠΉ разности ΠΏΠΎΡ‚Π΅Π½Ρ†ΠΈΠ°Π»ΠΎΠ² источника.
    2. Когда Π΄Π²Π° ΠΈΠ»ΠΈ Π±ΠΎΠ»Π΅Π΅ рСзистора соСдинСны встык, Ρ‚Π°ΠΊ Ρ‡Ρ‚ΠΎ Ρ‡Π΅Ρ€Π΅Π· ΠΊΠ°ΠΆΠ΄ΠΎΠ΅ ΠΈΠ· Π½ΠΈΡ… ΠΏΡ€ΠΎΡ‚Π΅ΠΊΠ°Π΅Ρ‚ ΠΎΠ΄ΠΈΠ½ ΠΈ Ρ‚ΠΎΡ‚ ΠΆΠ΅ Ρ‚ΠΎΠΊ, говорят, Ρ‡Ρ‚ΠΎ ΠΎΠ½ΠΈ соСдинСны ΠΏΠΎΡΠ»Π΅Π΄ΠΎΠ²Π°Ρ‚Π΅Π»ΡŒΠ½ΠΎ. Когда ΠΊ Π±Π°Ρ‚Π°Ρ€Π΅Π΅ ΠΏΠΎΠ΄ΠΊΠ»ΡŽΡ‡Π΅Π½Π° ΠΏΠΎΡΠ»Π΅Π΄ΠΎΠ²Π°Ρ‚Π΅Π»ΡŒΠ½Π°Ρ комбинация сопротивлСний, Ρ‡Π΅Ρ€Π΅Π· ΠΊΠ°ΠΆΠ΄ΡƒΡŽ ΠΈΠ· Π½ΠΈΡ… ΠΏΡ€ΠΎΡ‚Π΅ΠΊΠ°Π΅Ρ‚ ΠΎΠ΄ΠΈΠ½ ΠΈ Ρ‚ΠΎΡ‚ ΠΆΠ΅ Ρ‚ΠΎΠΊ (I).
      Когда ΠΊ Π±Π°Ρ‚Π°Ρ€Π΅Π΅ ΠΏΠΎΠ΄ΠΊΠ»ΡŽΡ‡Π΅Π½Π° ΠΏΠΎΡΠ»Π΅Π΄ΠΎΠ²Π°Ρ‚Π΅Π»ΡŒΠ½Π°Ρ комбинация сопротивлСний, Ρ‡Π΅Ρ€Π΅Π· ΠΊΠ°ΠΆΠ΄ΡƒΡŽ ΠΈΠ· Π½ΠΈΡ… ΠΏΡ€ΠΎΡ‚Π΅ΠΊΠ°Π΅Ρ‚ ΠΎΠ΄ΠΈΠ½ ΠΈ Ρ‚ΠΎΡ‚ ΠΆΠ΅ Ρ‚ΠΎΠΊ (I).
    3. Π—Π°ΠΊΠΎΠ½ ΠΊΠΎΠΌΠ±ΠΈΠ½Π°Ρ†ΠΈΠΈ сопротивлСний Π² сСрии: Π—Π°ΠΊΠΎΠ½ ΠΊΠΎΠΌΠ±ΠΈΠ½Π°Ρ†ΠΈΠΈ сопротивлСний Π² сСрии гласит, Ρ‡Ρ‚ΠΎ ΠΊΠΎΠ³Π΄Π° нСсколько сопротивлСний соСдинСны ΠΏΠΎΡΠ»Π΅Π΄ΠΎΠ²Π°Ρ‚Π΅Π»ΡŒΠ½ΠΎ, ΠΈΡ… эквивалСнтноС сопротивлСниС Ρ€Π°Π²Π½ΠΎ суммС ΠΎΡ‚Π΄Π΅Π»ΡŒΠ½Ρ‹Ρ… сопротивлСний. Π’Π°ΠΊΠΈΠΌ ΠΎΠ±Ρ€Π°Π·ΠΎΠΌ, Ссли R 1 , R 2 , R 3 … ΠΈ Ρ‚. Π”. ΠžΠ±ΡŠΠ΅Π΄ΠΈΠ½Π΅Π½Ρ‹ ΠΏΠΎΡΠ»Π΅Π΄ΠΎΠ²Π°Ρ‚Π΅Π»ΡŒΠ½ΠΎ, Ρ‚ΠΎ эквивалСнтноС сопротивлСниС (R) Π±ΡƒΠ΄Π΅Ρ‚ Ρ€Π°Π²Π½ΠΎ:
      R = R 1 + R 2 + R 3 +…….(i)
    4. Π’Ρ‹Π²Π΅Π΄Π΅Π½ΠΈΠ΅ матСматичСского выраТСния сопротивлСний Π² ΠΏΠΎΡΠ»Π΅Π΄ΠΎΠ²Π°Ρ‚Π΅Π»ΡŒΠ½ΠΎΠΉ ΠΊΠΎΠΌΠ±ΠΈΠ½Π°Ρ†ΠΈΠΈ :
      ΠŸΡƒΡΡ‚ΡŒ, R 1 , R 2 ΠΈ R 3 Π±ΡƒΠ΄ΡƒΡ‚ сопротивлСниями, Π²ΠΊΠ»ΡŽΡ‡Π΅Π½Π½Ρ‹ΠΌΠΈ ΠΏΠΎΡΠ»Π΅Π΄ΠΎΠ²Π°Ρ‚Π΅Π»ΡŒΠ½ΠΎ, I Π±ΡƒΠ΄Π΅Ρ‚ Ρ‚ΠΎΠΊΠΎΠΌ, ΠΏΡ€ΠΎΡ‚Π΅ΠΊΠ°ΡŽΡ‰ΠΈΠΌ ΠΏΠΎ Ρ†Π΅ΠΏΠΈ, Ρ‚. Π•. ΠŸΡ€ΠΎΡ…ΠΎΠ΄ΡΡ‰ΠΈΠΌ Ρ‡Π΅Ρ€Π΅Π· ΠΊΠ°ΠΆΠ΄ΠΎΠ΅ сопротивлСниС, Π° V 1 , V 2 ΠΈ V 3 – Ρ€Π°Π·Π½ΠΎΡΡ‚ΡŒ ΠΏΠΎΡ‚Π΅Π½Ρ†ΠΈΠ°Π»ΠΎΠ² Π½Π° R 1 , R 2 ΠΈ R 3 соотвСтствСнно. Π’ΠΎΠ³Π΄Π°, согласно Π·Π°ΠΊΠΎΠ½Ρƒ Ома,
      V 1 = IR 1 , V 2 = IR 2 ΠΈ V 3 = IR 3 … (ii)
    5. Если, V – Ρ€Π°Π·Π½ΠΎΡΡ‚ΡŒ ΠΏΠΎΡ‚Π΅Π½Ρ†ΠΈΠ°Π»ΠΎΠ² Π½Π° ΠΊΠΎΠΌΠ±ΠΈΠ½Π°Ρ†ΠΈΠΈ сопротивлСний, Ρ‚ΠΎΠ³Π΄Π°
      V = V 1 + V 2 + V 3 … (iii)
    6. Если R – эквивалСнтноС сопротивлСниС Ρ†Π΅ΠΏΠΈ, Ρ‚ΠΎ
      Π’ = IR… (iv)
    7. Π˜ΡΠΏΠΎΠ»ΡŒΠ·ΡƒΡ уравнСния (i) – (iv), ΠΌΡ‹ ΠΌΠΎΠΆΠ΅ΠΌ Π½Π°ΠΏΠΈΡΠ°Ρ‚ΡŒ:
      IR = V = V 1 + V 2 + V 3
      IR = IR 1 + IR 2 + IR 3
      IR = I (R 1 + R 2 + R 3 )
      R = R 1 + R 2 + R 3
      Π‘Π»Π΅Π΄ΠΎΠ²Π°Ρ‚Π΅Π»ΡŒΠ½ΠΎ, ΠΊΠΎΠ³Π΄Π° сопротивлСния ΠΎΠ±ΡŠΠ΅Π΄ΠΈΠ½Π΅Π½Ρ‹ ΠΏΠΎΡΠ»Π΅Π΄ΠΎΠ²Π°Ρ‚Π΅Π»ΡŒΠ½ΠΎ , эквивалСнтноС сопротивлСниС Π²Ρ‹ΡˆΠ΅, Ρ‡Π΅ΠΌ ΠΊΠ°ΠΆΠ΄ΠΎΠ΅ ΠΎΡ‚Π΄Π΅Π»ΡŒΠ½ΠΎΠ΅ сопротивлСниС.
    8. ЭквивалСнтная схСма ΠΏΠΎΠΊΠ°Π·Π°Π½Π° Π½Π° рисункС.

    НСкоторыС Ρ€Π΅Π·ΡƒΠ»ΡŒΡ‚Π°Ρ‚Ρ‹ ΠΎ ΠΊΠΎΠΌΠ±ΠΈΠ½Π°Ρ†ΠΈΠΈ сСрий:

    1. Когда Π΄Π²Π° ΠΈΠ»ΠΈ Π±ΠΎΠ»Π΅Π΅ рСзистора соСдинСны ΠΏΠΎΡΠ»Π΅Π΄ΠΎΠ²Π°Ρ‚Π΅Π»ΡŒΠ½ΠΎ, ΠΎΠ±Ρ‰Π΅Π΅ сопротивлСниС ΠΊΠΎΠΌΠ±ΠΈΠ½Π°Ρ†ΠΈΠΈ Ρ€Π°Π²Π½ΠΎ суммС всСх ΠΎΡ‚Π΄Π΅Π»ΡŒΠ½Ρ‹Ρ… сопротивлСний.
    2. Когда Π΄Π²Π° ΠΈΠ»ΠΈ Π±ΠΎΠ»Π΅Π΅ рСзистора соСдинСны ΠΏΠΎΡΠ»Π΅Π΄ΠΎΠ²Π°Ρ‚Π΅Π»ΡŒΠ½ΠΎ, ΠΎΠ΄ΠΈΠ½Π°ΠΊΠΎΠ²Ρ‹ΠΉ Ρ‚ΠΎΠΊ Ρ‚Π΅Ρ‡Π΅Ρ‚ Ρ‡Π΅Ρ€Π΅Π· ΠΊΠ°ΠΆΠ΄Ρ‹ΠΉ рСзистор.
    3. Когда нСсколько рСзисторов ΠΏΠΎΠ΄ΠΊΠ»ΡŽΡ‡Π΅Π½Ρ‹ ΠΏΠΎΡΠ»Π΅Π΄ΠΎΠ²Π°Ρ‚Π΅Π»ΡŒΠ½ΠΎ, напряТСниС Π½Π° ΠΊΠΎΠΌΠ±ΠΈΠ½Π°Ρ†ΠΈΠΈ
      (Ρ‚.Π΅.Π΅. напряТСниС Π±Π°Ρ‚Π°Ρ€Π΅ΠΈ Π² Ρ†Π΅ΠΏΠΈ), Ρ€Π°Π²Π½ΠΎ суммС ΠΏΠ°Π΄Π΅Π½ΠΈΠΉ напряТСния
      (ΠΈΠ»ΠΈ разности ΠΏΠΎΡ‚Π΅Π½Ρ†ΠΈΠ°Π»ΠΎΠ²) Π½Π° ΠΊΠ°ΠΆΠ΄ΠΎΠΌ ΠΎΡ‚Π΄Π΅Π»ΡŒΠ½ΠΎΠΌ рСзисторС.

    Π›ΡŽΠ΄ΠΈ Ρ‚Π°ΠΊΠΆΠ΅ ΡΠΏΡ€Π°ΡˆΠΈΠ²Π°ΡŽΡ‚

    ΠŸΡ€ΠΎΠ±Π»Π΅ΠΌΡ‹ Ρ†Π΅ΠΏΠ΅ΠΉ сСрии

    с Ρ€Π΅ΡˆΠ΅Π½ΠΈΡΠΌΠΈ
    1. Π’Ρ€ΠΈ рСзистора, R 1 , R 2 ΠΈ R 3 , ΠΏΠΎΡΠ»Π΅Π΄ΠΎΠ²Π°Ρ‚Π΅Π»ΡŒΠ½ΠΎ ΠΏΠΎΠ΄ΠΊΠ»ΡŽΡ‡Π΅Π½Ρ‹ ΠΊ Π±Π°Ρ‚Π°Ρ€Π΅Π΅ 6 Π’, ΠΊΠ°ΠΊ ΠΏΠΎΠΊΠ°Π·Π°Π½ΠΎ Π½Π° рисункС.

      РассчитайтС
      (a) эффСктивноС сопротивлСниС R Ρ†Π΅ΠΏΠΈ,
      (b) Ρ‚ΠΎΠΊ I Π² Ρ†Π΅ΠΏΠΈ,
      (c) Ρ€Π°Π·Π½ΠΎΡΡ‚ΡŒ ΠΏΠΎΡ‚Π΅Π½Ρ†ΠΈΠ°Π»ΠΎΠ² Π½Π° ΠΊΠ°ΠΆΠ΄ΠΎΠΌ рСзисторС, V 1 , V 2 ΠΈ V 3 .
      РСшСниС:

      ΠžΠ±Ρ€Π°Ρ‚ΠΈΡ‚Π΅ Π²Π½ΠΈΠΌΠ°Π½ΠΈΠ΅, Ρ‡Ρ‚ΠΎ Ρ‡Π΅ΠΌ большС сопротивлСниС, Ρ‚Π΅ΠΌ большС Ρ€Π°Π·Π½ΠΎΡΡ‚ΡŒ ΠΏΠΎΡ‚Π΅Π½Ρ†ΠΈΠ°Π»ΠΎΠ² Π½Π° Π½Π΅ΠΌ. Π‘ΡƒΠΌΠΌΠ° разности ΠΏΠΎΡ‚Π΅Π½Ρ†ΠΈΠ°Π»ΠΎΠ² Π½Π° ΠΊΠ°ΠΆΠ΄ΠΎΠΌ рСзисторС Ρ€Π°Π²Π½Π° разности ΠΏΠΎΡ‚Π΅Π½Ρ†ΠΈΠ°Π»ΠΎΠ² Π½Π° Π±Π°Ρ‚Π°Ρ€Π΅Π΅.

    Π’ Ρ€ΡƒΠ±Ρ€ΠΈΠΊΠ΅: Π€ΠΈΠ·ΠΈΠΊΠ° Π‘ Ρ‚Π΅Π³Π°ΠΌΠΈ: Ρ€Π°Π±ΠΎΡ‡ΠΈΠΉ лист ΠΊΠΎΠΌΠ±ΠΈΠ½ΠΈΡ€ΠΎΠ²Π°Π½Π½Ρ‹Ρ… Ρ†Π΅ΠΏΠ΅ΠΉ с ΠΎΡ‚Π²Π΅Ρ‚Π°ΠΌΠΈ, ΠΏΡ€ΠΎΠ±Π»Π΅ΠΌΡ‹ слоТных ΠΏΠΎΡΠ»Π΅Π΄ΠΎΠ²Π°Ρ‚Π΅Π»ΡŒΠ½Ρ‹Ρ… ΠΈ ΠΏΠ°Ρ€Π°Π»Π»Π΅Π»ΡŒΠ½Ρ‹Ρ… Ρ†Π΅ΠΏΠ΅ΠΉ, Ρ‚ΠΎΠΊ Π² ΠΏΠΎΡΠ»Π΅Π΄ΠΎΠ²Π°Ρ‚Π΅Π»ΡŒΠ½ΠΎΠΌ ΠΈ ΠΏΠ°Ρ€Π°Π»Π»Π΅Π»ΡŒΠ½ΠΎΠΌ цСпях, Π²Ρ‹Π²ΠΎΠ΄ ΠΏΠΎΡΠ»Π΅Π΄ΠΎΠ²Π°Ρ‚Π΅Π»ΡŒΠ½ΠΎΠΉ ΠΊΠΎΠΌΠ±ΠΈΠ½Π°Ρ†ΠΈΠΈ, Ρ„ΠΎΡ€ΠΌΡƒΠ»Π° эффСктивного сопротивлСния, элСктричСство, ΠΎΠΏΡ€Π΅Π΄Π΅Π»Π΅Π½ΠΈΠ΅ эквивалСнтного сопротивлСния, ΠΏΡ€ΠΎΠ±Π»Π΅ΠΌΡ‹ эквивалСнтного сопротивлСния, ΠΊΠ°ΠΊ Π²Ρ‹ рассчитываСтС ΠΎΠ±Ρ‰Π΅Π΅ сопротивлСниС ΠΏΠΎΡΠ»Π΅Π΄ΠΎΠ²Π°Ρ‚Π΅Π»ΡŒΠ½ΠΎΠΉ Ρ†Π΅ΠΏΠΈ ?, ΠΊΠ°ΠΊ Π½Π°ΠΉΡ‚ΠΈ эквивалСнтноС сопротивлСниС, ΠΊΠ°ΠΊ Ρ€Π΅ΡˆΠΈΡ‚ΡŒ ΠΏΡ€ΠΎΠ±Π»Π΅ΠΌΡ‹ ΠΏΠΎΡΠ»Π΅Π΄ΠΎΠ²Π°Ρ‚Π΅Π»ΡŒΠ½ΠΎΠΉ ΠΏΠ°Ρ€Π°Π»Π»Π΅Π»ΡŒΠ½ΠΎΠΉ Ρ†Π΅ΠΏΠΈ, ΠΎΠΏΡ€Π΅Π΄Π΅Π»Π΅Π½ΠΈΠ΅ ΠΏΠ°Ρ€Π°Π»Π»Π΅Π»ΡŒΠ½ΠΎΠΉ схСмы, ΠΏΡ€ΠΈΠΌΠ΅Ρ€Ρ‹ ΠΏΠ°Ρ€Π°Π»Π»Π΅Π»ΡŒΠ½ΠΎΠΉ схСмы, Ρ€Π΅ΡˆΠ΅Π½ΠΈΡ ΠΏΡ€ΠΎΠ±Π»Π΅ΠΌ ΠΏΠ°Ρ€Π°Π»Π»Π΅Π»ΡŒΠ½ΠΎΠΉ Ρ†Π΅ΠΏΠΈ, рСзисторы Π² ΠΏΠΎΡΠ»Π΅Π΄ΠΎΠ²Π°Ρ‚Π΅Π»ΡŒΠ½ΠΎΠΉ ΠΈ ΠΏΠ°Ρ€Π°Π»Π»Π΅Π»ΡŒΠ½ΠΎΠΉ цСпях, ΠΏΡ€ΠΎΠ±Π»Π΅ΠΌΡ‹ ΠΈ ΠΎΡ‚Π²Π΅Ρ‚Ρ‹ для ΠΏΠΎΡΠ»Π΅Π΄ΠΎΠ²Π°Ρ‚Π΅Π»ΡŒΠ½Ρ‹Ρ… ΠΈ ΠΏΠ°Ρ€Π°Π»Π»Π΅Π»ΡŒΠ½Ρ‹Ρ… Ρ†Π΅ΠΏΠ΅ΠΉ , ΠΎΠΏΡ€Π΅Π΄Π΅Π»Π΅Π½ΠΈΠ΅ ΠΏΠΎΡΠ»Π΅Π΄ΠΎΠ²Π°Ρ‚Π΅Π»ΡŒΠ½ΠΎΠΉ Ρ†Π΅ΠΏΠΈ, ΠΏΠΎΡΠ»Π΅Π΄ΠΎΠ²Π°Ρ‚Π΅Π»ΡŒΠ½Π°Ρ ΠΏΡ€ΠΈΠ½Ρ†ΠΈΠΏΠΈΠ°Π»ΡŒΠ½Π°Ρ схСма, ΠΏΡ€ΠΎΠ±Π»Π΅ΠΌΡ‹ ΠΏΠΎΡΠ»Π΅Π΄ΠΎΠ²Π°Ρ‚Π΅Π»ΡŒΠ½ΠΎΠΉ Ρ†Π΅ΠΏΠΈ с Ρ€Π΅ΡˆΠ΅Π½ΠΈΡΠΌΠΈ, ΠΏΠΎΡΠ»Π΅Π΄ΠΎΠ²Π°Ρ‚Π΅Π»ΡŒΠ½Π°Ρ Ρ†Π΅ΠΏΡŒ ΠΏΡ€ΠΎΡ‚ΠΈΠ² ΠΏΠ°Ρ€Π°Π»Π»Π΅Π»ΡŒΠ½ΠΎΠΉ Ρ†Π΅ΠΏΠΈ.ΠΏΡ€ΠΈΠΌΠ΅Ρ€Ρ‹ ΠΏΠΎΡΠ»Π΅Π΄ΠΎΠ²Π°Ρ‚Π΅Π»ΡŒΠ½ΠΎΠΉ Ρ†Π΅ΠΏΠΈ, ΠΏΠΎΡΠ»Π΅Π΄ΠΎΠ²Π°Ρ‚Π΅Π»ΡŒΠ½Π°Ρ комбинация, ΠΏΠΎΡΠ»Π΅Π΄ΠΎΠ²Π°Ρ‚Π΅Π»ΡŒΠ½Π°Ρ комбинация сопротивлСния, ΠΏΡ€ΠΈΠΌΠ΅Ρ€Ρ‹ ΠΏΠΎΡΠ»Π΅Π΄ΠΎΠ²Π°Ρ‚Π΅Π»ΡŒΠ½ΠΎΠΉ ΠΏΠ°Ρ€Π°Π»Π»Π΅Π»ΡŒΠ½ΠΎΠΉ схСмы, Ρ€Π°Π±ΠΎΡ‡ΠΈΠΉ лист ΠΏΠΎΡΠ»Π΅Π΄ΠΎΠ²Π°Ρ‚Π΅Π»ΡŒΠ½ΠΎΠΉ ΠΏΠ°Ρ€Π°Π»Π»Π΅Π»ΡŒΠ½ΠΎΠΉ схСмы, эффСктивноС сопротивлСниС рСзисторов, соСдинСнных ΠΏΠΎΡΠ»Π΅Π΄ΠΎΠ²Π°Ρ‚Π΅Π»ΡŒΠ½ΠΎ, напряТСниС Π² ΠΏΠΎΡΠ»Π΅Π΄ΠΎΠ²Π°Ρ‚Π΅Π»ΡŒΠ½ΠΎΠΌ ΠΈ ΠΏΠ°Ρ€Π°Π»Π»Π΅Π»ΡŒΠ½ΠΎΠΌ соСдинСнии

    РСзисторы Π² ΠΏΠ°Ρ€Π°Π»Π»Π΅Π»ΡŒΠ½ΠΎΠΉ Ρ„ΠΎΡ€ΠΌΡƒΠ»Π΅

    Π’ элСктричСских цСпях часто ΠΌΠΎΠΆΠ½ΠΎ Π·Π°ΠΌΠ΅Π½ΠΈΡ‚ΡŒ Π³Ρ€ΡƒΠΏΠΏΡƒ рСзисторов ΠΎΠ΄Π½ΠΈΠΌ эквивалСнтным рСзистором. Π­ΠΊΠ²ΠΈΠ²Π°Π»Π΅Π½Ρ‚Π½ΠΎΠ΅ сопротивлСниС ряда рСзисторов, Π²ΠΊΠ»ΡŽΡ‡Π΅Π½Π½Ρ‹Ρ… ΠΏΠ°Ρ€Π°Π»Π»Π΅Π»ΡŒΠ½ΠΎ, ΠΌΠΎΠΆΠ½ΠΎ Π½Π°ΠΉΡ‚ΠΈ, ΠΈΡΠΏΠΎΠ»ΡŒΠ·ΡƒΡ ΠΎΠ±Ρ€Π°Ρ‚Π½ΠΎΠ΅ сопротивлСниС, 1 / R.ΠžΠ±Ρ€Π°Ρ‚Π½Π°Ρ Π²Π΅Π»ΠΈΡ‡ΠΈΠ½Π° эквивалСнтного сопротивлСния Ρ€Π°Π²Π½Π° суммС ΠΎΠ±Ρ€Π°Ρ‚Π½Ρ‹Ρ… Π²Π΅Π»ΠΈΡ‡ΠΈΠ½ ΠΊΠ°ΠΆΠ΄ΠΎΠ³ΠΎ сопротивлСния. Π•Π΄ΠΈΠ½ΠΈΡ†Π΅ΠΉ измСрСния сопротивлСния являСтся Ом (Ом), ΠΊΠΎΡ‚ΠΎΡ€Ρ‹ΠΉ Ρ€Π°Π²Π΅Π½ Π’ΠΎΠ»ΡŒΡ‚ Π½Π° АмпСр (1 Ом = 1 Π’ / А). Π’Π°ΠΊΠΆΠ΅ распространСны рСзисторы большСго Ρ€Π°Π·ΠΌΠ΅Ρ€Π° с сопротивлСниСм ΠΊΠΈΠ»ΠΎΠΎΠΌ (1 кОм = 10 3 Ом) ΠΈΠ»ΠΈ ΠΌΠ΅Π³Π°ΠΎΠΌ (1 МОм = 10 6 Ом).

    R eq = эквивалСнтноС сопротивлСниС (Ом ΠΈΠ»ΠΈ Π±ΠΎΠ»Π΅Π΅ Π΅Π΄ΠΈΠ½ΠΈΡ†Ρ‹)

    R 1 = сопротивлСниС ΠΏΠ΅Ρ€Π²ΠΎΠ³ΠΎ рСзистора (Ом)

    R 2 = сопротивлСниС Π²Ρ‚ΠΎΡ€ΠΎΠ³ΠΎ рСзистора (Ом)

    R 3 = сопротивлСниС Ρ‚Ρ€Π΅Ρ‚ΡŒΠ΅Π³ΠΎ рСзистора (Ом)

    ΠŸΠ°Ρ€Π°Π»Π»Π΅Π»ΡŒΠ½Ρ‹Π΅ рСзисторы Вопросы ΠΏΠΎ Ρ„ΠΎΡ€ΠΌΡƒΠ»Π΅:

    1) КакоС эквивалСнтноС сопротивлСниС 1000 кОм ΠΈ 250 Ом.РСзистор 0 кОм ΠΏΠΎΠ΄ΠΊΠ»ΡŽΡ‡Π΅Π½ ΠΏΠ°Ρ€Π°Π»Π»Π΅Π»ΡŒΠ½ΠΎ?

    ΠžΡ‚Π²Π΅Ρ‚: Оба сопротивлСния Π²Ρ‹Ρ€Π°ΠΆΠ΅Π½Ρ‹ Π² ΠΊΠΈΠ»ΠΎΠΎΠΌΠ°Ρ…, поэтому Π½Π΅Ρ‚ нСобходимости ΠΈΠ·ΠΌΠ΅Π½ΡΡ‚ΡŒ ΠΈΡ… Π΅Π΄ΠΈΠ½ΠΈΡ†Ρ‹. Π­ΠΊΠ²ΠΈΠ²Π°Π»Π΅Π½Ρ‚Π½ΠΎΠ΅ сопротивлСниС ΠΌΠΎΠΆΠ½ΠΎ Π½Π°ΠΉΡ‚ΠΈ Π² кОм ΠΏΠΎ Ρ„ΠΎΡ€ΠΌΡƒΠ»Π΅:

    .

    ПослСдний шаг – ΠΈΠ½Π²Π΅Ρ€Ρ‚ΠΈΡ€ΠΎΠ²Π°Ρ‚ΡŒ значСния с ΠΎΠ±Π΅ΠΈΡ… сторон Ρ„ΠΎΡ€ΠΌΡƒΠ»Ρ‹, Ρ‡Ρ‚ΠΎΠ±Ρ‹ Π½Π°ΠΉΡ‚ΠΈ эквивалСнтноС сопротивлСниС:

    R экв = 200,0 кОм

    Π­ΠΊΠ²ΠΈΠ²Π°Π»Π΅Π½Ρ‚Π½ΠΎΠ΅ сопротивлСниС 1000 кОм ΠΈ 250 Ом.ΠŸΠ°Ρ€Π°Π»Π»Π΅Π»ΡŒΠ½ΠΎ ΠΏΠΎΠ΄ΠΊΠ»ΡŽΡ‡Π΅Π½Π½Ρ‹Π΅ рСзисторы 0 кОм ΡΠΎΡΡ‚Π°Π²Π»ΡΡŽΡ‚ 200,0 кОм.

    2) Π’Ρ€ΠΈ рСзистора соСдинСны ΠΏΠ°Ρ€Π°Π»Π»Π΅Π»ΡŒΠ½ΠΎ Π² ΡΠ»Π΅ΠΊΡ‚Ρ€ΠΈΡ‡Π΅ΡΠΊΡƒΡŽ Ρ†Π΅ΠΏΡŒ. Π˜Ρ… сопротивлСния ΡΠΎΡΡ‚Π°Π²Π»ΡΡŽΡ‚ 400 Ом, 40,0 кОм ΠΈ 4,00 МОм. КакоС эквивалСнтноС сопротивлСниС?

    ΠžΡ‚Π²Π΅Ρ‚: Π’Ρ€ΠΈ значСния сопротивлСния Π²Ρ‹Ρ€Π°ΠΆΠ΅Π½Ρ‹ Π² Ρ€Π°Π·Π½Ρ‹Ρ… Π΅Π΄ΠΈΠ½ΠΈΡ†Π°Ρ… измСрСния. ΠŸΠ΅Ρ€Π²Ρ‹ΠΉ шаг ΠΊ Π½Π°Ρ…ΠΎΠΆΠ΄Π΅Π½ΠΈΡŽ эквивалСнтного сопротивлСния – ΠΏΡ€Π΅ΠΎΠ±Ρ€Π°Π·ΠΎΠ²Π°Ρ‚ΡŒ ΠΈΡ… Π² ΠΎΠ±Ρ‰ΡƒΡŽ Π΅Π΄ΠΈΠ½ΠΈΡ†Ρƒ. Π”Π²Π° значСния ΠΌΠΎΠΆΠ½ΠΎ ΠΏΡ€Π΅ΠΎΠ±Ρ€Π°Π·ΠΎΠ²Π°Ρ‚ΡŒ Π² Ρ‚Ρƒ ΠΆΠ΅ Π΅Π΄ΠΈΠ½ΠΈΡ†Ρƒ, Ρ‡Ρ‚ΠΎ ΠΈ Ρ‚Ρ€Π΅Ρ‚ΡŒΠ΅. Π’ этом Ρ€Π΅ΡˆΠ΅Π½ΠΈΠΈ всС значСния Π±ΡƒΠ΄ΡƒΡ‚ ΠΏΡ€Π΅ΠΎΠ±Ρ€Π°Π·ΠΎΠ²Π°Π½Ρ‹ Π² Ом.

    Π”ΠΎΠ±Π°Π²ΠΈΡ‚ΡŒ ΠΊΠΎΠΌΠΌΠ΅Π½Ρ‚Π°Ρ€ΠΈΠΉ

    Π’Π°Ρˆ адрСс email Π½Π΅ Π±ΡƒΠ΄Π΅Ρ‚ ΠΎΠΏΡƒΠ±Π»ΠΈΠΊΠΎΠ²Π°Π½. ΠžΠ±ΡΠ·Π°Ρ‚Π΅Π»ΡŒΠ½Ρ‹Π΅ поля ΠΏΠΎΠΌΠ΅Ρ‡Π΅Π½Ρ‹ *